Notes - Inspire Academy - Volume-1 Cs Executive

You might also like

Download as pdf or txt
Download as pdf or txt
You are on page 1of 442

TAX LAWS - VOLUME-I BY SALEEM QURAISHEE

The Law stated in this book is amended


By the Finance Act 2019

6TH EDITION

ASSESSMENT YEAR 2020-21

DEC-2020

SALEEM QURAISHEE Mo: 9175664444 INSPIRE ACADEMY-8888881719


TAX LAWS - VOLUME-I BY SALEEM QURAISHEE

MODULE- 1 TAX LAWS (PAPER-4)


A. INCOME TAX - 50 MARKS
B. GOODS & SERVICE TAX- 50 MARKS

VOLUME-I

S.NO CHAPTER VOLUME PAGE-NO

1) BASICS CONCEPTS I 1.1-1.44

2) RESIDENTIAL STATUS I 2.1- 2.39

3) INCOME THAT IS EXEMPT FROM TAX I 3.1-3.26

4) INCOME FROM SALARY I 4.1-4.70

5) INCOME FROM HOUSE PROPERTY I 5.1-5.37

6) PROFITS AND GAINS OF BUSINESS PROFESSION I 6.1-6.123

7) INCOME FROM OTHER SOURCES I 7.1-7.39

8) CLUBBING OF INCOME I 8.1-8.30

9) SET OFF AND CARRY FORWARD OF LOSSES I 9.1-9.24

10) AGRICULTURE INCOME I 10.1-10.16

SALEEM QURAISHEE Mo: 9175664444 INSPIRE ACADEMY-8888881719


TAX LAWS - VOLUME-I BY SALEEM QURAISHEE

VOLUME-II

S.NO CHAPTER VOLUME

11) CAPITAL GAINS II

12) DEDUCTIONS II

13) ADVANCE TAX II

14) TDS & TCS II

15) TAXATION OF VARIOUS ENTITIES II

A. TAXATION OF INDIVIDUAL
B. ALTERNATE MINIMUM TAX
C. TAXATION OF HUF
D. TAXTAION OF FIRM
E. TAXATION OF AOP/BOI
F. TAXATION COOPERATIVE SOCEITY
G. TAXATION OF POLITICAL PARTY & ELECTORAL TRUST
H. TAXATION OF CHARITABLE TRUST

16) ASSESSMENT OF COMPANIES II

17) ASSESSMENT PROCEDURE II

18) APPEAL & REVISION II

19) REFUND & RECOVERY II

SALEEM QURAISHEE Mo: 9175664444 INSPIRE ACADEMY-8888881719


TAX LAWS - VOLUME-I BY SALEEM QURAISHEE

VOLUME-III
INDIRECT TAX
S.NO CHAPTER PAGE-NO

1) BASICS OF GST

2) INTRODUCTION TO CGST ACT 2017

3) SUPPLY

4) LEVY & COLLECTION

5) TIME AND VALUE OF GOODS AND SERVICE

6) INPUT TAX CREDIT & ISD

7) JOB WORK

8) REGISTRATIONS

9) TAX INVOICE, CREDIT AND DEBIT NOTE

10) ACCOUNTS AND RECORDS

11) RETURNS

12) PAYMENTS.

13) REFUND

14) AUDIT

15) E-WAY BILL

16) IGST

17) UTGST

18) GST COMPENSATION CESS

19) CUSTOMS LAW

INCORPORATING:
 COMPREHENSIVE COVERAGE OF GST & CUSTOMS LAW-
 COMBINATION OF PRACTICAL PROBLEMS, CASE STUDY &MCQ

SALEEM QURAISHEE Mo: 9175664444 INSPIRE ACADEMY-8888881719


BASIC CONCEPTS Page 1.1

1. TAX
A. Meaning of TAX- Tax is a fee charged by a Government on a product, income or activity. There are
two types of taxes – direct taxes and indirect taxes.
 Direct Taxes: If tax is levied directly on the income or wealth of a person, then, it is a direct
tax e.g. income-tax.
 Indirect Taxes: If tax is levied on the price of a good or service, then, it is an indirect tax e.g.
Goods and Services Tax (GST) or Custom Duty. In the case of indirect taxes, the person
paying the tax passes on the incidence to another person.

B. Why are Taxes Levied?-The reason for levy of taxes is that they constitute the basic source of
revenue to the Government. Revenue so raised is utilized for meeting the expenses of Government
like defense, provision of education, health-care, infrastructure facilities like roads, dams etc.

C. Power to levy taxes CONSTITUTION OF INDIA- The roots of every law in India lies in the
Constitution, therefore understanding the provisions of Constitution is foremost to have clear
understanding of any law. Let us first understand what it talks about tax:
 Article 265– No tax shall be levied or collected except by the Authority of Law.
 Article 246- Distributes legislative powers including taxation, between the Parliament of India and
the State Legislature.
 Schedule VII- Enumerates powers under three lists
• Union List – Powers of Central Government
• Legislative List- Powers of State Government
• Concurrent List- Both Central and state Government have powers, in case of conflict;
law made by Union Government prevails.

• POINT TO BE NOTED-Income-tax is the most significant direct tax. Entry 82 of the


Union List i.e., List I of Seventh Schedule to Article 246 of the Constitution of India has
given the power to Central Government to levy taxes on income other than
agricultural income

SALEEM QURAISHEE-9175664444 INSPIRE ACADEMY-8888881719


BASIC CONCEPTS Page 1.2

D. DEFINITIONS- There is no precise and accurate definition for the tax and the concept of tax has
been defined differently by different economists. Some definitions are as follows.
1) According to Prof Seligman – A tax is compulsory contribution from the person to the government
to defray the expense incurred in the common interest of all without reference to special benefits
conferred.
2) According to Bastable – A tax as a compulsory contribution of the wealth of a person, or body of
persons for the service of public powers.
3) Hugh Dalton – A tax is a compulsory charge imposed by a public authority irrespective of the exact
amount of service rendered to the tax payer in return and not imposed as a penalty for legal
offence.
4) Jom Bouvier defined a tax as “A pecuniary burden imposed for support of the government, the
enforced proportional contribution of persons and property of the government and for all public
needs”
5) According to Trussing, “The essence of Tax as distinguished from other charges by government is
the absence direct quid pro quo- tit for tat between the tax payers and the public authority”.

E. CHART-

SALEEM QURAISHEE-9175664444 INSPIRE ACADEMY-8888881719


BASIC CONCEPTS Page 1.3

2. CHARACTERISTICS OF TAX
1) TAX IS COMPULSORY – A tax is imposed by law. So tax is compulsory payment to the governments
from its citizens.
2) TAX IS CONTRIBUTION – Contribution means in order to help or provide something. Tax is
contribution from members of community to the Government.
3) TAX IS FOR PUBLIC BENEFIT – Tax is levied for the common good of society without regard to
benefit to special individual.
4) NO DIRECT BENEFIT – Government is compulsorily collecting all types of taxes and does not give
any direct benefits to the tax payer for taxes paid. The essence of tax as distinguished from other
charges by governments is the absence of a direct quid-pro-que between the taxpayer and the
public authority.
5) TAX IS PAID OUT OF INCOME OF THE TAX PAYER
6) GOVERNMENT HAS THE POWER TO LEVY TAX –
7) TAX IS FOR THE ECONOMIC GROWTH AND PUBLIC WELFARE.

3. CANONS OF TAXES
 Canons of taxation refer to the administrative aspect of a tax. They are related to the rate,
amount, method of levy, and collection of a tax.
 Despite the modern development of economic sciences, Adam smith‘s canons of taxation, still
continue to be widely accepted as providing a good basis by which to judge taxes and these
principles still apply today.
 The fundamental canons of taxation are as follows:-
1) CANON OF EQUITY: This canon implies that any tax system should be based on the principle of
social justice. Equity refers to both horizontal and vertical equity. Horizontal equity describes the
concept that, taxpayers with equal abilities to pay should pay the same amount of tax. Vertical
equity means that taxpayers with a greater ability to pay should pay more tax.
2) CANON OF CERTAINTY : The tax rules should clearly specify when the tax is to be paid, how it is to
be paid, and how the amount to be paid is to be determined. Objective of this canon is to create
trust between two parties, first party taxpayer who is to pay the tax and second party the authority
whom receipt tax.
3) CANON OF CONVENIENCE : A tax should be due at a time or in a manner that is most likely to be
convenient for the taxpayer. Convenience in paying a tax helps to ensure compliance. The
appropriate payment mechanism depends on the amount of the liability and the how easy or
difficult it is to collect.
4) CANON OF ECONOMY : This canon implies that decreasing the administrative cost of collection of
the tax at the lowest level. The costs to collect a tax should be kept to a minimum for both the
government and taxpayers. This principle considers the number of revenue officers needed to
administer a tax. Compliance costs for taxpayers should also be considered. This principle is closely
related to the principle of simplicity.

SALEEM QURAISHEE-9175664444 INSPIRE ACADEMY-8888881719


BASIC CONCEPTS Page 1.4

4. OBJECTIVES OF TAXATION-
The Objectives of taxation in brief are as under:-
1) REVENUE - The main objective of taxes is to raise revenue to meet the Govt. expenditures
adequately.
2) SOCIAL OBJECTIVES – Taxes became a main goal for some of social objectives.
a) REDISTRIBUTION OF INCOME AND WEALTH:
 Imposition of high rate tax upon luxury commodities.
 Applying progressive tax system when levying taxes from taxpayers.
 Imposition of tax exemption to basic goods.
b) SOCIAL WELFARE:
c) SAFETY OF SOCIETY FROM BAD AND INJURIOUS CUSTOMS:

3) ECONOMIC SIGNIFICANCE OF TAXES- Taxes are used from economic point of view, so taxation
helps to encourage some economic activities, and as a tool to solve some economics problems. Tax
is also a means for directing of scarce economic activities. Taxation helps to accelerate economic
growth, and taxation plays very important role in case of economic stability.

4) ECONOMIC GROWTH:
5) ENFORCING GOVERNMENT POLICY:
6) DIRECTING LIMITED SCARCE RESOURCES INTO EFFECTIVE AND ESSENTIAL CHANNELS:
7) ECONOMIC STABILITY:

SALEEM QURAISHEE-9175664444 INSPIRE ACADEMY-8888881719


BASIC CONCEPTS Page 1.5

5. MERITS & DEMERITS OF DIRECT TAX


A. MERITS OF DIRECT TAXES

1) EQUITY : - Direct taxes have equity of sacrifice, depend upon the volume of income. They are based
on the principle of progressive, so rates of tax increase as the level of income of a person rises

2) ELASTICITY AND PRODUCTIVITY : - Direct taxes have elasticity because when the government faces
some emergency, like earthquake, floods and famine the government can collect money for facing
those problems by direct tax.

3) CERTAINTY :- Direct taxes have certainty on both sides‘ tax-payer and government. The tax- payers
are aware of the quantity of tax. They have to pay and rate, time of payment, manner of payment,
and punishment from the side of government is also certain about the total amount they are
getting.

4) REDUCE INEQUALITY: - Direct taxes follow progressive principles so it is taxing the rich people with
higher of taxation and the poor people with a lower level of taxation.

5) GOOD INSTRUMENT IN THE CASE OF INFLATION.:-Tax policy as fiscal instrument plays important
role in the case of the inflation, so government can absorb the excess money by arising in the rate
of existing taxes or imposition of new taxes.

6) SIMPLICITY: - Direct taxes are simplicity, while levy the rules, procedures, regulations of income tax
are very clear and simple.

B. DEMERITS OF DIRECT TAXES


1) Evasion: - Direct tax is lump sum therefore tax payers try evasion.

2) Uneconomically.:-Expenses of collection are larger in the case of direct taxes, because they require
widely- spread staff for collection

3) Unpopular:-Direct tax is required to be paid in lump sum for the whole year, so the tax payers feel
the painful payment, these taxes are therefore unpopular.

4) Little incentive to work and save:-In direct taxes, rates are of progressive nature. A person with
higher earning is taxed more, in turn he is left little with amount. So the tax payer feels
disincentive to work hard and save money after reaching a certain level of income.

5) Not suitable to a poor country:- Direct taxes are not enough to meet its expenditure.

6) Arbitrary:-Due to absence of logical or scientific principle to determine the degree of progression


in the taxation, the direct taxes are arbitrary.

SALEEM QURAISHEE-9175664444 INSPIRE ACADEMY-8888881719


BASIC CONCEPTS Page 1.6

6. MERITS & DEMERITS OF INDIRECT TAXES


A. MERITS OF INDIRECT TAXES-
1) High revenue production :-Nature of indirect taxes is imposition on the commodities and services.
Here indirect taxes cover a large number of essential goods and luxurious goods which are
consumed by the mass both rich and poor people, these help in collecting a large revenue.

2) No evasion. Nature of indirect tax is that, it is included in the price of commodity, so tax evasion or
tax avoid is difficult.

3) Convenient:-Indirect taxes are small amount and indirect taxes are hidden in the price of goods and
service, hence the burden of these taxes is not felt very much by the tax-payers, and not lump sum
like direct taxes.

4) Economy - Indirect taxes are economical in collection and the administrations costs of collection
are very low, also the procedure of collection of these taxes is very simply

5) Wide coverage:-Indirect taxes cover almost all commodities like essential commodities, luxuries,
and harmful ones.

6) Elasticity:-Since a large number of commodities and services are covered by indirect taxation there
is great scope for modification of taxes, goods and tax rate, much depends on nature of goods and
on its demands.

B. DEMERITS OF INDIRECT TAXES

1) Regressive in effect:-Essential commodities are used from all members of community. When taxing
these commodities the burden would be equal, and no distinction is made between the rich and
poor people.

2) Uncertainty in collection- Discourage savings and Increase inflation:-Indirect taxes are payable
when people spent their income or when people buy goods and services, so tax authorities cannot
accurately estimate the total yield from different indirect taxes.

3) Discourage savings- Increase inflation:-Indirect taxes are included in the price of commodity, so
people have to spend more money on essential commodities, when levied indirectly. In this case
that means the customers cannot save some of their money.

4) Increase inflation:-Indirect taxes increase the cost of input and output, increase in production
cost, push the price of goods. These reflect in increase in the wages of the workers.

SALEEM QURAISHEE-9175664444 INSPIRE ACADEMY-8888881719


BASIC CONCEPTS Page 1.7

7. DIRECT TAX AND INDIRECT TAX-


BASIS Direct Tax Indirect Tax
1) Nature Direct Taxes are the taxes directly Indirect Taxes are paid by one person
incurred and paid by the person (Assessee / Dealer), but the same is
concerned. recovered from another person (Consumer).
2) Levied on Levied on the income / wealth of the Levied on Purchase / Sale / Manufacture of
Assessee. goods and Provision of Services.
3) Shifting of There is no shifting of tax burden. Tax burden is shifted to the subsequent user.
Burden Hence, it is directly borne by the
taxpayer.
4) Time of Direct Taxes are collected after the It is collected at the time of sale or purchase
Collection income for a year is earned. or rendering of services.
5) TYPES Income Tax. [Wealth Tax Abolished from Goods and Services Tax, Customs Duty
Financial Year 2016-2017]
6)
7)
8)

8. BACKGROUND OF TAXATION SYSTEM IN INDIA


A. TAXATION IN INDIA IN ANCIENT TIME-
 In India, the system of direct taxation as it is known today, has been in force in one form or
another even from ancient times. There are references both in Manu Smriti and Arthasastra to a
variety of tax measures.
 Manu, the ancient sage and law-giver stated that the king could levy taxes, according to Sastras.
The wise sage advised that taxes should be related to the income and expenditure of the subject.
He, however, cautioned the king against excessive taxation and stated that both extremes should
be avoided namely either complete absence of taxes or exorbitant taxation. According to him, the
king should arrange the collection of taxes in such a manner that the subjects did not feel the pinch
of paying taxes.
 He laid down that traders and artisans should pay 1/5th of their profits in silver and gold, while the
agriculturists were to pay 1/6th, 1/8th and 1/10th of their produce depending upon their
circumstances.
 The detailed analysis given by Manu Smriti and Arthasastra on the subject clearly shows the
existence of a well-planned taxation system, even in ancient times.
 Taxes were paid in the shape of gold-coins, cattle, grains, raw-materials and also by rendering
personal service.
 Most of the taxes of Ancient India were highly productive.
 The admixture of direct taxes with indirect taxes secured elasticity in the tax system, although
more emphasis was laid on direct tax.
 The tax-structure was a broad based one and covered most people within its fold.
 The taxes were varied and the large variety of taxes reflected the life of a large and composite
population”.

SALEEM QURAISHEE-9175664444 INSPIRE ACADEMY-8888881719


BASIC CONCEPTS Page 1.8

B. ESTABLISHMENT OF INCOME TAX IN MODERN INDIA INCOME TAX ACT, 1860-

 Consequent upon the financial difficulties created by the events of 1857. Income Tax was
introduced in India for the first time by the British in the year 1860.
 The Act of 1860 was passed only for five years and therefore it lapsed in 1865.
 It was replaced 1867 by a license tax on professions and trades and the latter was converted into a
certificate tax in the following year.
 It was latter abolished in 1873.
 License tax traders remained in operation till 1886 when it was merged in the income tax Act of
that year.

C. INCOME TAX ACT, 1886 The Act of 1886 levied a tax on the income of residents as well as non
residents in India. The Act defined agricultural income and exempted it from tax liability in view of
the already existing land revenue a kind of direct taxes. The Act of 1886 exempted life insurance
premiums paid by assessee policies of his own life. Another important provision of this Act Hindu
undivided family was treated as a distinct taxable entity.

D. INCOME TAX ACT, 1918 -The Act of 1918 brought under change also receipts of casual or non-
recurring nature pertaining to business or professions. Although income tax in India has been a
charge on net income since inception, it was in the Act of1918 that specific provisions were
inserted for the first time pertaining to business deductions for the purpose of computing net
income. The Act of 1918 remained in force for a short period and was replaced by new Act (Act XI
of 1922) in view of the reforms introduced by the Govt. of India Act, 1919 .

E. INCOME TAX ACT, 1922- The organizational history of the income tax department dates back to the
year 1922. “one of the important aspects of the 1922 Act was that, it laid down the basis, the
mechanism of administering the tax and the rates at which the tax was to be levied would be laid
down in annual finance acts. This is procedure brought in the much needed flexibility in adjusting
the tax rates in accordance with the annual budgetary requirements and in securing a degree of
elasticity for the tax system. Before 1922 the tax rate were determined by the Income tax act itself
and to revise the rates, the act itself had to be amended. The Income tax Act, 1922 gave for first
time a specific nomenclature to various income tax authorities and laid the foundation of a proper
system of administration as per provisions of income tax act 1922 thus, it is the income tax act
1961, which is currently operative in India.

F. INCOME TAX ACT 1961-The present law of income tax in India is governed by the Income Tax Act,
1961 which is amended from time to time by the annual finance Act and other legislations
pertaining to direct tax. The act which came into force on April 1, 1962, replaced the Indian income
tax Act, 1922, which had remained in operation for 40 years. Furthermore, A set of rules known as
Income Tax Rules, 1962 have been framed for implementing the various provisions of the Act.

SALEEM QURAISHEE-9175664444 INSPIRE ACADEMY-8888881719


BASIC CONCEPTS Page 1.9

9. LAW
A. LAW

B. ADMINSTRATION

C. COLLECTION

SALEEM QURAISHEE-9175664444 INSPIRE ACADEMY-8888881719


BASIC CONCEPTS Page 1.10

10. COMPONENTS OF INCOME TAX LAW-


An understanding of the Income tax law requires a study of the following.
A. INCOME TAX ACT 1961.
I. The main source of income-tax law is the Income-tax Act, 1961.
II. It came into force on 1st April, 1962.
III. It contains 298 sections and XIV schedules.
IV. The Act is divided into sections; the sections are grouped under Chapters. The Act also contains
Schedules.
V. Many of the sections are further divided into sub-sections, clauses and sub-clauses, which are
denoted within brackets.

B. INCOME TAX RULES 1962


I. Rules are necessary for carrying out the purposes of the Act.
II. The Act gives power to the authority responsible for implementation of the Act to make appropriate
rules.
III. The Central Board of Direct Taxes (CBDT) governs the administration of Income-tax Act, 1961 in
India, for which purpose it frames rules from time to time.
IV. As per Sec. 295, the Board may, subject to the control of the Central Government, make rules for the
whole or any part of India for carrying out the purposes of the Act.
V. Such rules are made applicable by notification in the Gazette of India.
VI. Since then, many new rules have been framed or existing rules have been amended from time to
time and the same has been incorporated in the aforesaid rules
VII. These rules together form the Income-tax Rules, 1962.

C. Annual Finance Act-


I. The Finance Bill is introduced in the Parliament every year for implementing the tax proposals in the
Union Budget.
II. When the Finance Bill is approved by the Parliament and gets the assent of the President, it becomes
the Finance Act.
III. The amendments are made every year to the Income-tax Act, 1961 and other tax laws by the
Finance Act.
IV. The First Schedule to the Finance Act contains FOUR parts which specify the rates of tax –
Part I of the First Schedule to the Finance Act specifies the rates of tax applicable for the
current Assessment Year.
Part II specifies the rates at which tax is deductible at source for the current Financial Year.
Part III gives the rates for calculating income-tax for deducting tax from income chargeable
under the head "Salaries" and computation of advance tax.
Part IV gives the rules for computing net agricultural income.

D. Circulars/Notifications
I. Circulars are issued by the CBDT to clarify the meaning and scope of certain provisions contained in
the Act.
II. Notifications are issued by the Central Government to give effect to the provisions of the Act.
III. U/s 119, CBDT may issue certain circulars and clarifications from time to time, which have to be
followed and applied by the Income tax authorities.
IV. These are binding upon the Income tax authorities, but the same are not binding on the assessee.
However, assessee can claim benefit under such circulars.
V. Note: These circulars are not binding on the Income Tax Appellate Tribunal or on the Courts.

SALEEM QURAISHEE-9175664444 INSPIRE ACADEMY-8888881719


BASIC CONCEPTS Page 1.11

E. JUDICIAL DECISIONS-
I. The various issues which arise out of the provisions of the Act are decided by judicial forums.
II. The decisions of the Courts interpreting the provisions of the law also form an important constituent
of income-tax law.

POINT TO BE NOTED-
1) Decision of the Supreme Court: Any decision given by the Supreme Court shall be applicable as law till there is
any change in law by the Parliament. Such decision shall be binding on all the Courts, Tribunals, Income tax
authorities, assessee, etc.
2) Contradiction in the decisions of the Supreme Court: In case, there is apparently contradiction in two decisions,
the decision of larger bench, whether earlier or later, shall always prevail. However, where decisions are given,
by benches having equal number of judges, the decision of the recent case shall be applicable.
3) Decisions given by a High Court or ITAT: Decisions given by a High Court or ITAT are binding on all assessee and
Income tax authorities, which fall under their jurisdiction, unless it is over ruled by a higher authority

11. EFFECTIVE DATES OF AMENDMENT IN FINANCE BILL


A. DIRECT TAXES-Date of notification in official gazette or in the Finance act.
B. INDIRECT TAXES-Midnight of date of presentation of Finance bill.

12. ASSESSMENT YEAR [SEC. 2(9)] & PREVIOUS YEAR {SEC 3}

A. ASSESSMENT YEAR
I. "Assessment year" means the period starting from April 1 and ending on March 31 of the next
year.
II. Income of previous year on an assessee is taxed during the next following assessment year at the
rates prescribed by the relevant Finance Act

B. PREVIOUS YEAR
I. Income earned in a year is taxable in the next year.
II. The year in which income is earned is known as previous year and the next year in which income
is taxable is known as assessment year.

C. UNIFORM PREVIOUS YEAR


I. From the assessment year 1989-90 onwards, all assesses are required to follow financial year (i.e.
April 1 to March 31) as the previous year.
II. This uniform previous year has been followed for all sources of income.

SALEEM QURAISHEE-9175664444 INSPIRE ACADEMY-8888881719


BASIC CONCEPTS Page 1.12

 PY IN THE CASE OF NEWLY SET-UP BUSINESS/PROFESSION

In the case of a newly set-up business/profession or in the case of a new source of income, the previous
year is determined as follows –
First Previous Year Second and subsequent
previous years

Starting point It commences on the date of setting up of the April 1


business/profession or on the date when the new
source of income comes into existence.

Ending point Immediately following March 31. March 31 of the following


year

Duration of previous 12 months or less 12 months


year
A financial year has a double role to play – It is a previous year as well as an assessment year:

 WHEN INCOME OF PREVIOUS YEAR IS NOT TAXABLE IN THE


IMMEDIATELY FOLLOWING ASSESSMENT YEAR –

a) Shipping business of non-residents [Sec. 172]:


I. The assessee is a nonresident.
II. He owns a ship or ship is chartered by a nonresident.
III. The ship carries passengers, livestock, mail or goods at a port in India.
IV. The nonresident may or may not have an agent in India.
 If all the aforesaid conditions are satisfied, 7.5 per cent of amount received (or receivable) on
account of such carriage (including demurrage charge or handling charge or similar amount) by
the non-resident shall be deemed to be the income the non-resident.
 Tax is payable at the rate applicable to a foreign company.

PROBLEM:1 STAR Ltd. is a foreign company which is non-resident in India. It is operating ships.
One of the ships had come to India and wants to leave Indian port on 25/12/2019. The company has
collected total fare and freight of Rs. 15 Lakh. Discuss its tax implications and how tax has to be paid.
ANSWER:1 As per Section 44B, 7.5% of Rs. 15,00,000 i.e. Rs. 1,12,500 shall be the deemed income of
STAR Ltd. On this amount, it shall be liable to pay tax. The Captain of the ship has to pay the amount
of tax before the ship has to leave Indian port and has to file the Income Tax Return thereof.

SALEEM QURAISHEE-9175664444 INSPIRE ACADEMY-8888881719


BASIC CONCEPTS Page 1.13

b) Person leaving India [Sec. 174]:


I. If appears to the Assessing Officer that an individual may leaves India during the current
assessment year or shortly after its expiry.
II. He has no present intention of returning to India.
III. The total income of such individual up to the probable date of his departure from India shall
be chargeable to tax in that assessment year.

PROBLEM:2 Mr. JACK a foreign citizen is resident in India since 2001. While completing
assessment for the PY 2019-2020 i.e. AY 2020-2021, on 5/3/2021, the Assessing Officer comes to
know that Mr. JACK will leave India on 20/3/2021 with no intention of returning back.
ANSWER:2 In this case, the Assessing Officer will make two assessments in the AY 2020-2021:
(a) Assessment for the income of PY 2019-2020, and
(b) Assessment for the income of period 1/4/2020 to 20/3/2021
For making the first assessment, the rates shall be as applicable for AY 2020-2021 and for second
assessment the rates shall be as applicable for the AY 2020-2021.

c) Association/bodies formed for short duration [Sec. 174A]:


I. There is an association of persons or a body of individuals or an artificial juridical person,
formed or established or incorporated for a particular event or purpose.
II. If appears to the Assessing Officer that the above mentioned association, body, etc., is likely
to be dissolved in the assessment year (i.e., April to March) in which such association of
persons or body of individuals or artificial juridical person was formed or established or
immediately after such assessment year.
III. The total income of such association or body or juridical person for the period from the
expiry of the previous year for that assessment year up to the date of its dissolution shall be
chargeable to tax in that assessment year.

PROBLEM:3 Mr. JOY and Mr. DEAN form an Association of Person on 10/6/2018 to fulfill the
contract given by DMRC Ltd. The contract has to be completed by 10/6/2019 and on the same date
this association shall be dissolved. AOP has filed its return of income for the PY 2018-2019 i.e. AY
2019-2020.While doing the assessment on 4/4/2019 the Assessing Officer comes to know about the
date of dissolution.
ANSWER:3 Assessing Officer will do two different assessments. One for the period 10/6/2018 till
31/3/2019.For this period rate of tax shall be the rate applicable for the AY 2019-2020. The second
assessment shall be done for the period from 1/4/2019 till 10/6/2019. For this period, rate of tax
shall be the rate applicable for the AY 2019-2020.

SALEEM QURAISHEE-9175664444 INSPIRE ACADEMY-8888881719


BASIC CONCEPTS Page 1.14

d) Person likely to transfer property to avoid tax [Sec. 175]:


I. If appears to the Assessing Officer during any assessment year that a person is likely to
charge, sell, transfer, dispose of (or otherwise part with) any of his asset.
II. Such asset may be movable or immovable.
III. The taxpayer is likely to part with asset with a view to avoiding payment of any liability under
the Income-tax Act.
IV. The total income of such person from the first day of the assessment year to the date when
proceeding is started under section 175 is taxable in that assessment year.

PROBLEM:4 On 19/12/2019 Assessing Officer comes to know that AJAY Ltd. is likely to dispose of
a house property during January 2020 with a view to avoiding payment of income-tax. A notice is
issued by the Assessing Officer on 28/12/2019 under section 175 to submit return of income of the
period commencing on 1/4/2019 to 28/12/2019.
ANSWER:4 In this case, income from 1/4/2019 to 28/12/2019 is chargeable to tax in the AY 2019-2020.

e) Discontinued business [Sec. 176]:


I. A business or profession is discontinued in any assessment year.
II. Income of the business/ profession from April 1 of the assessment year (in which the
business profession is discontinued.
III. The above income is taxable at the discretion of the Assessing Officer in assessment year in
which business is discontinued or it may be taxed in the normal assessment year (i.e.,
assessment year immediately fooling the previous year).

PROBLEM:5 Mr. SANDEEP has discontinued his cloth business on 31/12/2019.


ANSWER:5 The Assessing Officer, at his discretion, can tax the income earned from 1/4/2019 till
31/12/2019 in the PY 2019-2020 i.e. AY 2020-2021 at the rate of tax applicable for AY 2019-2020 or
wait for the AY 2020-2021 to start, then do the assessment by using the rate of taxes applicable for
the AY 2020-2021

A. Points to be noted.

SALEEM QURAISHEE-9175664444 INSPIRE ACADEMY-8888881719


BASIC CONCEPTS Page 1.15

13. PERSON- [SEC. 2(31)]

PERSON INCLUDES-

I. Individual- The term ‘individual’ means only a natural person, i.e., a human being.
 It includes both males and females.
 It also includes a minor or a person of unsound mind. But the assessment in such a case may be made
on the guardian or manager of the minor or lunatic.
 In the case of deceased person, assessment would be made on the legal representative.

II. HUF-Under the Income-tax Act, 1961, a Hindu undivided family (HUF) is treated as a separate entity
for the purpose of assessment. It is included in the definition of the term “person” under section
2(31). The levy of income-tax is on “every person”. Therefore, income-tax is payable by a HUF.

III. Company [Section 2(17)]- For all purposes of the Act the term ‘Company’, has a much wider
connotation than that under the Companies Act. Under the Act, the expression ‘Company’ means:
1) any Indian company as defined in section 2(26); or
2) anybody corporate incorporated by or under the laws of a country outside India, i.e., any
foreign company; or
3) any institution, association or body which is assessable or was assessed as a company for any
assessment year under the Indian Income-tax Act, 1922 or for any assessment year commencing
on or before 1.4.1970 under the present Act; or
4) any institution, association or body, whether incorporated or not and whether Indian or non-
Indian, which is declared by a general or special order of the CBDT to be a company for such
assessment years as may be specified in the CBDT’s order.

Classes of Companies
A. Domestic company [Section 2(22A)] - means an Indian company or any other company which, in
respect of its income liable to income-tax, has made the prescribed arrangements for the declaration and
payment of dividends (including dividends on preference shares) within India, payable out of such
income.
Indian company [Section 2(26)]- Two conditions should be satisfied so that a company can be
regarded as an Indian company -
a) The company should have been formed and registered under any law relating to companies
which was or is in force in any part of India, and
b) The registered office or the principal office of the company should be in India.
The expression ‘Indian Company’ also includes:
I. A corporation established by or under a Central, State or Provincial Act (like Financial Corporation or
a State Road Transport Corporation),
II. An institution or association or body which is declared by the Board to be a company under section
2(17)(iv) provided its registered or principal office is in India.
III. In the case of the State of Jammu and Kashmir, a company formed and registered under any law for
the time being in force in that State.
IV. In the case of any of the Union territories of Dadra and Nagar Haveli, Goa, Daman and Diu, and
Pondicherry, a company formed and registered under any law for the time being in force in that
Union territory.

SALEEM QURAISHEE-9175664444 INSPIRE ACADEMY-8888881719


BASIC CONCEPTS Page 1.16

B. Foreign company [Section 2(23A)]- Foreign company means a company which is not a domestic
company

IV. Firm [Section 2(23)]- The terms ‘firm’, ‘partner’ and ‘partnership’ have the same meanings as
assigned to them in the Indian Partnership Act, 1932. In addition, the definitions also include the
terms limited liability partnership, a partner of limited liability partnership as they have been
defined in the Limited Liability Partnership Act, 2008. However, for income-tax purposes a minor
admitted to the benefits of an existing partnership would also be treated as partner.

V. Association of Persons (AOP) When persons combine together for promotion of joint enterprise
they are assessable as an AOP when they do not in law constitute a partnership. In order to
constitute an association, persons must join for a common purpose or action and their object must
be to produce income; it is not enough that the persons receive the income jointly. Co-heirs, co-
legatees or co-donees joining together for a common purpose or action would be chargeable as an
AOP.
VI. Body of Individuals (BOI) It denotes the status of persons like executors or trustees who merely
receive the income jointly and who may be assessable in like manner and to the same extent as the
beneficiaries individually. Thus, co-executors or co-trustees are assessable as a BOI as their title and
interest are indivisible. Income-tax shall not be payable by an assessee in respect of the receipt of
share of income by him from BOI and on which the tax has already been paid by such BOI.
 AOP vs BOI
Basis AOP BOI
1) Creation Created voluntarily. Created by operation of Law.
2) Members AOP may consist of Individuals or non-individuals. BOI consists of Individuals only.
3) Condition AOP means two or more persons joining together The business run by a widow on her
for a common purpose to earn income, and without behalf or on behalf of her children
an intention to form Partnership. would be assessed as BOI.
4) Examples Co-Heirs, Co-Legatees or Co-Donees joining Co-Executors, Co-Trustees are
together for common purpose/action shall be assessable as BOI.
chargeable as AOP.
POINT TO BE NOTED-
I. No restriction on number of Members: The number of members is not restricted for AOP or BOI.
II. Share of Member: The share of each member need not be definite or ascertainable.
III. Assessment: The Income Tax Act does not give any option to the Assessing Officer to assess either
AOP or its Members individually.

VII. Local Authority The term means a municipal committee, district board, body of port commissioners
or other authority legally entitled to or entrusted by the Government with the control or
management of a municipal or local fund.

VIII. Artificial Persons-This category could cover every artificial juridical person not falling under other
heads. An idol, or deity would be assessable in the status of an artificial juridical person.

SALEEM QURAISHEE-9175664444 INSPIRE ACADEMY-8888881719


BASIC CONCEPTS Page 1.17

PROBLEM:6 Determine the status of the following:


1. Delhi University
2. DCM. Ltd.
3. Delhi Municipal Corporation.
4. Taxman Publication (P.) Ltd.
5. Laxmi Commercial Bank Ltd.
6. ABC Group Housing Co-operative Society.
7. XY & Co., firm of X and Y.
8. A joint family of X, Mrs. and their sons A and B
9. X and Y who are legal heirs of Z (Z died in 1996 and X and Y carry on his business without entering into
partnership).
ANSWER:6

14. ASSESSEE [SEC. 2(7)]

1) Assessee means any person who is liable to pay any tax or any other sum under the Income Tax Act,
1961.
2) Assessee includes -
a) Every person in respect of whom any proceedings has been taken for the assessment of –
 His Income or Fringe Benefits, or Income of any other person.
 Loss sustained by him or other person.
 Refund due to him or such other person.
b) Every person who is deemed to be an Assessee under the Act
c) Every person who is deemed to be an Assessee in Default under the Act.

3) Deemed Assessee-“Deemed Assessee” means a person who is treated as an Assessee under the IT
Act. This would include -
I. Trustee of a Trust,
II. Legal Representative of a Deceased Person u/s 159,
III. Representative Assessee of a Non-Resident u/s 160(2) (Agent of a Non-Resident),
IV. Legal Guardian or Manager entitled to receive the Income on behalf of a Minor, Lunatic or
Idiot,
V. Court of Wards/Official Trustee/Receiver entitled to receive Income on behalf of any other
person.

4) Assessee in Default-Assessee in Default: Assessee in Default includes persons who -


I. Fail to deduct and remit TDS (Sec. 191).
II. Fail to pay tax and any other sum demanded (Sec. 220).

SALEEM QURAISHEE-9175664444 INSPIRE ACADEMY-8888881719


BASIC CONCEPTS Page 1.18

15. HOW TO CHARGE TAX ON INCOME- SECTION 4

I. Annual tax – Income-tax is an annual tax on income.


II. Tax rate of assessment year – Income of previous year is chargeable to tax in the next following
assessment year at the tax rates applicable for the assessment year.
III. Rates fixed by Finance Act – Tax rates are fixed by the annual Finance Act and not by the Income-
tax Act.
IV. Tax on person – Tax is charged on every person
V. Tax on total income – Tax is levied on the "total income"

16. "INCOME" UNDER THE INCOME-TAX ACT-

The definition of the term "Income" in section 2(24) is inclusive and not exhaustive.
Meaning of income –
I. Income is a periodical monetary return with some sort of regularity.
II. It may be recurring in nature.
III. It may be broadly defined as the true increase in the amount of wealth which comes to a person
during a fixed period of time.

SALEEM QURAISHEE-9175664444 INSPIRE ACADEMY-8888881719


BASIC CONCEPTS Page 1.19

17. CONCEPT OF INCOME

1) Income should come from regular and definite source.


2) Income can be in any form i.e. in cash or in kind.
3) Income also includes illegal income.
4) Disputed title to income does not make any difference.
5) Income should be real and not fictional.
6) A contingent income is not an income.
7) Receipt in lump sum or in installment does not make any difference. Difference between capital
receipt and revenue receipt must be kept in mind.
8) Income can be taxed either on receipt or accrual basis.
9) Personal gifts received on occasions like birthdays, marriage is not income.
10) Pin money received by a lady for personal/household expenditure is not an income.
11) Surplus from mutual activity: A person cannot make taxable profit out of a transaction with himself.
Income must, therefore, come from outside. A surplus arising to a mutual concern cannot be
regarded as income chargeable to tax.

12) Income includes loss: Income includes loss. While income, profits and gains represent "plus
income", losses represent "minus income".

13) Appropriation of payment between capital and interest: Where interest is due on a capital sum and
the creditor gets an open payment from the debtor, the creditor is at liberty to appropriate the
payment towards principal. If, however, neither the debtor nor the creditor makes any
appropriation of payment as between capital and interest, the Income-tax Department is entitled to
treat the payment as applicable to the outstanding interest and assess it as income.

14) Same income cannot be taxed twice:

15) Application of Income Vs Diversion of Income


 APPLICATION OF INCOME-
1) Income accrues to the assessee.
2) Income reaches the assessee.
3) Income is applied to discharge an obligation.

 DIVERSION OF INCOME-
An obligation to apply the income in a particular way before it is received by the assessee or before it
has arisen or accrued to assessee results in diversion of income.
1) Income is diverted at source.
2) There is an overriding charge or title for such diversion; and
3) The charge or the obligation is on the source of income and not on the receiver.

SALEEM QURAISHEE-9175664444 INSPIRE ACADEMY-8888881719


BASIC CONCEPTS Page 1.20

18. DIFFERENCE BETWEEN EXEMPTION AND DEDUCTION


 EXEMPTION-
I. If an income is exempt from tax, it is not included in the computation of income.
II. Exemption can never exceed the amount of income.
 DEDUCTION-
I. Deduction is generally given from income chargeable to tax.
II. Deduction can be less than or equal to or more than the amount of income.
III. If the amount deductible is more than the amount of income, the resulting amount will be taken
as loss.

19. CAPITAL AND REVENUE RECEIPTS


An amount referable to fixed capital is a capital receipt whereas a receipt referable to circulating capital
would be a revenue receipt. While the latter is chargeable to tax, the former is not subject to income-tax
unless otherwise expressly provided.
A. Type of Capital
1) Fixed capital-Fixed capital is that which is not involved directly in the process of business but
remains unaffected by the process.
2) Circulating Capital-Circulating capital is that part of the capital which is turned over in the business
and which ultimately results in profit or loss. For instance, the proceeds of sale of stock-in-trade is
a revenue receipt while the sale proceeds of building, machinery or plant will be capital receipt.

B. DIFFERENCE BETWEEN CAPITAL RECEIPT AND REVENUE RECEIPT


Capital Receipts Revenue Receipts
1) Any amount received towards fixed capital or 1) Amount received towards circulating capital
for fixed asset is Capital Receipt. or for floating asset is a Revenue Receipt.
2) Any receipt towards substitution of a source of 2) Any receipt towards substitution of income is
income is a capital receipt. a revenue receipt.
3) The amount received as a compensation for 3) Any compensation received for the loss of
surrender of any rights of ownership is a capital future profit is a revenue receipt.
receipt. 4) Profits arising from sale of a Trading Asset are
4) Profit from Sale of Capital Asset shall be taxable as Business Income u/s 28.
chargeable to tax under the head Capital Gains
u/s 45
CHART

SALEEM QURAISHEE-9175664444 INSPIRE ACADEMY-8888881719


BASIC CONCEPTS Page 1.21

20. METHOD OF ACCOUNTING -SEC 145

I. The profits from business and profession and income under the head 'Income from Other
Sources' are to be computed in accordance with the method of accounting regularly
employed by the assessee.
II. As per section 145 of the Income tax Act only one of the following two methods of accounting
can be followed Mercantile system OR Cash system.
III. The income chargeable under the head PGBP/OS shall be computed in accordance with
INCOME COMPUTATION AND DISCLOSURE STANDARDS to be notified by the Central
Government from time to time
IV. If income has not been computed as per above said ICDS the AO may make assessment in the
manner provided under section 144.
V. These standards will not be applicable for maintenance of books of account but shall be
relevant for computation of total income and disclosure of information in the return
Method of accounting irrelevant in some cases:
1) Income from salary
2) Income from house property
3) Income under the head capital gains

21. INDIA [SECTION 2(25A)


The term 'India' means –
I. The territory of India as per Article 1 of the Constitution,
II. Its territorial waters, seabed and subsoil underlying such waters.
III. Continental shelf.
IV. Exclusive economic zone or
V. Any other specified maritime zone and the air space above its territory and territorial waters.
Specified maritime zone means the maritime zone as referred to in the Territorial Waters,
Continental Shelf, Exclusive Economic Zone and other Maritime Zones Act, 1976

SALEEM QURAISHEE-9175664444 INSPIRE ACADEMY-8888881719


BASIC CONCEPTS Page 1.22

22. COMPUTATION OF TOTAL INCOME AND TAX LIABILITY-

NAME-
PY-2019-20
AY-2020-21
PARTICULARS AMOUNT AMOUNT
1) Income from salaries XXXX
2) Income from house property XXXX
3) Profits and gains of business or profession XXXX
4) Capital gains XXXX
5) Income from other sources XXXX
Total [(1) +(2)+(3)+ (4)+ (5)] XXXX
Less: Adjustment on account of set-off and carry forward of losses XXXX
GROSS TOTAL INCOME XXXX
Less: Deduction u/s 80C to 80U XXXX
TOTAL INCOME/NET INCOME/TAXABLE INCOME XXXX

COMPUTATION OF TAX LIABLITY


TAX ON NET INCOME XXXX
LESS –REBATE U/S 87A XXXX
TAX XXXX
ADD-SURCHARGE XXXX
TAX AND SURCHARGE XXXX
LESS-Marginal relief (if any) XXXX
ADD-HEC XXXX
TAX XXXX
LESS- RELIEF U/S 86/89 XXXX
RELIEF U/S 90 XXXX
RELIEF U/S 90A XXXX
RELIEF U/S 91 XXXX
TAX XXXX
LESS-PREPAID TAXES
 ADVANCE TAX XXXX
 TDS XXXX
 TCS XXXX
TAX PYABLE OR REFUND +-XXXX

23. ROUNDING-OFF OF INCOME AND TAX (SEC. 288A & 288B)


I. The taxable income shall be rounded off to the nearest multiple of ten rupees and for this purpose any
part of a rupee consisting of paisa shall be ignored
II. If such amount is not a multiple of ten, then, if the last figure in that amount is five or more, the amount
shall be increased to the next higher amount which is a multiple of ten
III. If the last figure is less than five, the amount shall be reduced to the next lower amount which is a
multiple of ten.
Tax liability actually worked out (Rs.)
Tax liability as rounded off (Rs.)

SALEEM QURAISHEE-9175664444 INSPIRE ACADEMY-8888881719


BASIC CONCEPTS Page 1.23

24. WHAT IS AMALGAMATION [SEC.2 (18)]


For a merger to qualify as an "amalgamation" for the purpose of the Income-tax Act, it has to satisfy the following
conditions:

1 All the properties of the amalgamating company immediately before the amalgamation should become the
property of the amalgamated company by virtue of the amalgamation.

2 All liabilities of the amalgamating company immediately before the amalgamation should become the
liabilities of the amalgamated company by virtue of the amalgamation.

3 Shareholder holding not less than three-fourths (in value) of the shares in the amalgamating company (other
than shares already held by the amalgamated company or by its nominee) should become shareholders of the
amalgamated company by virtue of the amalgamation.

25. WHAT IS "DEMERGER" [SEC.2 (19AA)].


1. All the property of the undertaking, being transferred by the demerged company, becomes the property
of the resulting company.
2. All the liabilities relatable to the undertakings being transferred by the demerged company become the
liabilities of the resulting company.
3. The property and the liabilities of the undertaking being transferred by the demerged company are
transferred at values appearing in its books of account immediately before the demerger. For this
purpose, any change in the value of assets consequent to their revaluation shall be ignored.
4. The resulting company issues shares to the shareholders of the demerged company on a proportionate
basis as a consideration for demerge
5. The shareholders holding not less than three-fourths in value of the shares in the demerged company
(other than the shares already held therein immediately before the demerger, or by a nominee for, the
resulting company or, its subsidiary) become shareholders of the resulting company.
6. The transfer of the undertaking is on a going concern basis.
7. The demerger is in accordance with the conditions, if any, notified under section 72A(S)

SALEEM QURAISHEE-9175664444 INSPIRE ACADEMY-8888881719


BASIC CONCEPTS Page 1.24

26. "TAX RATES ARE NOT GIVEN UNDER THE INCOME-TAX ACT, 1961 BUT
BY THE ANNUAL FINANCE ACT", ----

I. Provisions for computation of taxable income are given by the Income-tax Act.
II. Tax rates are not given by the Income-tax Act, but by the Finance Act which is passed by
Parliament along with budget for the Central Government every year.
III. The Finance Act, 2019, provides tax rates in the First Schedule (Parts I, II and III) as follows –

 Part I of the First Schedule to the Finance Act, 2019: It gives income-tax rates for different
assessee for the assessment year 2019-20 ie PY 2018-19.

 Part II of the First Schedule to the Finance Act, 2019 : It gives rates for deduction of tax at source
applicable for the financial year 2019-20.

 Part III of the First Schedule to the Finance Act, 2019: It gives tax rates for different assesses for
payment of advance tax during the financial year 2019-20(i.e., for the assessment year 2019-20).
The same rates are applicable, for the tax deduction from salary payment during the financial year
2019-20.

 Generally, Part III of the First Schedule of a Finance Act becomes Part I of the First Schedule of the
subsequent Finance Act. For instance, Part III of the First Schedule to the Finance Act, 2019 will
become Part I of the First Schedule to the Finance Act, 2020.

SALEEM QURAISHEE-9175664444 INSPIRE ACADEMY-8888881719


BASIC CONCEPTS Page 1.25

27. TAX RATES- FINANCE ACT 2019

A. INDIVIDUAL/HUF/AOP/BOI/ARTIFICIAL JUDICIAL PERSON-


1. NORMAL RATES AY-2020-21
INDIVIDUAL/HUF/AOP/BOI/AJP SENIOR CITIZEN (60 OR MORE) SUPER SENIOR CITIZEN (80 OR MORE)
INCOME - RATE INCOME- RATE INCOME- RATE
0- 250000 = Nil 0- 300000 = Nil 0- 500000 = Nil
Next 250000 = 5% Next 200000 = 5% Next 500,000 = 20%
Next 500,000 = 20% Next 500,000 = 20% Balance = 30%
Balance = 30% Balance = 30%

POINT TO BE NOTED-
I. CLARIFICATIONS REGARDING ATTAINING PRESCRIBED AGE OF 60/80 YEARS ON 31st MARCH ITSELF-
 IN CASE OF SENIOR/VERY SENIOR CITIZENS WHOSE DATE OF BIRTH FALLS ON 1st APRIL:
CIRCULAR No. 28/2016, dated 27-7-2016 .
 CBDT has clarified that a person born on 1st April would be considered to have attained a
particular age on 31st March, the day preceding the anniversary of his birthday.

II. NRI not to be treated senior citizen.

2. REBATE- 87A - Income tax rebate shall be provided to the persons who fall under the lower income
tax slab if following conditions are satisfied:
I. Allowed to Individual only (Resident in India)
II. T.I does not exceed ` 5,00,000
III. 100% I.T payable or ` 12500 2500 (whichever
( is less)
POINT TO BE NOTED-
I. Rebate under section 87A is, however, not available in respect of tax payable @10% on long-term
capital gains taxable under section 112A.
II. Health and Education Cess shall be imposed only after allowing rebate under Section 87A

SALEEM QURAISHEE-9175664444 INSPIRE ACADEMY-8888881719


BASIC CONCEPTS Page 1.26

3. SURCHARGE- INDIVIDUAL/HUF/AOP/BOI/ AJP (Amended from AY 2020-2021)


Type of Income Income

Income of Rs. Income more Income more Income more Income more
`50 lakhs or than `50 lakhs than `100 lakhs than `200 lakhs than `500 lakhs
less but less than or but less than or but less than or
equal to `100 equal to `200 equal to `500
lakhs lakhs lakhs

Long-term capital NIL 10% 15% 15% 15%


gain covered under
Section 112A

Short-term capital NIL 10% 15% 15% 15%


gain covered under
Section 111A

Unexplained 25% 25% 25% 25% 25%


income chargeable
to tax under
Section 115BBE

Any other Income NIL 10% 15% 25% 37%

4. H & EC- @ 4% of (Tax + SC)

 POINT TO BE NOTED-
A. SPECIAL BENEFITS FOR ONLY RESIDENT INDIVIDUALS/HUF-
1) Deficiency in Normal Income: This benefit is available to a resident individual/HUF provided his
normal income is below the exemption limit (2,50,000/3,00,000/5,00,000).
2) The difference between the normal income and the exemption limit is referred to as deficiency.
3) Such deficiency is allowed to be set-off against the special income and tax is charged @ 20% or 15%
or 10% (as the case may be) on the balance income remaining after adjustment of deficiency.
4) Deficiency needs to be adjusted in the following order:
 Firstly, against LTCG
 Balance against STCG u/s 111A/LTCG-112A
 No deficiency can be adjusted against Casual Income
B. Special income-
I. LTCG-20%
II. STCG-111A-15%/LTCG-112A
III. Casual income-30%
 No deduction under section 80C-80U is allowed from above three special income-

SALEEM QURAISHEE-9175664444 INSPIRE ACADEMY-8888881719


BASIC CONCEPTS Page 1.27

B. FIRM (INCLUDING LLP)/LOCAL AUTHORITY


1. FLAT RATE- 30%
2. SURCHARGE- @12% OF TAX IF T.I EXCEEDS 1 CR
3. H & EC- @ 4% of (Tax + SC)

C. CO-OPERATIVE SOCEITY-
1.
First 10000 -10%
Next 10000 -20%
Balance -30%
2. SURCHARGE- @12% OF TAX IF T.I EXCEEDS 1 CR
3. H & EC- @ 4% of (Tax + SC)

D. COMPANY-
1) A Domestic company whose total turnover or Gross receipts ………..not exceeding 400Cr in
PY 2017-18……….25%

2) Any other Domestic Company -30%


3) Foreign Company- 40%
4) SURCHARGE-
If Net income does If N.I exceeds 1Cr but does not exceed 10 Cr. If N.I exceeds 10 Cr
not exceed 1 Cr
DOMESTIC CO. NIL 7% 12%
FOREIGN CO. NIL 2% 5%

5) H &EC- @ 4% of (Tax + SC)

E. MARGINAL RELIEF-
1) Marginal Relief is a reduction from the Tax Payable by the Assessee.
2) Relief from Tax Payable shall be given, where the Tax Payable together with Surcharge exceeds
the Income earned by an Assessee in excess of `50 Lakhs or `1 Crore. Such Relief is known as
Marginal Relief.
3) The principle in Marginal Relief is that the Additional Amount of Income Tax Payable with
Surcharge in excess of Income over `50 Lakhs or `1 Crore, should not be more than the
amount in excess of `50 Lakhs or `1 Crore
4) Computation of Marginal Relief
Marginal Relief = Tax on Total Income including Surcharge
Less: (Total Income – `50 Lakhs or`1 Crore) + (Tax on `50 Lakhs or `1 Crore
excluding Surcharge)
Tax Payable = Tax on Total Income including Surcharge
Less: Marginal Relief as computed above

SALEEM QURAISHEE-9175664444 INSPIRE ACADEMY-8888881719


BASIC CONCEPTS Page 1.28

F. SPECIAL RATES OF TAX-( FIXED BY INCOME TAX ACT)


SECTION NATURE OF INCOME RATE
111A Short term capital gains from transfer of securities on which STT has charged 15%

112 Long term capital gains (other than LTCG taxable as per section 112A) 20%

112A Long term capital gains on transfer of – 10%


I. Equity shares in a company [On LTCG>
II. Unit of an equity-oriented fund ` 1 lakh]
III. Unit of business trust
Condition for availing the benefit of this concessional rate is Securities
Transaction tax should have been paid–
I. Equity share in a company – Both at the time of acquisition and transfer
II. Unit of an equity-oriented fund- At the time of transfer
III. Unit of business trust- At the time of transfer
Note: LTCG upto ` 1 lakh is exempt. LTCG exceeding ` 1 lakh is taxable @10%.

115BB Winnings from lotteries, cross word puzzles, races, horse races, card games and 30%
others games of any sort or gambling or betting of any form or nature whatsoever.
115BBE 1) Where the TI referred in section 68/69/69A/69B/69C/69D and reflected in ROI
furnished under section 139(1) or
2) Determined by AO
60%
I. Cash credit [Sec. 68]-
II. Unexplained investments [Sec. 69]- +
III. Unexplained money, etc. [Sec. 69A] – 25%
IV. Amount of investments, etc., not fully disclosed in books of account [Sec. (Surcharge)
69B]- +
V. Unexplained expenditure, etc. [Sec. 69C] – 4% HEC
VI. Amount borrowed or repaid on hundi [Sec. 69D] –
Point to be noted-
I. No basic exemption or allowance or expenditure shall be allowed to the
assessee under any provision of the Income-tax Act, 1961 in computing such
deemed income.
II. Further, no set off of any loss shall be allowable against income brought to
tax under sections 68 or section 69 or section 69A or section 69B or section
69C or section 69D.
115BBDA Income by way of dividend in excess of ` 10 Lakhs in case of an I/HUF/Firm who is 10%
resident in India

115BBF Income by way of royalty in respect of patent developed and registered in India in 10%
respect of person who is resident in India

115BBG Income from transfer of carbon credits- 10%

SALEEM QURAISHEE-9175664444 INSPIRE ACADEMY-8888881719


BASIC CONCEPTS Page 1.29

G. SPECIAL RATES OF TAX FOR COMPANIES-

1) Tax on income of certain domestic manufacturing companies.115BA.


I. 25% tax in case of Domestic manufacturing company subject to other provisions of chapter XII
(other than 115BAA/115BAB)
II. Conditions-
a) The company has been set-up and registered on or after the 1st day of March, 2016.
b) The company engaged in the business of manufacture or production of any article or thing.
c) Company should not have claimed benefit of section 10AA/32AD/ADDITIONAL DEP/33AB/
33ABA /35 (1) (ii) /(iia)(iii) /35(2AA)/35(2AB)/35CCC/35CCD or any deduction in respect of
income under chapter VI-A other than the provisions of section 80JJAA;
d) Option has to be exercised due date of return filing.
e) Provided that once the option has been exercised for any previous year, it cannot be
subsequently withdrawn for the same or any other previous year.
f) Provided further that where the person exercises option under section 115BAA, the option
under this section may be withdrawn.

2) SECTION 115BAA -BY THE TAXATION LAWS (AMENDMENT) ORDINANCE, 2019


I. In case of domestic company for assessment year 2020-21 and future assessment years the tax on
total income of domestic company shall be computed at the rate of 22 percent + 10% SC+4%
HEC.
II. At the option of the company
III. Option to be exercised by due date of filing of return under section 139(1)
IV. Option once exercised will apply to subsequent assessment years
V. Option once exercised cannot be withdrawn for same or subsequent assessment year
VI. Assessee company shall not claim deductions
 under section 10AA
 under sections 80IA to section 80RRB except section 80JJAA
 under section 32(l)(iia) i.e. additional depreciation
 under section 32AD i.e. investment allowance
 under section 33AB Tea/Coffee/Rubber Development Account
 under section 35(1)(ii) i.e. 150% deduction on donations to scientific research institution
 under section 35(1)(iii) i.e. 100% deductions on donations for social science and statistical research
 under section 35(2AA) i.e. research by IIT and specified person (150% deduction)
 under section 35(2AB) i.e. in house research laboratory (150% deduction)
 under section 35CCC
 under section 35CCD
VII. Companies which opt for section 115BAAshall not be required to pay MAT.

SALEEM QURAISHEE-9175664444 INSPIRE ACADEMY-8888881719


BASIC CONCEPTS Page 1.30

3) SECTION 115BAB -BY TAXATION LAWS (AMENDMENT) ORDINANCE, 2019-

CONDITIONS FOR AVAILING THE BENEFIT OF SECTION 115BAB A domestic company can avail of
the benefit of Section 115BAB when it fulfills following conditions:

I. The domestic company should be incorporated on or after 1/10/2019


II. It should commence the manufacturing on or after 1/10/2019 but before 31/3/2023
III. It must be engaged in the business of manufacture or production of any article or thing and
research in relation to, or distribution of, such article or thing manufactured or produced by it
IV. The option to avail of the benefit of section 115BAB must be exercised on or before the due date
u/s 139(1) for furnishing of first ITR. This option once exercised cannot be withdrawn
subsequently
V. It must not be formed by splitting up or reconstruction of an existing business. However, this
condition is not applicable in case of an undertaking formed as a result of re-establishment,
reconstruction or revival in accordance with the provisions of section 33B.
VI. It does not use any building which was previously used as a hotel or a convention centre
VII. It does not use any second hand plant or machinery. However, maximum of 20% of total value of
plant or machinery can be second hand plant or machinery.
VIII. Any plant or machinery used outside India shall not be treated as second hand plant or
machinery if following conditions are satisfied:
 Before the date of installation, they were not used in India
 These assets were imported into India and
 No deduction on account of depreciation has been allowed on such plant and machinery before
they were installed by the assessee
IX. It cannot claim deductions as referred in section 115BAA if it opts for 15% rate
X. Company shall not be allowed to set-off of any loss carried forward from earlier assessment year
if such loss is attributable to any of the aforesaid deductions.
XI. Companies which opt for section 115BABshall not be required to pay MAT
XII. Provided that once the option has been exercised for any previous year, it cannot be
subsequently withdrawn for the same or any other previous year.

SALEEM QURAISHEE-9175664444 INSPIRE ACADEMY-8888881719


BASIC CONCEPTS Page 1.31

28. KEY POINTS


1) MAXIMUM MARGINAL RATE OF TAX: It means the rate of income-tax (including surcharge on
income-tax) applicable in relation to the highest slab of income in the case of an individual,
association of persons or body of individuals as specified in the Finance Act of the relevant year
SECTION 2(29C):

2) AVERAGE RATE OF INCOME TAX- As per Section 2(10), average rate of income-tax means the rate
arrived at by dividing the amount of income-tax calculated on the total income, by such total
income.

3) RATES-

4) TAX,CESS & SURCHARGE-

SALEEM QURAISHEE-9175664444 INSPIRE ACADEMY-8888881719


BASIC CONCEPTS Page 1.32

29. SECTION SUMMARY


SECTIONS PARTICULARS

SALEEM QURAISHEE-9175664444 INSPIRE ACADEMY-8888881719


BASIC CONCEPTS Page 1.33

PRACTICAL QUESTIONS
PROBLEM:7 Compute the tax liability in the following cases for Assessment year 2020-21.
I. Mr Raj has total income of Rs 8,00,000
II. Mr Raj has total income of Rs 10,00,000
III. Mr RAJ has total income of Rs 13,00,000
ANSWER:7

PROBLEM:8 Mr. Santosh has income as given below:


I. Income under the head Salary 4,00,000
II. Income under the head House Property 5,00,000
III. Income under the head Business/Profession 6,30,250
IV. Deductions allowed under section 80C to 80U are Rs. 1,10,000.
Compute total income the tax liability for previous year 2019-20.
ANSWER:8

PROBLEM:9 Compute tax liability in the following cases for the assessment year 2020-21.
I.Mr. X (resident) has total income of Rs. 14,00,000
II. Mr. X (non-resident) has total income of Rs. 16,00,000
III. Mrs. X (resident) has total income of Rs. 14,00,000
IV.Mrs. X (non-resident) has total income of Rs. 18,00,000
V.Mr. X (resident), aged 60 years has total income of Rs. 16,00,000
VI.Mrs. X (resident), aged 60 years has total income of Rs. 19,00,000
VII. Mr. X (non-resident), aged 60 years has total income of Rs. 11,00,000
VIII. Mrs. X (non-resident), aged 60 years has total income of Rs. 12,00,000
IX. Mr. X (resident), aged 80 years has total income of Rs. 16,50,000
X. Mrs. X (resident), aged 80 years has total income of Rs. 11,00,000
XI. Mr. X (non-resident), aged 80 years has total income of Rs. 13,00,000
XII. Mrs. X (non-resident), aged 80 years has total income of Rs. 12,00,000
ANSWER:9

PROBLEM:10 Compute tax liability in the following cases for Assessment Year 2020-21-
I. Mr A (resident) has total income of Rs 18,00,000
II. Mr A (non-resident) has total income of Rs 22,00,000
III. Mr A (resident), aged 60 years has total income of Rs 25,00,000
IV. Mr A (non-resident), aged 60 years has total income of Rs 24,60,000
V. Mr A (resident), aged 80 years has total income of Rs 21,50,000
VI. Mr A (non-resident), aged 80 years has total income of Rs 22,00,000
ANSWER:10

PROBLEM:11 Compute tax liability in the following cases for Assessment Year 2020-21.
I. Mrs A (resident) has total income of Rs 50,50,000
II. Mrs A (Non -resident) has total income of Rs 52,00,000
III. Mrs A (resident), aged 60 years has total income of Rs 51,80,000
IV. Mrs A (resident), aged 80 years has total income of Rs 52,50,000
V. Mrs A (resident) has total income of Rs 1,01,00,000
VI. Mrs A (resident) has total income of Rs 1,02,00,000
VII. Mrs A (resident) has total income of Rs 1,03,00,000
ANSWER:11

SALEEM QURAISHEE-9175664444 INSPIRE ACADEMY-8888881719


BASIC CONCEPTS Page 1.34

PROBLEM:12 Compute the tax liability of Mr. A having total income of Rs 3,48,000.
ANSWER:12

PROBLEM:13 Compute the tax liability of Mr. A for Assessment Year 2020-21.
I. Gross total income Rs 4,90,000
II. Deductions allowed u/s 80C to 80U are Rs 1,30,000
ANSWER:13

PROBLEM:14 Compute the tax liability of Mrs. A, aged 64 years for PY 2019-20 (AY 2020-21):
Gross total income Rs 4,44,000; Deductions allowed u/s 80C to 80U are Rs 88,000"
a) Case 1 Mrs. A is a resident
b) Case 2: Mrs. A is a non-resident
ANSWER:14

PROBLEM:15 Mr. X has income under the head Salary Rs.5,00,000 and casual income Rs. 300000
and deduction under section 80C to 80G Rs. 2,00,000, in this case his tax liability shall be
ANSWER:15

PROBLEM:16 Mr. X has casual income of Rs.102,00,000 and deduction allowed under section
80C to 80U are Rs. 1,00,000, in this case his tax liability shall be
ANSWER:16

PROBLEM:17 Mr. X has income under the head Salary Rs.70,000 and casual income Rs.2,50,000
and deduction u/s 80C to 80U Rs.40,000. Compute his tax liability assessment year 2020-21.
ANSWER:17

PROBLEM:18 Compute the tax liability of Mr Z under the following two cases if he has salary
income of Rs 6,00,000 and casual income of Rs 2,00,000.
I. Deductions u/s 80C to 80U Rs 1,80,000.
II. Deductions u/s 80C to 80U Rs 7,00,000.
ANSWER:18

PROBLEM:19 Compute the tax liability of Mr. Y having casual income of Rs 100 lakhs and
deductions allowed u/ 80C to 80U are Rs 2,50,000.
ANSWER:19

PROBLEM:20 Compute the tax liability of Mr. Y having casual income of Rs 101 lakhs and
deductions allowed u/ 80C to 80U are Rs 2,50,000.
ANSWER:20

PROBLEM:21 Compute the tax liability of Mr. Y having casual income of Rs 102 lakhs and
deductions allowed u/ 80C to 80U are Rs 2,50,000.
ANSWER:21

SALEEM QURAISHEE-9175664444 INSPIRE ACADEMY-8888881719


BASIC CONCEPTS Page 1.35

PROBLEM:22 Mr. MANOJ has incomes as given below;


I. Income under the head salary 35,000
II. Income under the head house property 45,000
III. Income under the head business/profession 30,000
IV. Long term capital gains 1,10.000
V. Long term capital gains u/s 112A 5,00,000
VI. Short term capital gains 25,000
VII. Short term capital gains u/s 111A 7,00,000
VIII. Casual Income (winnings of lottery) 55,000
IX. Other income 3,000
X. Deductions allowed under section 80C to 80U 2,00,000
Compute his tax liability for the assessment year 2020-21.
a) Case 1: Mr MANOJ is resident.
b) Case 2: Mr MANOJ is resident and aged about 68 years.
c) Case 3- Mr MANOJ is resident and aged about 81 years.
d) Case 4: Mr MANOJ is non-resident.
ANSWER:22

PROBLEM:23 Mr X has income under the head house property Rs.5,00,000 and LTCG 112A Rs.
1,00,000. Compute tax payable.
ANSWER:23

PROBLEM:24 Compute tax liability for the assessment year 2020-21 in the following situations:
I. Mr. X is resident in India and has income under the head house property Rs.40,000 and income
under the head salary Rs.30.000 and long term capital gains Rs.4,80,000.
II. Presume in the above situation the assessee is Mrs. X.
III. Presume in the above situation the assessee is Mrs. X and she is aged about 70 years.
IV. Presume in the above situation the assessee is Mr, X and he is aged about 70 years.
V. Presume in the above situation the assessee is Mrs. X and she is aged about 85 years.
VI. Presume in the above situation the assessee is Mr. X and he is aged about 85 years.
VII. Presume in all the above situations, the assessee is non-resident in India.
ANSWER:24

PROBLEM:25 Compute tax liability in the following cases


I. Mr. X a resident has long term capital gains Rs.3,50,000.
II. Mr. X a resident has casual income Rs.3,50,000.
III. Mr. X a resident has short term capital gains u/s 111A Rs.3,50.000
IV. Mr. X a non-resident has long term capital gains Rs.3,50,000
V. Mr. X a non-resident has casual income-Rs.3,50,000.
VI. Mr. X a non-resident has short term capital gains u/s 11 1A Rs.3,50,000.
VII. Mr. X a non-resident aged 61 years has long term capital gains Rs.3,50,000.
VIII. Mr. X a non-resident aged 61 years has casual income Rs.3,50,000.
IX. Mr. X a non-resident aged 61 years has short-term capital gains u/s 111A Rs.3,50.000.
ANSWER:25

SALEEM QURAISHEE-9175664444 INSPIRE ACADEMY-8888881719


BASIC CONCEPTS Page 1.36

PROBLEM:26 Compute the tax liability of X Ltd., a domestic company, assuming that the total
income of X Ltd. is Rs. 1,01,00,000 and the total income does not include any income in the nature
of capital gains.
ANSWER:26

PROBLEM:27
Calculate income tax liability for AY 2020-21

Mr. A (age 45) Mrs. B (age 62) Mr. C (age 81) Mr. D (age 82)

Status Resident Non-Resident Resident Non -Resident

Total income other than LTCG `2,80,000 `3,30,000 `5,90,000 ` 4,80,000

LTCG `15,000 from `10,000 from sale `60,000 from Nil


sale of land of listed equity sale of
shares on which agricultural land
STT is paid in rural area

ANSWER:27

PROBLEM:28 Compute the tax liability of X Ltd,, a domestic company, assuming that the total
income of X Ltd. is Rs. 1,01,00,000 and the total income does not include any income in the nature
of capital gains.
ANSWER:28

PROBLEM:29 Compute tax liability of ABC Ltd. a domestic company in the following situations:
I. The company has income under the head Business/Profession Rs.60,000.
II. The company has income under the head Business/Profession Rs.2.00,000.
III. The company has income under the head Business/Profession Rs.600,00,000.
IV. The company has income under the head Business/Profession Rs.1 00,00,000.
V. The company has long term capital gains of Rs.70,000.
VI. The company has long term capital gains of Rs.300,00,000.
VII. The company has long term capital gains of Rs.5,00,000.
VIII. The company has long term capital gains of Rs.10,20,000.
ANSWER:29

SALEEM QURAISHEE-9175664444 INSPIRE ACADEMY-8888881719


BASIC CONCEPTS Page 1.37

29. MCQ
PROBLEM:1 Taxes can be broadly divided into ................
(a) Direct taxes (b) Indirect taxes (c) Direct or Indirect taxes (d) Ancillary taxes

PROBLEM:2 Direct Taxes are taxes that are ................


(a) Directly paid to the government by the taxpayer
(b) Applied on the manufacture or sale of goods and services
(c) Both (a) and (b) (d) Either (a) or (b)

PROBLEM:3 Indirect Taxes are taxes that are ................


(a) Directly paid to the government by the taxpayer
(b) Applied on the manufacture or sale of goods and services
(c) Paid to the government by an intermediary who collects from the taxpayer (d) Both (b) and (c)

PROBLEM:4 Which of the following is the characteristic of Income Tax?


(a) Direct tax on the person on whom it is levied, Le. its burden cannot be shifted
(b) Various kinds of income on which income tax is levied are divided into five heads
(c) The rate of tax could be different for different kind of income
(d) All of the above

PROBLEM:5 Which of the following is not a characteristic of Income Tax?


(a) Direct tax on the person on whom it is levied, ie. its burden cannot be shifted
(b) There could be certain deductions from income or the entire income may be taxed at a flat rate
(c) Income tax is collected by the Government department, called Income tax Department
(d) None of the above

PROBLEM:6 Income tax is ................


(a) A “Quarterly Tax” (b) A “Half Yearly Tax”
(c) An “Annual Tax”, which may be paid in instalments (d) An “Annual Tax” paid at the end of the year

PROBLEM:7 Income Tax of Previous Year, is charged to tax in the immediately following year called the
................
(a) Assessment Year (b) Succeeding Year (c) Preceding year (d) Taxable year

PROBLEM:8 Tax is levied at rates of income tax, fixed by the relevant ................
(a) Finance Act (b) Finance Bill (c) Both of the above (d) Either of the above

PROBLEM:9 In case, the Finance Act has not been passed as on April 1 of the relevant AY, rates of
income tax in the provision of the ................ applies to ascertain tax on income.
(a) Finance Bill (b) Previous Finance Act
(c) Finance Bill/Previous Finance Act, whichever are more beneficial to taxpayer
(d) Finance Bill/Previous Finance Act, whichever are more beneficial to tax office

PROBLEM:10Law as on ................ is applied for income tax purpose.


(a) April 1 of the relevant AY (b) April 1 of the relevant PY
(c) March 31 of the relevant AY (d) March 31 of the relevant PY

SALEEM QURAISHEE-9175664444 INSPIRE ACADEMY-8888881719


BASIC CONCEPTS Page 1.38

PROBLEM:11In order to understand the Income Tax Law, a person has to study which of the following?
(a) Income-tax Act, 1961 and the Income-tax Rules, 1962
(b) Notifications, Circulars and Clarifications issued by (Central Board of Direct taxes)
(c) Judicial decision by various Courts (d) All of the above

PROBLEM:12The rate at which total income of an assessee are taxable are categorized into two parts,
one is slab rate which is applicable on ................ and the other is flat rate which is applicable on
................
(a) Individuals, Companies (b) Companies, Individuals
(c) Partnership Firm, Companies (d) Companies, Partnership Firm

PROBLEM:13Income-tax law is passed, in which of the following sequence?


i. FM presents Finance Bill ii. Passed by Both Houses of Parliament
iii. Receives Assent of the President iv. Becomes Finance Act
(a) i, ii, iii, iv ( b) i, iii, iv, ii
(c) i, iv, ii, iii,
(d) iii, iv, ii, i.

PROBLEM:14Finance Bill is presented by the ................


(a) Prime Minister (b) Defense Minister (c) Finance Minister (d) Either of the above

PROBLEM:15Finance Bill contains amendments to, and provides for ................


(a) Exiting provisions of Income-tax Act, 1961 (b) Direct and Indirect tax rates
(c) Direct tax rates (d) Both (a) and (b)

PROBLEM:16Income-tax Act, 1961 came into force as on ................


(a) 1.4.1962 (b) 1.4.1961 (c) 31.3.1961 (d) 31.3.1962

PROBLEM:17 There are ................ parts in the First Schedule.


(a) 1 (b) 2 (c) 3 (d) 4

PROBLEM:18Income-tax Act provisions are applicable ................


(a) Throughout India including Jammu and Kashmir
(b) Throughout India excluding Jammu and Kashmir
(c) India and the country where a non-resident who earns from India lives
(d) All of the above

PROBLEM:19Income-tax Act, 1961 is divided into ................


(a) 298 section and 14 Schedules (b) 14 section and 298 Schedules
(c) 291 section and 14 Schedules (d) 298 section and 24 Schedules

PROBLEM:20Income-tax Act contains provisions relating to ................


(a) Determination of taxable income (b) Determination of tax liability
(c) Income-tax procedures relating to assessment, appeals and penalties
(d) All of the above

PROBLEM:21Income-tax Act is subject to amendments from time to time which are brought about by
various
(a) Finance Acts (b) Finance bill (c) Finance Minister (d) All of the above

SALEEM QURAISHEE-9175664444 INSPIRE ACADEMY-8888881719


BASIC CONCEPTS Page 1.39

PROBLEM:22CBDT has power to make rules u/s ................


(a) 295 (b) 296 (c) 297 (d) 298

PROBLEM:23CBDT has power to make rules which are necessary for the ................
(a) Administration of the tax office (b) Manner in which the court has to give decisions
(c) Conduct of taxpayer (d) Implementation of the provisions of the Income-tax Act, 1961

PROBLEM:24Notification is a subordinate legislation and is issued under powers delegated by


................
(a) Parliament (b) Supreme Court (c) High Court (d) Central Government

PROBLEM:25Income Tax Notifications lay down the general law which deals with ................ aspects of
the Income-tax Act.
(a) Procedural (b) Legal (c) Material (d) Immaterial

PROBLEM:26Income Tax Notifications can be issued by ................


(a) The Central Government (b) CBDT (c) Either (a) or (b) (d) Both (a) and (b)

PROBLEM:27Income Tax Notifications are binding on ................


(a) Both the assessee as well as the Income-tax authorities (b) Assessee
(c) Income-tax authorities (d) Either (b) or (c)

PROBLEM:28CBDT to issue circulars from time to time, to ................


(a) Clarify doubts regarding the meaning and scope of the various provision of Act
(b) Lay down procedural provision (c) Both (a) and (b) (d) Neither (a) nor (b)

PROBLEM:29As per section 2(31), person includes ................


(a) HUF (b) Local Authority (c) Company (d) All of the above

PROBLEM:30As per section 2(31), person includes ................


(a) Individual (b) Limited Liability Partnership
(c) Partnership firm (d) All of the above

PROBLEM:31As a general rule, income of minor is taxable in the hands of ................


(a) Minor (b) Legal guardian including Parents (c) Either (a) or (b) (d) Both (a) and (b)

PROBLEM:32HUF includes ................


(a) All person vertically descended from a common ancestor
(b) All person lineally descended from a common ancestor
(c) All person upwardly descended from a common ancestor
(d) All of the above

PROBLEM:33In the case of an AOP, two or more persons join together for
(a) Common purpose (b) Earning income
(c) Both (a) and (b) (d) Either (a) or (b)

PROBLEM:34The provision relating to computation of income are same in the case of which of the
following two entities?
(a) AOP and BOI (b) AOP and Partnership firm
(c) AOP and LLP (d) BOI and LLP

SALEEM QURAISHEE-9175664444 INSPIRE ACADEMY-8888881719


BASIC CONCEPTS Page 1.40

PROBLEM:35AOP may consist of ................, but BOI has to consist of ................


(a) Individuals, Non-individuals (b) Non-individuals, Individuals
(c) Either (a) or (b) (d) None of the above

PROBLEM:36Which of the following are considered as a "Local Authority’’?


(a) Panchayat (b) Municipality and Cantonment Board
(c) Municipal committee entrusted by Government to control Municipal Funds
(d) All of the above

PROBLEM:37Assessment Year is a period of ................


(a) 12 months (b) 15 months (c) 18 months (d) Either (a) or (b)

PROBLEM:38Assessment year generally starts on ................


(a) 1st January every year (b) 1st April every year
(c) The date on which business commences its operations (d) Either (a) or (b)

PROBLEM:39Generally, Previous Year is a period of ................


(a) 12 months before the Assessment Year (b) 12 months after the Assessment Year
(c) 12 months parallel to the Assessment Year (d) 12 months coinciding with the Assessment Year

PROBLEM:40The first previous year of a newly started business ................


(a) Must be equal to 12 months (b) Can exceed 12 months
(c) Can be equal to or less than 12 months (d) None of the above

PROBLEM:41Section 172 deals with the taxation of ................ of non-residents.


(a) Aviation business (b) Shipping business
(c) Transport business (d) Trading business

PROBLEM:42Section 172 deals with the taxation of shipping business of non-residents, casts an
obligation on ................ to file return of full amount earned from India.
(a) Master of ship (b) Taxpayer
(c) Owner of ship (d) All of the above

PROBLEM:43As per section 172, ................ of the amount of fare shall be deemed to be income of non-
resident carrying on shipping business on which income-tax will be charged.
(а) 5% (b) 7.5% (c) 9% (d) 10%

PROBLEM:44Delhi University is considered as ................


(a) Artificial Juridical Person (b) Association of Persons
(c) Body of Individuals (d) Local Authority

PROBLEM:45In which of the following case/s, income of the previous year is assessed in the same year?
(a) Shipping business of non-residents
(b) Income of persons leaving India not having intention to return to India
(c) Assessment of persons likely to transfer property to avoid tax
(d) All of the above

SALEEM QURAISHEE-9175664444 INSPIRE ACADEMY-8888881719


BASIC CONCEPTS Page 1.41

PROBLEM:46SAMEER, intends to permanently settle in USA, and intends to depart on December 13th,
2019. The AO, having jurisdiction on Sunil, came to know about this fact and proceeded to tax Sunil
on the current financial year income. The action of the Assessing Officer is ................
(a) Justified (b) Not justified
(c) Can be justified provided Sunil gives an undertaking to return to India
(d) None of the above

PROBLEM:47Part 1 of schedule I of the Finance Act, 2019 gives rate of income tax for the
(a) AY 2019-2020 (b) AY 2020-2021
(c) AY 2018-2019 (d) AY 2017-2018

PROBLEM:48Part 2 of schedule of I of the Finance Act, 2019 gives rate of tax deductible of source for
the
(a) PY 2019-2020 (b) PY 2020-2021 (c) PY 2018-2019 (d) PY 2017-2018

PROBLEM:49Part 3 of schedule I of The Finance Act, 2019 gives rate of advance tax payable for the
(a) AY 2019-2020 (b) AY 2020-2021 (c) AY 2018-2019 (d) AY 2017-2018

PROBLEM:50The rates of income-tax are mentioned in—


(a) The Income-tax Act, 1961 only(b) Both Income-tax Act, 1961 and Income-tax Rules, 1962
(c) The First Schedule to the Annual Finance Act
(d) Both Income-tax Act, 1961 and the First Schedule to the Annual Finance Act

PROBLEM:51The rate of Health and Education Cess for the AY 2020-2021 i.e. PY 2019-2020 is
(a) 2% (b) 2% + 1%
(c) 3% (d) 4%

PROBLEM:52 AJAY is a software professional aged 30 years. The exemption limit for A for AY 2020-21
is ...
(a) Rs. 1,80,000 (b) Rs. 2,50,000 (c) Rs. 3,00,000 (d) Rs. 5,00,000

PROBLEM:53 BINU is a medical professional aged 65 years. The exemption limit for B for AY 2020-21 is
(a) Rs. 1,80,000 (b) Rs. 2,50,000 (c) Rs. 3,00,000 (d) Rs. 5,00,000

PROBLEM:54 C is a retired teacher aged 85 years. The exemption limit for C for AY 2020-21 is …….
(a) Rs. 1,80,000 (b) Rs. 2,50,000 (c) Rs. 3,00,000 (d) Rs. 5,00,000

PROBLEM:55 Alex is a non-resident individual aged 65 years. The exemption limit for Alex for AY 2020-
21 is ................
(a) Rs. 1,80,000 (b) Rs. 2,50,000 (c) Rs. 3,00,000 (d) Rs. 5,00,000

SALEEM QURAISHEE-9175664444 INSPIRE ACADEMY-8888881719


BASIC CONCEPTS Page 1.42

PROBLEM:56 Kalpesh has a taxable income of Rs. 85,00,000. The surcharge is applicable at the rate of
................ on his income.
(a) 5% (b) 10% (c) 15% (d) 20%

PROBLEM:57Surcharge at the rate of 15% is applicable, if the taxable income of the individual exceeds
................
(a) Rs. 5 lakhs (b) Rs. 1 crore(c) Rs. 5 crores(d) Rs. 10 crores

PROBLEM:58 Sunil wants to claim rebate u/s 87A of Rs. 12,500. It shall be available, if he is having total
income upto ................
(a) Rs. 3,00,000 (b) Rs. 3,50,000 (c) Rs. 5,00,000 (d) Rs. 4,50,000

PROBLEM:59The surcharge applicable in case of a Play Peon Private Limited, an Indian company, whose
total income is Rs. 12 crores is ................

PROBLEM:60The applicable rate of Income-tax in case of a ATUL Private Limited, an Indian company,
whose place of management is outside India is ................
(a) 10% (b) 20% (c) 30% (d) 40%

PROBLEM:61The applicable rate of Income-tax in case of Alpha Inc., a foreign company is ................
(a) 10% (b) 20% (c) 30% (d) 40%

PROBLEM:62The applicable rate of income-tax in case of Branch office of Alpha Inc., a Foreign company
is ................
(a) 10% (b) 20% (c) 30% (d) 40%

PROBLEM:63Every assessee is a person, every person ................ be an assessee.


(a) Must (b) Need not (c) Either (a) or (b) (d) None of the above

PROBLEM:64The gross total income must be rounded off to the nearest multiple of ................
(a) 10 rupees (b) 100 rupees (c) 1,000 rupees (d) NIL

PROBLEM:65The applicable rate of Income-tax in case of a firm is ................


(a) 10% (b) 20% (c) 30% (d) 40%

PROBLEM:66The maximum amount on which income tax is not chargeable for the AY 2020-2021 i.e. PY
2019- 2020 in case of an individual who is resident in India other than senior citizen is:
(a) Rs. 1,90,000 (b) Rs. 2,00,000
(c) Rs. 2,50,000 (d) Rs. 1.80,000

PROBLEM:67The maximum amount on which income tax is not chargeable for the AY 2020-2021 i.e. PY
2019- 2020 in case of an individual who is resident in India and is more than 60 year old but is less
than 80 years old is:
(a) Rs. 2,50,000 (b) Rs. 3,00,000
(c) Rs. 2,00,000 (d) Rs. 5,00,000

SALEEM QURAISHEE-9175664444 INSPIRE ACADEMY-8888881719


BASIC CONCEPTS Page 1.43

PROBLEM:68The maximum amount on which income tax is not chargeable for the AY 2020-2021 i.e. PY
2019- 2020 in case of an individual who is resident in India and is more than 80 years old is:
(a) Rs. 2,00,000 (b) Rs. 5,00,000 (c) Rs. 2,50,000 (d) Rs. 3,00,000

PROBLEM:69Surcharge on income tax is payable by:


(a) All assesses except a foreign company. (b) Individual and HUF only
(c) A company , domestic or foreign (d) All assesses

PROBLEM:70Surcharge on incomes tax is payable by a Domestic Company at the rate of:


(a) 1% of the income-tax payable provided its total incomes exceeds Rs. 10 lakhs
(b) 7% of incomes-tax payable irrespectively or the amount of its income
(c) 7% of incomes-tax payable provided its total incomes exceeds Rs. 1 crore and 12% of incomes-
tax if total income exceeds Rs. 10 Crores.
(d) 7% of incomes-tax payable provided its total incomes exceeds Rs. 1 crore.

PROBLEM:71Surcharge in case of a foreign company is payable at the rate of:


(a) 2% of the income-tax payable provided its total incomes exceeds Rs. 1 crore and 5 % of income
tax if total income exceeds Rs. 10 crore
(b) 5% of incomes-tax payable provided its total incomes exceeds Rs. 1 crore.
(c) 2 % of the incomes-tax payable provided its total income exceeds Rs. 1 crore
(d) 2% of the income-tax payable irrespective of the amount of its total income

PROBLEM:72The maximum amount on which income tax is not chargeable in case HUF is:
(a) Rs. 1,80,000 (b) Rs. 2,00,000
(c) Rs. 2,20,000 (d) Rs. 2,50,000

PROBLEM:73The maximum amount on which income tax is not chargeable in case of firm is:
(a) Rs. 3,00,000 (b) Rs. 5,00,000
(c) Rs. 2,50,000 (d) Nil

PROBLEM:74The maximum amount on which income tax is not chargeable in case of a co-operative
society is:
(a) Rs. 5,00,000 (b) Rs. 2,50,000 (c) NIL (d) Rs. 3,00,000

PROBLEM:75Health and Education Cess is leviable on:


(a) Income tax (b) Income tax + surcharge if applicable
(c) Only Surcharge (d) Not applicable at all

SALEEM QURAISHEE-9175664444 INSPIRE ACADEMY-8888881719


BASIC CONCEPTS Page 1.44

PROBLEM:76In case of an individual Health and Education Cess is leviable only when total income of
such assessee:
(a) Exceed Rs. 10,00,000 (b) Exceed Rs. 1 crore
(c) Exceed Rs. 2,50,000 (d) None of the above

PROBLEM:77 PY 2019-2020, a firm is subject to income tax at a flat rate of:


(a) 30% + surcharge @ 12% if the total income exceeds Rs. 1 crore + Health and Education Cess @ 4%
(b) 30% + surcharge @ 10% if the total income exceeds Rs. 1 crore + Health and Education
Cess @ 4%
(c) 30% + surcharge @ 5% if the total income exceeds Rs. 1 crore + Health and Education
Cess @ 4%
(d) 30% + surcharge @ 7% if the total income exceeds Rs. 1 crore + Health and Education Cess @ 4%

PROBLEM:78The surcharge applicable in the case of an individual for incomes other than section 112A,
section 111A and section 115BBE shall be
(a) 10% of tax payable if total income exceeds Rs. 50 lakhs but does not exceed Rs. 1 crore
(b) 10% of tax payable if total income exceeds Rs. 1 crore
(c) 15% of tax payable if total income exceeds Rs. 1 crore but does not exceed Rs. 2 crore
(d) Both (a) and (c), as the case may be.

PROBLEM:79In case of a domestic company whose gross receipts for the PY 2017-2018 is Rs. 451
Crores, the rate of tax applicable for AY 2020-2021 i.e. PY 2019-2020
(a) 29% (b) 25% (c) 30% (d) None of the above

PROBLEM:80The rate of tax applicable to a partnership firm for AY 2020-2021 i.e. PY 2019-2020 is—
(a) 25% (b) 30% (c) 35% (d) 40%

SALEEM QURAISHEE-9175664444 INSPIRE ACADEMY-8888881719


RESIDENTIAL STATUS- Page 2.1

1. INTRODUCTION
 Residential status of an assessee determines the scope of chargeability of his income.
 Whether a person will be charged to a particular income or not, depends on his residential status.
 Sec. 6 provides the test for residential status for the persons which can be categorized as under-

GENERAL POINTS TO BE KEPT IN MIND REGARDING RESIDENTIAL STATUS OF A PERSON

1) Different residential status for each previous year: Residential status is determined in respect of
each previous year. In other words, residential status of a person may vary from one previous year
to another previous year.
2) Single Status for each source of income: A person can have only one residential status for a
previous year i.e. he cannot be a resident for one source of income and non-resident for another
source.
3) Impact of citizenship : Citizenship and residential status are two different concepts. A citizen of
India may not be a resident in India.
4) Country Specific: A person can have same residential status in more than one country.

2. INDIAN INCOME & FOREIGN INCOME

SALEEM QURAISHEE-9175664444 INSPIRE ACADEMY-8888881719


RESIDENTIAL STATUS- Page 2.2

3. RESIDENTIAL STATUS OF AN INDIVIDUAL [SEC. 6]

 RESIDENT IN INDIA- SECTION 6(1)


An individual will be 'resident' in any previous year, if he satisfies any one or both of the following two
basic conditions.
1) He is in India in the previous year for a period of 182 days or more OR
2) He is in India for a period of 60 days or more during the previous year and 365 days or more during the
four years immediately preceding the previous year.

Exceptions to basic condition No.2


In the following cases, condition (2) of Basic condition is irrelevant:
1. An Indian citizen, who leaves India during the previous year for employment purpose.
2. An Indian citizen, who leaves India during the previous year as a member of crew of an Indian ship.
3. Individual, being Indian Citizen or person of Indian Origin engaged outside India in an employment or a
Business or Profession or in any other vocation, and visiting India during the relevant Previous Year.

 Person of Indian origin: A person is deemed to be of Indian origin if he or either of his parents or
grandparents were born in undivided India. Here, grand parents may be paternal or maternal.

 RESEDENT AND ORDINARILY RESIDENT -SECTION 6


ADDITIONAL CONDITION-

1) He has been in India in at least 2 out of 10 previous years immediately preceding the relevant previous
year.
2) He has been in India for a period of 730 days or more during 7 years immediately preceding the
relevant previous year.

 RESIDENT BUT NOT ORDINARY RESIDENT-SECTION 6


If a person fulfills any one or none of above conditions he is treated as resident but not ordinary resident.

 NON RESIDENT
'Non-Resident' means a person who is not a "resident". An individual is a non-resident in India if he
satisfies none of the basic conditions.

POINT TO BE NOTED-
I. Stay at same place in India is not necessary.
II. Continuous stay in India is not necessary.
III. A person shall be deemed to reside in India, if he is on the territorial waters of India.
IV. In computing the period of 182 days, the day the individual enters the India and the day, he
leaves India should both be treated as a stay in India.

SALEEM QURAISHEE-9175664444 INSPIRE ACADEMY-8888881719


RESIDENTIAL STATUS- Page 2.3

How to determine period of stay in India for an Indian citizen, being a crew member?
Determination of Residential Status of Crew Member of a Ship: In the case of an Individual, being a
Indian Citizen and a Member of the Crew of a Foreign-bound Ship leaving India, the period(s) of stay in
India shall, in respect of such voyage shall be determined in the manner and subject to such prescribed
conditions. For determining the period of Stay in India, the following period shall not be included-
Period beginning From Period ending to
Date entered into the Continuous Discharge Date entered into Continuous Discharge Certificate in
Certificate in respect of joining the ship by respect of the signing off by that individual from the
the said individual for the eligible voyage ship in respect of such voyage.
Meaning of Terms:
a) Continuous Discharge Certificate shall have the meaning assigned to it in the Merchant Shipping
(Continuous Discharge Certificate – Cum-Seafarer’s Identity Document) Rules, 2001 under
Merchant Shipping Act, 1958.
b) Eligible Voyage shall mean a voyage undertaken by a ship engaged in the carriage of passengers or
freight in international traffic where-
I. For the voyage having originated from any port in India, has its destination as any port
outside India and
II. For the voyage Originated from any port outside India, has its destination as any port in
India.

PROBLEM:1 Mr. Anand is an Indian citizen and a member of the crew of a Singapore bound Indian ship
engaged in carriage of passengers in international traffic departing from Chennai port on 6th June,
2019. From the following details for the P.Y. 2019-20, determine the residential status of Mr. Anand
for A.Y. 2020-21, assuming that his stay in India in the last 4 previous years (preceding P.Y. 2019-20)
is 400 days and last seven previous years (preceding P.Y.2019-20) is 750 days:
Particulars Date
Date entered into Continuous Discharge Certificate in respect of joining the ship by th
6 June 2019
Mr Anand
Date entered into Continuous Discharge Certificate in respect of signing off the ship 9th December 2019
by Mr Anand
ANSWER:1

PROBLEM:2 During the last four years preceding the financial year 2019-20, Mr. Damodhar, a Citizen of
India, was present in India for 430 days. During the last seven previous years preceding the previous
year 2019-20, he was present in India for 830 days.
Mr. Damodhar is a Member of Crew of a Dubai bound Indian Ship, carrying passengers in the
international waters, which left Kochi Port in Kerala, on 12th August 2019. Following details are
made available to you for the previous year 2019-20:
Particulars Date
Date entered into the Continuous Discharge Certificate in respect of joining the ship by 12th Aug, 2019
Mr. Damodhar
Date entered into the Continuous Discharge Certificate in respect of signing of the ship 21st Jan 2020
by Mr. Damodhar
In May 2019, he had gone out of India to Singapore and Malaysia on a private tour for a continuous period
of 29 days. You are required to determine the Residential Status of Mr. Damodhar for the Previous Year
2019-20.
ANSWER:2
SALEEM QURAISHEE-9175664444 INSPIRE ACADEMY-8888881719
RESIDENTIAL STATUS- Page 2.4

PROBLEM:3 Mr. A, an Indian Citizen, is living in Mumbai since 1950, he left for China on July 1, 2014 and
comes back on August 7, 2019. Determine his residential status for the assessment year 2020-21.
ANSWER:3

PROBLEM:4 Dr. A, an Indian Citizen and a Professor in IIM, Lucknow, left India on September 15, 2019
for USA to take up Professors job in MIT, USA. Determine his residential status for the assessment year
2020-21
ANSWER:4

PROBLEM:5 Mr. X is a foreign citizen. His father was born in Mumbai in 1960 and mother was born in
USA in 1965. His grandfather was born in Chennai in 1935. Mr. X is coming to India to see Taj Mahal
and visit other historical places in India. He comes to India on 1st November, 2019 for 200 days. He has
never come to India before. Determine his residential status for AY 2020-21.
ANSWER:5

PROBLEM:6 Mr. Anil, an Indian citizen, leaves India on 22nd September, 2019 for the first time to work
as an Engineer in France. Determine his residential status for AY 2020-21.
ANSWER:6

PROBLEM:7 Brett Lee, an Australian cricket player visits India for 100 days in every financial year. This
has been his practice for the past 10 financial years. Find out his residential status for the assessment
year 2020-21.
ANSWER:7

PROBLEM:8 Mr. X, the Australian cricketer comes to India for 105 days every year. Find out his
residential status for the A.Y. 2020-21.
ANSWER:8

PROBLEM:9 On 01.06.2017 Mr. X, a Malaysian citizen leaves India after stay of 10 years. During the
financial year 2018-19 lie comes to India for a period of 46 days. Later, he returns to India for one year
on 10.20.19.
ANSWER:9

PROBLEM:10 Mr. Ram, an Indian citizen, left India on 22.09.2019 for the first time to work as an officer of a
company in Germany. Determine the residential status of Ram for the assessment year 2020-21.
ANSWER:10

PROBLEM:11 Mr. B, a Canadian citizen, comes to India for the first time during PY 2015-16. During FY
2015-16, FY 2016-17, FY 2017-18, FY 2018-19 & FY 19-20 he was in India for 55 days, 60 days, 90 days,
150 days and 70 days, respectively. Determine his residential status for AY 2020-21.
ANSWER:11

PROBLEM:12 Mr. Ram an Indian citizen left India on 22.09.2019 for the first time to work as an officer of a
company in Germany. Determine (he residential status of Ram for the assessment year 2020-21 and
explain the conditions to be fulfilled for the same under the Income-tax Act 1961.
ANSWER:12

SALEEM QURAISHEE-9175664444 INSPIRE ACADEMY-8888881719


RESIDENTIAL STATUS- Page 2.5

PROBLEM:13 In the year P.Y. 2019-20, a sailor has remained on ship for a private company owning
ocean going ships as follows:
I. Outside the territorial waters of India for 183 days.
II. Inside the territorial waters of India for 182 days .
Is he considered to be resident or not for the Assessment Year 2020-21. Comment.
ANSWER:13

PROBLEM:14 Mrs. Kapoor, is a Hollywood actress. Her passport reveals the Following Information
about her stay in India.

2019-20 From April 3rd to July 11th

2018-19 From June 22nd to July 11th

2017-18 From Feb 10th to March 26th

2016-17 From Sept 7th to March 26th

2015-16 From May 17th to September 30"'

2014-15 From April 3rd to July 11th

2013-14 From April 3rd to July 11th

2012-13 From April 3rd to July 11th

2011-12 From April 3rd to July 11th

Find out her residential status for the assessment year 2020-21.
ANSWER:14

SALEEM QURAISHEE-9175664444 INSPIRE ACADEMY-8888881719


RESIDENTIAL STATUS- Page 2.6

4. RESIDENTIAL STATUS OF A HUF [SEC. 6(2)]


Residential status of HUF depends on whether control and management of its affairs is wholly or partly
situated in India.
Place of Control and Management Residential status of family
Wholly situated in India Resident
Partly situated in India Resident
Wholly outside India Non Resident

 RESIDENT AND ORDINARY RESIDENT


A 'resident' HUF is treated as 'Resident and Ordinarily resident' if Karta of HUF satisfies following two
additional conditions -
1) He has been in India in at least 2 out of 10 previous years immediately preceding the relevant previous
year.
2) He has been in India for a period of 730 days or more during 7 years immediately preceding the relevant
previous year.

 RESIDENT BUT NOT ORDINARY RESIDENT-SECTION 6


If a KARTA OR MANAGER of resident HUF fulfills any one or none of above conditions he is treated as
resident but not ordinary resident.

 POINT TO BE NOTED-
i. For determining whether HUF is a resident or not, the residential status of its Karta for the relevant
previous year is of no relevance.
II. However for determining whether HUF is ordinarily resident in India or not, Karta’s status for the
preceding years becomes relevant.

PROBLEM:15 An HUF, whose affairs of business are completely controlled from India. Determine its
Residential status for AY 2020-21 (a) if Karta is ROR in India for that year (b) If Karta is NR in India but he
satisfies both the additional conditions (c) If Karta is RNOR in India.
ANSWER:15

PROBLEM:16 Karta of an Hindu Undivided Family comes to India every year for a minimum period of 60
days and maximum 91 days. Determine residential status of the HUF for the assessment year 2020-21.
ANSWER:16

PROBLEM:17 Hindu Undivided Family is being managed partly from Mumbai and partly from Japan. The
Karta of HUF is a foreign citizen and comes to visit in India every year since 1980 in the month of April
for 105 days. Determine residential status of HUF for AY 2020-21.
ANSWER:17

PROBLEM:18 The business of a HUF is transacted from Australia and all the policy decisions are taken
there. Mr. E, the Karta of the HUF, who was born in Kolkata, visits India during the P.Y. 2019-20 after 15
years. He comes to India on 1.4.2019 and leaves for Australia on 1.12.2019. Determine the residential
status of Mr. E and the HUF for A.Y. 2020-21.
ANSWER:18

SALEEM QURAISHEE-9175664444 INSPIRE ACADEMY-8888881719


RESIDENTIAL STATUS- Page 2.7

5. RESIDENTIAL STATUS OF FIRM AND AOP SEC 6(2)


Residential status of Firm/AOP/BOI depends on whether control and management of its affairs is wholly or
partly situated in India.
Place of Control and Management Residential status
Wholly situated in India Resident
Partly situated in India Resident
Wholly outside India Non Resident

 POINT TO BE NOTED-
I. Firm/AOP/BOI cannot be “ordinarily” or “not ordinarily resident”.
II. The status of the partners/ members of the Firm/AOP/BOIis not relevant in determining the
status of the firm/association.

PROBLEM:19 AB & Co. is a partnership firm whose operations are carried out in India. However, all meetings of partners
take place outside India as all the partners are settled abroad. Determine Residential status of firm for AY 2020-21.
ANSWER:19

6. RESIDENTIAL STATUS OF A COMPANY [SEC. 6(3)]


1) Indian company is always considered as a resident in India irrespective of the fact where its
control & management lies. In other words, a company registered in India would always be a
resident even if:
I. its entire business is outside India; or
II. Its entire control A management lies outside India.

2) Foreign company is said to be resident in India if its place of effective management (POEM) is in
India at any time during the relevant previous year.

 Place of effective management' has been defined to mean a place where key management and
commercial decisions that are necessary for the conduct of the business of an entity as a whole are,
in substance, made.

CASE-1 XYZ Ltd is incorporated in India but it has most of its business outside India. XYZ Ltd would
always be considered as a resident.

CASE-2 RAJ Ltd is incorporated outside India but its place of effective management is in India. RAJ
Ltd would also be considered as a resident

PROBLEM:20 TATA limited is a company incorporated in India and which has its registered office in
Mumbai. For the AY 2020-2021 i.e. PY 2019-2020 it holds all four BOD meetings in Nepal. Determine its
residential status for the AY 2020-2021 i.e. PY 2019-2020.
ANSWER:20 TATA limited is an Indian company and therefore is always a resident of India. We do not
have to check the place of effective management (POEM).

SALEEM QURAISHEE-9175664444 INSPIRE ACADEMY-8888881719


RESIDENTIAL STATUS- Page 2.8

PROBLEM:21 D limited is incorporated in Delhi - India but it holds all BOD meetings in Nepal. K limited is
incorporated in Kathmandu - Nepal but holds all BOD meetings in India. Discuss the residential status in
India for D limited and K limited.
ANSWER:21 D limited is incorporated in India and therefore is a resident of India. K limited is a foreign
company and has POEM in India and therefore it becomes resident of India.

PROBLEM:22 B Ltd., a foreign company and it carries on majority of its operations and decision making
activities from Calcutta and Assam but some part of operational activities and few decisions are being
taken from the place at which registered office of B Ltd. is located, i.e. Dhaka. Determine its residential
status for the assessment year 2020-21.
ANSWER:22 B Ltd. is a foreign company and its place of effective management is in India. Hence B Ltd. is
resident in India for the assessment year 2020-21.

7. RESIDENTIAL STATUS OF OTHER PERSONS- SEC 6(4)


A. Local authority, artificial juridical person, etc would be treated as resident in India if the control and
management of its affairs is situated wholly or partially in India.

B. Local authority, artificial juridical person, etc would be treated as non-resident in India if the
control and management of its affairs is situated wholly outside India.

SALEEM QURAISHEE-9175664444 INSPIRE ACADEMY-8888881719


RESIDENTIAL STATUS- Page 2.9

8. SCOPE OF TOTAL INCOME (SEC-5)

9. RECEIPTS VS. REMITTANCE.


I. The "receipt" of income refers to the first occasion when the recipient gets the money under his
control.
II. Once an amount is received as income, any remittance or transmission of the amount to another
place does not result in "receipt" at the other place.

10. CASH VS. KIND


I. It is not necessary that income should be received in cash.
II. Income may be received in cash or in kind.

11. RECEIPT VS. ACCRUAL


I. Receipt is not the sole test of chargeability to tax.
II. If an income is not taxable on receipt basis, it may be taxable on accrual basis.

12. ACCRUAL OF INCOME


I. Income accrued in India is chargeable to tax in all cases irrespective of residential status of an
assessee.
II. The words "accrue" and "arise" are used in contradistinction to the word "receive".
III. Income is said to be received when it reaches the assessee; when the right to receive the income
becomes vested in the assessee, it is said to accrue or arise.

SALEEM QURAISHEE-9175664444 INSPIRE ACADEMY-8888881719


RESIDENTIAL STATUS- Page 2.10

13. INCOME FROM BUSINESS CONNECTION [SEC. 9]


Business connection’ shall include any business activity carried out through a person acting on behalf of
the non-resident [Explanation 2 to section 9(1)(i)]
For a business connection to be established, the person acting on behalf of the non-resident –

I. Must have an authority, which is habitually exercised in India, to conclude contracts on behalf of
the non-resident or habitually concludes contracts or plays the principal role leading to
conclusion of contracts by that non-resident and such contracts should be

 In the name of the non-resident; or


 For the transfer of the ownership of, or for the granting of the right to use, property owned by that
non-resident or that non-resident has the right to use; or
 For the provision of services by that non-resident.

II. In a case, where he has no such authority, but habitually maintains in India a stock of goods or
merchandise from which he regularly delivers goods or merchandise on behalf of the non-
resident, or
III. Habitually secures orders in India, mainly or wholly for the non-resident.

Further, there may be situations when the person acting on behalf of the non-resident secure order
for other non-residents. In such situation, business connection for other non-residents is
established if,
a) such other non-resident controls the non-resident or
b) such other non-resident is controlled by the non-resident or
c) such other non-resident is subject to same control as that of non-resident.
In all the three situations, business connection is established, where a person habitually secures
orders in India, mainly or wholly for such non-residents

SIGNIFICANT ECONOMIC PRESENCE CONSTITUTES BUSINESS CONNECTION:


I. Business Connection: Significant Economic Presence of a non-resident in India shall also
constitute k Business Connection in India.

II. MEANING OF SIGNIFICANT ECONOMIC PRESENCE (SEP):


a) Transactions exceeding prescribed limit: is for the Transaction in respect of any goods, services or
property carried out by a non-resident in India, including provision of download of data or software
in India, if the aggregate of payments arising from such transactions during the PY exceeds the
prescribed amount, or
b) Digital Transactions: Systematic and Continuous Soliciting of Business activities, or engaging in
interaction with prescribed number of users in India, through digital means.
POINT TO BE NOTED-
1) Transactions or Activities shall constitute significant economic presence in India, whether or not -
I. The Agreement for such transaction or activities is entered in India or
II. The Non-resident has a residence or place of Business in India or
III. The Non-resident renders Services in India
2) While computing Income based on the Significant Economic Presence, only so much of income as is
attributable to the above referred transactions or activities, shall be deemed to accrue or arise in
India.

SALEEM QURAISHEE-9175664444 INSPIRE ACADEMY-8888881719


RESIDENTIAL STATUS- Page 2.11

OPERATIONS NOT TAKEN AS BUSINESS CONNECTIONS-


I. Where all operations are not carried out in India - If all business operations are not carried out
in India, the income of the business deemed to accrue or arise in India shall be only such part of
income as is reasonably attributable to the operations carried out in India.

II. Purchase of goods for export - In the case of non-resident no income shall be deemed to accrue
or arise in India to him through or from operations which are confined to the purchase of goods
in India for the purpose of export.

III. Collection of news and views - No income arises to a non-resident from the activity of collection
of news and views in India for transmission out of India.

IV. Shooting of cinematograph films in India -Shooting of cinematographic films in India by a non-
resident provided the film is not shown in India and conditions listed below are satisfied:
(If the film is shown in India, it would amount to a business connection)
I. Shooting is done by non-resident individual: The individual should not be a citizen of India.
II. Shooting is done by non-resident firm: All the partners should be non-resident in India and none
of them should be a citizen of India.
III. Shooting is done by non-resident company: All the shareholders should be non-resident in India
and none of them should be a citizen of India.

V. Display of uncut and unasserted diamond in a notified special zone. In the case of a foreign
company engaged in the business of mining of diamonds , no income shall be deemed to accrue
or arise in India through or from the activities which are confined to the display of uncut and
unasserted diamond (without any sorting or sale) in any special zone notified by the central
government.

14. INCOME FROM ANY PROPERTY, ASSET OR SOURCE OF INCOME IN INDIA


[SEC. 9] -
I. Income from any property, asset or source of income in India is deemed to accrue or arise in
India.
II. The term "source" means a real source of income.
III. The term "property" does not refer to merely a house property but includes any tangible or
immovable property.
IV. The term "asset" includes all intangible rights and, consequently, interests, patent and copyright,
royalties, rents, etc., will find room in four corners of the term "asset".

SALEEM QURAISHEE-9175664444 INSPIRE ACADEMY-8888881719


RESIDENTIAL STATUS- Page 2.12

15. INCOME FROM THE TRANSFER OF CAPITAL ASSET SITUATED IN INDIA


[SEC. 9(1)(I)]-
I. Any capital gain, within the meaning of section 45, earned by a person by transfer of any capital
asset situated in India, is deemed to accrue or arise in India.
II. An asset or a capital asset (being any share or interest in a company or entity registered or
incorporated outside India) shall be deemed to be and shall always be deemed to have been
situated in India if the share or interest derives, directly or indirectly, its value substantially from
the assets located in India".

16. INCOME UNDER THE HEAD "SALARIES" [SEC. 9(1) (II)] –


I. Income of an individual which falls under the head "Salaries" is deemed to accrue or arise in India
if service is rendered in India.
II. Any salary payable for rest period or leave period which is both preceded and succeeded by
service in India, will also be regarded as salary earned in India.

17. SALARY PAYABLE ABROAD BY THE GOVERNMENT TO A CITIZEN OF


INDIA-
I. Salary received by Indian nationals from the Indian Government, out of India, is deemed to
accrue or arise in India.
II. Section 10(7)- any allowance or perquisite paid abroad is, however, fully exempt from tax

18. INCOME BY WAY OF INTEREST, ROYALTY AND TECHNICAL FEES -


These are deemed to accrue or arise in India in the following cases:
Rule 1: When received from Government:
I. Interest, royalty or technical fees received from the Central Government/ any State
Government is deemed to accrue or arise in India.

Rule 2: When received from a person resident in India –


I. Interest, royalty or technical fees received from a resident person, is deemed to accrue or
arise in India in the hands of recipient.
II. However, this rule is not applicable in the following cases-

a. If borrowed money is utilized by the payer for carrying on a business/profession outside India
or for earning any income outside India; or
b. Payment of royalty /technical fees pertains to a business /profession carried on by the payer
outside India or earning any income outside India.

Rule 3 - When received from a non-resident –


I. Interest, royalty or technical fees received from a non-resident, is deemed to accrue or
arise in India in the hands of recipient, in the following cases -
a. Borrowed money is utilized by the payer for carrying on a business /profession in India; or
b. Payment of royalty /technical fees pertains to a business /profession carried on by the payer
in India or earning any income in India.
SALEEM QURAISHEE-9175664444 INSPIRE ACADEMY-8888881719
RESIDENTIAL STATUS- Page 2.13

19. SECTION SUMMARY-

SECTION PARTICULAR

PROBLEM:23 Examine with reasons whether the following transactions attract income-tax in India in
the hands of recipients:
(i) Salary paid by Central Government to Mr. John, a citizen of India ` 7,00,000 for the services rendered
outside India.
(ii) Interest on moneys borrowed from outside India ` 5,00,000 by a non-resident for the purpose of
business within India say, at Mumbai.
(iii) Post office savings bank interest of ` 19,000 received by a resident assessee, Mr. Ram.
(iv) Royalty paid by a resident to a non-resident in respect of a business carried on outside India
(v) Legal charges of ` 5,00,000 paid to a lawyer of United Kingdom who visited India to represent a case at
the Delhi High Court.
ANSWER:23

SALEEM QURAISHEE-9175664444 INSPIRE ACADEMY-8888881719


RESIDENTIAL STATUS- Page 2.14

PROBLEM:24 From the following particulars of income furnished by Mr. Anirudh pertaining to the year
ended 31.3.2020, compute the total income for the assessment year 2020-21, if he is:
(i) Resident and ordinary resident;
(ii) Resident but not ordinarily resident;
(iii) Non-resident
Particulars `

I. Short term capital gains on sale of shares in Indian Company 15,000


received in Germany
II. Dividend from a Japanese Company received in Japan 10,000

III. Rent from property in London deposited in a bank in London, later 75,000
on remitted to India through approved banking channels

IV. Dividend from RP Ltd., an Indian Company 6,000

V. Agricultural income from land in Gujarat 25,000

ANSWER:24

PROBLEM:25 From the following incomes earned by Mr. Rajesh during the financial year 2019-20,
determine his total income for the assessment year 2020-21 if he is (1) resident and ordinarily
resident; (2) not ordinarily resident: (3) non-resident.
a) Profits from a business in Bombay managed from London. 1,60,000

b) Pension for services rendered in India but received in Burma 15,000

c) Interest on U.K. Government Bonds half of which is received in India 4,000

d) Income from property situated in Pakistan received there. 20,000

e) Past foreign untaxed income brought to India during the previous year 7,000

f) Income from agricultural land in Nepal received there and then brought to India. 30,000

g) Income from profession in Kenya which was set up in India, received there. 12,000
ANSWER:25 ` 241000, 189000, 177000.

SALEEM QURAISHEE-9175664444 INSPIRE ACADEMY-8888881719


RESIDENTIAL STATUS- Page 2.15

PROBLEM:26 Ram has the following incomes during the financial year 2019-20
Particulars of income `

(i) Royalty earned in India, received on 3-5-2019 in Nepal 46,000


(ii) Dividend from a foreign company received in Nepal on 16-7-2019 56,000
(iii) Share of profit of a business situated in Nepal, received in Burma on 14-6-2019 but
controlled from India 42,000
(iv) Rent of 2019-20 of a land situated in Nepal, and received there on 31-12-2019 1,65,000
(v) Speculation profit earned and received outside India on 15-4-2020 60,000
(vi) Profit on sale of machinery at Delhi received on 30-3-2020 (one- half is received in
London) 50,000
From the above particulars ascertain the Gross Total Income of R for the assessment year 2020-21 if he is
(a) a resident and ordinarily resident (b) a not ordinarily resident (c) a non-resident.
ANSWER:26 ` 359000, 138000, 96000.

PROBLEM:27 'A' earns the following income during the financial year 2019-20
(a) Interest paid by an Indian company but received in London. 2,00,000

(b) Pension from former employer in India, received in USA 8,000

(c) Profits earned from business in Paris which is controlled in India, half of the profits
being received in India 40,000

(d) Income from agriculture in Bhutan and remitted to India. 10,000

(e) Income from property in England and received there 8,000

(f) Past foreign untaxed income brought to India 20,000

Determine the total income of 'A' for the assessment year 2020-21 if he is –

(i) Resident and ordinarily resident,

(ii) Not ordinarily resident, and

(iii) Non-resident in India.


ANSWER:27 ` 266000, 248000, 228000.

SALEEM QURAISHEE-9175664444 INSPIRE ACADEMY-8888881719


RESIDENTIAL STATUS- Page 2.16

PROBLEM:28 From the following particulars, compute the taxable income of R when his residential
status is (i) resident and ordinarily resident, (ii) not ordinarily resident, and (iii) non-resident.
Particulars of income `

(i) Interest for debentures in an Indian company received in London 5,000

(ii) Interest on a company deposit in India but received in Germany 22,000

(iii) Interest on U.K. Development Bonds 50% on interest received in India 40,000

(iv) Dividend from British Co. received in London 10,000

(v) Profits on sale of plan at Germany 50% of profits are received in India 60,000

(vi) Income earned from business in Germany which is controlled from Delhi, ` 40,000 is
received in India 70,000

(vii) Profits from a business in Delhi but managed entirely from London 45,000

(viii) Rent from property in London deposited in a Indian Bank at London, brought to
India 50,000

(ix) Interest received in London on money lent to a resident in India in London but the
same money was used in India 46,000

(x) Fees for technical services rendered in India but received in London 25,000

(xi) Royalty received in London for a right given to non-resident in India to be used for
business in India 34,000
ANSWER:28 ` 407000, 297000, 267000.

PROBLEM:29 Following are the particulars of taxable income of R for the PY ended 31-3-2020.
(i) Royalty received from Government of India ` 24,000.
(ii) Income from business earned in Afghanistan ` 25,000 of which ` 15,000 were received in India.
(iii) Interest received from G a non-resident against a loan provided to him to run a business in India `
5,000.
(iv) Royalty received in India from S a resident for technical services provided to run a business outside
India ` 20,000.
(v) Income from business in Jaipur ` 40,000. This business is controlled from France, ` 20,000 were
remitted to France.
Find out Gross Total Income of R for assessment year 2020-21, if he is----
(a) resident and ordinarily resident of India (b) not ordinarily resident of India and
(c) non-resident of India in previous year.
ANSWER:29 Rs1,14,000,1,04,000,1,04,000

SALEEM QURAISHEE-9175664444 INSPIRE ACADEMY-8888881719


RESIDENTIAL STATUS- Page 2.17

PROBLEM:30 _______________________ Mr. X and Mr. Y are brothers and they earned the following
incomes during the financial year 2019-20. Mr. X settled in America in the year 2006 and Mr. Y settled
in Mumbai. Mr. X visits India for 20 days every year. Mr. Y also visits America every year for a month.
Compute their total income for the Assessment year 2020-21 from the following information.
SI. No. Particulars Mr. X Mr. Y

1. Interest on American Development bonds, 50% of interest received in 46,000 18,000


India.
2. Dividend from a Japanese Company received in America. 10,000 15,000
3. Short term capital gains on sale of shares of an Indian company received in 45,000 75,000
India.
4. Profit from a business in Mumbai, but managed directly from America. 10,000 -
5. Income from a business in Mumbai. 32,000 28,000
6. Fees for technical, services rendered in America and received in America. 1,50,000
The services were, however; utilized in India.
7. Interest on fixed deposit with State Bank of India, Mumbai. 4,500 12,000
8. Income from house property at Mumbai. 67,200 38,500
ANSWER:30

PROBLEM:31 Mrs. X and Mrs. Y are sisters and they earned the following income during the Financial Year 2019-
20. Mrs. X is settled in Malaysia since 2006 and visits India for a month every year. Mrs. Y is settled in Indore
since her marriage in 2006. Compute the total income of Mrs. X and Mrs. Y for the assessment year 2019-20:
SI. Particulars Mrs. X Rs. Mrs. Y
No. Rs.
(i) Income from Profession in Malaysia, (set up in India) received there 15,000 -
(ii) Profit from business in Delhi, but managed directly from Malaysia 40,000 -
(iii) Rent {computed) from property in Malaysia deposited in a Bank at Malaysia, later on 1,20,000 -
remitted to India through approved banking channels.
(iv) Dividend from PQR Ltd. an Indian Company 5,000 9,000
(v) Dividend from a Malaysian company received in Malaysia 15,000 8,000
th
(vi) Cash gift received from a friend on Mrs. Y’s 50 birthday 51,000
(vii) Agricultural income from land in Maharashtra 7,500 4,000
(viii) Past foreign untaxed income brought to India 5,000 -
(ix) Fees for technical services rendered in India received in Malaysia 25,000
(x) Income from a-business in Pune (Mrs. X receives 50% of the income in India) 12,000 15,000
(xi) Interest on debentures in an Indian company (Mrs. X received the same in Malaysia) 18,500 14,000
(xii) Short-term capital gain on sale of shares of an Indian company 15,000 25,500
(xiii) Interest on Fixed Deposit with SB! in India 12,000 8,000
ANSWER:31

SALEEM QURAISHEE-9175664444 INSPIRE ACADEMY-8888881719


RESIDENTIAL STATUS- Page 2.18

20. MCQ-
PROBLEM:1 An individual may be ................
(a) Resident in India (b) Non-resident in India
(c) Either (a) or (b) (d) None of the above

PROBLEM:2 A resident Hindu Undivided Family (HUF) can be ................


(a) Resident and ordinarily resident in India (b) Resident but not ordinarily resident in India
(c) Non-resident in India (d) Either (a) or (b)

PROBLEM:3 An individual is said to be resident in India, if ................


(a) He is in India for a period of at least 182 days during the previous year
(b) He is in India for a period of 60 days or more during the relevant previous year
(c) He has been in India for a period of at least 365 days during 4 previous year immediately
preceding the relevant previous year
(d) He has been in India for at least Rs. 30 days during 7 previous year immediately preceding the
relevant previous year
(a) Either (i) or [(ii) & (iii)]
(b) Either (i) or [(iii) & (iv)]
(c) Either (ii) or [(i) & (iv)]
(d) Either (ii) or [(iii) & (iv)]

PROBLEM:4 AJAY has been in India for a period of 184 days during the previous year 2019-20 but he
was not in India for a continuous period of 184 days. Determine his residential status.
(a) He is a resident in India during PY 2019-20
(b) He is not a resident in India during PY 2019-20
(c) He is resident and ordinary resident in India during PY 2019-20
(d) He is both resident and non-resident in India during PY 2019-20

PROBLEM:5 While calculating the number of days in order to ascertain the residential status of an
individual, the day when he enters India and the day when he leaves India, both are ................
(a) Included (b) Excluded
(c) The date of entry is included and date of exit is excluded (d) None of the above

PROBLEM:6 An individual is said to be resident and ordinary resident in India, if he has been resident
in India for at least 2 out of 10 previous years and ................
(a) He has been in India for a period of 700 days or more during the 7 previous years immediately
preceding the relevant previous year
(b) He has been in India for a period of 730 days or more during the 7 previous years immediately
preceding the relevant previous year
(c) He has been in India for a period of 700 days or more during the 10 previous years immediately
preceding the relevant previous year
(d) He has been in India for a period of 730 days or more during the 10 previous years immediately
preceding the relevant previous year

SALEEM QURAISHEE-9175664444 INSPIRE ACADEMY-8888881719


RESIDENTIAL STATUS- Page 2.19

PROBLEM:7 As per section 6(3), company is said to be resident in India in any previous year, if its place
of effective management, in that year, is in India or ................
(a) It is an Indian company (b) It is a Foreign company
(c) Either (a) or (b) (d) Both (a) and (b)

PROBLEM:8 As per section 7, which of following income is/are deemed to be received in India even in
the absence of actual receipt?
(a) Contribution made by the employer to Recognised Provident Fund in excess of 12% of salary of
the employee (b) Interest credited to RPF of employee in excess of 12% p.a.
(c) Both (a) & (b) (d) None of the above

PROBLEM:9 Sameer , a non-resident, transfers an immovable property (located in Delhi) to Mayur,


resident in India. The amount received by SAMEER will be ................
(a) Taxable in India (b) Non-taxable in India
(c) 50% of the amount will be taxable in India (d) None of the above

PROBLEM:10 A resident HUF will be considered as resident and ordinarily resident in India, if its
................ is an ordinary resident in India.
(a) Members (b) Karta (c) Karta and any one member (d) Karta and all members

PROBLEM:11 Fees for technical services, paid by the Central Government will be taxable in case of
................
(a) Resident and ordinarily resident in India (b) Resident but not ordinarily resident in India
(c) Non-resident (d) All of the above

PROBLEM:12 Rent from a property (situated in Kerala) received in Australia is taxable in the hands of
................
(a) Resident and ordinarily resident (b) Resident but not ordinarily resident
(c) Non-resident (d) All of the above

PROBLEM:13 As per ................ , total income means the total amount of income referred to in section
5, computed in the manner laid down in the Income-tax Act, 1961.
(a) Section 2(41) (b) Section 2(43) (c) Section 2(45) (d) Section 2(47)

PROBLEM:14 Rent received in England from property situated in England is taxable in the hands of
……………………….
(a) Resident and ordinarily resident (b) Resident but not ordinarily resident
(c) Non-resident (d) Both (a) and (b)

PROBLEM:15 Amount received as rent from the property situated in France during FY 2018-19 in France
in said year but remitted to India in FY 2019-20 will be taxable in the hands of ................
(a) Foreign Company (b) Resident but not ordinarily resident
(c) Non-resident (d) None of the above

SALEEM QURAISHEE-9175664444 INSPIRE ACADEMY-8888881719


RESIDENTIAL STATUS- Page 2.20

PROBLEM:16 Agricultural income from operations outside India and received in England during the FY
2019-20 and the same has been remitted to India in same FY, is taxable in the hands of ................
(a) Resident and ordinarily resident (b) Resident but not ordinarily resident
(c) Non-resident (d) All of the above

PROBLEM:17 Under Residence based taxation, taxation of an individual is based on ................


(a) Country of their residence
(b) Whether the source of income is in or outside the country of residence
(c) Both (a) and ( b) (d) Neither (a) nor (b)

PROBLEM:18 Under Residence based taxation, taxation of a companies is based on ................


(a) Place where they are incorporated (b) Place of Effective Management
(c) Both (a) and (b) (d) Neither (a) nor (b)

PROBLEM:19 POEM in the context of residential status of a company stands for ................
(a) Place of Effective Management (b) Place of residence of Effective Manager
(c) Place of Employee Matters (d) Place of Employee Management

PROBLEM:20 Domestic company means a company which is ................


(a) An Indian company
(b) Company which has made prescribed arrangements for the declaration and payment of
dividends (including dividend on preference shares) within India, out of its income taxable in India
(c) Both (a) and (b) (d) None of the above

PROBLEM:21 Alpha Inc., a company incorporated in USA, has made prescribed arrangement for
declaration and payment of dividends within India. Alpha Inc. would be a ................
(a) Foreign company for Indian Income-tax Act purpose
(b) Domestic company for Indian Income-tax Act purpose

PROBLEM:22 XYZ India Private Limited, is a company incorporated in India, but its place of effective
management is in UAE. XYZ India would be a …………………
(a) Foreign company for Indian Income-tax Act purpose
(b) Domestic company for Indian Income-tax Act purpose

PROBLEM:23 Non-resident means ................


(a) a person who is not a resident
(b) a person who is a resident
(c) For the purposes of sections 92, 93 and 168 includes a person who is not ordinarily resident
within section 6(6)
(d) Both (a) and (c)

SALEEM QURAISHEE-9175664444 INSPIRE ACADEMY-8888881719


RESIDENTIAL STATUS- Page 2.21

PROBLEM:24 Alpha Inc., a company incorporated in USA, has an Indian subsidiary, Beta India Private
Limited. Alpha Inc. exchanged the shares of Beta India with shares of Gama Singapore. Such a
transaction ................
(a) Would constitute a transfer and liable to capital gains tax in India;
(b) Would constitute a transfer and liable to capital gains tax in India, subject to any Treaty benefits
(c) Would not constitute a transfer and would not be liable to capital gains tax in India

PROBLEM:25 Resident taxpayers are liable to pay tax on their ................


(a) Global income (b) Only India sourced income (c) Only Foreign income

PROBLEM:26 Non-resident taxpayers are liable to pay tax on their ................


(a) Global income (b) Only India sourced income (c) Only Foreign income

PROBLEM:27 An individual is said to be resident in India in any previous year, if he satisfies any
................ of the basic conditions of residential status.
(a) One (b) Both
(c) Both the Basic conditions and at least one additional condition

PROBLEM:28 Which one of the following is a basic condition to determine residential status of an
individual?
(a) He has been in India during the previous year for a total period of 182 days or more
(b) He has been in India during the previous year for a total period of 182 days or less
(c) He has been in India during the 4 years immediately preceding the previous year for total period
of 365 days or more and has been in India for at least 60 days in the previous year
(d) Both (a) and (c)

PROBLEM:29 AJAY , an Indian citizen, left India during previous year 2019-20, for the purpose of
employment outside India on August 31, 2019. He was in India during the 4 years immediately
preceding the previous year for total period of more than 365 days. He would be a ................
(a) Resident in India (b) Non-resident in India

PROBLEM:30 REHAN , an Indian citizen, left India during previous year 2019-20, as a member of the
crew of an Indian ship for the purposes of employment outside India, on September 15,2019. He was
in India during the 4 years immediately preceding the previous year for total period of more than 365
days. He would be a ................
(a) Resident in India (b) Non-resident in India

PROBLEM:31 Samir, an Indian citizen was living outside India since 2013 and comes on a visit to India in
previous year 2019-20 for 180 days. In such a case, he would be ................
(a) Resident in India (b) Non-resident in India

SALEEM QURAISHEE-9175664444 INSPIRE ACADEMY-8888881719


RESIDENTIAL STATUS- Page 2.22

PROBLEM:32 A person is said to be of Indian origin, if ................


(a) He was born in undivided India (b) Either of his parents were born in undivided India
(c) Either of his grandparents were born in undivided India. (d) Either of the above

PROBLEM:33 A person, who is a resident, is said to be resident and ordinarily resident, if he satisfies
................ of the additional conditions specified u/s 6(6).
(a) One (b) Both (c) None

PROBLEM:34 Which one of the following is the additional conditions specified u/s 6(6) for a resident, to
be a resident and ordinarily resident?
(a) Individual has been resident in India in 2 out of 10 previous years immediately preceding the
relevant previous year
(b) Individual has been resident in India in 3 out of 10 previous years immediately preceding the
relevant previous year
(c) Individual has been resident in India in 4 out of 10 previous years immediately preceding the
relevant previous year

PROBLEM:35 Which one of the following is the additional conditions specified u/s 6(6) for a resident, to
be a resident and ordinarily resident?
(a) Individual has during the 7 previous years preceding the relevant previous year been in India for
a period of 730 days or more
(b) Individual has during the 7 previous years preceding the relevant previous year been in India for
a period of 830 days or more
(c) Individual has during the 8 previous years preceding the relevant previous year been in India for
a period of 730 days or more

PROBLEM:36 An individual is said to be a resident but not ordinarily resident, if he ................ any of the
additional conditions.
(a) Fails to satisfy (b) Satisfies

PROBLEM:37 Which of the following can be classified into RNOR/ROR?


(a) Individuals (b) HUFs (c) Company (d) Both (a) and (b)

PROBLEM:38 Residential status is to be determined on ................


(a) Every five years (b) Year to year basis
(c) Initial year of incorporation (d) Both (a) and (b)

PROBLEM:39 “Stay in India” to determine residential status includes ................


(a) Stay in the territorial waters of India
(b) Stay in a ship or boat moored in the territorial waters of India
(c) Date of departure as well as the date of arrival in India
(d) All of the above

SALEEM QURAISHEE-9175664444 INSPIRE ACADEMY-8888881719


RESIDENTIAL STATUS- Page 2.23

PROBLEM:40 Which of the following are not relevant in determining the residential status of an
individual for Income-tax purpose?
(a) Citizenship, place of birth or domicile. (b) Whether period of stay is continuous
(c) Whether stay is at the usual place of residence, business or employment of the individual.
(d) All of the above

PROBLEM:41 RAJAT is an Indian citizen working at a MNC in Mumbai. He was given an opportunity to
work on an assignment in UK for such MNC. He had never visited any foreign country earlier and left
India on June 30, 2019. His residential status for FY 2019-20 would be?
(a) Resident (b) Non-resident (c) Resident but not ordinarily resident.

PROBLEM:42 JOHN is a foreign citizen outside India, whose parents were born in undivided India. He
comes on visit to India on January 1,2020. He left India on April 1,2020. His residential status for FY
2019-20 would be?
(a) Resident (b) Non-resident (c) Resident but not ordinarily resident

PROBLEM:43 Who amongst the following persons, who would be resident in India, if the control and
management of their affairs is situated wholly or partly in India?
(a) HUF (b) Firm (c) AOP/BOI
(d) Local authorities and artificial juridical persons (e) All of the above

PROBLEM:44 Who amongst the following persons, who would be a non-resident in India if the control
and management of their affairs is situated wholly outside India?
(a) HUF (b) Firm (c) AOP/BOI
(d) Local Authorities and Artificial Juridical Persons (e) All of the above

PROBLEM:45 Control and Management of HUF is with ................


(a) Its Karta (b) Owner (c) Any member of the family (d) None of the above

PROBLEM:46 Control and Management of Firm/ AOP is with its ................


(a) Partners/Members. (b) Lender
(c) Any member of the partners family (d) None of the above

PROBLEM:47 An HUF, which is resident in India, can be ................


(a) Resident and ordinarily resident (b) Resident and not ordinarily resident
(c) Non-resident (d) All of the above

PROBLEM:48 Whether an HUF is a resident and ordinarily resident in India or otherwise would depend
upon ................
(a) Whether the Karta of resident HUF satisfies the additional conditions applicable to individuals
(b) Residential status of the HUF
(c) Both the above (d) None of the above

SALEEM QURAISHEE-9175664444 INSPIRE ACADEMY-8888881719


RESIDENTIAL STATUS- Page 2.24

PROBLEM:49 TEJAS, a non-resident, is a Karta of resident HUF. The HUF would be ................
(a) Resident and ordinarily resident
(b) Resident and not ordinarily resident (c) Non-resident

PROBLEM:50 MANOJ, a resident and ordinarily resident, is a Karta of resident HUF. The HUF would be
................
(a) Resident and ordinarily resident (b) Resident and not ordinarily resident (c) Non-resident

PROBLEM:51 SANTOSH, is a Karta of a resident HUF. He was a resident of India in 3 out of 10 previous
years immediately preceding relevant previous year and stayed in India during last 5 previous year
immediately preceding relevant previous year for 740 days. The HUF would be ................
(a) Resident and ordinarily resident (b) Resident and not ordinarily resident
(c) Non-resident

PROBLEM:52 Firms, Association of Persons, Local Authorities and other Artificial Juridical persons can
be ................
(a) Resident and ordinarily resident (b) Resident and not ordinarily resident
(c) Non-resident (d) Resident (e) Both (c) and (d)

PROBLEM:53 The head office of XYZ, HUF was situated in UK. What would be the residential status of
HUF for PY 2019-20, when affairs of HUF are managed from UK?
(a) Resident and ordinarily resident (b) Resident and not ordinarily resident
(c) Non-resident (d) Resident

PROBLEM:54 The head office of XYZ, HUF was situated in UK. What would be the residential status of
HUF for PY 2019-20 when affairs of HUF are managed from partly from India? (Assume status of
Karta is not known)
(a) Resident and ordinarily resident (b) Resident and not ordinarily resident
(c) Non-resident (d) Resident

PROBLEM:55 The importance of residential status of a company is that, it helps in establishing whether
................
(a) For resident companies, global income is taxable
(b) For non-resident companies, India sourced income is taxable
(c) Both (a) and (b) (d) Neither (a) nor (b)

PROBLEM:56 From AY 2020-21, a company is said to be resident in India in any previous year, if it is
................
(a) An Indian company
(b) During that year, the control and management of its affairs is situated wholly in India
(c) Place of Effective Management (‘POEM’), in that year, is in India
(d) Both (a) and (c)

SALEEM QURAISHEE-9175664444 INSPIRE ACADEMY-8888881719


RESIDENTIAL STATUS- Page 2.25

PROBLEM:57 POEM, is an ................ test to determine, whether a company incorporated in a foreign


jurisdiction, is a tax resident of another country.
(a) Internationally acceptable (b) India based
(c) Both (a) and (b) (d) Neither (a) nor (b)

PROBLEM:58 The scope of total income of an assessee depends upon which of the following
considerations?
(a) Residential status of assessee (Le., resident or non-resident or ordinarily not resident)
(b) Place of accrual or receipt of income (in India or outside India), whether actual or deemed;
(c) Point of time of accrual or receipt of income by or on behalf of the assessee
(d) All of the above

PROBLEM:59 Income of non-resident includes ................


(a) Income received in India in the previous year
(b) Income deemed to be received in India in the previous year
(c) Both (a) and (b) (d) Neither (a) nor (b)

PROBLEM:60 Income of non-resident includes ................


(a) Income which accrues or arises in India during the PY
(b) Income which is deemed to accrue or arise in India in the previous year
(c) Both (a) and (b) (d) Neither (a) nor (b)

PROBLEM:61 Income of the non-resident includes ................


(a) Indian income (b) Foreign income (c) Both (a) and (b) (d) Neither (a) nor (b)

PROBLEM:62 Mr. AJAY is a non-resident individual. He received an interest income of Rs. 2,40,000 in
India during the previous year 2019-20. Such income ................ included in the total income of Mr.
AJAY under the IT Act, 1961.
(a) Would be (b) Would not be (c) Can’t Say

PROBLEM:63 Mr. JOY is a non-resident ass- see. He received interest income of Rs. 1,55,000 outside
India during the previous year 2019-20, but the interest accrued and arose in India. Such interest
income ................ in the total income of Mr. JOY for Indian tax purpose.
(a) Would be included (b) Would not be included (c) Can’t Say

PROBLEM:64 Income received or deemed to be received in India during the previous year is taxable in
hands of ................
(a) Resident (b) Non-resident (c) Both (a) and (b) (d) Neither (a) nor (b)

PROBLEM:65 Income received or deemed to be received in India during the previous year is taxable, if it
................
(a) Accrues in India (b) Accrues outside India (c) Both (a) and (b) (d) Neither (a) nor (b)

SALEEM QURAISHEE-9175664444 INSPIRE ACADEMY-8888881719


RESIDENTIAL STATUS- Page 2.26

PROBLEM:66 Income received in India during the previous year is taxable. In such a case, receipt of
income refers to ................
(a) Remittance from one place or person to another
(b) First occasion when recipient gets money under his control
(c) Transmission from one place or person to another (d) None of the above

PROBLEM:67 Mr. Ram was a non-resident for Indian tax purposes. He sold his property situated in
Singapore and credited the amount to his bank account in India. Subsequently, such sale proceeds
were transferred to his bank account in UK. Whether such sale proceeds are taxable in India?
(a) Would be taxable in India as receipt were to his bank account in India
(b) Would not be taxable in India as sale was in Singapore
(c) Would be deemed to accrue in India
(d) Would be deemed to be received in India

PROBLEM:68 Once an item of income is included in the assessee’s total income and subjected to tax on
the ground of its accrual/deemed accrual or receipt, it ................ again, be included in the person’s
total income and subjected to tax on the ground of its ................ receipt.
(a) Can be, Actual (b) Can be, or Deemed
(c) Cannot be, Actual or deemed (d) All of the above

PROBLEM:69 Section 9 Le. income deemed to accrue or arise in India, is meant for ................
(a) Non-residents (b) Residents
(c) Both (a) and (b) (d) Neither (a) nor (b)

PROBLEM:70 Non-residents would be required to pay tax on income which is deemed to accrue or arise
in India ................
(a) Only if it actually accrues in India (b) Only if it actually arises in India
(c) Even though it may actually accrue or arise outside India (d) Both (a) and (b)

PROBLEM:71 Any income accruing or arising in any place outside India shall be deemed to accrue or
arise in India if, it accrues through or from ................
(a) Any business connection in India
(b) Any business connection outside India
(c) Through or from any property outside India
(d) Through or from any asset or source of income outside India

PROBLEM:72 “Business Connection” shall include ................


(a) Any charitable activity carried out through a person acting on behalf of the non-resident
(b) Any business activity carried out through a person acting on behalf of the non-resident
(c) Any business activity carried out through a person acting on his own behalf
(d) None of the above

SALEEM QURAISHEE-9175664444 INSPIRE ACADEMY-8888881719


RESIDENTIAL STATUS- Page 2.27

PROBLEM:73 In case of a non-resident engaged in shooting of cinematograph, no income shall be


deemed to accrue or arise in India from operations confined to the shooting of any cinematograph
film in India, if such non-resident is ................
(a) An individual, who is a citizen of India
(b) A firm which has at least one partner who is a citizen of India or who is resident in India
(c) A company which does not have any shareholder who is a citizen of India or who is resident in
India (d) All of the above

PROBLEM:74 All income which accrues or arises, whether directly or indirectly through or from any
property shall include income from ................
(a) Movable property in India (b) Immovable property in India
(c) Tangible and Intangible property in India (d) All of the above

PROBLEM:75 Salary earned by Indian citizen for services rendered outside India would be deemed to
accrue or arise in Indian, when salary is payable by ................
(a) Indian Government (b) Indian Company (c) Indian Charitable organisation (d) Any of the above

PROBLEM:76 Pension payable ................ India by the Government to its officials and judges, who
permanently reside outside India shall ................ deemed to accrue or arise in India.
(a) In, Be (b) Outside, Be (c) In, Not Be (d) Outside, Not Be

PROBLEM:77 Which of the following interest would be deemed to accrue or arise in India?
(a) Interest payable by the Indian Government to any non-resident
(b) Interest payable by a person resident in India, unless used for Indian business or for earning
foreign income
(c) Interest payable by non-resident in respect of any debt incurred or moneys borrowed and used
for the purpose of a business or profession carried on by such person in India (d) All the above.

PROBLEM:78 Mr. JOY is a citizen of Australia and has been staying in India since 1/1/2013. He leaves
India on 16/7/2019 to visit Australia and returns on 4/1/2020. Determine his residential status for
the AY 20020.
(a) Resident and ordinary resident (b) Resident and not ordinary resident
(c) Non resident (d) Person of Indian origin

PROBLEM:79 HUF is situated in Delhi whose Karta is Mr. JATIN. Mr. JATIN has been in India since
1/1/2015 and before that he was in UK. Mr. JATIN takes all decisions regarding the working of HUF
in India. Determine residential status of HUF for the AY 2020-21 .
(a) Resident and not ordinary resident (b) Resident and ordinary resident
(c) Indian citizen (d) None of the above

SALEEM QURAISHEE-9175664444 INSPIRE ACADEMY-8888881719


RESIDENTIAL STATUS- Page 2.28

PROBLEM:80 HUF is situated in Mumbai. Its karta is Mr. JAY. who is of 95 years of age has delegated
power to his eldest son Mr. Y. Mr. JAY. is in USA for his medical treatment and left India for the first
time on 18/9/2018. Mr. Y has full controls over affairs of HUF. Mr. Y excises partial control from India
and partially from Nepal. Calculate residential status of HUF and Karta Mr. JAY for the AY 2020-21 i.e.
PY 2019-20—
(a) HUF is resident, Mr. JAY is non resident (b) HUF is non-resident, Mr. JAY is non
resident
(c) HUF is resident and ordinary resident, Mr. JAY is non resident
(d) HUF is resident and ordinary resident, Mr. JAY is resident and ordinary resident

PAST EXAMINATION MCQS :

PROBLEM:81 Dec 2014 : The following additional conditions are to be satisfied by a person to be
resident and ordinarily resident in India __________
A. He is a resident in at least any two out of the ten previous years immediately preceding the
relevant previous year
B. He has been in India for 730 day or more during the seven previous years immediately
preceding the relevant previous year
C. Both (A) and (B) of above
D. None of the above

PROBLEM:82 Dec 2014 : X, an Indian citizen, who is living in Delhi since 1982, left for Japan on 1st July,
2019 for employment. He came back to India on 2nd January, 2020 on visit and stayed for 4 months.
His residential status for the AY 2020-21 would be _______
A. Resident and ordinarily resident
B. Not ordinarily resident
C. Non resident
D. Resident

PROBLEM:83 Dec 2014: Abhay earns the following income during the previous year ended 31st March,
2019.
Interest on U.K. development Bonds (1/4th being received in India ): Rs. 2,00,000: profits on sale of a
building in India but received in Holland: Rs. 2,00,000.
The income liable to tax for the AY 2020-21 if Abhay is resident and not ordinarily resident in India
is __________

PROBLEM:84 Dec 2014: If Karta is resident and ordinarily resident in India but control and
management of HUF is situated partly outside India in the previous year, the HUF is _______
A. Resident and ordinarily resident
B. No ordinarily resident
C. Non resident
D. Resident

SALEEM QURAISHEE-9175664444 INSPIRE ACADEMY-8888881719


RESIDENTIAL STATUS- Page 2.29

PROBLEM:85 Dec 2014: Atul is a foreign citizen. His father was born in Delhi in 1951 and mother was
born in England in 1950. His grandfather was born in Delhi in 1922. Atul visited India to see Taj Mahal
and visit other historical places. He came to India on 1st November, 2019 for 200 days. He has never
come to India before. His residential status for AY 2020-21 will be________

PROBLEM:86 June 2015: Paresh, a software engineer at ABC Ltd, left India on 10th August, 2019 for
the treatment of his wife. For income- tax purpose, his residential status for the AY 2020-21 will
be______
A. Resident
B. Non – resident
C. Not ordinarily resident
D. Cannot be determined from the given information

PROBLEM:87 Dec 2015: Residential status of an Indian company is resident and ordinarily resident for
the year__________
A. If the entire control and management is wholly in India
B. If part of the control and management is in India
C. Regardless of the place of control and management
D. If it is listed on recognized stock exchange

PROBLEM:88 Dec 2015: HUF of Ashwin consisting of himself, his wife and 2 sons is assessed to income
tax. The residential status of HUF would be non – resident, when ________
A. The management and controls of its affairs is wholly in India
B. The management and control of its affairs is wholly outside India
C. The status of Karta is non – resident for that year
D. When majority of the members are non – residents

PROBLEM:89 Dec 2015: Ram who was born and brought up in India left for employment in Dubai on
20th August, 2019. His residential status in respect of the AY 2020-21 would be ______
A. Resident and ordinarily resident
B. Non – resident
C. Not ordinarily resident
D. None of the above

PROBLEM:90 Dec 2015: Income of non – resident when attributed from operations in India relating to
the following is taxable in India.
1. Profits of business
2. Fee for technical services
3. Royalty
4. Income from house property in India
Select the correct answer from the options given below______
A. (1) and (4)
B. (1),(3) and (4)
C. (1) and (3)
D. (1),(2),(3) and (4)

SALEEM QURAISHEE-9175664444 INSPIRE ACADEMY-8888881719


EXEMPTED INCOME- Page 3.1

1. INTRODUCTION-
I. Incomes, which do not form part of the total income, are also called the income exempt from tax.
Therefore, these incomes are not included in the total income of the assessee.
II. The burden of proving that income is exempt and therefore is covered in the Section 10 is upon the
assessee and not on the income tax department.

2. EXEMPTION U/S 10 VIS-A-VIS DEDUCTION UNDER CHAPTER VI-A


I. The various items of income referred to in the different clauses of section 10 are excluded from the
total income of an assessee.
II. These incomes are known as exempted incomes. Consequently, such income shall not enter into
the computation of taxable income.
III. Moreover, there are certain other incomes which are included in Gross total income but are wholly
or partly allowed as deductions under Chapter VI-A in computation of total income

SALEEM QURAISHEE Mo: 917566444 INSPIRE ACADEMY-8888881719


EXEMPTED INCOME- Page 3.2

3. AGRICULTURAL INCOME [SECTION 10(1)]


I. Agricultural income is totally exempt, provided it falls within the definition of agricultural income
given under section 2(1A).
II. Agricultural income, though exempt, is to be aggregated in case of certain assesses for the purpose
of determining the rate of tax on non-agricultural income.

4. SUM RECEIVED BY A MEMBER FROM HUF [SECTION 10(2)]


Subject to the provisions of section 64(2), any sum received by an individual, as member of a Hindu
undivided family, shall be exempt in the hands of the member provided the following conditions are
satisfied.
I. Such sum has been received out of the income of the family or
II. In case of any impartible estate, such sum has been paid out of the income of the estate belonging
to the family.
Who can claim exemption: The following persons can only claim exemption under this section:—
I. Members of HUF who on partition would be entitled to demand a share; or
II. Members who are entitled to maintenance under Hindu law.
Thus only those members who have an interest in the joint income of the property are entitled to this
exemption.

5. SHARE OF PROFIT OF A PARTNER FROM A FIRM [SECTION 10(2A)]


I. In case of a person being a partner of a firm which is separately assessed as such, his share of profit
is the total income of the firm shall be exempt from tax.

6. INTEREST ON NON-RESIDENT (EXTERNAL) ACCOUNT [SECTION 10(4)]


In the case of an individual assessee any income by way of interest on money standing to his credit in a
Non-Resident (External) Account in any bank in India shall also be exempt if the following conditions are
satisfied.
I. Money has been deposited in the Non-Resident (External) Account in accordance with the Foreign
Exchange Regulation Act, 1973 and the rules made there under.
II. Such individual is a person resident outside India as defined in section 2(q) of the said Act or he has
been permitted by the Reserve Bank of India to maintain the aforesaid account.

SALEEM QURAISHEE Mo: 917566444 INSPIRE ACADEMY-8888881719


EXEMPTED INCOME- Page 3.3

7. REMUNERATION TO PERSONS WHO ARE NOT CITIZENS OF INDIA [SECTION


10(6)]
In case of an individual who is not a citizen of India, the following income shall be exempt:
I. REMUNERATION RECEIVED BY DIPLOMATS, ETC. [SECTION 10(6)(II)]: Any remuneration received
by a foreign national as an official by whatever name called of an embassy, high commission,
legation, commission, consulate, trade representation of a foreign state, or as a member of staff of
any of these officials for service in such capacity shall be exempt.
• In the case of the members of the staff, the exemption will be available only if the following
two conditions are satisfied:
(a) They are subjects of the country represented; and
(b) They are not engaged in any business or profession or employment in India otherwise
than as members of such staff.

II. REMUNERATION RECEIVE BY A FOREIGN NATIONAL AS EMPLOYEE OF A FOREIGN ENTERPRISE


ANY remuneration received by an individual (non-citizen) as an employee of a foreign enterprise for
services rendered by him during his stay in India, provided the following conditions are fulfilled:
(a) the foreign enterprise is not engaged in any trade or business in India;
(b) his stay in India does not exceed in the aggregate a period of ninety days in such previous year;
and
(c) such remuneration is not liable to be deducted from the income of the employer chargeable
under this Act;

III. NON-RESIDENT EMPLOYED ON A FOREIGN SHIP [SECTION 10(6)- Any salary received by an
individual (non-citizen foreign national) being a non-resident employed on a foreign ship provided
his stay in India does not exceed in the aggregate a period of 90 days in the previous year.

IV. REMUNERATION OF EMPLOYEE OF FOREIGN GOVERNMENT DURING HIS TRAINING IN INDIA - Any
remuneration received by an individual (non-citizen) as an employee of the Government of a
foreign State during his stay in India in connection with his training in any establishment or office
of, or in any undertaking owned by:
(a) the Government; or
(b) any company in which the entire paid up share capital is held by the Central Government or any
State Government or Governments, or partly by the Central Government and partly by one or
more State Governments; or
(c) any company which is a subsidiary of a company referred to in item (b) above; or
(d) any corporation established by or under a Central, State or Provincial Act; or
(e) any society registered under the Societies Registration Act, 1860, or under any other
corresponding law for the time being in force and wholly financed by the Central Government,
or any State Government or State Governments, or partly by the Central Government and partly
by one or more State Governments.

SALEEM QURAISHEE Mo: 917566444 INSPIRE ACADEMY-8888881719


EXEMPTED INCOME- Page 3.4

8. TAX PAYABLE ON ROYALTY OR FEES FOR TECHNICAL SERVICE ON BEHALF


OF FOREIGN COMPANY [SECTION 10(6A)]

1) The taxpayer is a foreign company.


2) It has income by way of royalty or fees for technical services.
3) Such royalty is received from the Central Government or State Government or an Indian
concern under an agreement made after March 31, 1976 but before June 1, 2002.
4) The above agreement is in accordance with the industrial policy of the Indian Government or
it is approved by the Central Government.
5) The payer of income pays tax liability of the foreign company in respect of the above income.
If all the aforesaid conditions are satisfied, the tax liability of the foreign company borne by the payer is not
taxable in the hands of the foreign company (income will not be grossed up).

9. ROYALTY INCOME OR FEES FOR TECHNICAL SERVICES RECEIVED FROM


NATIONAL TECHNICAL RESEARCH ORGANISATION (NTRO) [SECTION
10(6D)]
Income arising to non-corporate non-residents and foreign companies, by way of royalty from or fees from
technical services rendered in or outside India to, the National Technical Research Organisation (NTRO) is
exempt.

10. PAYMENTS TO BHOPAL GAS VICTIMS [SECTION 10(10BB)]


I. Any payment made to a person under Bhopal Gas Leak Disaster (Processing of Claims) Act, 1985
and any scheme framed there under will be fully exempt.
II. However, payments made to any assessee in connection with Bhopal Gas Leak Disaster to the
extent he has been allowed a deduction under the Act on account of any loss or damage caused to
him by such disaster will not be exempted.

11. EXEMPTION FOR COMPENSATION RECEIVED OR RECEIVABLE ON


ACCOUNT OF ANY DISASTER [SECTION 10(10BC)]
I. Any amount received or receivable from CG or a SG or a LA by an individual or his legal heir by way of
compensation on account of any disaster shall be exempt.
II. However, the exemption is not allowable in respect of amount received or receivable to the extent
such individual or his legal heir has been allowed a deduction under the Income-tax Act on account
of any loss or damage caused by such disaster.

SALEEM QURAISHEE Mo: 917566444 INSPIRE ACADEMY-8888881719


EXEMPTED INCOME- Page 3.5

12. AMOUNT RECEIVED UNDER A LIFE INSURANCE POLICY (10(10D)]

Nature of policy Whether exemption is available under section


10(10D)
1. Any sum received under section 80DD (3) Exemption not available
2. Keyman insurance policy Exemption not available
3. Any other policy (sum received on the death Exemption available, nothing is chargeable to tax
of a person)
4. Any other policy (not being the case when
sum received on the death of a person)
a) Policy issued before April 1, 2003 Exemption available, nothing is chargeable to tax
b) Policy issued on or after April 1, 2003 but Exemption available only when annual premium
before April 1, 2012 payable is not more than 20% of sum assured
c) Policy issued during 2012-13 Exemption will be available only when annual
premium payable is not more than 10% of sum
assured
d) Policy issued on or after April 1,2013 Exemption available only when annual premium
payable is not more than 10%/15%.
Note:15% in case of Policy on life of any person who is
person with disability(80U) or suffering from disease
(80DDB)

13. INTEREST AND WITHDRAWALS FROM SUKANYA SAMRIDHI A/C (SECTION


10(11A)
I. Any payment from an account opened in accordance with Sukanya Samriddhi account rules 2014
made under the Government saving bank act 1873 shall not be included in the total income of the
assessee.
II. Interest on deposits in and withdrawals from any account under the scheme would be exempt.

14. PAYMENT FROM NATIONAL PENSION SYSTEM TRUST [SECTION 10(12A)]


I. Any payment from National Pension System Trust referred to in Section 80CCD, to an assessee—
 Due to closure of the pension scheme; or
 his opting out of the pension scheme
shall be exempt from tax upto 60% of the total amount payable to him. (Amended from AY 2020-2021)
II. However, the full amount received by the nominee, on death of the assessee shall be exempt
from tax.

SALEEM QURAISHEE Mo: 917566444 INSPIRE ACADEMY-8888881719


EXEMPTED INCOME- Page 3.6

15. TAX EXEMPTION TO PARTIAL WITHDRAWAL FROM NPS BY AN


EMPLOYEE SECTION 10(12B) W.E.F AY 2018-19
Any payment from the National Pension System Trust to an employee under the pension scheme referred
to in section 80CCD, on partial withdrawal made out of his account in accordance with the terms and
conditions, specified under the Pension Fund Regulatory and Development Authority Act, 2013 (23 of
2013) and the regulations made there under, to the extent it does not exceed twenty-five per cent of the
amount of contributions made by him;

16. SCHOLARSHIPS TO MEET THE COST OF EDUCATION [SECTION 10(16)]


Scholarships granted to meet the cost of education are exempt.

17. DAILY AND CONSTITUENCY ALLOWANCE, ETC. RECEIVED BY MPS AND


MLAS [SECTION 10(17)]
Clause (17) of section 10 provides exemption to Members of Parliament and State Legislature in respect
of the following allowances:
I. Daily allowance: Entire amount is exempt
II. Any other allowance received by a Member of Parliament under the Members of Parliament
(Constituency Allowance) Rules, 1986: Entire amount is exempt
III. Constituency allowance received by any person by reason of his membership of any State
Legislature: Entire amount is exempt

18. AWARD OR REWARD [SECTION 10(17A)]


Any payment made, whether in cash or in kind, shall be exempt from tax provided it is made:
I. in pursuance of any award instituted in the public interest by the Central Government or any State
Government or instituted by any other body and approved by the Central Government in this
behalf, or
II. as a reward by the Central Government or any State Government for such purposes as may be
approved by the Central Government in this behalf in the public interest.

19. PENSION RECEIVED BY RECEIPENT OF GALLANTARY AWARDS[SECTION


10(18)]
I. Income by way of pension/family pension received by an individual or any member of his family
shall be exempt from Income tax if
a) Such individual has been in the service of Central/State Government and
b) has been awarded Param Vir Chakra or Maha Vir Chakra or Vir Chakra or such other gallantry
award as may be notified.
II. In case of the death of such individual, any income by way of family pension received by any
member of the family of such individual shall also be exempt under this clause.

SALEEM QURAISHEE Mo: 917566444 INSPIRE ACADEMY-8888881719


EXEMPTED INCOME- Page 3.7

20. INCOME OF A LOCAL AUTHORITY [SECTION 10(20)]


The following income of a local authority shall be exempt.
I. Income which is chargeable under the head, ‘Income from house property’,
II. Income from ‘Capital gains’, or
III. Income from other sources’, or
IV. Income from a trade or business carried on by it which accrues or arises from the supply of:
a. water or electricity within or outside its own jurisdictional area, or
b. Any other commodity or service within its own jurisdictional area.
In other words, the entire income of a local authority shall be exempt from tax except the income
derived from the supply of commodity or service (other than water and electricity) outside its own
jurisdictional area.

21. INCOME OF SPECIFIED NEWS AGENCY [SECTION 10(22B)]


I. Any income of such news agency, set-up in India solely for collection and distribution of news, as the
Central Government may notify, in this behalf, is totally exempt.
II. The exemption is available, provided the news agency applies its income or accumulates it for
application solely for collection and distribution of news and does not distribute its income in any
manner to its member

22. INCOME OF PROFESSIONAL ASSOCIATION/INSTITUTIONS [SECTION


10(23A)]
Any income (other than income from house property, income received for rendering any specific service,
or income by way of interest or dividend on investments) of a professional institution is exempt from tax
if the following conditions are satisfied:
1) Professional institution is established in India for the purpose of control, supervision,
regulation or encouragement of the profession of law, medicine, accountancy, engineering or
architecture or any other notified profession [i.e., company secretary, chemistry, materials
management and town planning].
2) The institution applies its income or accumulates it for application solely to the object for
which it is established.
3) The institution is approved by the Central Government.

SALEEM QURAISHEE Mo: 917566444 INSPIRE ACADEMY-8888881719


EXEMPTED INCOME- Page 3.8

23. INCOME OF FUND ESTABLISHED FOR WELFARE OF EMPLOYEES


[SEC.10(23AAA)]
Exemption under section 10(23AAA) is available if the following conditions are satisfied:
1) There is a fund established for purposes as maybe notified by the Board, for the welfare of
employees or their dependents.
2) Employees are members of the fund.
3) The fund applies its income, or accumulates it for application wholly and exclusively to the
objects for which it is established.
4) The aforesaid fund shall invest its funds and contributions made by the employees and
other sums received by it in anyone or more of the forms or modes specified in section
11(5).
5) The said fund is approved by the Commissioner (Form No. 9 for application) in accordance
with the rules made in this behalf. At one time approval cannot be given for more than 3
assessment years.

24. INCOME OF CERTAIN NATIONAL FUNDS, EDUCATIONAL INSTITUTION &


HOSPITAL [SEC. 10(23C)]
Exemption given by section 10(23C) is given below:
INCOME OF CERTAIN NATIONAL FUNDS:
Any income received by any person on behalf of the following funds is exempt* from tax
I. the Prime Minister's National Relief Fund
II. the Prime Minister's Fund (Promotion of Folk Arts)
III. the Prime Minister's Aid to Student Fund
IV. the National Foundation for Communal Harmony
V. The Swach bharat kosh
VI. The clean ganga fund
VII. Chef Minister relief fund/ LG Relief fund. FA-2017
VIII. any other charitable fund or institution notified by the Central Government
IX. any trust or institution wholly for public religious purposes or wholly for public religious and
charitable purposes which is notified by the Central Government

25. INCOME OF EDUCATIONAL INSTITUTIONS


Income of the following educational institutions is exempt from tax under section 10(23C):
Cases 1 Any university or other educational institution existing solely for educational purposes and
not for purposes of profit, and which is wholly or substantially financed by the
Government
Cases 2 Any university or other educational institution existing solely for educational purposes and
not for purposes of profit if the aggregate annual receipts of such university or educational
institution do not exceed the amount of annual receipts as maybe prescribed (i.e. ` 1
crore)
Cases 3 Any university or other educational institution existing solely for educational purposes and
not for purposes of profit, other than those mentioned in Case 1 and Case 2 (supra) and
which is to be approved by the prescribed authority (i.e., the Chief Commissioner)
An educational institution mentioned under Case 3 will have to satisfy some conditions to claim
exemption.

SALEEM QURAISHEE Mo: 917566444 INSPIRE ACADEMY-8888881719


EXEMPTED INCOME- Page 3.9

26. INCOME OF HOSPITAL -


If the following conditions are satisfied, the income of a hospital is exempt from tax under section
10(23C):
1 Income arises to a hospital or other institution for the reception and treatment of persons:
a. suffering from illness or mental defectiveness; or
b. during convalescence or
c. requiring medical attention or rehabilitation.
2 The hospital or other institution exists solely for philanthropic purposes and not for the purpose
of profit.
3 The hospital or other institution is:
a. wholly or substantially financed by the Government
b. the aggregate annual receipts of such hospital or institution do not exceed the amount
prescribed (i.e., ` 1 crore)
c. approved by the prescribed authority (i.e., the Chief Commissioner) [one has to satisfy
certain conditions for getting approval].

27. REVERSE MORTGAGE SCHEME [SEC. 10(43)] –


I. Under reverse mortgage, the borrower is generally a senior citizen.
II. He owns a house property but does not having a regular source of income.
III. He mortgages his property with a scheduled bank or a housing finance company (i.e., lender).
IV. The lender in return pays periodic installments or lump sum to the borrower during his lifetime.
V. The borrower can continue to stay in the property during his lifetime and as well continue to
receive regular income from the lender.
VI. The borrower does not pay the principle as well as interest to the lender during his lifetime.
VII. The lender will recover the loan along with the accumulated interest by selling the house after
the death of the borrower.
VIII. However, before resorting to disposal of the property, an option will be given to the legal heirs to
repay the loan amount, along with the interest, and to get the mortgaged property released.Any
excess amount will be remitted back to the legal heirs of the borrower.

28. INCOME OF SOCIETY OR TRUST EXISTING FOR DEVELOPMENT OF KHADI


OR VILLAGE INDUSTRIES [SECTION 10(23B)]
Any income of an institution, constituted as a public charitable trust or registered under the Societies
Registration Act or under any law corresponding to that Act and existing solely for the development of
Khadi or Village Industries or both and not for the purpose of profit, to the extent such income is
attributable to the business of production, sale, marketing or Khadi or products of Village Industries, shall
be exempt

SALEEM QURAISHEE Mo: 917566444 INSPIRE ACADEMY-8888881719


EXEMPTED INCOME- Page 3.10

29. INCOME OF AUTHORITY KNOWS AS KHADI AND VILLAGE INDUSTRIES


BOARD, ETC. [SECTION 10(23BB)]
Any income of an authority (whether known as the Khadi and Village Industries Board or by any other
name) established in a State by or under a State or Provincial Act for the development of Khadi or Village
Industries in the State should be exempt.

30. INCOME OF INVESTOR PROTECTION FUND-[SECTION 10(23EA)]


I. Income by way of contribution received from recognized stock exchange and the members thereof of such
Investor Protection Fund set up by recognized stock exchanges in India, either jointly or separately, as the
Central Government may, by notification in the Official Gazette, specify in this behalf, shall be exempt.
II. where any amount standing to the credit of the Fund and not charged to income-tax during any previous year is
shared, either wholly or in part, with a recognized stock exchange, the whole of the amount so shared shall
be deemed to be the income of the previous year in which such amount is so shared and shall accordingly be
chargeable to income-tax.

31. EXEMPTION TO INCOME OF INVESTOR PROTECTION FUND OF


DEPOSITORIES [SECTION 10(23ED)]
I. An income, by way of contributions received from a depository, of such Investor Protection Fund set up in
accordance with the regulations by a depository as the Central Government may, by notification in the
Official Gazette, specify in this behalf:
II. Provided that where any amount standing to the credit of the Fund and not charged to income-tax during any
previous year is shared, either wholly or in part with a depository, the whole of the amount so shared shall
be deemed to be the income of the previous year in which such amount is so shared and shall, accordingly,
be chargeable to income-tax.

32. INCOMES OF CERTAIN BODIES - [SECTION 10(29A)]


Under this clause, any income accruing or arising to the following bodies is exempt from tax:
The Coffee Board constituted under section 4 of the Coffee Act, 1942,
The Rubber Board constituted under section 4(1) of the Rubber Board Act, 1947,
The Tea Board established under section 4 of the Tea Act, 1953,
The Tobacco Board constituted under the Tobacco Board Act, 1975,

33. TEA BOARD SUBSIDY [SECTION 10(30)] –


The amount of any subsidy received by any assessee engaged in the business of growing and
manufacturing tea in India through or from the Tea Board will be wholly exempt from tax.
Conditions-
I. The subsidy should have been received under any scheme for replantation or replacement of the
bushes or for rejuvenation or consolidation of areas used for cultivation of tea.
II. The assessee should furnish a certificate from the Tea Board as to the subsidy received by him
during the previous year to the Assessing Officer along with his return of the relevant assessment
year or within the time extended by the Assessing Officer for the purpose.

SALEEM QURAISHEE Mo: 917566444 INSPIRE ACADEMY-8888881719


EXEMPTED INCOME- Page 3.11

34. OTHER SUBSIDIES [SECTION 10(31)]


I. Amount of any subsidy received by an assessee engaged in the business of g rowing and
manufacturing rubber, coffee, cardamom or other specified commodity in India from or through
the Rubber Board, Coffee Board, Spices Board or any other Board in respect of any other
commodity under any scheme for replantation or replacement of rubber, coffee, cardamom or
other plants or for rejuvenation or consolidation of areas used for cultivation of all such
commodities will be exempt from income-tax

35. EXEMPTION OF SPECIFIED INCOME OF CORE SETTLEMENT GUARANTEE


FUND SET UP BY A RECOGNIZED CLEARING CORPORATION [SECTION
10(23EE)]
I. The Clearing Corporations are required, under the provisions of Securities Contracts (Regulation) (Stock
Exchanges and Clearing Corporations) Regulations, 2012 notified by SEBI, to establish a fund, called Core
Settlement Guarantee Fund (Core SGF) for each segment of each recognized stock exchange to
guarantee the settlement of trades executed in respective segments of the exchange.
II. Under sections 10(23EA), 10(23EC) and 10(23ED), income by way of contributions received from
recognized stock exchanges or commodity exchanges and the members thereof or depositories of Investor
Protection Fund set up by such recognized stock exchanges in India, or by commodity exchanges in India or
by such depository, respectively, as the Central Government may notify in this behalf, are exempt from
taxation.
III. On parallel lines, section 10(23EE) exempts any specified income of such Core SGF set up by a
Recognized Clearing Corporation in accordance with the regulations, notified by the Central Government in
the Official Gazette.
IV. However, where any amount standing to the credit of the Fund and not charged to income- tax during any
previous year is shared, either wholly or in part with the specified person, the whole of the amount so
shared shall be deemed to be the income of the previous year in which such amount is shared, and shall
accordingly be chargeable to income-tax

36. INCOME OF TRADE UNION [SECTION 10(24)]


Section 10(24) provides that ‘income from house property’ and ‘income from other source’ of a registered union,
within the meaning of the Indian Trade Unions Act, 1926 or an Association of such Trade Unions, formed primarily
for the purpose of regulating the relations between workmen and employers or between workmen and workmen, is
exempt from income-tax. In other words, two conditions have to be satisfied in order to obtain exemption under
this clause:
(i) that the assessee must be registered under the Indian Trade Union Act, 1926 or it is an Association of
such Trade Unions; and
(ii) That it should have been formed primarily for the purpose of regulating the relations between workmen
and employers or between workmen and workmen.

SALEEM QURAISHEE Mo: 917566444 INSPIRE ACADEMY-8888881719


EXEMPTED INCOME- Page 3.12

37. INCOME OF EMPLOYEES’ STATE INSURANCE FUND [SECTION 10(25A)]


Any income of the Employees’ State Insurance Fund set up under the provisions of the Employees’ State
Insurance Act, 1948 shall be exempt

38. INCOME AN INDIVIDUAL BEING A SIKKIMESE [SECTION 10(26AAA)]


The following income, which accrues or arises to a Sikkemese individual, shall be exempt from income-
tax—
(a) Income from any source in the State of Sikkim; or
(b) Income by way of dividend or interest on securities.
However, this exemption will not be available to a Sikkimese woman who, on or after 1-4-2008,
marries a non-Sikkimese individual.

39. EXEMPTION OF INCOME OF CORPORATIONS ESTABLISHED TO PROTECT


INTERESTS OF MINORITY COMMUNITY [SECTION 10(26BB)] –
Any income of a corporation established by the Central Government or any State Government for
promoting the interests of the members of a minority community will be exempt from income tax. Section
80G also provides tax relief in respect of donations made to these corporations.

40. EXEMPTION OF INCOME OF CORPORATION ESTABLISHED BY A


CENTRAL, STATE OR PROVINCIAL ACT FOR WELFARE OF EX-SERVICEMEN
[SECTION 10(26BBB)] –
This clause exempts any income of a corporation established by a Central, State or Provincial Act for the
welfare and economic up liftment of ex-servicemen, being citizens of India.

41. INCOME OF AGRICULTURAL PRODUCE MARKETING COMMITTEE OR


BOARD [SECTION 10(26AAB)]
Any income of an agricultural produce market committee or board constituted under any law for the time
being in force for the purpose of regulating the marketing of agricultural produce shall be exempt.

42. CO-OPERATIVES FOR SCHEDULED CASTES [SECTION 10(27)] –


Any income of a cooperative-society formed for promoting the interests of the members of either the
scheduled castes or scheduled tribes will be exempted from being included in the total income of the
society.
Conditions:
I. The membership of the co-operative society should consist of only other co-operative societies
formed for similar purposes,
II. The finances of the society shall be provided by the Government and such other societies.

SALEEM QURAISHEE Mo: 917566444 INSPIRE ACADEMY-8888881719


EXEMPTED INCOME- Page 3.13

43. INCOME OF A CORPORATION ETC. FOR THE PROMOTION OF INTERESTS


OF MEMBERS OF SCHEDULED CASTS OR TRIBES OR BOTH [SECTION
10(26B)] –
Any income of a corporation (established by a Central, State or Provincial Act) or other body, institution or
association (wholly financed by Government) formed for promotion of the interests of the members of
scheduled castes or tribes or backward classes or of any two or all of them is exempt from tax.

44. INCOME OF PRASAR BHARTI (BROADCASTING CORPORATION OF INDIA)


[SECTION 10 (23BBH)
Any income of the Prasar Bharati (Broadcasting Corporation of India) established under sub-section (1) of
section 3 of the Prasar Bharati (Broadcasting Corporation of India) Act, 1990 shall be exempt.

45. EXEMPTION OF SPECIFIED INCOME FROM INTERNATIONAL SPORTING


EVENT HELD IN INDIA [SECTION 10(39)]
Any specified income arising, from any international sporting event held in India, to the person or persons
notified by the Central Government in the Official Gazette, shall be exempt if such international sporting
event—
I.is approved by the international body regulating the international sport relating to such event;
II.has participation by more than two countries;
III.Is notified by the Central Government in the Official Gazette for the purpose of this clause.

46. EXEMPTION OF SPECIFIED INCOME OF NOTIFIED BODY OR AUTHORITY


OR BOARD OR TRUST OR COMMISSION [SECTION 10(46)]
Any specified income arising to a body or authority or Board or Trust or Commission (by whatever name
called) or a class thereof which—
(a) has been established or constituted by or under a Central, State or Provincial Act, or constituted by
the Central Government or a State Government, with the object of regulating or administering any
activity for the benefit of the general public;
(b) is not engaged in any commercial activity; and
(c) Is notified by the Central Government in the Official Gazette for the purposes of this clause.
Shall be exempt.

47. EXEMPTION OF INCOME OF AN INFRASTRUCTURE DEBT FUND [SECTION


10(47)]
I. Clause (47) has been inserted in section 10 with effect from June 1, 2011 (i.e., with effect from
the assessment year 2012-13).
II. It provides enabling power to the Central Government to notify any infrastructure debt fund
which is set-up in accordance with the prescribed guidelines.
III. Once notified, the income of such a debt fund would be exempt from tax.
IV. However, such fund will have to submit return of income under section 139.

SALEEM QURAISHEE Mo: 917566444 INSPIRE ACADEMY-8888881719


EXEMPTED INCOME- Page 3.14

48. EXEMPTION IN RESPECT OF INCOME RECEIVED BY CERTAIN FOREIGN


COMPANIES [SECTION 10(48)
A new clause (48) has been inserted in section 10 of the Income-tax Act to provide for exemption in
respect of any income of a foreign company received in India in Indian currency on account of sale of crude
oil to any person in India subject to the following conditions being satisfied:
(i) The receipt of money is under an agreement or an arrangement which is either entered into by
the Central Government or approved by it.
(ii) The foreign company, and the arrangement or agreement has been notified by the Central
Government having regard to the national interest in this behalf.
(iii) The receipt of the money is the only activity carried out by the foreign company in India.
Amendment made by the Finance Act, 2013, w.e.f.A.Y. 2014-15
Besides the sale of crude oil, the Finance Act, 2013 has extended the exemption to:
(a) any other goods; or
(b) rendering of services as by the notified by the Central Government in this behalf.

49. EXEMPTION OF INCOME ACCRUING OR ARISING TO A FOREIGN


COMPANY ON ACCOUNT OF STORAGE OF CRUDE OIL IN A FACILITY IN
INDIA AND SALE OF CRUDE OIL THERE FROM TO ANY PERSON RESIDENT
IN INDIA [SECTION 10(48A)] F.A 2016
Any income accruing or arising to a foreign company on account of storage of crude oil in a facility in India
and sale of crude oil there from to any person resident in India, if, -
I. such storage and sale by the foreign company is pursuant to an agreement or an arrangement
entered into by the Central Government or approved by the Central Government; and
II. Having regard to the national interest, the foreign company and the agreement or arrangement are
notified by the Central Government in this behalf.

50. EXEMPTION OF INCOME OF FOREIGN COMPANY FROM SALE OF LEFT-


OVER STOCK OF CRUDE OIL ON TERMINATION OF AGREEMENT OR
ARRANGEMENT-SECTION 10 (48B)
Any income accruing or arising to a foreign company on account of sale of leftover stock of crude oil, if any,
from the facility in India after the expiry of the agreement or the arrangement referred to in clause (48A)
[or on termination of the said agreement or the arrangement, in accordance with the terms mentioned
therein, as the case may be,] subject to such conditions as may be notified by the Central Government in
this behalf;]

51. PERQUISITES/ALLOWANCES TO CHAIRMAN/MEMBERS OF UPSC [SEC.


10(45)]
The following allowances and perquisites are not chargeable to tax:
A. In case of serving Chairman and members of Union Public Service Commission (UPSC)
I. the value of rent free official residence;
II. the value of conveyance facilities including transport allowance;
III. the sumptuary allowance;
IV. The value of leave travel concession provided to a serving Chairman or member of the UPSC and
members of his family.

SALEEM QURAISHEE Mo: 917566444 INSPIRE ACADEMY-8888881719


EXEMPTED INCOME- Page 3.15

B. In case of the retired Chairman and retired members of UPSC -


I. a sum of maximum ` 14,000 per month for defraying the service of an orderly and for meeting
expenses incurred towards secretarial assistance on contract basis;
II. The value of a residential telephone free of cost and the number of free calls to the extent of 1,500
per month (over and above the number of free calls per month allowed by the telephone
authorities).

52. OTHER EXEMPTIONS-


I. Any Income of the National Financial Holdings Limited being a Company set up the Central
Government, of any previous year relevant to any assessment year commencing on or before 01-04-
2014.

II. Any Income arising from any specified service provided on or after the date on which the provisions
of Chapter VIII of the Finance act,2016 comes into force and chargeable to equalization levy under
that Chapter.(F.A 2016)

III. Any Income by way of distributed income referred to in section 115TA received from Securitasion
trust by any person being an investor of the trust.

IV. Income from Securitasion trust from the activity of Securitasion.

V. Any Income of a business trust by way of


a) Interest received or receivable from a special purpose vehicle.
b) Dividend received under section 115-O(7)

VI. Any distributed income referred to in section 115UA received by unit holder from Business trust ,
not being that proportion of the income which is of the same nature as the income referred to in
section 10(23FC).

SALEEM QURAISHEE Mo: 917566444 INSPIRE ACADEMY-8888881719


EXEMPTED INCOME- Page 3.16

53. EXPENDITURE INCURRED IN RELATION TO EXEMPT INCOME [SEC.14A]


1) No Deduction: In computation of Total Income under the Income Tax Act, the expenditure incurred
in relation to Exempted Income, will not be allowed as deduction.
2) Power of AO to determine expenditure: The Assessing Officer shall determine the amount of
expenditure incurred in relation to Exempt Income, in accordance with the prescribed method, in
the following situations
a) Having regard to the accounts of the Assessee, if the Assessing Officer is not satisfied with the
correctness of the claim of the Assessee in respect of such expenditure, or
b) The Assessee claims that no expenditure has been incurred in relation to Exempt Income.

3) Prescribed Method (Rule 8D): The Disallowed Expenditure will be the aggregate of—
a) The amount of expenditure directly relating to income which does not form part of total
income, and
b) An amount equal to 1% of the Annual Average of the monthly averages of the opening and
closing balances of the value of investment, income from which does not or shall not form part
of Total Income,
i.e.
1% x Monthly Average of Opening & Closing Balances of the Value of Investment/12 months
Point to be noted
I. The amount referred to in (a) and (b) shall not exceed the total expenditure claimed by the assessee.
II. Sec.14A provides for disallowance of the expenditure, even where taxpayer in a particular year has
not earned any Exempt Income.

54. UNITS SPECIAL ECONOMIC ZONE -[SEC 10AA]-


A. The following conditions should be satisfied:
I. The assessee is an entrepreneur, i.e., a person who has been granted a letter of approval by the
Development Commissioner to set a unit in a Special Economic Zone.
II. The unit in Special Economic Zone begins to manufacture or produce articles or things or provide
services during the financial year 2005-06 or any subsequent year.
III. It is not formed by the splitting up, or reconstruction, of a business already an existence.
IV. It is not formed by the transfer to a new business, of old plant or machinery. However, it can be
formed by transfer of old plant or machinery to the extent of 20 per cent.
V. The assessee has exported goods or provided services out of India from the Special Economic Zone
by land, sea, air or by any other mode, whether physical or otherwise.
VI. Books of account of the taxpayer should be audited.
VII. Deduction should be claimed in the return of income.

B. AMOUNT OF DEDUCTION: Deduction depends upon quantum of profit derived from export of
articles or things or services (including computer software). It is calculated as under:

SALEEM QURAISHEE Mo: 917566444 INSPIRE ACADEMY-8888881719


EXEMPTED INCOME- Page 3.17

a. Deduction for first five assessment years - 100 per cent of the profits and gains derived from
export of articles or things or from services is deductible for a period of 5 consecutive assessment
years. Deduction for the first year is available in the assessment year relevant to the previous
year in which the unit begins to manufacture or produce articles or things or provide services.

b. Deduction for sixth assessment year to tenth assessment year- 50 per cent of the profits and
gains derived from export of articles or things or from services is deductible for the next 5 years.

c. Deduction for eleventh assessment year to fifteenth assessment year- For the next 5 years, a
further deduction would be available to the extent of 50 per cent of the profit provided an
equivalent amount is debited to the profit and loss account of the previous year and credited to
Special Economic Zone Re-investment Allowance Reserve Account (hereinafter referred to as
Special Reserve Account).

d. The following conditions should be satisfied-


1. The Special Reserve Account should be utilized for the purpose of acquiring new plant and
machinery.
2. The new plant and machinery should be first put to use before the expiry of 3 years from the
end of the year in which the Special Reserve Account was created.
3. Until the acquisition of new plant and machinery the Special Reserve Account can be utilized
for the business purposes of the undertaking but it cannot be utilized for distribution of
dividends/ profits or for remittance outside India as profits or for creating an asset outside
India.
4. Prescribed particulars [Form No. 56FF] should be submitted in respect of new plant and
machinery. In the new income-tax return forms annexure is not possible. Form No. 56FF
should be retained by the taxpayer and can be submitted whenever the Assessing Officer
wants to examine it.
5. If the Special Reserve Account is misutilised, then the deduction would be taken back in the
year in which the Special Reserve Account is misutilised. If the Special Reserve Account is not
utilized for acquiring new plant and machinery within three years as stated above, then the
deduction would be taken back in the year immediately following the period of three years.

PROBLEM:1 Hans Ltd. is a unit located in SEZ. It has Export turnover of Rs. 500 lakhs; Total turnover of
Rs. 900 lakhs; Business profits of Rs. 80 lakhs. The said export turnover includes sales tax amounting to
Rs. 20 lakhs and excise duty amounting to Rs. 30 lakhs. Further, the assessee incurred freight
amounting to 50 lakhs outside India. Compute the deduction available under section 10AA.
ANSWER:1

SALEEM QURAISHEE Mo: 917566444 INSPIRE ACADEMY-8888881719


EXEMPTED INCOME- Page 3.18

PROBLEM:2 X Ltd. is located in SEZ. From the following information furnished by it, compute its total
income ,Total turnover for the year = Rs. 500 lakhs, Export turnover = Rs. 400 lakhs, Business profits -
50 lakhs. Here, export turnover represents export of computers @ Rs. 80,000 per computer.
ANSWER:2

PROBLEM:3 Y Co. Ltd. furnishes you the following information for the year ended 31.3.2020:

Particulars

Total turnover of Unit A located in Special Economic Zone 100

Profit of the business of Unit A 30

Export turnover of Unit A 50

Total turnover of Unit B located in Domestic Tariff Area (DTA) 200

Profit of the business of Unit B 20


Compute deduction under section 10AA for the A.Y. 2020-21.
ANSWER:3

PROBLEM:4 Rudra Ltd. has one unit at Special Economic Zone (SEZ) and other unit at Domestic Tariff
Area (DTA). The company provides the following details for the previous year 2019-20.

Particulars Rudra Ltd. (`) Unit in DTA (`)


Total Sales 6, 00, 00,000 2, 00, 00,000
Export Sales 4, 60, 00,000 1, 60, 00,000
Net Profit 80, 00,000 20, 00,000
Calculate the eligible deduction under section 10AA of the Income-tax Act, 1961, for the Assessment
Year 2020-21, in the following situations:
(i) If both the units were set up and start manufacturing from 22-05-2012.
(ii) If both the units were set up and start manufacturing from 14-05-2016.
ANSWER:4

SALEEM QURAISHEE Mo: 917566444 INSPIRE ACADEMY-8888881719


EXEMPTED INCOME- Page 3.19

PROBLEM:5 Mr. Pranay is running two Industrial Undertakings, one in a SEZ (Unit A) and another in a
DTA (Unit B). The brief details for the year ended 31.03.2020 are as under: (amounts ` in Lakhs)
Particulars Unit A Unit B
Domestic Turnover 10 100
Export Turnover 120 Nil
Gross Profit 20 10
Less: Expenses and Depreciation 07 06
Profits derived from the Units 13 04
The Brought Forward Business Loss pertaining to Assessment Year 2016-2017 for Unit B is `3.2
Lakhs. Briefly compute the Business Income of the Assessee.
ANSWER:5
PROBLEM:6 Mr. Suresh has set up an Undertaking in SEZ (Unit A) and another Undertaking in DTA (Unit
B) in Financial Year 2014-15. In the Previous Year 2019-20, Total Turnover of the Unit A is `180 Lakhs
and Total Turnover of Unit B is `120 Lakhs. Export Turnover of Unit A for the year is`150 Lakhs and the
Profit for the Unit A is `60 Lakhs. Calculate the deduction available, if any, to Mr. Suresh u/s 10AA for
Assessment Year 2020-21, if the manufacturing started in Unit A in Financial Year 2014-15.
ANSWER:6

PROBLEM:7 Examine whether the following are chargeable to tax-


I. Arvind received ` 20,000 as his share from the income of the HUF.
II. Mr. Xavier, a ‘Param Vir Chakra’ awardee, who was formerly in the service of the Central
Government, received a pension of ` 2,20,000 during the financial year 2019-20.
III. Agricultural income of ` 1,27,000 earned by a resident of India from a land situated in Malaysia.
IV. Rent of ` 72,000 received for letting out agricultural land for a movie shooting.
ANSWER:7
PROBLEM:8 Examine with reasons in brief whether the following statements are true or false with
reference to the provisions of the Income-tax Act, 1961:
I. Exemption is available to a Sikkimese individual, only in respect of income from any source in
the State of Sikkim.
II. Pension received by a recipient of gallantry award, who was a former employee of Central
Government, is exempt from income-tax.
III. Mr. A, a member of a HUF, received ` 10,000 as his share from the income of the HUF. The
same is to be included in his chargeable income.
ANSWER:8

SALEEM QURAISHEE Mo: 917566444 INSPIRE ACADEMY-8888881719


EXEMPTED INCOME- Page 3.20

55. MCQ
PROBLEM:1 Which of the following income/s do not form part of Total Income?
(a) Income of newly established units in Special Economic Zones
(b) Income of Political Parties (c) Income of an Electoral Trust (d) All of the above

PROBLEM:2 As per section 10(2), any sum received by an individual as a member of HUF is ................
in the hands of such member.
(a) Taxable (b) Exempt (c) Partly Taxable (d) None of the above

PROBLEM:3 As per section 10, which of the following income/s is/are not exempt from tax?
(a) Share or profit of a partner in total income of the firm
(b) Interest received by an individual on account of amount standing to his credit in a non-resident
account in India
(c) Salary received by a non-resident, non-Indian citizen employee on a foreign ship whose stay in
India is 92 days in the previous year
(d) Remuneration received by foreign government employee during his training in India

PROBLEM:4 As per section 10(6)(viii), a non-resident employed on a foreign ship must stay in India for
a period not exceeding ................ , in order to exempt his income from tax.
(a) 90 days (b) 100 days (c) 182 days (d) 212 days

PROBLEM:5 Any sum received under life insurance policy (by way of Keyman Insurance Policy) is
................ u/s 10(10D).
(a) Not exempt from tax (b) Exempt from tax
(c) Taxable upto specified amount (d) None of the above

PROBLEM:6 As per section 10(12A), amount payable at the time of closure of National Pension Scheme
is exempt upto ................
(a) 25% of the total amount payable (b) 30% of the total amount payable
(c) 40% of the total amount payable (d) 60% of the total amount payable

PROBLEM:7 As per section 10(12A), amount pay able at the time of closure of National Pension
Scheme is exempt upto 40% of the total amount payable in case of assessee being an ................
(a) Employee (b) Non-Employee
(c) Both (a) & (b) (d) None of the above

PROBLEM:8 Any payment received by the employee from National Pension System Trust, on partial
withdrawal upto ................ of the amount of contributions made by him is exempt from tax.
(а) 10% (b) 15% (c) 25% (d) 30%

SALEEM QURAISHEE Mo: 917566444 INSPIRE ACADEMY-8888881719


EXEMPTED INCOME- Page 3.21

PROBLEM:9 As per section 10(18), pension received by family member of an individual will be exempt,
if
(a) Such individual has been in the service of Central/State Government
(b) Such individual has been awarded Param Vir Chakra/Maha Vir Chakra/any other gallantry award
(c) Both (a) and (b) (d) None of the above

PROBLEM:10 As per section 10(20), which of the following income of a local authority is not exempt
from tax?
(a) Income from Other Sources (b) Income from Capital Gains
(c) Income from stile of any commodity outside its jurisdictional area
(d) Income from sale of electricity or water outside its jurisdictional area

PROBLEM:11 Income of a member of Scheduled Tribe residing in Manipur is ................


(a) Exempt from Tax (b) Partly Taxable
(c) Fully Taxable (d) None of the above

PROBLEM:12 As per section 10(26AA4), which of the following income is not exempt from tax for
Sikkimese individual?
(a) Income from any source in the State of Sikkim (b) Income by way of dividend
(c) Income by way of interest on securities (d) None of the above

PROBLEM:13 Section 10(44) deals with ................


(a) Exemption of amount received by individual as loan under reverse mortgage scheme
(b) Exemption of income of New Pension Scheme Trust
(c) Exemption in respect of income received by certain foreign companies (d) None of the above

PROBLEM:14 Exemption u/s 10(48B) to the extent of ................ in respect of income of Foreign
Company from sale of leftover stock of crude oil on termination of agreement or arrangement.
(a) 100% is available (b) 50% is available (c) Not available (d) None of the above

PROBLEM:15 Which of the following income is not exempt under section 10 –


(a) Share in total income of firm (b) Income from agriculture in Lahore (c) Bonus on life insurance (d)
Income from mutual funds.

PROBLEM:16 Mr. X received compensation of Rs. 2,60,000 from the Central Government on account of
disaster. He claimed Rs. 1,20,000 as a deduction on account of loss or damage caused by such
disaster under this Act. What amount of compensation received shall be exempt?
(a) Rs. 2,60,000 (b) Rs. 1,40,000 (c) Rs. 1,20,000 (d) Nil

SALEEM QURAISHEE Mo: 917566444 INSPIRE ACADEMY-8888881719


EXEMPTED INCOME- Page 3.22

PROBLEM:17 If in the above case, Mr., X claimed Rs. 2, 60,000 as a deduction on account of loss or
damage caused by such disaster under this Act then what amount of compensation received shall be
exempt?
(a) Rs. 12,60,000 (b) Rs. 1,40,000 (c) Rs. 1,20,000 (d) Nil

PROBLEM:18 Any income of a business trust by way of interest received or receivable from a special
purpose vehicle shall ____.
(a) Be unconditionally exempt (b) Be conditionally exempt
(c) Be taxable (d) Be taxable after deduction of 50%.

PROBLEM:19 The amount received including bonus under Life Insurance Policy issued on 01-4-2017 in
respect of which the premium payable for the year during the term of the policy was 15% of the
actual capital sum assured. What shall be the not exempt amount?
(a) Premium payable exceeds 15% of actual capital sum assured.
(b) Premium payable exceeds 10% of actual capital sum assured.
(c) Premium payable exceeds 20% of actual capital sum assured. (d) No exemption

PROBLEM:20 If in the above case, the Life Insurance Policy is issued on life of any person, who is a
person with disability or a person with severe disability as referred to in section 8OU; or suffering
from disease or ailment as specified in the rules made under section 80DDB then what shall be the
not exempt amount?
(a) Premium payable exceeds 15% of actual capital sum assured.
(b) Premium payable exceeds 10% of actual capital sum assured.
(c) Premium payable exceeds 20% of actual capital sum assured. (d) No exemption

PROBLEM:21 The amount received including bonus under Life Insurance Policy issued on 01-4-2012 in
respect of which the premium payable for the year during the term of the policy was 15% of the
actual capital sum assured. What shall be the not exempt amount?
(a) Premium payable exceeds 15% of actual capital sum assured.
(b) Premium payable exceeds 10% of actual capital sum assured.
(c) Premium payable exceeds 20% of actual capital sum assured. (d) No exemption

SALEEM QURAISHEE Mo: 917566444 INSPIRE ACADEMY-8888881719


EXEMPTED INCOME- Page 3.23

PROBLEM:22 The sum received under Life Insurance Policy including sum allocated by way on bonus on
the policy which shall not be exempt includes:
(a) Any sum received u/s 80DD (3) and 80 DDA (3)
(b) Any sum received under Keyman Insurance Policy.
(c) Any sum received under policy issued on or after 1-4-2012 in respect of which premium payable for
any year exceeds 10% of actual sum assured. (d) ANY of the above.

PROBLEM:23 Any income of a business trust by way of interest received or receivable from a special
purpose vehicle is ____.
(a) Fully exempt from tax (b) Partly exempt from tax (c) Fully taxable (d) None of the above.

PROBLEM:24 Notified awards and rewards instituted by the Central/State Government for the specified
purposes is exempt if received for:
(a) Literary work (b) Artistic work (c) Scientific work (d) All of the above

PROBLEM:25 Incomes exempt include:


a) All incomes of local authority except income from supply of commodities or services (other than
water or electricity) outside its jurisdiction
b) Income of Central Electricity Regulatory Commission,
c) Contributions received from commodity exchanges and members, by any notified Investor
Protection Fund set up by commodity Exchanges in India. When exempted income of Fund is shared
with commodity exchanges in any previous year, shared amount shall be deemed as taxable
income. (d) All of the above,

PROBLEM:26 Incomes exempt include:


a) Income of member of Scheduled Tribes residing in specified areas or states, which accrue or arise
from any source in that specified area or state; or by way of interest/dividends on securities
b) Subsidy from commodity board, received by assessee engaged in business of growing and
manufacturing tea, rubber, coffee, cardamom or other notified commodity in India, after receiving
certificate regarding subsidy paid to it.
c) Any income of Prasar Bharati (Broadcasting Corporation of India) established under Prasar Bharati
(Broadcasting Corporation of India) Act, 1990 is exempt from tax.
d) All of the above.

SALEEM QURAISHEE Mo: 917566444 INSPIRE ACADEMY-8888881719


EXEMPTED INCOME- Page 3.24

PROBLEM:27 Specified income of certain bodies or authorities is exempt. Conditions to be fulfilled for
such exemption includes:
a) Such body or authority should have been established or constituted or appointed under a treaty or
an agreement entered into by the Central Government with 2 or more countries or a convention
signed by the Central Government;
b) Such body or authority is established not for the purposes of profit.
c) Any of (a) or (b).
d) Both (a) and (b).

PROBLEM:28 Income received by certain foreign companies in India in Indian currency from sale of
crude oil or any other goods or rendering of services, as may be notified by the Central Government
in this behalf, to any person is exempt. Condition to be fulfilled to claim such exemption:
a) The money has been received under an agreement or arrangement entered into, or approved by,
the Central Government.
b) The foreign company, as well as the arrangement or agreement, are notified by the Central
Government having regard to the national interest.
c) The foreign company is not engaged in any other activity in India, except receipt of income in India
under such arrangement or agreement.
d) All of the above,

PROBLEM:29 Any income of the National Financial Holdings Company Limited, being a company set up
by the Central Government shall be exempt, if relates to any previous year relevant to any
assessment year:
(a) Commencing on or before 01-04-2014. (b) Commencing on or after 01-04-2014.
(c) Commencing on or after 01-04-2012. (d) Any of the above.

PROBLEM:30 Agricultural income is —


a) Fully exempt b) fully taxable
c) Although fully exempt but it is to be aggregated in case of certain assessees for the purpose of
determining the rate of tax on non-agricultural income d) none of the above

PROBLEM:31 Income which accrues or arises to a individual is exempt —


a) In all cases
b) To the extent such income is derived from any source in the State of Sikkim
c) To the extent such income is derived from any source in the State of Sikkim and income by way of
dividend or interest on securities. d) None of these

SALEEM QURAISHEE Mo: 917566444 INSPIRE ACADEMY-8888881719


EXEMPTED INCOME- Page 3.25

PROBLEM:32 Any sum received by an individual as a member of HUF from the income of HUF shall be:
a) Fully taxable b) fully exempt c) included in the total income of the member for rate
purpose
d) None of these

PROBLEM:33 In case of a partner, the share of the profits from the firm shall be:
a) Fully taxable b) fully Exempt
c) included in the total income of the partner and relief of income tax u/s 86 shall be allowed.
d) None of the above

PROBLEM:34 A local authority has earned income from the supply of water or electricity outside its own
jurisdictional area. Such income is:
a) Exempt b) taxable c) casual income d) none of the above

PROBLEM:35 A local authority has earned income from the supply of commodities outside its own
jurisdictional area. Such income is:
a) Exempt b) taxable c) casual income d) none of the above.

PROBLEM:36 An income under the head capital gain to a local authority is:
a) Exempt b) taxable c) casual income d) none of the above

PROBLEM:37 An income under the head capital gain to a trade union is:
a) Exempt b) taxable c)casual income d) none of the above

PROBLEM:38 Income of local authority from house property, capital gain or from other sources is:
a) Exempt b) taxable c) casual income
d) None of the above

PROBLEM:39 Income of trade union from a house property and from other sources is:
a) Exempt b) taxable c) casual income d) none of the above

PROBLEM:40 A subsidy received from the Tea Board by an assessee carrying on business of growing and
manufacturing tea for re-plantation or replacement of tea bushes is:
a) Taxable b) exempt c) casual income d) none of the above

PROBLEM:41 Subsidy received by the assessee from Rubber Board for re-plantation or replacement of
rubber plant is:
a) Exempt b) taxable c) casual income d) none of these

PROBLEM:42 Any sum received under a Life Insurance Policy including bonus shall be exempt:
a) In all kinds of policies
b) In all kinds of policies except when received under a Keyman Insurance Policy
c) In all kinds of policies except when received under Keyman Policy or under a policy covered under
section 80DDA (3).
d) in all kinds of policies except when received under Keyman Insurance Policy or such policy as is
covered under section 80DD(3) or policy issued, if the premium paid for any year exceeds 10%/15% as the
case may be, of actual capital sum assured, except on death.

SALEEM QURAISHEE Mo: 917566444 INSPIRE ACADEMY-8888881719


EXEMPTED INCOME- Page 3.26

PROBLEM:43 Any pension received by an individual or family pension received by any member of his
family where such individual is in the service of Central or State Government and was awarded
Parmvir Chakra, Mahavir Chakra or Vir Chakra or any other notified gallantry award shall be:
a) exempt b) taxable c) casual income d) none of these

PROBLEM:44 Any amount received or receivable from the Central Government or a State Government
or a local authority by an individual or his legal heir by way of compensation on account of any
disaster shall be:
a) taxable b) Fully exempt
c) Exempt except to the extent the amount received or receivable has been allowed as deduction on
account of any loss or damage caused by such disaster.
d) None of these

PROBLEM:45 In case of an individual, any income by way of interest on any money standing to his credit
in a Non-Resident (External) Account in any bank in India shall be —
a) exempt b) fully taxable c) exempt upto Rs 13,000 d) none of these

SALEEM QURAISHEE Mo: 917566444 INSPIRE ACADEMY-8888881719


INCOME UNDER HEAD SALARY- Page 4.1

1. BASIC ELEMENTS-
1) Every payment made by an Employer to his Employee for service rendered, would be chargeable to
tax as Income from Salaries.

2) Salary includes both Monetary and Non-Monetary facilities –


a) Monetary Facilities – Basic Salary, Bonus, Commissions, Allowances, etc.
b) Non-Monetary Facilities – Housing Accommodation, Medical Facility, Perquisites, etc.

3) Employer-Employee Relationship: To be taxable under “Salaries” -


(a) There should be an Employer – Employee relationship or Master- servant
(b) The Employment may be part-time or full-time employment.

4) Employer may be an individual, firm, and association of persons, company, corporation, Central
Government, State Government, public body or a local authority.

5) Employer may be operating in India or abroad.

6) Employee may be a full-time employee or part-time employee.

7) A Member of Parliament or of State Legislature is not treated as an employee of the Government.


Salary and allowances received by him are, therefore, not chargeable to tax under the head "Salaries"
but are chargeable to tax under section 56 under the head "Income from other sources".

8) If an individual receives salary from more than one employer during the same previous year, salary
from each source is taxable under the head "Salaries

9) Salary Remuneration received (or due) during the previous year is chargeable to tax under the head
"Salaries" irrespective of the fact whether it is received from a former, present or prospective
employer.

SALEEM QURAISHEE Mo: 9175664444 INSPIRE ACADEMY-8888881719


INCOME UNDER HEAD SALARY- Page 4.2

2. CONTRACT OF SERVICE VS CONTRACT FOR SERVICE

Particulars Contract of Service Contract for Service


1) Meaning Employer-Employee relationship is vital. In this contract, a person offers his
The Employee does the work for his services to any person who is willing to
master. pay the charges therefore.
2) Control Control and Supervision vests in the The day-to-day control is normally
Master (employer). The Servant absent in the case of contract for
(employee) is bound to follow the service.
master’s directions.
3) Execution of Employee works under the close The person executing the job is
work supervision of his Employer who answerable only for the work to be
determines the manner of execution of carried out in accordance with the terms
work. (Control over What should be of contract. He has discretion to do the
done and How) work in his own way. (Control over
What should be done and not How to
do it)
4) Remuneration An employee works for remuneration, The person rendering the service is
which may be paid monthly, or in lump entitled to the fruits of his labour, and
sum or on any suitable basis as per also liable for the losses.
agreement.
5) Head Income from Salaries. Profits & Gains of Business / Profession.

3. SALARY & OTHER HEADS

SALEEM QURAISHEE Mo: 9175664444 INSPIRE ACADEMY-8888881719


INCOME UNDER HEAD SALARY- Page 4.3

4. FOREGOING OF SALARY -
I. Section 15 taxes salary on "due" basis even if it is not received.
II. If, therefore, an employee foregoes his salary, it does not mean that salary so foregone is not
taxable.
III. Once salary has accrued to an employee its subsequent waiver does not make it exempt from tax
liability.
IV. Such voluntary waiver or foregoing by an employee of salary due to him is merely an application of
income and is nonetheless chargeable to tax.

CASE-1 An employee instructs his employer that he is not interested in receiving his salary for April
2019 and the same might be donated to a Trust. In this case, the employee cannot claim that he
cannot be charged in respect of the salary for April 2019. It is only due to his instruction that the
donation was made to a Trust by his employer. It is only an application of income. Hence, the salary
for the month of April 2019 will be taxable in the hands of the employee.

5. SURRENDER OF SALARY -
I. If an employee opts to surrender his salary to the Central Government under section 2 of the
Voluntary Surrender of Salaries (Exemption from Taxation) Act, 1961, the salary so surrendered
would be excluded while computing his taxable income.
II. Benefit of tax exemption in respect of salary so surrendered is available to all employees whether
they are employed in private sector or public sector.

6. SALARY PAID TAX-FREE –


I. If salary is paid tax-free by the employer, the employee has to include in his taxable income not
only salary received but also amount of tax paid by the employer
II. It does not make any difference whether tax is paid under terms of contract by the employer or
voluntarily

7. SALARY UNDER SECTION 17(1)-


Salary is defined to include the following-
I. Wages, any annuity or pension any gratuity
II. Any fees, commission, perquisites or profits in lieu of or in addition to any salary or wages;
III. Any advance of salary;
IV. Any payment received by an employee in respect of any period of leave not availed by him;
V. The portion of the annual accretion in any previous year to the balance at the credit of an employee
participating in a recognized provident fund to the extent it is taxable;
VI. Transferred balance in a recognized provident fund to the extent it is taxable; and
VII. The contribution made by the Central Government or any other employer to the account of an
employee under a notified pension scheme referred to in section 80CCD.

SALEEM QURAISHEE Mo: 9175664444 INSPIRE ACADEMY-8888881719


INCOME UNDER HEAD SALARY- Page 4.4

8. PLACE OF ACCRUAL OF SALARY -


1) The place of accrual of salary is the place of employment.
2) Salary earned in India (Service rendered in India) is deemed to accrue or arise in India even if -
a) it is paid outside India,
b) it is paid or payable after the contract of employment in India comes to an end.
3) If an Employee gets pension paid abroad in respect of services rendered in India, the same will be
deemed to accrue or arise in India.
Exceptions: Any Pension payable outside India by the Government to its Officials and Judges who
permanently reside outside India shall not be deemed to accrue or arise in India. Salary of an
Employee in the UNO or in its Constituent Bodies is exempt under United Nations (Privilege and
Immunity) Act.
4) Leave Salary paid abroad in respect of leave earned in India is deemed to accrue or arise in India.
5) Salary paid by Government for Service rendered outside India:
a) Sec.9(1)(iii) provides that income chargeable under the head “Salaries” payable by the
Government to a citizen of India for service rendered outside India, will be deemed to accrue or
arise in India.
b) If the Non-Resident is not an Indian Citizen, Salary paid by the Government of India shall not be
deemed to accrue or arise in India, and consequently not chargeable to tax.
c) U/s 10(7), any Allowance or Perquisites paid or allowed outside India by the Government to a
Citizen of India for rendering services outside India will be fully exempt.

9. KEYMAN INSURANCE POLICY


1) Keyman Insurance Policy (KIP) is a Life Insurance Policy taken by a person on the life of another
person, who is or was an Employee of the first-mentioned person, or is or was connected in any
manner with the business of the first-mentioned person.
2) The sums received under a Keyman Insurance Policy, including the sum allotted by way of bonus on
such policy, is considered as Income u/s 2(24)(xi).
3) Tax treatment-
a) For Employees: Sum received by an employee under KIP is deemed to be Profits in lieu of Salary
and taxable under the head “Salaries”. [Sec.17 (3)].
b) For Other Persons: Sum received under KIP is deemed to be income chargeable under the head
“Profits and Gains of Business or Profession” [Sec.28 (vi)] or under the head “Income from
Other Sources” [Sec.56 (2)(iv)].

SALEEM QURAISHEE Mo: 9175664444 INSPIRE ACADEMY-8888881719


INCOME UNDER HEAD SALARY- Page 4.5

10. BASIS OF CHARGE OF SALARY INCOME-SECTION 15


Basis of charge as per section 15 - As per section 15, salary consists
I. any salary due from an employer (or a former employer) to an assessee in the previous year,
whether actually paid or not;
II. any salary paid or allowed to him in the previous year by or on behalf of an employer (or a
former employer), though not due or before it became due; and
III. Any arrears of salary paid or allowed to him in the previous year by or on behalf of an
employer (or a former employer), if not charged to income-tax for any earlier previous year.
NOTE

CASE-2 If A draws his salary in advance for the month of April 2020 in the month of March 2020
itself, the same becomes chargeable on receipt basis and is to be assessed as income of the P.Y.2019-
20 i.e., A.Y.2020-21. However, the salary for the A.Y.2020-21 will not include that of April 2020.

CASE-3 If the salary due for March 2020 is received by A later in the month of April 2020, it is still
chargeable as income of the P.Y.2019-20 i.e., A.Y.2020-21 on due basis. Obviously, salary for the
A.Y.2021-22 will not include that of March 2020.

PROBLEM:1 X joins a company on June 1, 2019 on monthly salary of ` 30,000 (he was not in
employment prior to June 1, 2019). As per the terms of employment, salary becomes due on the first
day of the next month and is paid on the seventh day of the next month. Determine the amount of
salary chargeable to tax for the assessment year 2020-21.
ANSWER:1 ` 2, 70,000.

PROBLEM:2 Suppose in problem P-1, the salary due for payment on last day of each month, find out
taxable salary for the AY 2020-21.
ANSWER:2 ` 3,00,000

PROBLEM:3 X joins a company on December 1, 2016 in the pay scale of ` 10,000 – `1,000 –` ` 25,000
(salary at the time of joining is fixed at ` 12,000). As per the terms of employment salary becomes
"due" on the first day of the next month, and it is generally paid on the fifth day of the next month. Find
out the salary taxable for the assessment year 2020-21.
ANSWER:3 `

PROBLEM:4 Assume in above problem salary becomes due on the last day of each month. Find out
salary chargeable to tax for AY 2020-21.
ANSWER:4 `

PROBLEM:5 Up till June 30, 2019, X is in the employment of A Ltd. on the fixed salary of ` 25,000 per
month which becomes "due" on the first day of the next month. On July 1, 2019, X joins B Ltd. (salary
being ` 30,000 per month which becomes "due" on the last day of each month). Salary is actually paid
on the seventh day of the next month in both cases. Find out the amount of salary chargeable to tax for
the assessment year 2020-21.
ANSWER:5 `

SALEEM QURAISHEE Mo: 9175664444 INSPIRE ACADEMY-8888881719


INCOME UNDER HEAD SALARY- Page 4.6

PROBLEM:6 Assume in problem P-5 that salary, becomes due on the last day of the month, in the case
of A Ltd. and on the first of the next day of month in the case of B Ltd., find out the taxable salary for the
assessment year 2020-21.
ANSWER:6 `

PROBLEM:7 Mr. A joins a job on 1-7-2019 at monthly salary or Rs. 25,000. His salary becomes due on last day
of each month. His taxable salary for AY 2020-21 will be-
ANSWER:7
PROBLEM:8 Mr. A joins a job on 1-7-2019 at monthly salary of Rs. 20,000. His salary becomes due on first day
of next month. His taxable salary for AY 2020-21 will be-
ANSWER:8
PROBLEM:9 Mr. A joins a job on 1-7-2018 at monthly salary of Rs. 20,000 in A Ltd. He got an increment of Rs.
1,000 in the month of July 2019. His salary becomes due on last day of each month. His taxable salary for AY
2020-21 will be-
ANSWER:9
PROBLEM:10 Mr. A joins a job on 1-7-2018 at monthly salary of Rs. 20,000 in A Ltd. He got an increment of Rs.
1,000 in the month of July 2019 . His salary becomes due on first day of next month. His taxable salary for AY
2020-21 will be-
ANSWER:10
PROBLEM:11 Salary of S (Rs. 40,000 per month) becomes due on the last day of the month but is paid on 7th of
next month. Also, salary of April, 2020 and May, 2020 is received in advance in March,2020. What will be his
gross income for A.Y. 2020-21 ?

PROBLEM:12 Mr. A joins a job in the grade of Rs. 20,000-500-25000-1,000-40,000-1,500-60,000 on 1-7-2008


which becomes due on last day of each month. His taxable salary for AY 2020-21 will be –
ANSWER:11

PROBLEM:13 Mr. Anil was employed on 1/4/2009 in the grade of Rs. 15,000-Rs. 500-Rs. 17,000-Rs.
750-Rs. 21,500-Rs. 1,000- Rs. 31,500. His gross salary for the AY 2020-2021 i.e. PY 2019-2020 shall be:
(a) Rs. 2,70,000 (b) Rs. 2,58,000 (c) Rs. 2,16,000 (d) Rs. 1,80,000

PROBLEM:14 Salary of Mr. Karan becomes due on 1st of next month and it is paid on 7th of that month.
For AY 2020-2021 i.e. PY 2019-2020, the salary of Mr. Karan shall be taken from:
(a) April 2019 to March 2020 (b) March 2019 to February 2020
(c) Any of the above at the option of the employer
(d) Any of the above at the option of the employee

SALEEM QURAISHEE Mo: 9175664444 INSPIRE ACADEMY-8888881719


INCOME UNDER HEAD SALARY- Page 4.7

11. ADVANCE SALARY V/S ADVANCE AGAINST SALARY


A. ADVANCE SALARY- Salary for a future period is taken in advance by the employee.
I. Advance salary is taxable on receipt basis in the assessment year relevant to the previous year in which it is
received, irrespective of incidence of tax in the hands of the employee.
II. The recipient can, however, claim relief in terms of section 89.

B. Advance Against Salary:


I. Advance against salary is different from advance salary.
II. It is an advance taken by the employee from his employer.
III. This advance is generally adjusted with his salary over a specified time period.
IV. It cannot be taxed as salary.

12. ARREAR SALARY –


I. It is taxable on receipt basis, if the same has not been subjected to tax earlier on due basis.
II. Recipient can claim relief under section 89.

13. SALARY IN LIEU OF NOTICE PERIOD –


I. It is taxable under section 15 on receipt basis.

14. SALARY TO A PARTNER


I. Salary paid to a partner by a firm is an appropriation of profits.
II. It is, therefore, not chargeable under the head "Salaries" but is taxable under the head "Profits and gains of
business or profession

15. FEES AND COMMISSION


I. Fees and commission are taxable as salary irrespective of the fact that they are paid in addition to or in lieu
of salary.
II. Commission paid to a director (not being an employee) for his giving guarantee for repayment of loan, etc.,
is taxable under the head "Income from the other sources".

16. BONUS
I. It is taxable in the year of receipt if it has not been taxed earlier on the basis.
II. If bonus is received in arrears, the assessee can claim relief in terms of section 89.

17. ANNUITY [SEC. 17(1)(II)]


I. An annuity payable by a present employer is taxable as salary even if it is paid voluntarily without any
contractual obligation of the employer.
II. An annuity received from an ex-employer is taxed as profit in lieu of salary.

SALEEM QURAISHEE Mo: 9175664444 INSPIRE ACADEMY-8888881719


INCOME UNDER HEAD SALARY- Page 4.8

18. REMUNERATION FOR EXTRA DUTIES -


I. Where an employee agrees to do something outside the duties of his office, thereby enlarging scope of his
office, for which he is given extra payment, that payment is taxable as salary.

19. SALARY RECEIVED FROM A UNITED NATIONS ORGANIZATION –


I. Salary received from a United Nations Organization is not taxable in India.

20. HOW TO COMPUTE SALARY INCOME-


NAME-
PY-
AY-
Particulars Details Amount
Basic Salary XXXX
B/F/C XXXX
Gratuity XXXX
Leave Encashment XXXX
Pension XXXX
Retrenchment Compensation XXXX
Compensation received under Voluntary Retirement Scheme
Allowances:
Dearness Allowance (DA) /Dearness Pay (DP)
House Rent Allowance
Children Education Allowance
Hostel Expenditure Allowance
Entertainment Allowance
Medical Allowance
Conveyance Allowance
City Compensatory Allowance
Uniform Allowance
Transport Allowance
Other Allowances XXXXX
Perquisites u/s 17(2)
Any Obligation of Employee paid by Employer
Accommodation
Shares and securities issued under ESOP
Employer's Contribution to Superannuation Fund
Gas, Electricity & Water
Education Facility
Medical Facility
Other fringe benefits ..
Leave Travel Concession
Contribution of Employer to Provident Fund
Interest on Recognized Provident Fund
Any other item
Gross Salary XXXX
Less: Deduction u/s 16
I. Standard deduction xxxx
II. Entertainment Allowance XXXX
III. Tax on employment/Professional tax XXXX
INCOME FROM SALARY XXXX

SALEEM QURAISHEE Mo: 9175664444 INSPIRE ACADEMY-8888881719


INCOME UNDER HEAD SALARY- Page 4.9

PROBLEM:15 Which is the charging section of Income under the head salaries?
ANSWER:15 ……………
PROBLEM:16 Which of the following income is chargeable to income tax under the head salaries?
(a) Salary due (b) Advance Salary (c) Arrears of Salary (d) All of these

PROBLEM:17 Any salary due from an employer or a former employer to an assessee in the previous year whether
paid or not is known as –
(a) Salary due (b) Advance Salary (c) Arrears of Salary (d) All of these

PROBLEM:18 Income is taxable under the head salaries only if there exists______ relationship between the payer
and payee.
(a) Employer-Employee (b) Principal-Agent (c) Agent-Principal (d) All of the above

PROBLEM:19 A teacher receives remuneration for setting question paper for examination. What is the
chargeability position of this remuneration? Choose the most appropriate answer.
(a) It will be chargeable under any head. (b) It will be allowed as deduction
(c) It will not be charged to tax under any head. (d) It will be allowed as deduction

PROBLEM:20 R, a chartered accountant is employed with R Ltd., as an internal auditor and requests the
employer to call the remuneration as internal audit fee. R shall be chargeable to tax for such fee under
the head:
a) Income from Salaries
b) Profit and gains from Business and Profession c) Income from other Sources.

PROBLEM:21 Which section gives the definition of salary?


(a) Section 15 (b) Section 17(1) (c) Section 10 (d) None of these

PROBLEM:22 Which amongst the following is not included under the inclusive definition of salary?
(a) Wages (b) Any annuity or pension
(c) Employer’s contribution in RPF in excess of 12% of salary (d) None of these

PROBLEM:23 Which of the following income is taxable under the head ‘income from salary’
(a) Salary received by a partner from firm (b) Salary received by a Member of Parliament
(c) Salary of a Government Officer (d) None of the above

SALEEM QURAISHEE Mo: 9175664444 INSPIRE ACADEMY-8888881719


INCOME UNDER HEAD SALARY- Page 4.10

PROBLEM:24 An income can be taxed under the head salaries only if there is relationship of employer-
employee between the payer and payee. In which of the following cases, such relationship would be
deemed to exist?
(a) When a payer directs payee what has to be done and payee is doing that work independently
without any control of employer
(b) When a payer directs payee what has to be done, how it has to be done and payee is bound to
follow the instructions of payer
(c) Both (a) and (b) (d) None of the above

PROBLEM:25 In which of the following cases, income of payee would be taxable under the head
“Salaries”?
(a) When payer is making payment to consultant for certain services
(b) When firm is making payment of salary to its partners
(c) When payer is making payment to persons working under direct supervision and control of
payer (d) Both (a) and (b)

PROBLEM:26 What would be the taxability of salary surrendered by employee to Central Government
for the help of earthquake victims?
(a) It would be taxable under the head salaries (b) It would not be included under the head salaries
(c) It would be taxable as income from other sources
(d) Employee would get deduction of amount surrendered u/s 80C

PROBLEM:27 In which of the following cases, salary would be deemed to accrue or arise in India?
(a) When salary is received by private employee outside India but services are rendered in India
(b) When salary is received by private employee in India and the services are rendered outside India
(c) When salary is received outside India by Government employee who is a citizen of India for
services rendered outside India (d) Both (a) and (c)

PROBLEM:28 When salary is received outside India by Indian Government employee, who is a
.................. , then such salary shall be deemed to accrue or arise in India?
(a) Non-resident and not a Citizen of India (b) Citizen of India
(c) Not a citizen of India (d) Both (a) and (c)

PROBLEM:29 Suman, a Government employee has rendered services in India and retired in 2011. He
then shifted and settled in Singapore. The pension received by him for such services would be
..................
(a) Deemed to accrue or arise outside India (b) Deemed to accrue or arise in India
(c) Deemed to accrue or arise in India only when it is received in India
(d) Deemed to accrue or arise in India only when it is received outside India

PROBLEM:30 Sumit receives net salary of Rs. 5,00,000 from his employer on which TDS amounting
Rs. 50,000 is borne by employer. In such case, income taxable under the head “Salary” would be?
(a) Rs. 5,00,000 (b) Rs. 5,50,000 (c) Rs. 4,50,000 (d) None of the above

SALEEM QURAISHEE Mo: 9175664444 INSPIRE ACADEMY-8888881719


INCOME UNDER HEAD SALARY- Page 4.11

PROBLEM:31 Select the correct statement from the following?


(a) Salary paid by foreign enterprise to its employees serving in India would be taxable in India
under the head “Salaries”
(b) Salary paid by foreign enterprise to its employees serving in India would be taxable only in the
country of residence
(c) Salary paid by foreign enterprise to its employees serving outside India would be taxable in India
under the head salaries (d) None of the above

PROBLEM:32 Sam, a partner in firm Chaman & Co. received salary and bonus from the partnership firm.
Such income would be taxable as ..................
(a) Income from Other Sources (b) Salary income
(c) Profit and Gains from Business or Profession (d) None of the above

PROBLEM:33 Mr. Sahil is getting salary of Rs. 20,000 per month. Apart from his normal salary which
becomes due on last day of the month, he has also received advance salary of one month of Rs.
20,000 during PY 2019-20. Compute his taxable salary income for the PY 2019-20?
(a) Rs. 2,40,000 (b) Rs. 2,60,000 (c) Rs. 2,20,000 (d) None of the above

PROBLEM:34 Which of the following shall be included in taxable salary?


(a) Wages (b) Profits in lieu of salary (c) Leave encashment (d) All of the above

PROBLEM:35 Which of the following shall be included in taxable salary?


(a) Employer’s contribution to recognised provident fund in excess of 10% of employee salary
(b) Employer’s contribution to recognised provident fund in excess of 12% of employee salary
(c) Interest credited to recognized provident fund in excess of 5% (d) All of the above

PROBLEM:36 Bonus is taxable on ..................


(a) Receipt basis (b) Due basis
(c) Receipt or due whichever is earlier (d) None of the above

PROBLEM:37 XY India Limited wants to terminate services of Mr. SANJAY. As per the terms of
employment, Mr. SANJAY can be removed by giving three month’s notice. XY India removed Mr.
SANJAY with immediate effect from March 1, 2020 by giving him three months salary of Rs.
3,00,000. Such salary would be taxable ..................
(a) Proportionately during March- May, 2019 (b) In the PY 2018-19
(c) In the PY 2019-20 (d) Either (b) or (c)

PROBLEM:38 SAMEER was working for an Indian company, but was posted abroad at a client place
where he rendered his services for half of the year. The remaining services were rendered in India. If
he draws the total remuneration of Rs. 50,00,000, then “Income taxable under the head Salaries” in
India would be ..................
(a) Rs. 50,00,000 (b) Rs. 25,00,000 (c) Rs. 37,50,000 (d) Nil

SALEEM QURAISHEE Mo: 9175664444 INSPIRE ACADEMY-8888881719


INCOME UNDER HEAD SALARY- Page 4.12

PROBLEM:39 RAJAT was working for a US company, but was posted in India at a client place where he
rendered his services. If he draws the total remuneration of Rs. 50,00,000 then. “Income taxable
under the head Salaries” in India would be ..................
(a) Rs. 50,00,000 (b) Rs. 25,00,000 (c) Rs. 37,50,000 (d) Nil

PROBLEM:40 The amount paid by employer as salary in lieu of notice period is ..................
(a) Fully taxable (b) Not taxable
(c) Depends upon the time period of notice period (d) Either (b) or (c)

PROBLEM:41 Payment made by employer to employee for working beyond office hours is ..................
“Gross Salary”.
(a) Included in (b) Excluded from (c) Only 75% of such payment is included
(d) Both (a) and (c)

PROBLEM:42 When annuity is received from present employer, then it is ..................


(a) Taxed as profit in lieu of salary (b) Taxed as salary
(c) Exempt (d) None of the above

21. DEDUCTIONS FROM SALARY INCOME [SEC. 16]


The income chargeable under the head "Salaries" is computed after making the following deductions:
1) Standard Deduction-Section 16(ia)
2) Entertainment allowance-Section 16(ii)
3) Professional Tax- Section 16(iii)

1) STANDARD DEDUCTION- 16( )


I. Standard Deduction is allowed to meet various expenses that employee has to meet while in the
employment like expenses on travelling to office and back to home, expenses of stationary, medical
expenses etc.
II. A standard deduction of ` 50,000 or the amount of salary, whichever is lower, is to be provided to
the employees.

PROBLEM:43 What is the ceiling for Standard deduction u/s 16(ia) in respects of salaried taxpayer?
(a) Amount of salary or Rs. 50,000, whichever is less (b) Rs. 50,000
(c) Amount of salary or Rs. 15,000 whichever is less (d) Rs. 15,000

SALEEM QURAISHEE Mo: 9175664444 INSPIRE ACADEMY-8888881719


INCOME UNDER HEAD SALARY- Page 4.13

2) ENTERTAINMENT ALLOWANCE SEC. 16 (II)


Entertainment allowance is first included in salary income under the head salary and thereafter a
deduction is given-
A. Allowed to Government employee only-(Minimum of the following is deductible)
I. Amount of EA actually received.
II. ` 5000 pa.
III. 20 % of Basic salary.

B. Non government employee fully chargeable to tax.

PROBLEM:44 Who is eligible to claim deduction of entertainment allowance?


(a) Non-Government employee (b) Government employee
(c) Both (a) and (b) (d) None of the above

PROBLEM:45 Mr. JOY has received basic salary of Rs. 30,000 per month (excluding any allowances or
perquisites). He has received entertainment allowance of Rs. 8,000 per month. Calculate the
amount of exemption u/s 10 on entertainment allowance?
(a) Rs. 8,000 per month (b) Rs. 6,000 per month
(c) Rs. 5,000 per month (d) Rs. 10,000 per month

PROBLEM:46 Select the correct statement from the following in case of entertainment allowance.
(a) Assessee is eligible for exemption and not deduction u/s 16
(b) Assessee is eligible for deduction and not exemption u/s 16
(c) Non-Government employee is also eligible for deduction (d) None of the above

PROBLEM:47 The deduction of entertainment allowance is allowed to ..................


(a) Only Government employees (b) Only Non-Government employees
(c) Both Government & Non- Government employees (d) None of the above

PROBLEM:48 The entertainment allowance received by a Government employee is exempt upto the
lower of actual amount received, ..................
(a) 1 /5th of basic salary and Rs. 10,000 (b) 1 /5th of basic salary or Rs. 5,000
(c) 1/10th of basic salary and Rs. 10,000 (d) 1 / 10th of basic salary or Rs. 5,000

PROBLEM:49 Which of the following allowances are first included in the gross salary and thereafter a
deduction u/s 16(n) is allowed only to a Government employee?
(a) House rent allowance (b) Medical allowance
(c) Conveyance allowance (d) Entertainment allowance

PROBLEM:50 X, Government employee, gets ` 8, 00,000 per annum as basic pay. In addition, he receives
`8,500 as entertainment allowance. His actual expenditure on entertainment for official purposes,
however, exceeds ` 9,000. Can he claim deduction of actual amount spent by him on entertainment?
ANSWER:50 ` 5000

SALEEM QURAISHEE Mo: 9175664444 INSPIRE ACADEMY-8888881719


INCOME UNDER HEAD SALARY- Page 4.14

PROBLEM:51 X, an officer of the Punjab Government, gets ` 40,000 per month as salary and ` 600 per
month as entertainment allowance. Besides, he gets dearness allowance and house rent allowance as per
the Government's rules. During the relevant previous year, he has utilized the entire entertainment
allowance for private purpose. The Assessing Officer is, therefore, of the view that the entertainment
allowance is fully taxable. Is he legally correct? If not, determine the amount of entertainment allowance
deductible from salary.
ANSWER:51 5000

PROBLEM:52 X who joined the ABC (P.)Ltd. in 1954, receives ` 88,200 as basic salary and ` 4000 per
month as entertainment allowance during the previous year 2019-20. Determine the amount of income
chargeable under the head "Salaries"
ANSWER:52 Rs. 11,06,400

PROBLEM:53 Mr. Ankush is an employee of Haryana Government and gets salary of Rs. 18,000 pm. He
receives Entertainment Allowance of Rs. 2,000 pm and DA of 100% of Basic Salary. Calculate his taxable
salary for the AY 2020-2021 i.e. PY 2019-2020.
ANSWER:53

PROBLEM:54 Mr. Lokesh is working with the Central Government and is getting basic salary of Rs.
10,000 per month and an entertainment allowance of Rs. 500 per month. Calculate the deduction
of entertainment allowance u/s 16?
(a) Rs. 6,000 (b) Rs. 24,000 (c) Rs. 5,000 (d) Rs. 10,000

PROBLEM:55 Mr. SANJAY is getting basic salary of Rs. 10,000 per month and an entertainment
allowance of Rs. 500 per month. He has spent entertainment allowance of Rs. 600 per month.
Calculate the deduction of entertainment allowance u/s 16, assuming he is working with the Central
Government.
(a) Rs. 6,000 (b) Rs. 24,000 (c) Rs. 5,000 (d) Rs. Rs. ,200

PROBLEM:56 Mr. KARAN is getting basic salary of Rs. 10,000 per month and an entertainment
allowance of Rs. 500 per month. He has spent entertainment allowance of Rs. 400 per month.
Calculate the deduction of entertainment allowance u/s 16.
(a) Rs. 4,800 (b) Rs. 6,000 (c) Rs. 24,000 (d) Rs. 5,000

3) PROFESSIONAL TAX OR TAX ON EMPLOYMENT [SEC. 16 (III)]


I. Professional tax or tax on employment, levied by a State under article 276 of the Constitution, is
allowed as deduction.
II. Deduction is available only in the year in which professional tax is paid.
III. If the professional tax is paid by the employer on behalf of an employee, it is first included in the
salary of the employee as a "perquisite" (since it is an obligation of the employee discharged by the
employer) and then the same amount is allowed as deduction on account of "professional tax"
from gross salary.
IV. There is no monetary ceiling under the Income-tax Act.

SALEEM QURAISHEE Mo: 9175664444 INSPIRE ACADEMY-8888881719


INCOME UNDER HEAD SALARY- Page 4.15

PROBLEM:57 Mr. MANOJ has salary income of Rs. 80,000 pm and State Government charges
Professional Tax of Rs. 2,500 from him. Calculate taxable salary for the AY 2020-2021 i.e. PY 2019-
2020 in following cases:
I. Professional Tax paid by Mr. MANOJ on 31/3/2020.
II. Professional Tax paid by Mr. MANOJ on 5/4/2020.
III. Half of Professional Tax is paid by Mr. MANOJ on 31/3/2020 and half on 2/4/2020.
IV. Professional Tax of two years ie current year and preceding year are paid by Mr. MANOJ on
31/3/2020 and interest is charged by State Government for the amount of Rs. 400.
ANSWER:57

PROBLEM:58 X is employed by A Ltd. (basic salary being ` 38,750 per month). Besides, he gets ` 3,000
per month as entertainment allowance. He pays professional tax of ` 1,000. Find out the salary
chargeable to tax for the assessment year 2019-20. Does it make any difference if the professional tax is
paid by A Ltd.
ANSWER:58

PROBLEM:59 Mr. VIJAY is an employee of ABC limited receiving monthly salary of Rs. 25,000 and monthly
Entertainment Allowance of Rs. 6,000. He is liable to Professional Tax of Rs. 200 pm but it is paid by the
company on his behalf. Compute the Income under the head Salary for the AY 2020-2021 i.e. PY 2019-
2020
ANSWER:59

22. ALLOWANCES-

SALEEM QURAISHEE Mo: 9175664444 INSPIRE ACADEMY-8888881719


INCOME UNDER HEAD SALARY- Page 4.16

SALEEM QURAISHEE Mo: 9175664444 INSPIRE ACADEMY-8888881719


INCOME UNDER HEAD SALARY- Page 4.17

PROBLEM:60 X, who resides in Madras, gets ` 3, 00,000 per annum as basic salary. He receives ` 50,000
per annum as house rent allowance. Rent paid by him is ` 40,000 per annum. Find out the amount of
taxable house rent allowance for the assessment year 2020-21?
ANSWER:60 `

PROBLEM:61 X (who is posted in Delhi but resides in NOIDA) gets ` 60,000 per month as basic pay. He
gets ` 9,000 per month as house rent allowance, though he pays ` 12,000 per month as rent. During the
previous year 2019-20 , he receives ` 5,000 as advance salary of April 2020. Can he claim the entire
amount of house rent allowance as exempt from tax for the assessment year 2020-21 ?
ANSWER:61 `

PROBLEM:62 X, a resident of Ajmer, receives ` 1, 92,000 per annum as basic salary during the previous
year 2019-20. In addition, he gets ` 19200 per annum as dearness allowance forming part of basic
salary for computation of all retirement benefits, 7 per cent commission on sales made by him (sale
made by X during the relevant previous year is ` 86000) and ` 24000 per annum as house rent
allowance. He, however, pays ` 2, 1500 per annum as house rent. Determine the quantum of house
rent allowance exempt from tax.
ANSWER:62 ` Nil

PROBLEM:63 R furnishes the following information for the PY 2019-20.

Particulars `

Basic salary 15000pm

Dearness allowance (60% which is part of salary for retirement benefit) 6000pm

Entertainment allowance 500pm

HRA 6000pm

Actual rent paid for a house in Delhi 7000pm

Education allowance for 3 children 200 pm per child

Transport allowance for commuting between office and residence 2400 pm

Medical allowance (He spends ` 5000 for his medical treatment) 1000 pm

Lunch allowance 200 pm.

Compute taxable salary for the AY 2020-21.


ANSWER:63 `

PROBLEM:64 _____ is granted to an employee by his employer to meet expenditure actually incurred on
payment of rent.
(a) Dearness Allowance (b) House Rent Allowance (c) City Compensatory Allowance (d) None of the

SALEEM QURAISHEE Mo: 9175664444 INSPIRE ACADEMY-8888881719


INCOME UNDER HEAD SALARY- Page 4.18

PROBLEM:65 Which amongst the following is a fully taxable allowance?


(a) Conveyance allowance (b) Travelling allowance (c) Medical allowance (d) Research allowance

PROBLEM:66 For an employee in receipt of hostel expenditure allowance for his three children, the maximum
annual allowance exempts under section 10(14) is –
(a) Rs. 10,800 (b) Rs. 7,200 (c) Rs. 9,600 (d) Rs. 3,600

PROBLEM:67 For an employee in receipt of education allowance for his three children, the maximum annual
allowance exempt under Section 10(14) is –
(a) Rs. 1,200 (b) Rs. 2,400 (c) Rs. 4,800 (d) Rs. 3,600

PROBLEM:68 Sneha is an employee in a private company. In the previous year she received salary Rs. 60,000.
She is residing with her parents. Her taxable house rent allowance will be

PROBLEM:69 Calculate the exempt HRA from the following details; X is entitled to a basic salary of Rs. 50,000
p.m. and dearness allowance of Rs. 10,000 p.m., 40% of which forms part of retirement benefits. He is also
entitled to HRA of Rs. 20,000 p.m. He actually lives with his parents in Mumbai and pays rent of Rs. 20,000
p.m.

PROBLEM:70 Y received children education allowance of Rs. 500 pm for 1 of his children. Calculate taxable
amount of children education allowance for the assessment year 2020-21 if entire Rs. 500 is spent by Y.

PROBLEM:71 Arun, a resident of Meerut, receives Rs. 38,000 per annum as basic salary. In addition, he gets Rs.
12,000 p.a. as dearness allowance, which does not form part of basic salary, 5% commission on turnover
achieved by him (turnover achieved by him during the relevant previous year is Rs. 6,00,000) and Rs. 7,000 per
annum as house rent allowance. He, however, pays Rs. 8,000 per annum as house rent. The quantum of house
rent allowance exempt from tax is – June-2007

PROBLEM:72 Ramesh is resident of Jaipur, receives Rs. 25,000 p.m. as basic salary. In addition, he gets Rs.
10,000 p.m. as dearness allowance, which form part of basic salary and Rs.18,000 p.m. as house rent
allowance. He, however, pays Rs. 20,000 p.m. as house rent. The quantum of house rent allowance taxable is-

SALEEM QURAISHEE Mo: 9175664444 INSPIRE ACADEMY-8888881719


INCOME UNDER HEAD SALARY- Page 4.19

PROBLEM:73 The maximum exemption in respect of transport allowance granted to an employee to meet his
expenditure for the purpose of commuting between the place of his residence and the place of his duty shall
be-
(a) Rs. 1600 per month (b) Rs. 700 per month (c) Rs. 800 per month (d) Rs. 900 per month

PROBLEM:74 The maximum exemption in respect of transport allowance granted to an blind employee to meet
his expenditure for the purpose of commuting between the place of his residence and the place of his duty
shall be-
(a) Rs. 2400 per month (b) Nil (c) Rs. 1,600 per month (d) Rs. 3200 per month

PROBLEM:75 Rajesh an employee of Transport Company receives Rs. 25,000 p.m. as basic salary. In addition, he
gets Rs. 12,000 p.m. as transport allowance to meet his personal expenditure incurred in course of his official
duty of running the transport from one place to another. He has expended Rs. 60,000 for the said purpose
during the previous year. He is not in receipt of daily allowance. The quantum of transport allowance taxable .
ANSWER:75

PROBLEM:76 Kamlesh an employee of XYZ Ltd. receives Rs. 25,000 p.m. as basic salary. In addition, he gets Rs.
12,000 p.m. as dearness allowance and Rs. 15,000 p.m. as uniform allowance. He has expended Rs. 60,000 to
meet expenditure incurred on purchase and maintenance of uniform during the previous year. His taxable
salary
ANSWER:76

PROBLEM:77 Sahil an employee of XYZ Ltd. receives Rs. 25,000 p.m. as basic salary. In addition, he gets Rs.
15,000 p.m. as dearness allowance and Rs. 240 p.m. as for three children as education allowance. His taxable
salary is-
ANSWER:77

PROBLEM:78 Naveen an employee of XYZ Ltd. receives Rs. 30,000 p.m. as basic salary. In addition, he gets Rs.
10,000 p.m. as dearness allowance, not forming part of salary and Rs. 2,000 p.m. as fixed medical allowance.
His taxable salary is-
ANSWER:78

PROBLEM:79 Mr. RAJAT is entitled to a gross salary of Rs. 30,000 per month and commission of 5% of
net profit of the company amounting Rs. 1,00,000 during the year. Calculate income taxable in the
hands of RAJAT , without any deductions?
(a) Rs. 3,60,000 (b) Rs. 4,60,000 (c) Rs. 2,60,000 (d) Rs. 3,00,000

SALEEM QURAISHEE Mo: 9175664444 INSPIRE ACADEMY-8888881719


INCOME UNDER HEAD SALARY- Page 4.20

PROBLEM:80 Mr. Manish is entitled to a basic salary of Rs. 20,000 per month and dearness allowance
of Rs. 10,000 per month. He is getting HRA of Rs. 30,000 per month. He is paying rent of Rs.
23,000 per month for house in Mumbai. Calculate amount of HRA exempt u/s 10(13A)?
ANSWER:80

PROBLEM:81 Mr. Manish is entitled to a basic salary of Rs. 20,000 per month and dearness allowance
of Rs. 10,000 per month. He is getting HRA of Rs. 30,000 per month. He is paying rent of Rs.
15,000 per month for house in Dehradun. Calculate amount of HRA exempt u/s 10(13A)?
ANSWER:81

PROBLEM:82 Mr. Aman is entitled to a basic salary of Rs. 20,000 per month and dearness allowance of
Rs. 10,000 per month. He is getting HRA of Rs. 30,000 per month. He paid rent of Rs. 15,000 per
month for house in Dehradun upto 30.09.2018 and Rs. 20,000 per month thereafter. Calculate
amount of HRA exempt u/s 10(13A)?
ANSWER:82

PROBLEM:83 Mr. Kalpesh has three children and he has received children education allowance of Rs.
500 per month. Calculate the amount of exemption u/s 10?
(a) Rs. 2,400 (b) Rs. 6,000 (c) Rs. 1,200 (d) Rs. 3,600

PROBLEM:84 Mr. Sanjay has one-child and he has received children education allowance of Rs. 300
per month. Calculate the amount of exemption u/s 10?
(a) Rs. 2,400 (b) Rs. 3,600 (c) Rs. 2,200 (d) Rs. 1,200

PROBLEM:85 Mr. Ramesh has one-child and he has received hostel expenditure allowance of Rs. 400
per month. Calculate the amount of exemption u/s 10?
(a) Rs. 2,400 (b) Rs. 3,600 (c) Rs. 4,800 (d) Rs. Rs. ,200

PROBLEM:86 A met with an accident and lost his eyesight. He has received a transport allowance of Rs.
3,500 per month. The amount exempt would be?
(a) Up to Rs. 800 per month (b) Up to Rs. 1,600 per month
(c) Up to Rs. 2,000 per month (d) Up to Rs. 3,200 per month

PROBLEM:87 In which of the following cases, transport allowance is exempt up to Rs. 3200 per
month?
(a) Blind (b) Deaf and Dumb (c) Orthopedically handicapped (d) All of the above

PROBLEM:88 Mr. Raj is working in a transportation system and in order to meet his expenditure during
the course of such transport he is entitled to an allowance of Rs. 15,000 per month. The amount of
allowance exempt from tax would be?
ANSWER:88

SALEEM QURAISHEE Mo: 9175664444 INSPIRE ACADEMY-8888881719


INCOME UNDER HEAD SALARY- Page 4.21

PROBLEM:89 Which of the following allowances are fully taxable?


(a) Allowances to High Court judges (b) Allowances received by employee of UNO
(c) Dearness allowance
(d) Allowances to citizen of India, who is a Government employee and rendering services outside
India

PROBLEM:90 Which of the following allowances are exempt from tax?


(a) Overtime allowance (b) Servant allowance
(c) Warden allowance (d) Sumptuary allowances given to High Court and Supreme Court judges

PROBLEM:91 An employee receives children education allowance of Rs. 1,000 per month per child for
3 children. Calculate the limit of exemption u/s 10?

PROBLEM:92 House rent allowance, subject to deductions, if any is ..................


(a) Fully exempt (b) Fully taxable (c) Fully or partly taxable (d) Actual rent paid is exempt

PROBLEM:93 The maximum exemption available on HRA in metropolitan cities is ..................


(a) 10% of salary (b) 40% of salary (c) 50% of salary (d) Not exempt, fully taxable

PROBLEM:94 The maximum exemption available on HRA in non-metropolitan cities is


(a) 10% of salary (b) 40% of salary (c) 50% of salary (d) Not exempt, fully taxable

PROBLEM:95 The cities which are considered as metropolitan cities for the purpose of section 10(73/1)
are ..................
(a) Mumbai, Kolkata, Delhi & Chennai
(b) Mumbai, Kolkata, Delhi, Chennai & Bangalore
(c) Mumbai, Kolkata, Delhi & Bangalore

PROBLEM:96 The maximum exemption allowed u/s 10(73/1) in respect of HRA in case of a metropolitan
city is lower of ..................
(i) Actual allowance received
(ii) Rent paid in excess of 10% of salary
(iii) 40% of salary
(iv) 50% of salary
(a) (i), (ii), (iii)
(b) (i), (ii), (iv)
(c) (i), (iii), (iv)
(d) (i), (ii), (iii), (iv)

SALEEM QURAISHEE Mo: 9175664444 INSPIRE ACADEMY-8888881719


INCOME UNDER HEAD SALARY- Page 4.22

PROBLEM:97 The maximum exemption allowed u/s 10(13A) in respect of HRA in case of a non-
metropolitan city is lower of ..................
(i) Actual allowance received
(ii) Rent paid in excess of 10% of salary
(iii) 40% of salary (iv) 50% of salary
(a) (i), (ii), (iii)
(b) (i), (iii (iv)
(c) (i), (iii), (iv)
(d) (i), (ii), (iii), (iv)

PROBLEM:98 A receives an HRA of Rs. 10,000 per month but he stays in a house owned by him. The
maximum exemption available u/s 10(13A) is ..................

PROBLEM:99 ANIL receives HRA of Rs. 10,000 per month and a dearness allowance of Rs. 5,000 per
month. He also received a fixed commission of Rs. 50,000. Salary of ANIL, for the purpose of section
10(13A) would be ..................

PROBLEM:100 MANOJ receives an HRA of Rs. 15,000 per month and a dearness allowance of Rs.
5,000 per month. He also received a fixed commission as a percentage of turnover amounting Rs.
1,50,000. Salary of MANOJ , for the purpose of section 10( 13A) would be ..................

PROBLEM:101 Sumptuary allowance has to be treated as a part of ..................


(a) House rent allowance (b) Perquisites (c) Entertainment allowance (d) Daily allowance

SALEEM QURAISHEE Mo: 9175664444 INSPIRE ACADEMY-8888881719


INCOME UNDER HEAD SALARY- Page 4.23

23. PERQUISITE-SEC 17(2)-


Meaning of Perquisite: - In simple words, perquisites are the benefits or facilities provided by an
employer to his employee in addition to the normal salary.

1) Rent free accommodation - Section 17(2) (i) & Rule 3(1)or Accommodation at concessional
rent - Section 17(2)(ii) & Rule 3(1)

2) Perquisites taxable in case of specified employees only - Section 17(2)(iii) & Rules 3(2) to 3(6)
A. Motor car facility - Section 17(2)(iii) 4 Rule 3(2)
B. Facilities of gardener, watchman, sweeper, servant, etc. - Section 17(2)(iii) & Rule 3(3)
C. Facilities of gas, electricity and water - Section 17(2)(iii) & Rule 3(4)
D. Free education facility - Section 17(2)(iii) & Rule 3(5)
E. Free transport facility - Section 17(2)(iii) & Rule 3(6)
POINT TO BE NOTED-
‘Specified Employee'; An employee shall be treated as a specified employee, if he falls under any of the
following circumstances:
I. The employee is a director of the company (whether full-time or part-time): or
II. The employee has a substantial interest in the company (ie the employee should be the beneficial
holder of at least 20% equity shares of the company): or
III. The monetary income of the employee u/h salary for the relevant previous year should be more
than `50,000 (monetary income u/h salary means income u/h salary computed in accordance with
the provisions of the Income Tax Act, 1961, however the value of non-monetary perquisites shall
not be taken into consideration).

3) Any obligation of the employee discharged/met by the employer - Section 17(2)(iv)

4) Payment of life insurance premium by the employer on behalf of the employee - Section
17(2)(v)
5) Specified securities or sweat equity shares allotted/transferred by the employer to his
employees free of cost or at concessional rates - Section 17(2)(vi)

6) Employer's contribution to approved superannuation fund - Section 17(2)(vii)

7) Any other fringe benefit - Section 17(2)(viii) 4 Rule 3(7)

A. Interest free/concessional loans - Section 17(2)(viii) 4 Rule 3(7)(i)


B. Facility of travelling, touring, accommodation, etc - Section 17(2)(viii) 4 Rule 3(7)(ii)
C. Free food/refreshment - Section 17(2)(viii) & Rule 3(7)(iii)
D. Gifts to employees - Section 17(2)(viii) & Rule 3(7)(iv)
E. Expenses on credit cards (ie credit card facility) - Section 17(2)(viii) & Rule 3(7)(v)
F. Club facilities - Section 17(2)(viii) 4 Rule 3(7)(vi)
G. Use of employer s moveable assets by employee - Section 17(2)(viii) & Rule 3(7)(vii)
H. Sale of moveable assets by employer to employee - Section 17(2)(viii) & Rule 3(7)(viii)
I. Any other fringe benefit - Section 17(2)(viii) & Rule 3(7)(ix)

8) Medical facility - Proviso to Section 17(2)


9) Leave Travel Concession (LTC)/Leave Travel Assistance (LTA) - Section 10(5) & Rule 2B

SALEEM QURAISHEE Mo: 9175664444 INSPIRE ACADEMY-8888881719


INCOME UNDER HEAD SALARY- Page 4.24

PROBLEM:102 Any benefit or an amenity provided to the employee by the employer directly or indirectly
whether in cash on in kind in addition to salary & wages is known as____.
(a) Perquisite (b) Allowance (c) Deduction (d) Exemption

PROBLEM:103 Who amongst the following is considered to be a specified employee?


(a) A director employee of the company (b) An employee having the substantial interest in the company
(c) An employee whose income chargeable under the head salary excluding value of all non-monetary benefits
exceeds Rs. 50,000 (d) All of the above

PROBLEM:104 Rent free accommodation provided to employee by the employer is


(a) Exempt from tax (b) Taxable in case of specified employees only
(c) Taxable in case of non-specified employee
(d) Taxable whether the employee is a specified employee or non-specified employee

PROBLEM:105 Any sum paid by the employer in respect of any obligation, for which such payment would have
been payable by the employee is ………………………………………………………………..

PROBLEM:106 Perquisites are the benefits or amenities in .................. provided by employer to


employee.
(a) Kind (b) Cash
(c) Any mode which is not convertible into money (d) All of the above

PROBLEM:107 Which of the following shall be included in the definition of perquisite?


(a) Value of rent-free accommodation provided by employer
(b) Value of concessional rent in respect of an accommodation
(c) Value of any benefit granted free of cost or at concessional rate
(d) All of the above

PROBLEM:108 Which of the following would be deemed as perquisite for all employees?
(a) Watchman is engaged by employee and his wages are reimbursed by employer
(b) Free education facility provided by employer in his educational institute
(c) Free air tickets provided by airline company to its employees (d) None of the above

PROBLEM:109 Which of the following would be deemed as perquisite for specified employees?
(a) Sweeper is engaged by an employee and his wages are reimbursed by employer
(b) Free education facility provided by employer to his employees in his educational institute
(c) Free or concessional air tickets provided by airline company to its employees
(d) Both (b) and (c)

SALEEM QURAISHEE Mo: 9175664444 INSPIRE ACADEMY-8888881719


INCOME UNDER HEAD SALARY- Page 4.25

PROBLEM:110 Which of the following perquisites are not taxable in the hands of all category of
employees?
(a) Concession in rent for the accommodation provided by the employer to the employee
(b) Rent free accommodation provided by the employer to the employee
(c) Sum paid by the employer in discharging the monetary obligation of the employee
(d) Services of sweeper, gardener or watchman

PROBLEM:111 Which of the following perquisites are taxable in the hands of specified category of
employees?
(a) Free or concessional educational facilities
(b) Free or concessional use of gas and water for household consumption
(c) Personal journey provided free of cost to an employee (d) All of the above

PROBLEM:112 Which of the following fringe benefits are taxable in the hands of all employees?
(a) Interest free loans (b) Value of free-food and non-alcoholic beverages
(c) Value of any gift, voucher or token (d) All of the above

1) RENT FREE ACCOMODATION OR ACCOMODATION AT CONCESSIONAL RATE


I. Accommodation provided to employee may be (a) unfurnished (b) furnished.
II. Such accommodation may be provided (a) rent free (b) at concessional rate
III. Accommodation includes house, flat, farm house, hotel, motel, guest house, caravan, mobile home
etc.
IV. Furniture includes radio sets, television, refrigerator, air conditioner and other house hold
appliances

(A) Accommodation provided by Government to its employees-

Point to be noted-
RFA not taxable in case of the following-
I. Accommodation provided to judge of a High court/Supreme court.
II. Accommodation provided to an official of parliament.
III. Accommodation provided to Union minister, Leader of opposition.
IV. Accommodation provided to serving chairman/Member of UPSC

SALEEM QURAISHEE Mo: 9175664444 INSPIRE ACADEMY-8888881719


INCOME UNDER HEAD SALARY- Page 4.26

(B) Accommodation provided by any other Employer

(C) A FURNISHED ACCOMMODATION IN A HOTEL –


The value of the perquisite is determined on the basis of lower of the following two
I. 24 per cent of salary paid or payable for the period during which such accommodation is provided
in the previous year
II. Actual charges paid or payable by the employer to such hotel

Exception: If an accommodation is provided in a hotel and the following two conditions are satisfied,
nothing is chargeable to tax.
I. The hotel accommodation is provided for a total period not exceeding to aggregate 15 days in a
previous year
II. Such accommodation is provided on an employee's transfer from one place to another place
Note: If in the aforesaid case, the hotel accommodation is provided for more than 15 days, then the
perquisite is not taxable for the first 15 days. After that it is chargeable to tax.

Points to be noted-
I. Meaning of salary-Basic salary +Dearness allowance (if terms of employment so provide) +All
taxable allowances from more than one employer
II. Salary shall be calculated on accrual basis.
III. Advance salary and arrears of salary shall not be considered for RFA.

RFA not taxable in case of following-


1) Above rules are not applicable to any accommodation located outside in a remote area i.e.
• an area located at least 40 Km away from town having population less than 20000
• Temporary accommodation (of area 800 sq.ft or less) located 8 km away from local limit of
municipality or cantonment board.
Provided to an employee working at mining/onshore oil exploration/project execution/dam/power
generation/offshore site

2) Rent Free Accommodation Provided at Two Places: Where an employee has been transferred from
one place to another and he is provided with accommodation at the new place of posting while
retaining the accommodation at the original place, perquisite value shall be determined as under:
 For the initial 90 days: Perquisite value of only one of the accommodations shall be taxable.
Obviously, the employee would prefer to choose the accommodation with lower perquisite value.
 After 90 days: Perquisite value of both the accommodations shall be taxable.

SALEEM QURAISHEE Mo: 9175664444 INSPIRE ACADEMY-8888881719


INCOME UNDER HEAD SALARY- Page 4.27

PROBLEM:113 X an officer of the Government of MP, draws a salary of ` 45000 per month as basic salary.
The government has provided him a rent free unfurnished flat whose market rent is ` 9800 per month.
Though as per government rule license fee of the flat is ` 1250 per month. Determine the value of the
perquisite in respect of rent free flat for the AY 2020-21?
ANSWER:113 `

PROBLEM:114 X, an employee of ABC (P.)Ltd., posted at Ajmer (population: 18 lakhs), drawing ` 3,00,000
as basic salary, ` 10,000 as dearness allowance (forming part of salary for all retirement benefits) and `
5,000 as commission. Besides, the company provides a rent-free unfurnished accommodation in Ajmer.
The house is owned by the company. Fair rent of the accommodation is ` 50,000 per annum. Determine
the taxable value of the perquisite for the assessment year 2020-21.
ANSWER:114 `

PROBLEM:115 Manav receives Rs. 50,000 p.m. as basic salary from the government during the financial
year 2019-20 and has been provided rent free accommodation in Jaipur (population exceeds 25
lakhs). The license fee determined by the Government for such accommodation is Rs. 1,000 p.m. The
market rent of such accommodation is Rs.5, 000 p.m. The taxable value of rent free accommodation
will be-
ANSWER:115

PROBLEM:116 Rahul receives Rs. 25,000 p.m. as basic salary from ABC Ltd. He is also provided dearness allowance
of Rs. 10,000 p.m. (not forming part of retirement benefit) and has been provided rent free accommodation in
Jaipur (population exceeds 25 lakhs) which is owned by employer. The market rent of such accommodation is Rs.
5,000 p.m. The taxable value of rent free accommodation will be-
ANSWER:116

PROBLEM:117 Rakesh receives Rs. 25,000 p.m. as basic salary from ABC Ltd. He is also provided dearness allowance
of Rs. 10,000 p.m. forming part of retirement benefit and has been provided rent free accommodation, which is
owned by employer, in Tonk (population is 8 lakhs). The market rent of such accommodation is Rs. 1,000 p.m.
The taxable value of rent free accommodation will be-
ANSWER:117

PROBLEM:118 Sunny receives Rs. 15,000 p.m. as basic salary from ABC Ltd. He is provided dearness allowance of
Rs. 5,000 p.m. forming part of retirement benefit and has been provided rent free accommodation, which is
owned by employer, in Tonk (population is 8 lakhs). The market rent of such accommodation is Rs. 1,000 p.m.
The taxable value of rent free accommodation will be –
ANSWER:118

SALEEM QURAISHEE Mo: 9175664444 INSPIRE ACADEMY-8888881719


INCOME UNDER HEAD SALARY- Page 4.28

PROBLEM:119 Ravi receives Rs. 25,000 p.m. as basic salary from ABC Ltd. He is provided dearness allowance of Rs.
15,000 p.m. forming part of retirement benefit and has been provided rent free accommodation, which is taken
on rent; in Tonk (population is 8 lakhs). The employer has paid monthly rent of Rs. 4,000. The taxable value of
rent free accommodation will be-
ANSWER:119

PROBLEM:120 Ravi receives Rs. 25,000 p.m. as basic salary from ABC Ltd. he is provided dearness allowance of Rs.
15,000 p.m. forming part of retirement benefit and has been provided, accommodation, which is taken on rent,
in Delhi (population is 25 lakhs). The employer has charged rent of Rs. 4,000 p.m. from Ravi. The taxable value of
concessional accommodation will be-
ANSWER:120

PROBLEM:121 C receives Rs. 25,000 p.m. as basic salary from ABC Ltd. She is also provided dearness allowance of
Rs. 15,000 p.m. forming part of retirement benefit and has been provided, accommodation owned by employer
in Delhi (population exceeds 25 lakhs). The employer has charged rent of Rs. 4,000 p.m. from C. The taxable
value of concessional accommodation will be-
ANSWER:121

PROBLEM:122 Mr. C is a Finance Manager in ABC Ltd. The company has provided him with rent-free unfurnished
accommodation in Mumbai. He gives you the following particulars:
Basic salary ` 6,000 p.m.
Dearness Allowance` 2,000 p.m. (30% is for retirement benefits)
Bonus ` 1,500 p.m.
Even though the company allotted the house to him on 1.4.2019, he occupied the same only from 1.11.2019.
Calculate the taxable value of the perquisite for A.Y.2020-21.
ANSWER:122

PROBLEM:123 Neeraj has a salary of Rs. 5,00,000 and is provided with a rent-free unfurnished
accommodation by his employer M/s ABC Pvt. Ltd. at New Delhi, which is owned by the employer.
The value of perquisite in the hands of Neeraj would be ..................

PROBLEM:124 KUNDAN Ltd. owned a premise, which it provided as rent free accommodation to its
employee Mr. Kamal in Manesar Haryana, where total population is 7,50,000. The value of perquisite
would be ..................

PROBLEM:125 AMAR Ltd. owned a premises, and provided it as rent free accommodation to its
employee Mr. Kamal in NASHIK (MH) , where total population is 15,00,000. The value of perquisite
would be ..................

SALEEM QURAISHEE Mo: 9175664444 INSPIRE ACADEMY-8888881719


INCOME UNDER HEAD SALARY- Page 4.29

PROBLEM:126 LOVELY Ltd. has leased a premise and then provided it as rent-free accommodation to its
employee Mr. Amar in SATARA (MH), where total population is 8,50,000. The value of perquisite
would be lower of ..................
(a) Rent paid or 20% of salary (b) Rent paid or 15%of salary
(c) Rent paid or 10% of salary (d) Rent paid or 7.5% of salary

PROBLEM:127 ABC Ltd. leased premises, and then provided it as rent-free accommodation to its
employee Mr. KARAN in a city where total population is 50,00,000. The value of perquisite would be
lower of ..................
(a) Rent paid or 20% of salary (b) Rent paid or 15% of salary
(c) Rent paid or 10% of salary (d) Rent paid or 7.5% of salary

PROBLEM:128 ATUL Ltd. has leased premises and then provided it as rent-free accommodation to its
employee Mr. Amar in AURANGABAD, where total population is 18,50,000. The value of perquisite
would be lower of ..................
(a) Rent paid or 20% of salary (b) Rent paid or 15% of salary
(c) Rent paid or 10% of salary (d) Rent paid or 7.5% of salary

PROBLEM:129 ANIL , a Government employee, was provided regular accommodation by his employer in
a hotel. The value of perquisite will be ..................
(a) 10%of salary paid/payable or the actual charges paid/payable to such hotel, whichever is lower
(b) 12% of salary paid/payable or the actual charges paid/payable to such hotel, whichever is lower
(c) 24% of salary paid/payable or the actual charges paid/payable to such hotel, whichever is lower
(d) Nil, as the accommodation is being provided in a hotel

SALEEM QURAISHEE Mo: 9175664444 INSPIRE ACADEMY-8888881719


INCOME UNDER HEAD SALARY- Page 4.30

2) PERQUISITES TAXABLE IN CASE OF SPECIFIED) EMPLOYEES ONLY -


SECTION 17(2)(III)
 PERQUISITES TAXABLE IN CASE OF SPECIFIED EMPLOYEE ONLY
A. Motor car facility - Section 17(2)(iii) 4 Rule 3(2)
B. Facilities of gardener, watchman, sweeper, servant, etc - Section 17(2)(iii) & Rule 3(3)
C. Facilities of gas, electricity and water - Section 17(2)(iii) & Rule 3(4)
D. Free education facility - Section 17(2)(iii) & Rule 3(5)
E. Free transport facility - Section 17(2)(iii) & Rule 3(6)

 Monetary Perquisite V/S Non-Monetary Perquisite:


I. If the facility has been arranged by the employer himself, such perquisite is known as nonmonetary
perquisite.
II. If the facility has been arranged by the employee himself but payment for the same is made by the
employer on behalf of the employee or the employer reimburses the expenditure incurred by the
employee, such perquisite is known as monetary perquisite.
III. Section 17(2)(iii) applies only if the facilities mentioned above have been provided by way of non-
monetary perquisite.
IV. If the above facilities are provided by way of monetary perquisites, such facilities shall not be
covered u/s 17(2)(iii). Instead, such facilities would get covered u/s 17(2)(iv) and would be taxable
in case of specified as well as non-specified employees.

SALEEM QURAISHEE Mo: 9175664444 INSPIRE ACADEMY-8888881719


INCOME UNDER HEAD SALARY- Page 4.31

A. VALUATION OF PERQUISITE IN RESPECT OF MOTOR CAR-

1) CAR OWNED OR HIRED BY AN EMPLOYER-

A. RUNNING & MAINTENACE MET BY EMPLOYER-

I. OFFICIAL PURPOSE- NIL

II. PERSONAL PURPOSE- Actual expenditure XXXX


Add- R/M expenses XXXX
Add-10% of AC XXXX
XXXX
Less –Amount recovered from employee XXXX
TAXABLE VALUE OF PERQ IN THE HANDS OF EMPLOYEE XXXX

III. PARTLY OFFICAL/PERSONAL-

 `1800 PER MONTH (1600 CC OR LESS) XXXX


 `2400 PER MONTH (above 1600 cc) XXXX
 IF DRIVER IS PROVIDED `900 PM XXXX
TAXABLE VALUE OF PERQ- XXXX
NOTE- Expenditure recovered from employee is not deductible

B. RUNNING & MAINTENING EXPENSES MET BY EMPLOYEE-

I. OFFICIAL PURPOSE- NIL

II. PERSONAL PURPOSE- 10% OF ACTUAL COST


OR HIRE CHARGES XXXX
XXXX
Less –amount recovered from employee XXXX
TAXABLE VALUE OF PERQ XXXX

III. PARTLY OFFICIAL/PERSONAL-

 `600 PER MONTH (1600 CC OR LESS) XXXX


 `900 PER MONTH (above 1600 cc) XXXX
 IF DRIVER IS PROVIDED `900 PM XXXX
TAXABLE VALUE OF PERQ- XXXX
NOTE- Expenditure recovered from employee is not deductible

SALEEM QURAISHEE Mo: 9175664444 INSPIRE ACADEMY-8888881719


INCOME UNDER HEAD SALARY- Page 4.32

2) CAR OWEND AND HIRED BY EMPLOYEE-

A. RUNNING & MAINTENACE MET BY EMPLOYER-


I. OFFICIAL PURPOSE- NIL

II. PERSONAL PURPOSE- Actual expenditure XXXX


Add- R/M expenses XXXX

XXXX
Less –Amount recovered from employee XXXX
TAXABLE VALUE OF PERQ IN THE HANDS OF EMPLOYEE XXXX

III. PARTLY OFFICAL/PERSONAL-


ACTUAL EXPENDITURE INCURRED XXXX
Less-official purpose
 `1800 PER MONTH (1600 CC OR LESS) XXXX
 `2400 PER MONTH (above 1600 cc) XXXX
 IF DRIVER IS PROVIDED 900 PM XXXX
TAXABLE VALUE OF PERQ- XXXX
NOTE- Expenditure recovered from employee is not deductible

B. RUNNING & MAINTENING EXPENSES MET BY EMPLOYEE- (NOT TREATED AS PERQ)

Point to be noted-
1) CONDITIONS TO BE SATISFIED IF CAR IS USED FOR OFFICIAL PURPOSES -
a) The employer has maintained complete details of journey undertaken for official purposes which
may include date of journey, destination, mileage and the amount of expenditure incurred
thereon.
b) The employer gives a certificate to the effect that the expenditure was incurred wholly and
exclusively for the performance of official duties.

2) 'Month'- Meaning of – The word "month" in the table denotes completed month according to the
English calendar and a part of the month is left out of consideration.

3) If the employer has provided more than one car for partly official and partly personal purposes then
it will be a case of pools of cars. In such a case any one car will be taxed as per the rules of partly
official and partly personal use and remaining car(s) will be taxed as per the rules of personal use.

4) If employer has provided any other vehicle other than car then its perquisite value shall be Rs. 900
pm

5) CAR FACILITY NOT CHARGEABLE TO TAX-


I. Car facility between office and residence – The use of motor car by an employee for the
purposes of going from his residence to the place where the duties of employment are to be
performed or from such place back to his residence, is not chargeable to tax.
II. Conveyance facility to judges –
III. Conveyance facility to Chairman/members of UPSC –.

SALEEM QURAISHEE Mo: 9175664444 INSPIRE ACADEMY-8888881719


INCOME UNDER HEAD SALARY- Page 4.33

PROBLEM:130 Employer provides a car (below 1.6 Ltr. Capacity) along with a driver to MANOJ partly for official and
partly for personal purpose. The expenses incurred by the company are Running and maintenance expenses –
Rs. 32,000 & Driver’s salary- Rs. 36,000. The taxable value for assessment year 2020-21 will be-
………………………….

PROBLEM:131 Garima receives Rs. 2,500 p.m. as basic salary from ABC Ltd. She is also provided dearness allowance
of Rs. 1,500 p.m. forming part of retirement benefit. She has been provided motor car (engine capacity exceeds
1.6 lts.) owned by employer for her personal use. Expenditure incurred by the employer on running and
maintenance of the motor car during the relevant previous year amounted Rs. 15,000. The taxable value of car
facility for AY 2020-21 will be-…………………………………

PROBLEM:132 RITA receives Rs. 50,000 p.m. as basic salary from ABC Ltd. She is also provided dearness allowance
of Rs. 25,000 p.m. forming part of retirement benefit. She has been provided motor car (engine capacity exceeds
1.6 lts) owned by employer for her personal use. The original cost of car is Rs. 6,00,000 (WDV Rs. 5,10,000).
Expenditure incurred by the employer on running and maintenance of the motor car during the relevant
previous year amounted Rs. 75,000. The salary of driver paid by the employer Rs. 96,000. The taxable value of
car facility for AY 2020-21 will be- ………………………………………….

PROBLEM:133 S receives Rs. 50,000 p.m. as basic salary from ABC Ltd. She is also provided dearness allowance of
Rs. 25,000 p.m. forming part of retirement benefit. She has been provided motor car (engine capacity exceeds
1.6 lts) owned by employer for her personal use as well as official use. The original cost of car is Rs. 6,00,000
(WDV Rs.5,10,000). Expenditure incurred by the employer on running and maintenance of the motor car during
the relevant previous year amounted Rs. 75,000. The salary of driver paid by the employer Rs. 96,000. The
taxable value of car facility for AY 2020-21 will be- …………………………………

PROBLEM:134 Karishma receives Rs. 25,000 p.m. as basic salary from ABC Ltd. She is also provided dearness
allowance of Rs. 15,000 p.m. forming part of retirement benefit. She has been provided motor car (engine
capacity does not exceed 1.6 lts) owned by employer for her personal use. The original cost of car is Rs. 3,50,000
(WDV Rs.3,15,000). Expenditure incurred by the employer on running and maintenance of the motor car during
the relevant previous year amounted Rs. 25,000. The salary of driver paid by the employer – Rs. 48,000. The
taxable value of car facility for AY 2020-21 will be- …………………………………..

SALEEM QURAISHEE Mo: 9175664444 INSPIRE ACADEMY-8888881719


INCOME UNDER HEAD SALARY- Page 4.34

PROBLEM:135 N receives Rs. 25,000 p.m. as basic salary from ABC Ltd. She is also provided dearness allowance of
Rs. 15,000 p.m. forming part of retirement benefit. She has been provided motor car (engine capacity does not
exceed 1.6 lts) owned by employer for her personal use. on running and maintenance of the motor car during
the relevant previous year amounted Rs. 25,000. The salary of driver paid by the employer Rs. 48,000. The
employer recovers Rs. 2,000 p.m. from the employee. The taxable value of car facility for AY 2020-21 will be-
…………………………….

PROBLEM:136 S receives Rs. 30,000 p.m. as basic salary from ABC Ltd. She is also provided dearness allowance of
Rs. 15,000 p.m. forming part of retirement benefit. She has been provided motor car (engine capacity does not
exceed 1.6 lts) owned by employer for her official use. The original cost of car is Rs. 3,50,000 (WDV Rs. 3,15,000).
Expenditure incurred by the employer on running and maintenance of the motor car during the relevant
previous year amounted Rs. 25,000. The salary of driver paid by the employer Rs. 48,000. The taxable value of
car facility for AY 2020-21 will be- ……………………………………..

PROBLEM:137 K receives Rs. 30,000 p.m. as basic salary from ABC Ltd. She is also provided dearness allowance of
Rs. 15,000 p.m. forming part of retirement benefit. She has been provided motor car (engine capacity does not
exceed 1.6 lts) owned by employer for commuting between office and residence. The original cost of car is Rs.
3,50,000 (WDV Rs. 3,15,000). Expenditure incurred by the employer on running and maintenance of the motor
car during the relevant previous year amounted Rs. 25,000. The salary of driver paid by the employer Rs.
48,000. The taxable value of car facility for AY 2020-21 will be- ……………………………………….

PROBLEM:138 P receives Rs. 30,000 p.m. as basic salary from ABC Ltd. She is also provided dearness allowance of
Rs. 15,000 p.m. forming part of retirement benefit. She has been provided motor car (engine capacity does not
exceed 1.6 lts) owned by employer for her personal use . The car is self driven by her. The original cost of car is
Rs. 3,50,000 (WDV Rs. 3,15,000). Expenditure incurred by P on running and maintenance of the motor car
during the relevant previous year amounted Rs. 25,000. The taxable value of car facility for AY 2020-21 will be-
……………………………………………….

PROBLEM:139 S receives Rs. 30,000 p.m. as basic salary from ABC Ltd. he is also provided dearness allowance of Rs.
15,000 p.m. forming part of retirement benefit. he has been provided motor car (engine capacity does not
exceed 1.6 lts) owned by employer for her personal as well as official use. The car is self driven by him. The
original cost of car is Rs. 3,50,000 (WDV Rs. 3,15,000). Expenditure incurred by S on running and maintenance of
the motor car during the relevant previous year amounted Rs. 25,000. The taxable value of car facility for AY
2020-21 will be-……………………………………….

SALEEM QURAISHEE Mo: 9175664444 INSPIRE ACADEMY-8888881719


INCOME UNDER HEAD SALARY- Page 4.35

PROBLEM:140 P receives Rs. 30,000 p.m. as basic salary from ABC Ltd. he is also provided dearness allowance of Rs.
15,000 p.m. forming part of retirement benefit. he has been provided motor car (engine capacity does not
exceed 1.6 lts) owned by employer for his personal as well as official use. The original cost of car is Rs. 3,50,000
(WDV Rs. 3,15,000). Expenditure incurred by P on running and maintenance of the motor car during the relevant
previous year amounted Rs. 25,000. The salary of driver paid by the employer amounted Rs. 48,000. The taxable
value of car facility for AY 2020-21 will be- ……………………………………………

PROBLEM:141 S receives Rs. 30,000 p.m. as basic salary from ABC Ltd. he is also provided dearness allowance of Rs.
15,000 p.m. forming part of retirement benefit. He owns a motor car (engine capacity does not exceed 1.6 lts)
which is used by him for personal purpose. The original cost of car is Rs. 3, 50,000. Expenditure incurred by
employer on running and maintenance of the motor car during the relevant previous year amounted Rs. 25,000.
The salary of driver paid by the employer amounted Rs. 48,000. The taxable value of car facility for AY 2020-21
shall be…………..

PROBLEM:142 Ankush receives Rs. 30,000 p.m. as basic salary from ABC Ltd. he is also provided dearness allowance
of Rs. 15,000 p.m. forming part of retirement benefit. He owns a motor car (engine capacity does not exceed 1.6
lts) which is used by him for personal purpose as well as official purpose. The original cost of car is Rs.3, 50,000.
Expenditure incurred by employer on running and maintenance of the motor car during the relevant previous
year amounted Rs. 25,000. The salary of driver paid by the employer amounted Rs. 48,000. The taxable value of
car facility for AY 2020-21 will be-

PROBLEM:143 T receives Rs. 30,000 p.m. as basic salary from ABC Ltd. She is also provided dearness allowance of
Rs. 15,000 p.m. forming part of retirement benefit. He owns a motor car (engine capacity does not exceed 1.6
lts) which is used by him for personal purpose as well as official purpose .The original cost of car is Rs. 3,50,000.
Expenditure incurred by employer on running and maintenance of the motor car during the relevant previous
year amounted Rs. 25,000. The salary of driver paid by the employer amounted Rs. 48,000. The taxable value of
car facility for AY 2020-21 will be-

PROBLEM:144 Mr. A is provided with two cars, to be used for official and personal work by his employer ABC Ltd.
Car 1 Rs. Car 2 Rs.

Engine Capacity 1.8 Lts. 1.6 Lts.


Cost of the car 6,00,000 4,00,000
Running & maintenance (Borne by the company) 40,000 28,000
Salary of driver (Borne by the company) 28,000 24,000

The taxable monetary emoluments of Mr. A are Rs. 90,000. The taxable ‘Perk’ in respect of Cars on the
assumption car 2, is exclusively used by ‘A’ for personal purpose will be-
ANSWER:144

SALEEM QURAISHEE Mo: 9175664444 INSPIRE ACADEMY-8888881719


INCOME UNDER HEAD SALARY- Page 4.36

PROBLEM:145 Rajat , who was working with RELIANCE Limited was provided a motor car with cubic
capacity of 2.5 liters. The car was owned by RELIANCE Limited for fulfilling the official as well as
personal needs of Ram. The value of perquisites will be .................. , if the expenses of running and
maintenance are met by RELIANCE Limited.
(a) Rs. 600 p.m. + Rs. 900 p.m. (if chauffeur is provided)
(b) Rs. 900 p.m. + Rs. 900 p.m. (if chauffeur is provided)
(c) Rs. 1,800 p.m. + Rs. 900 p.m. (if chauffeur is provided)
(d) Rs. 2,400 p.m.+ Rs. 900 p.m. (if chauffeur is provided)

PROBLEM:146 Raj, an employee of FINACLE Pvt. Ltd., has been provided a Santro car (without a driver)
with a cubic capacity of 1 liter which he uses for personal as well as private purposes, and expenses
for private use are fully met by Raj himself. What would be the amount of perquisite in the hands of
Raj if the car is owned by FINACLE Pvt. Ltd.?
(a) Rs. 600 p.m. (b) Rs. 900 p.m. (c) Rs. 1,200 p.m. (d) Rs. 1,800 p.m.

PROBLEM:147 Salman, an employee of FAB Pvt. Ltd., has been provided a Santro Car (CC 1 litre) for travel
from office to home and home to office. What would be the amount of perquisite in the hands of
Salman if car is owned by FAB Pvt. Ltd.?
(a) Rs. 600 p.m. (b) Nil (c) Rs. 1,200 p.m. (d) Rs. 1,800 p.m.

PROBLEM:148 Conveyance facility provided to the High Court Judges and Supreme Court Judges is
.................. as Salary.

PROBLEM:149 SAMEER , an employee of Radio FAB Pvt. Ltd., has been provided two cars. Which of the
following shall form perquisites in the hands of SAMEER?
(a) One car is exclusively for official purpose and one partly for office and partly for personal
purpose (b) Both car are partly for office and partly for personal purpose
(c) None of the car is exclusively for personal purpose
(d) One car is exclusively for personal purpose and one partly for office and partly for personal
purpose

PROBLEM:150 Mr. John is working with A ltd. drawing a gross salary of Rs. 45,000 pa. The company
provides Motor Car of 1.7 liters engine capacity. This car is used for both personal and official use.
However, Mr. John pays the petrol bills. Employer also provides Gas, Water and Electricity facilities
for which employer’s expenditure during the year amounts to Rs. 24,000. Calculate the value of
taxable perquisite.
ANSWER:150

SALEEM QURAISHEE Mo: 9175664444 INSPIRE ACADEMY-8888881719


INCOME UNDER HEAD SALARY- Page 4.37

B. DOMESTIC SERVANTS -

 If employer is owner of the RFA and employer has provided facility of Gardner along with RFA to the
employee, then perquisite value of gardener shall not be taxed for the employee

PROBLEM:151 Mr. X and Mr. Y are working for M/s. Gama Ltd. As per salary fixation norms, the following
perquisites were offered:
I. For Mr. X, who engaged a domestic servant for ` 500 per month, his employer reimbursed the
entire salary paid to the domestic servant i.e. ` 500 per month.
II. For Mr. Y, he was provided with a domestic servant @ ` 500 per month as part of remuneration
package.
You are required to comment on the taxability of the above in the hands of Mr. X and Mr. Y, who
are not specified employees.
ANSWER:151

C. GAS, ELECTRIC ENERGY OR WATER SUPPLY PROVIDED FREE OF COST –

PROBLEM:152 The Gardner, Sweeper and the watchman are employed by the employee but their salary of
Rs500 p.m. per person is paid by the employer. The valuation of this perquisite shall be:
a) Rs 4,320 b) Rs 18,000 c) Rs 1,960 d) Rs 6,000

PROBLEM:153 R Ltd. has taken a house on rent which has been provided to its employee along with the
Gardner. The Gardner salary paid shall be:
a) Tax free perquisite b) taxable to the extent of Rs 120 p.m. c)fully taxable
d) Tax free perquisite but will be added to the fair rental value

SALEEM QURAISHEE Mo: 9175664444 INSPIRE ACADEMY-8888881719


INCOME UNDER HEAD SALARY- Page 4.38

PROBLEM:154 JATIN receives Rs. 30,000 p.m. as basic salary from ABC Ltd. he is also provided dearness allowance
of Rs. 15,000 p.m. forming part of retirement benefit. He has been provided free electricity facility for which the
actual cost to employer is Rs. 58,000. The employer has recovered Rs. 8,000 from JATIN on account of such
facility. The taxable value of electricity perquisite will be-………………………………….

PROBLEM:155 B receives Rs. 30,000 p.m. as basic salary from ABC Ltd. he is also provided dearness allowance of Rs.
15,000 p.m. forming part of retirement benefit. He has been provided free electricity facility for which the actual
cost to employer is Rs. 58,000 p.m. The employer has recovered Rs. 8,000 from B on account of such facility. The
taxable value of electricity perquisite will be- ………………….

PROBLEM:156 Find out the value of the perquisite in respect of gas in the cases given below.
1. X is employed by a gas supply company to whom free gas (manufacturing cost :`6,000) is supplied by
the employer.
2. Y is employed by A Ltd. which supplies free gas to Y. Gas bills are issued in the name of A Ltd. (total
bills of the previous year 2019-20:` 10,000).
3. Z is employed by B Ltd. which supplies free gas to Z. Gas bills are issued in the name of Z (total bill of
the previous year 2019-20 :` 15,000).
ANSWER:156

D. VALUATION OF PERQUISITE IN RESPECT OF FREE EDUCATION–


1) TAXABLE ONLY IN CASE OF SPECIFIED EMPLOYEES.
2) Expenditure incurred by the employer for providing free education facility or training to the employee
is NOT TAXABLE.
3) Taxability of education facility provided to other members of the employee household shall be as
follows:
a) Educational institution is owned by the Employer
 Children: Cost of education in a similar institution in or near the locality shall be taken as
perquisite value. However, exemption of Rs 1,000 per month shall be provided per child without
any limit on the number of children.

 Other Members: Cost of education in a similar institution in or near the locality shall be taken as
perquisite value. No exemption is available in case of other family members.

 Any amount recovered from the employee can be deducted in both the cases.

b) EDUCATIONAL INSTITUTION IS NOT OWNED BY THE EMPLOYER

 Children: Actual expenditure incurred by the employer shall be taken as perquisite value.
(Exemption of Rs 1,000)

 Other Members: Actual expenditure incurred by the employer shall be taken as perquisite value.
No exemption is available in case of other family members.
 Any amount recovered from the employee can be deducted in both the cases.

SALEEM QURAISHEE Mo: 9175664444 INSPIRE ACADEMY-8888881719


INCOME UNDER HEAD SALARY- Page 4.39

4) Section 17(2)(iv) (Monetary Perquisite


 Taxable in case of both specified as well as non-specified employees
 Perquisite Value = Amount reimbursed/paid by the employer
 Any amount recovered from the employee can be deducted.

Point to be noted-
1) Rs.1000 per month per child shall be exempted without any restriction on number of children.
2) Child includes adopted child, stepchild of the assessee, but does not include grandchild or illegitimate
child.
3) Any amount charged from the employee for such facility shall be reduced from the above value.
4) Contribution made under an Educational Trust, created for the children of particular group of
employees, is not taxable.
5) Training to employees not chargeable to tax.
6) Scholarship given by Employer Company to children of employee is not assessable as perquisite

PROBLEM:157 Find out the taxable value of the perquisite for the assessment year 2020-21 in the
following cases:
1. X is an employee in the Accounts Department of A Ltd. On November 27, 2019, he attends a seminar
on "Perquisite Valuation". Seminar fees of ` 7,500 is paid by A Ltd.
2. Y's son is a student of ninth class of DPS, Noida. ` 17,800 being tuition fees of Y's son is
paid/reimbursed by B Ltd. where Y is employed. There is no arrangement between B Ltd. and DPS,
Noida.
ANSWER:157

PROBLEM:158 Mahesh receives Rs. 30,000 p.m. as basic salary from Delhi Public School. He is also provided
dearness allowance of Rs. 15,000 p.m. forming part of retirement benefit. His son is studying in the school for
which the employer charges Rs. 200 p.m. The cost of such education in a similar institution in the locality is Rs.
3,500 p.m. The taxable value of education facility will be- ………………………………………

PROBLEM:159 Rajesh receives Rs. 30,000 p.m. as basic salary from Delhi Public School. He is also provided dearness
allowance of Rs. 15,000 p.m. forming part of retirement benefit. His son is studying in the school for which the
employer charges Rs. 200 p.m. The cost of such education in a similar institution in the locality is Rs. 1,200 p.m.
The taxable value of education facility will be-……………………………………………

PROBLEM:160 Ravi’s 5 year old son studies free in DPS Public School, which is being maintained and
owned by his employer. General fee of a similar school is Rs. 3,000 per month. Compute the
monthly value of perquisite taxable in the hands of Ravi?
(a) Nil (b) Rs. 1,000 (c) Rs. 2,000 (d) Rs. 3,000

PROBLEM:161 RAVI ’s 15 year old son studies free in DPS Public School, which is owned by his employer
and where all kinds of all employee study free. General fee of a similar school is Rs. 800 per month.
Compute the monthly value of perquisite taxable in the hands of Ravi?
(a) Nil (b) Rs. 1,000 (c) Rs. 2,000 (d) Rs. 800

SALEEM QURAISHEE Mo: 9175664444 INSPIRE ACADEMY-8888881719


INCOME UNDER HEAD SALARY- Page 4.40

E. FREE TRANSPORT FACILITY


1. Transportation facilities provided by airlines/railways to their employees or members of his household-NIL
2. Other employees employed by an employer engaged in carriage of goods or passengers
 Transport facilities provided free of cost- Value at which such benefit or amenity is offered by
the employer to the public
 Transport facilities provided at concessional rate - Value at which such benefit or amenity is
offered by the employer to the public (-) Amount recovered from employee

3) EMPLOYEE'S OBLIGATION MET BY EMPLOYER [SEC. 17(2)(IV)] –


I. Amount paid by an employer in respect of any obligation which otherwise would have been
payable by the employee is taxable in all cases.

PROBLEM:162 1. X is general manager in A Ltd. He engages a domestic servant on monthly salary of `


2,000. The entire salary (i.e., ` 24,000 is paid by A Ltd. to the domestic servant (or salary is paid by X and
A Ltd. reimburses the entire amount).
2. Y takes a loan of ` 1 lakhs from a bank. The loan is later on paid by his employer on behalf of Y.
ANSWER:162 Rs. 24,000, Taxable

4) AMOUNT PAYABLE BY EMPLOYER TO EFFECT AN ASSURANCE ON THE LIFE


OF EMPLOYEE [SEC. 17(2)(V)] –
I. Life insurance premium paid by the employer to LIC or any other insurance company for a life
insurance policy taken in the name of the employee or any member of his family shall be taxable
in the hands of the employee as a perquisite u/s 17(2)(v).

II. The employee shall be entitled to claim deduction u/s 80C in respect of the life insurance premium
paid by the employer subject to the conditions specified u/s 80C.

III. Other insurance premiums like accident insurance premium, group insurance premium, etc shall
not be regarded as a perquisite for the employee because such schemes are generally for the
benefit of the employer and not the employee.

IV. Proviso to Section 17(2): Payment/reimbursement of Mediclaim insurance premium by an


employer for a policy taken in the name of the employee or his family member is exempt in the
hands of the employee.

5) APPROVED SUPERANNUATION FUND-


I. Employer’s contribution to an approved superannuation fund is exempt in the hands of the
employee to the extent of Rs 1, 50,000.
II. Employer s contribution in excess of Rs 1, 50,000 shall be taxable as a perquisite u/s 17(2)(vii).

SALEEM QURAISHEE Mo: 9175664444 INSPIRE ACADEMY-8888881719


INCOME UNDER HEAD SALARY- Page 4.41

6) SWEAT EQUITY SHARES – SECTION 17(2)(VI)


I. Value of perquisite u/s 17(2)(vi) shall be calculated in the following manner:

FMV of Shares/Securities on the Amount Recovered No. of Shares/Securities


(Minus)
Date of Exercising the Option from the Employees X Allotted to the Employees

II. If such shares/securities are subsequently sold by the employee, the cost of acquisition of such
shares/securities shall be the FMV of the shares/securities as on the date of exercising the option.

III. COMPUTATION OF FMV OF SHARES/SECURITIES


a) Listed shares (listed on a I. FMV shall be the average of the opening price and closing price of
recognized stock the day on which the option is exercised.
exchange) II. If shares are listed on more than one recognized stock exchange on
the date of exercising the option, opening price and closing price of
that stock exchange shall be taken into consideration which records
the highest volume of trading in the share on that day.
b) Unlisted shares I. FMV in such situations shall be the value as determined by a
Securities (whether listed merchant banker.
or unlisted)

PROBLEM:163 AB Co. Ltd. allotted 1000 sweat equity shares to Sri Chand in June 2019.The shares were
allotted at ` 200 per share as against the fair market value of ` 300 per share on the date of exercise of
option by the allottee viz. Sri Chand. The fair market value was computed in accordance with the
method prescribed under the Act.

I. What is the perquisite value of sweat equity shares allotted to Sri Chand?
II. In the case of subsequent sale of those shares by Sri Chand, what would be the cost of acquisition
of those sweat equity shares?

PROBLEM:164 Mohan was working with the company and was provided certain sweat equity shares. On
the date when he exercise the option for taking the shares, the opening value of the shares was Rs.
50 and the closing was Rs. 60. The average value of the share on that day was Rs. 53. If Mohan did
not pay anything for getting the shares, the value of perquisite in his hands would be ..................

PROBLEM:165 Mohan was working with the company and was provided certain sweat equity shares. On
the date when he exercises the option for taking the shares, the shares are not listed on any Stock
Exchange. In such a case the value of perquisite in his hands would be determined by the ..................
(a) Stock broker (b) Merchant banker (c) Chartered Accountant (d) Lending banker

PROBLEM:166 L was working with the company and was provided certain preference shares. On the date
when he exercise the option for taking the shares, the shares are not listed on any Stock Exchange. In
such a case the value of perquisite in his hands would be determined by the . ..................
(a) Stock broker (b) Merchant banker (c) Chartered Accountant (d) Lending banker

PROBLEM:167 In order to ascertain the value of unlisted equity shares given free of cost to the
employees, A Limited hired the services of merchant banker, with the valuation date of September
30, 2019. The merchant banker cannot consider the specified date for purpose of valuation as ..........
SALEEM QURAISHEE Mo: 9175664444 INSPIRE ACADEMY-8888881719
INCOME UNDER HEAD SALARY- Page 4.42

7) FRINGE BENEFITS-
A. Interest free loan or loan at concessional rate of interest -Interest rates charged by SBI on April 1,
2018. WHEN PERQUISITE IS NOT CHARGEABLE TO TAX -
I. If a loan is made available for medical treatment in respect of diseases specified in rule 3A
(the exemption is, however, not applicable to so much of the loan as has been reimbursed to
the employee under any medical insurance scheme.

II. Where the amount of original loan (or loans) does not exceed in the aggregate ` 20,000.

PROBLEM:168 Determine the taxable value of the perquisite in the following cases:
1. X is employed by A Ltd. On June 1, 2019, the company gives an interest-free housing loan of `
14, 00,000 loan is repayable within 5 years.
2. Y is employed by B Ltd. On April 1, 2019, he takes a personal loan of ` 12, 50,000 from B Ltd..
Recovers interest @ 9.5 per cent per annum from Y.
3. C Ltd. gives the following interest-free loan to Z, an employee of the company— ` 15,000 for child's
education and ` 5,000 for purchasing a refrigerator. No other loan is given by C Ltd.
ANSWER:168

B. TRAVELLING / TOURING / HOLIDAY HOME EXPENDITURE ON HOLIDAY –


Case Taxable value of perquisite
Where such facility is maintained by employer and is Value at which such facilities are offered by other
not available uniformly to all employee agencies to the public.
Where the employee is on official tour and the The amount of expenditure so incurred for the
expenses are incurred in respect of any member of accompanying member of his household.
his household accompanying him
Where any official tour is extended as a vacation The value will be limited to the expenses incurred in
relation to such extended period of stay or vacation.
In any other case Amount incurred by the employer.

C. LUNCH/REFRESHMENT, ETC. –
Case Tax Treatment
Tea, snacks or other non-alcoholic beverages in Nil
the form of light refreshment provided during
office hours (including over-time)
Free meals provided during office hours in: Nil
 Remote area; or
 An offshore installation
Free meals provided by the employer during Expenditure on free meals in excess of Rs.50 per meal shall be
office hours: taxable perquisite to the extent of excess amount in hands of all
 At office or business premises; or employees.
 Through paid vouchers which are not
transferable and usable only at eating
joints.
In any other case Actual expenditure to employer - Amount charged from
employee

SALEEM QURAISHEE Mo: 9175664444 INSPIRE ACADEMY-8888881719


INCOME UNDER HEAD SALARY- Page 4.43

PROBLEM:169 The employer provides tea/coffee to X in office (employer's expenditure being ` 6,000 per
annum). Besides the employer provides free lunch during office hours (cost being `120 per meal for 300
working days, amount recovered from X is ` 20 per meal).
ANSWER:169

D. PERQUISITE IN RESPECT OF GIFT, VOUCHER OR TOKEN –


I. Gifts made in cash or convertible into money (like gift cheques) are not exempt).
II. Gifts-in-kind up to ` 5,000 in aggregate per annum would be exempt, beyond which it would be
taxable.

PROBLEM:170 The employer provides a cash gift/gift cheques of ` 3,000 to X. Besides, he gets a wrist
watch of ` 18,000 as Diwali gift from the employer.
ANSWER:170 Rs. 3,000

E. VALUATION OF PERQUISITE IN RESPECT OF CREDIT CARD -


Case Tax Treatment
Where such credit card is used wholly and exclusively for Nil
office purpose and specified conditions# are satisfied.
Where expenses (including membership and annual fees) If directly paid by the employer
are incurred by the employee or any member of his Any amount incurred by the employer as reduced by
household, which is charged to a credit card (including any amount charged from the employee shall be taxable in
add-on card) provided by the employer or otherwise, are the hands of all employees
paid or reimbursed by the employer. If amount reimbursed by the employer
Any amount reimbursed by the employer shall be
taxable in the hands of all employees.

F. VALUATION OF PERQUISITE IN RESPECT OF CLUB EXPENDITURE


Case Tax Treatment
Where such expenses are incurred wholly and Nil
exclusively for office purpose
Where health club, sports and similar facilities are Nil
provided uniformly to all employee’s at employers
premises.
Where the employer has obtained corporate Amount incurred by employer for such facility shall be
membership of the club and the facility is enjoyed by the taxable perquisite in the hands of all employees. However,
employee or any member of his household initial fees paid for obtaining corporate membership shall
not be a taxable perquisite.
Any payment or reimbursement by the employer of any If directly paid by the employer
expenditure incurred (including the amount of annual or Any amount incurred by the employer as reduced by
periodical fee) in a club by employee or any member of amount charged from the employee shall be taxable in the
his household hands of all employees.
If amount reimbursed by the employer
Any amount reimbursed by the employer shall be taxable
in the hands of all employees.

SALEEM QURAISHEE Mo: 9175664444 INSPIRE ACADEMY-8888881719


INCOME UNDER HEAD SALARY- Page 4.44

G. USE OF MOVABLE ASSETS -

H. TRANSFER OF MOVEABLE ASSETS-

Particulars Computer & Car Other Movable


Electronic Gadgets Assets
Method of Depreciation WDV WDV SLM
Rate of Depreciation for every completed 50% 20% 10%
year
Actual Cost xxxxx XXXXX XXXXX
Less: Depreciation for completed years xxxxx xxxxx xxxxx
WDV at the end of completed years xxxxx xxxxx xxxxx
Less: Sale Value taken from Employee xxxxx xxxxx xxxxx
Taxable Value of Perquisite xxxxx xxxxx xxxxx
Point to be noted-
I. Electronic Gadgets include Computer, Digital Diaries and Printers, but exclude Washing Machines,
Microwave Ovens, Mixers, Hot Plates, etc.
II. Transfer of Movable Assets which are 10 years old shall not attract taxability.
III. Completed Year means actual completed year from the date of acquisition of the asset to the date
of transfer of such asset to the Employees.

PROBLEM:171 Find out the taxable value of the perquisite in the following cases for the assessment year
2020-21:
1. X is given a laptop by the employer-company for using it for office and private purpose
(ownership is not transferred). Cost of the laptop to the employer is ` 96,000.

2. On October 15, 2019, the company gives its music system to Y for domestic use. Ownership is not
transferred. Cost of music system (in 2000) to the employer is ` 15000.

3. The employer company sells the following assets to the employees on January 1, 2020 –
Name of employee Z A B
Asset sold Car Computer Fridge
Cost of the asset to employer ` 6,96,000 ` 1,17,000 ` 40,000
Date of purchase (put to use on the same day May 15, 2017 May 15, 2017 May, 2017
Sale price ` 2,10,000 ` 24,270 ` 1,000
Before sale on January 1, 2020, these assets were used for business purpose by the employer.
ANSWER:171 Nil, Rs. 690, ( Car-` 2,35,440, Computer-` 4,980, Fridge-` 31,000)

SALEEM QURAISHEE Mo: 9175664444 INSPIRE ACADEMY-8888881719


INCOME UNDER HEAD SALARY- Page 4.45

I. ANY OTHER BENEFITS AMENITY, ETC. -


I. Telephone/mobile phone: The perquisite in respect of telephone/ mobile phone is not taxable in the
case of any employee of any organization.

8) VALUATION OF MEDICAL FACILITIES –


I. Fixed medical allowance is always chargeable to tax.
II. For the purpose of valuation of the perquisite in respect of medical facilities, "family" means –
 the spouse and children of the individual; and
 The parents, brothers and sisters of the individual or any of them, wholly or mainly
dependent on the individual.

(A) MEDICAL FACILITIES IN INDIA- The provisions are given below:


a. The perquisite in respect of medical facility provided by an employer in the following
hospitals/clinic is not chargeable to tax –
I. hospital owned/maintained by the employer,
II. hospital of Central Government/State Government/local authority,
III. private hospital if it is also recommended by the Government for the treatment of Government
employees,

IV. Specified medical facility (given in rule 3A) in a hospital approved by the Chief Commissioner.

a. Health insurance premium: Medical insurance premium paid or reimbursed by the employer is not
chargeable to tax.
b. Any other facility in India: Any other expenditure incurred or reimbursed by the employer for
providing medical facility in India is not chargeable to tax up to ` 15,000 in aggregate per
assessment year

(B) MEDICAL FACILITIES OUTSIDE INDIA-


I. Any expenditure incurred by the employer (or reimbursement of expenditure incurred by the
employee) on medical treatment of the employee or any member of the family of such employee
outside India subject to the conditions given below:
Perquisite not chargeable to tax Condition to be satisfied

Medical treatment of employee or any member of Expenditure shall be excluded from perquisite only to
family of such employee outside India. the extent permitted by the Reserve Bank of India.

Cost on travel of the employee/any member of his Expenditure shall be excluded from perquisite only in
family and one attendant who accompanies the the case of an employee whose gross total income,
patient in connection with treatment outside India. as computed before including therein the
expenditure on travelling, does not exceed `
2,00,000.

Cost of stay abroad of the employee or any member Expenditure shall be excluded from the perquisite
of the family for medical treatment and cost of stay only to the extent permitted by the Reserve Bank of
of one attendant who accompanies the patient in India.
connection with such treatment.

SALEEM QURAISHEE Mo: 9175664444 INSPIRE ACADEMY-8888881719


INCOME UNDER HEAD SALARY- Page 4.46

PROBLEM:172 Compute the taxable value of following perquisite received by Mr. G from his employer
during PY19-20(AY 20-21):
Particulars Amount (Rs)
Medical premium paid for insuring health of Mr G 7,000
Treatment of Mr G by his family doctor 5,000
Treatment of Mrs G in a Government hospital 25,000
Treatment of Mr G's grandfather in a private clinic 12,000
Treatment of Mr G's mother (68 years and dependent) by family doctor 8,000
Treatment of Mr G's sister (dependent) in a nursing home 3,000
Treatment of Mr G's brother (independent) 6,000
Treatment of Mr G's father (75 years and dependent) abroad 50,000
Expenses of staying abroad of the patient; and 30,000
Limit specified by RBI 75,000
ANSWER:172

PROBLEM:173 Ms Rakhi is an employee in a pvt company. She receives the following medical benefits
during PY 2019-20:
S No Particulars Amount (Rs)
1 Reimbursement of following medical expenses incurred by Ms Rakhi:
a) On treatment of her self-employed daughter in a private clinic 4,000
b) On treatment of herself by family doctor 8,000
c) On treatment of her mother-in-law dependent on her in a nursing 5,000
home
2 Payment of premium on Mediclaim policy taken on her health 7,500
3 Medical allowance 2,000 p.m.
4 Medical expenses reimbursed on her son‘s treatment in a government 5,000
hospital
5 Expenses incurred by company on the treatment of her minor son abroad (Rs 1,50,000
1,00,000 permitted by RBI)
6 Expenses incurred by company on the stay of Ms Rakhi and her son abroad 70,000
(Rs 50,000 permitted by RBI)
7 Expenses incurred by company in relation to foreign travel of Ms Rakhi and 95,000
her son
Discuss about the taxability of above benefits and allowances in the hands of Ms Rakhi.
ANSWER:173

PROBLEM:174 In which of the following cases, the expenditure incurred by the employer for employees
on medical treatment shall be fully tax free?
(a) Expenditure incurred by employee for treatment of his family member in a hospital maintained
by Government
(b) Expenditure incurred by employee for treatment of his family member for prescribed disease as
per rule 3A of Income-tax Rules in hospital approved by Chief Commissioner of Income- tax
(c) Neither (a) nor (b) (d) Both («) and (b)

SALEEM QURAISHEE Mo: 9175664444 INSPIRE ACADEMY-8888881719


INCOME UNDER HEAD SALARY- Page 4.47

PROBLEM:175 RAJAT, whose gross total income was Rs. 5,00,000, had a disease due to which he has to
go to Netherlands for medical treatment. His wife, also accompanied him for such treatment. Since
he was working with the company for a long duration his travel expense and his wife’s travel
expenses were paid by the employer amounting Rs. 3,00,000. The exempt perquisite in such a case
would be ..................
(a) Rs. 3,00,000 (b) Rs. 5,00,000 (c) Rs. 1,50,000 (d) Nil

PROBLEM:176 In case of medical treatment out side India, the travel expense of patient and one
attendant shall be tax-free in case of employees whose gross total income does not exceed
..................
(a) Rs. 2,00,000 (b) Rs. 2,50,000 (c) Rs. 3,00,000 (d) Rs. 4,00,000

PROBLEM:177 Alpha Limited has taken insurance policy in respect of all its 100 employees for Rs.
10,00,000 for their health and the health of their family. In such a case, the taxable perquisite in the
hands of employees would be ..................
(a) Rs. 10,000 (b) Rs. 5,000 (c) Rs. 15000 (d) Nil

PROBLEM:178 Alpha Limited, reimbursed Arun for the cost of insurance policy for his own health and the
health of his family amounting Rs. 10,000. In such a case, the taxable perquisite in the hands of
Arun would be ..................
(a) Rs. 10,000 (b) Rs. 5,000 (c) Rs. 15,000 (d) Nil

PROBLEM:179 RUBY hospitals provided medical treatment to the family of the housekeeping manager,
who is on a contractual assignment with the hospital amounting Rs. 2,00,000. The value of such
benefit provided to manager will be ..................
(a) Taxable (b) Tax free (c) Partly taxable

PROBLEM:180 TATA hospitals provided free medical facility to ROHIT Jain’s family in the hospital
amounting Rs. 2,00,000 for a concessional price of Rs. 1,00,000. The perquisite taxable in hands of
ROHIT would be ..................
(a) Rs. 1,50,000 (b) Rs. 2,00,000 (c) Tax free (d) Rs. 1,00,000

SALEEM QURAISHEE Mo: 9175664444 INSPIRE ACADEMY-8888881719


INCOME UNDER HEAD SALARY- Page 4.48

9) VALUATION OF LEAVE TRAVEL CONCESSION IN INDIA [SEC. 10(5)-


I. Leave travel assistance extended by an employer to an employee for going anywhere in India
along with his family exempt on the basis of provisions given in the table below.
Journey performed Maximum exempted fare
By Air Air economic class fare of shortest route
By Rail Air conditioned 1st class fare of shortest route
When the place of origin and destination is connected Same as above
by rail but journey is performed by any other mode of
transport
When the place of origin and destination is not connected by rail:
Where a recognised public transport system exists First class or deluxe class fare, as the case may be, on such
transport.
Where no recognised public transport system exists Amount equivalent to air-conditioned 1st class rail fare, for
the distance of the journey by the shortest route, as if
journey had been performed by rail.
POINT TO BE NOTED-

I. No exemption can be claimed without performing journey and incurring expenses thereon.
II. Block-period: Exemption is available in respect of 2 journeys performed in a block of 4 calendar years
commencing from 1st January 1986.
III. Carry-forward facility: Where concession is not availed during the preceding block (whether on one
occasion or both), then any one journey performed in the first calendar year of the immediately
succeeding block will be additionally exempted (i.e. not counted in two journey limit)
IV. Family: Family here means -
 Spouse and children of the individual; and
 Parents, brothers and sisters of the individual, who are wholly or mainly dependent on him.
V. Restriction on number of children: Exemption can be claimed for any number of children born on or
before 30/9/98. In addition, exemption is available only for 2 surviving children born on or after
1/10/98. However, children born out of multiple birth, after the first child, will be treated as one
child only.
VI. Exemption is based upon actual expenditure.
VII. Exemption is available in respect of fare .(shortest route)
VIII. No other expenses, like scooter or taxi charges at both ends, porterage expenses during the journey
and lodging/boarding expenses are qualified for exemption.

PROBLEM:181 Mr. D went on a holiday on 25.12.2019 to Delhi with his wife and three children (one son –
age 5 years; twin daughters – age 2 years). They went by flight (economy class) and the total cost of
tickets reimbursed by his employer was ` 60,000 (` 45,000 for adults and ` 15,000 for the three minor
children). Compute the amount of LTC exempt. Will there be any difference if among his three children
the twins were 5 years old and the son 3 years old? Discuss.
ANSWER:181

PROBLEM:182 Mr. B went to Srinagar on a holiday on 15.11.2019 with his wife and three children (one son
- age 6 years; twin daughters - age 3 years). They went by aero plane (economy class) and the total cost
of tickets by his employer was Rs 58,000 (Rs 43,000 for adults and Rs 15,000 for the three minor
children).
Case 1: Compute the amount of exemption available.
Case 2: Will there be any difference if the twins were 6 years old and the son 3 was years old. Discuss
ANSWER:182

SALEEM QURAISHEE Mo: 9175664444 INSPIRE ACADEMY-8888881719


INCOME UNDER HEAD SALARY- Page 4.49

PROBLEM:183 Leave travel concession is available under ..................


(a) Section 10(2) (b) Section 10(3) (c) Section 10(5) (d) Section 10(7)

PROBLEM:184 Ram, working with Alpha LLP wants to know how many times, can he claim exemption u/s
10(5) in respect of Leave Travel Concession?
(a) 1 journey every year (b) 2 journeys every year
(c) 1 journey in a block of 2 year (d) 2 journeys in a block of 4 years

PROBLEM:185 Sunita has three children. She wants to claim exemption for Leave Travel Concession
received from her employer for all the children. Such exemption shall apply to a maximum of
.................. children.
(a) One (b) Two (c) Three (d) Any number of children

24. RETIREMENT BENEFITS

1) LEAVE SALARY-
WHAT IS LEAVE SALARY –
I. As per service rules, an employee get different leaves.
II. An employee has to earn leave in the first instance and only when he has leave to his credit, he can
apply for leave.
III. If a leave (standing to his credit) in not taken within a year, as per the service rules, it may lapse or
it may be enchased or it may be accumulated.
IV. The accumulated leaves standing to the credit of an employee may be availed by the employee
during his service time or, subject to service rules such leaves may be encashed at the time of
retirement or leaving the job.
V. Encashment of leave by surrendering leave standing to one's credit is known as "leave salary".

SALEEM QURAISHEE Mo: 9175664444 INSPIRE ACADEMY-8888881719


INCOME UNDER HEAD SALARY- Page 4.50

Points to be noted:
I. Where leave salary or leave encashment is received by a non-Government employee from two or
more employers (maybe in the same year or different years), the maximum amount of exemption
under section 10(10AA) during the lifetime of the concerned employee cannot exceed ` 3,00,000.
II. Even if there is any voluntary retirement from service by way of resignation, the provisions of
section 10(10AAA) would apply.
III. Relief under section 89 would be admissible in respect of encashment of leave salary by an
employee when in service—
IV. Salary paid to the legal heirs of the deceased employee in respect of privilege leave standing to the
credit of such employee when in service.

PROBLEM:186 X, a non- Government employee, receives ` 375000, as leave salary at the time of
retirement on February 20, 2020. On the basis of the following information, determine the amount
of taxable leave salary: Basic pay: ` 15,000 per month since 2007; duration of service : 26 years;
leave at the credit of X at the time of retirement : 25 months; entitlement of leave salary: 60 days'
salary for every year of service and leave availed while in service : 27 months.
ANSWER:186 ` 3, 75,000

PROBLEM:187 R an employee of XYZ Ltd. retired from service w.e.f. 1-1-2020 after serving for 16 years
and 7 months. At the time of retirement he received a sum of ` 50,000 as leave encashment for
unavailed leave of 300 days. He was entitled to 40 days leave for each year of completed service. He
was getting a salary of ` 5,000 per month at the time of the retirement. He received increment of `
500 w.e.f. 1-7-2019.Compute the amount of leave encashment exempt from tax.
ANSWER:187 `

PROBLEM:188 X was employed by PQR Ltd. up to March 15, 1990. At the time of leaving PQR Ltd., he
was paid ` 350000 as leave salary out of which ` ` 57,000 was exempt from tax under section
10(10AA)(ii). Thereafter he joined ABC(P.) Ltd. and received ` 4,12,200 as leave salary at the time of
his retirement on December 31, 2019. Determine the amount of taxable leave salary -
Salary at the time of retirement (per month) ` 22,900

Average salary received during 10 months ending on December 31, 2019

- From March 1, 2019 to July 31, 2019 (per month) ` 22,600

- From August 1, 2019 to December 31, 2019 (per month) ` 22,900

Duration of service (a) 14¾ years

Leave entitlement for every year of service (b) 45 days

Leave availed while in service (c) 90 days

Leave at the credit of employee at the time of retirement [(14 X 45-90) ÷30] 18 months

Leave salary paid at the time of retirement at the rate of ` 22,900 per month ` 4,12,200
(i.e.,` 22,900 X 18)
ANSWER:188 Taxable ` 184700

SALEEM QURAISHEE Mo: 9175664444 INSPIRE ACADEMY-8888881719


INCOME UNDER HEAD SALARY- Page 4.51

PROBLEM:189 The maximum exemption under section 10(10AA) in case of leave encashment is-
(a) Rs. 3, 50,000 (b) Rs. 3, 00,000 (c) Rs. 10, 00,000 (d) Rs. 5, 00,000

PROBLEM:190 For the purpose of calculation of average salary in case of exemption of leave encashment the
average salary drawn during the period of 10 months immediately preceding the _____ of retirement.
(a) Month (b) Date (c) Week (d) Year

PROBLEM:191 What is the annual leave entitlement specified in the Income Tax Act?
(a) 1.5 Months (b) 40 days (c) 30 days (d) None of these

PROBLEM:192 The amount of exemption for leave encashment in case of Government employee is –
(a) Actual amount of leave encashment received (b) Fully exempted from tax
(c) Rs. 3, 00,000 (d) 10 months average salary proceeding the month of retirement

PROBLEM:193 Salary received in lieu of unveiled leave during service shall be-
(a) Fully taxable (b) Fully exempted (c) Partially taxable (d) None of the above

PROBLEM:194 Mr. Bansal was employed in M/s. ABC & Associates. After completing 40 years and 7 months of
service he retired on 31st October 2019. The particulars are as under – CALCULATE EXEMPTION
(i) Salary at the time of retirement – Rs. 10,000 p.m.
(ii) Average monthly salary for 10 months ending on 31st October 2019 – Rs. 9,500 p.m.
(iii) Leave entitlement – 1 ½ months for each completed year of service.
(iv) Leave encashment received for 25 months on basis of salary at the time of retirement – Rs. 2,50,000.
(a) Rs. 2, 50,000 (b) Rs. 95,000 (c) Rs. 47,500 (d) Rs. 3, 00,000

PROBLEM:195 Mr. Kamal was employed in M/s. ABC & Associates. He received Rs. 1, 00,000 as earned leave
salary. He retired on 16th February 2020 after rendering 24 years and 7 months of service. His basic salary was
Rs. 15,000 p.m. His dearness allowance was Rs. 10,000 p.m. (forms part of retirement benefit). Leave
entitlement – 1 ½ months for each completed year of service. The amount of taxable earned leave salary.
(a) Nil (b) Rs. 1, 00,000 (c) Rs. 25,000 (d) Rs. 50,000

PROBLEM:196 Sunil is a private employee and receives leave salary during the tenure of his service
amounting Rs. 30,000. The leave salary is exempt upto ..................
(a) Fully exempt (b) Nil
(c) Exempt upto the minimum of amount actually received or Rs. 20,000 (d) 50%

SALEEM QURAISHEE Mo: 9175664444 INSPIRE ACADEMY-8888881719


INCOME UNDER HEAD SALARY- Page 4.52

PROBLEM:197 MANOJ is a Government employee and receives leave salary during the tenure of his
service amounting Rs. 30,000. The Leave Salary is exempt upto ..................
(a) Fully exempt (b) Nil
(c) Exempt upto the minimum of amount actually received or Rs. 20,000 (d) 50%

PROBLEM:198 The relief for leave salary received by an employee during his tenure of service can be
claimed under ..................

2) GRATUITY [SEC.10 (10)] –

POINT TO BE NOTED-
I. Gratuity received during the period of service is fully taxable.
II. Where gratuity is received from 2 or more employers in the same year then aggregate amount of
gratuity exempt from tax cannot exceed ` 20,00,000 as the case may be..
III. Where gratuity is received in any earlier year from former employer and again received from
another employer in a later year, the limit of ` 20,00,000 as the case may be, will be reduced by the
amount of gratuity exempt earlier.
IV. The exemption in respect of gratuities would be available even if the gratuity is received by the
widow, children or dependents of a deceased employee.

PROBLEM:199 X, an employee of the Central Government, receives ` 7, 86,000 as gratuity at the time of
his retirement on September 30, 2019. Is gratuity fully exempt from tax?
ANSWER:199 Exempt

PROBLEM:200 X, an employee PQ Co. Ltd., receives ` 78,000 as gratuity. He is covered by the Payment
of Gratuity Act, 1972. He retires on December 12, 2019 after rendering service of 38 years and 8 months.
At the time of retirement his monthly basic salary and dearness allowance was ` 2,400 and ` 800,
respectively. Is the entire amount of gratuity exempt from tax?
ANSWER:200 ` 72,000

SALEEM QURAISHEE Mo: 9175664444 INSPIRE ACADEMY-8888881719


INCOME UNDER HEAD SALARY- Page 4.53

PROBLEM:201 X, an employee of LMN Ltd., receives ` 45,000 as gratuity under the Payment of Gratuity
Act, 1972. He retires on November 10, 2019 after rendering service of 30 years and 4 months. At the
time of retirement monthly salary was ` 2,340 (inclusive of dearness allowance of ` 200 per month).
Calculate amount of gratuity chargeable to tax.
ANSWER:201 ` 4,500

PROBLEM:202 X, who is not covered by the Payment of Gratuity Act, 1972, retires on November 20, 2019
from ABC Ltd. and receives ` 1,86,000 as gratuity after service of 38 years and 10 months. His salary
is `8,000 per month up to July 31, 2019 and ` 9,000 per month from August 1, 2019. Besides, he
gets ` 500 per month as dearness allowance (69 per cent of which is part of salary for computing all
retirement benefits but 100% of dearness allowance is considered provident fund). What amount of
gratuity will be exempt from tax?
ANSWER:202 `

PROBLEM:203 Mr. Ravi retired on 15.6.2019 after completion of 26 years 8 months of service and
received gratuity of ` 6,00,000. At the time of retirement, his salary was:

Basic Salary : ` 5,000 p.m.


Dearness Allowance ` 3,000 p.m. (60% of which is for retirement benefits)
Commission : 1% of turnover (turnover in the last 12 months was ` 12,00,000)
Bonus : ` 12,000 p.a.

Compute his taxable gratuity assuming:


(a) He is private sector employee and covered by the Payment of Gratuity Act 1972.
(b) He is private sector employee and not covered by Payment of Gratuity Act 1972.
(c) He is a Government employee.
ANSWER:203

PROBLEM:204 X, not being covered by the Payment of Gratuity Act,1972, retires on January 6, 2020 from
PQR and receives ` 1,24,000 as gratuity after service of 29 years and 11 months. His average
monthly salary during March 1, 2019 to December 31, 2019 is ` 8,500. Besides, he gets ` 2,000 per
month as dearness allowance but it is considered only for calculating provident fund and not gratuity
or pension. Determine the amount of: (a) taxable gratuity, (b) gratuity exempt from tax for the
assessment year 2020-21.
ANSWER:204 ` 750, Rs. 1,23,250

PROBLEM:205 In which of the following case the gratuity received is not taxable?
(a) Government employees (b) Employees covered by Payment of Gratuity Act, 1872
(c) Any other employee (d) None of these

PROBLEM:206 Under Section 10(10) of the Income tax Act, 1961, the maximum amount of gratuity received
which is not chargeable to tax shall be-
(a) Rs. 3,50,000 (b) Rs. 3,00,000 (c) Rs. 2,50,000 (d) Rs. 10,00,000

SALEEM QURAISHEE Mo: 9175664444 INSPIRE ACADEMY-8888881719


INCOME UNDER HEAD SALARY- Page 4.54

PROBLEM:207 What is the specified limit in case of employees covered by the Payment of Gratuity Act, 1972?
(a) Rs. 3,00,000 (b) Rs. 3,50,000 (c) Rs. 10,00,000 (d) Rs. 5,00,000

PROBLEM:208 Saurabh is an employee of XYZ Ltd. he retired on 17th Nov. 2019 after rendering service of 38
years, 6 month and 1 day. Calculate the number of completed years if Saurabh is covered under the Payment
of Gratuity Act.
(a) 37 years (b) 39 years (c) 38 years (d) 38 years 6 months 7 days

PROBLEM:209 Palak is an employee of ABC Pvt. Ltd. She receives Rs. 50,000 p.m. as basic salary. She is also
provided dearness allowance of Rs. 15,000 p.m. forming part of retirement benefit. She received gratuity of
Rs. 25,000 during the continuity of employment. The taxable portion of gratuity will be-
(a) Rs. 25,000 (b) Rs. 15,000 (c) Rs. 12,500 (d) Nil

PROBLEM:210 Mr. Sunil is a private sector employee. He received Rs. 12, 00,000 as gratuity. He retired on 16th
February 2020 after rendering 25 years and 7 months of service. His basic salary was Rs. 30,000 p.m. His
dearness allowance was Rs. 15,000 p.m. (40% forms part of retirement benefit). He is not covered under
payment of Gratuity Act, 1972. The amount of gratuity exempt will be-
(a) Rs. 5, 40,000 (b) Rs. 10, 00,000 (c) Rs. 4, 50,000 (d) Rs. 12, 00,000

PROBLEM:211 Mr. Raj is a government employee. He received Rs. 25, 00,000 as gratuity. He retired on 16th
February 2020 after rendering 25 years and 7 months of service. His basic salary w.e.f. 01-10-2019 was Rs.
30,000 per month (prior to that Rs. 25,000 p.m.). His dearness allowance was Rs. 15,400 p.m. The amount of
gratuity taxable will be-
(a) Nil (b) Rs. 18, 19,000 (c) Rs. 20, 85,000 (d) Rs. 25, 00,000

PROBLEM:212 Mr. Kalicharan is a private sector employee. He received Rs. 18, 00,000 as gratuity. He retired on
16th February 2020 after rendering 25 years and 7 months of service. His basic salary was Rs. 30,000 p.m. His
dearness allowance was Rs. 15,000 p.m. He is covered under Payment of Gratuity Act, 1972. The amount of
gratuity exempt will be-
(a) Nil (b) Rs. 10, 00,000 (c) Rs. 18, 00,000 (d) Rs. 6, 75,000

PROBLEM:213 Gratuity received by legal heir of the deceased employee shall be taxable under the head
..................
(a) Income from Salaries (b) Income from Business or Profession
(c) Income from Other Sources (d) Not taxable

SALEEM QURAISHEE Mo: 9175664444 INSPIRE ACADEMY-8888881719


INCOME UNDER HEAD SALARY- Page 4.55

PROBLEM:214 Ms. Pooja is working as a Government employee for the last 20 years. At the time of his
retirement, he received certain amount as death cum retirement gratuity. The amount received by
him would be ..................
(a) Fully exempt (b) Partly taxable
(c) Fully taxable (d) 1 / 4th of the amount received is taxable

PROBLEM:215 Suman is working as a Government employee for the last 20 years. He died due to heart
attack and his legal heir received certain amount as death cum retirement gratuity. The amount
received by him would be ..................
(a) Fully exempt (b) Partly taxable
(c) Fully taxable (d) 1 /4th of the amount received is taxable

PROBLEM:216 The amount of gratuity received by the employees who are covered under the Payment of
Gratuity Act, 1972 is minimum of actual amount received, 15 days salary for every completed year of
service or part thereof in excess of 6 months or Rs. ..................
(a) Rs. 3,00,000 (b) Rs. 5,00,000 (c) Rs. 10,00,000 (d) Rs. 15,00,000

PROBLEM:217 The amount of gratuity received by the employees covered under the Payment of Gratuity
Act, 1972 is exempt to least of actual amount received, Rs. 20,00,000 or ..................
(a) Half month’s average salary for previous 10 years of service
(b) 15 days salary for every completed year of service or part thereof in excess of 6 months
(c) 15 days salary for every completed year of service
(d) 15 days salary for previous 10 years of service

PROBLEM:218 AJAY is working as an employee for the last 20 years 3 months with a private organization
which is not covered under the Payment of Gratuity Act, 1972. At the time of his retirement the
number of completed years of service would be
(a) 20 years (b) 21 years (c) 12 years (d) 22 years

PROBLEM:219 Average salary, for the purpose of calculating exempt gratuity, for non-government
employees or employees not covered under the Payment of Gratuity Act, 1972, is calculated on the
basis of average salary for ..................
(a) 6 months immediately preceding the month in which retirement/ death takes place
(b) 10 months immediately preceding the month in which retirement/ death takes place
(c) 12 months immediately preceding the month in which retirement/ death takes place
(d) Total salary received for completed year of service divided by total no. of completed years of
service

PROBLEM:220 SANJAY is working as an employee for the last 10 years with a private organization which
is not covered under the Payment of Gratuity Act, 1972. Prior to this, he had worked with an earlier
organisation, when he had claimed exemption from gratuity amounting Rs. 1,00,000. For the
purpose of calculating the exemption, the general limit of Rs. 10,00,000 would ..................
(a) Remain Rs. 10,00,000 (b) Reduced to Rs. 9,00,000
(c) Neither of the above (d) Either of the above

SALEEM QURAISHEE Mo: 9175664444 INSPIRE ACADEMY-8888881719


INCOME UNDER HEAD SALARY- Page 4.56

3) PENSION -

PROBLEM:221 Mr. Sagar retired on 1.10.2019 receiving ` 5,000 p.m. as pension. On 1.2.2020, he
commuted 60% of his pension and received ` 3,00,000 as commuted pension. You are required to
compute his taxable pension assuming:
(a) He is a government employee.
(b) He is a private sector employee, receiving gratuity of ` 5,00,000 at the time of retirement.
(c) He is a private sector employee and is not in receipt of gratuity at the time of retirement.
ANSWER:221

PROBLEM:222 Determine the amount of pension taxable for the assessment year 2020-21 in the
following cases on the assumption that it becomes due on the last day of each month:
1. X receives ` 18,250 per month as pension from the Central Government during the previous year
2019-20.
2. X receives ` 21,000 per month as pension from the Government of Punjab during the previous year
2019-20.
3. X receives ` 20,000 per month as pension from ABC Ltd., a public limited company in the private
sector, during the previous year 2019-20.
4. X retires from the Central Government service on May 31, 2018. He get pension of ` 15,000 per
month up to November 30, 2019 (i.e., `15,000 X 6). With effect from December 1, 2019 he gets one-
third of his pension commuted for ` 7, 18,000.
5. X retires from ABC Co. on June 30, 2018. He get pension of ` 20,000 per month up to January 31,
2020. With effect from February 1, 2020 he gets 60 per cent of pension commuted for ` 10, 71,000.
Does it make any difference if he also gets gratuity of ` 40,000. At the time of retirement?
ANSWER:222 Rs. 2,19,000, Rs. 2,52,000, Rs. 2,40,000, Rs. 1,30,000, (Rs.1,56,000 , Rs. 4,76,000)

PROBLEM:223 Pension foregone by an employee in exchange of lump sum amount is known as


..................
(a) Commuted Pension (b) Uncommuted Pension (c) Surrendered Pension (d) None of the above

SALEEM QURAISHEE Mo: 9175664444 INSPIRE ACADEMY-8888881719


INCOME UNDER HEAD SALARY- Page 4.57

PROBLEM:224 .................. is fully taxable in the hands of all the employees, whether government or non-
government.
(a) Commuted Pension (b) Uncommuted Pension (c) Both (a) and (b) (d) Neither (a) nor (b)

PROBLEM:225 Commuted pension received by government employees (including Judges of Supreme


Court and High Court) is ..................
(a) Fully Taxable (b) Partly Taxable (c) Fully Exempt (d) Exempt up to a specific limit

PROBLEM:226 A is a non-government employee and receives commuted pension of Rs. 3,00,000 along
with gratuity of Rs. 5,00,000. The commuted pension is exempt upto
(a) Rs. 3,00,000 (b) Rs. 1,00,000 (c) Rs. 1,50,000 (d) Rs. Rs. 5,000

PROBLEM:227 Sunil is a non-government employee and receives commuted pension of Rs. 3,00,000 but
no gratuity is received since he didn’t meet the minimum tenure condition. The commuted pension is
exempt up to ..................
(a) Rs. 3,00,000 (b) Rs. 1,00,000 (c) Rs. 1,50,000 (d) Rs. Rs. 5,000

4) RETRENCHMENT COMPENSATION [SEC. 10(10B)] –


A. Meaning –
I. The retrenchment compensation means the compensation paid under Industrial Disputes Act,
1947 or under any Act, Rule, Order or Notification issued under any law.
II. It also includes compensation paid on transfer of employment under section 25F or closing down
of an undertaking under section 25FF of the Industrial Disputes Act, 1947.
III. It may be noted that compensation on account of termination and due to modification in terms
and conditions of employment would be taxable as “profits in lieu of salary”.
IV. However, the retrenchment compensation would be exempt under section 10(10B), subject to
following limits.

a) Amount calculated in accordance with the provisions of section 25F of the Industrial Disputes Act,
1947 i.e., 15 days average pay x completed years of service and part thereof in excess of 6 months

b) An amount, not less than` 5, 00,000 as may be notified by the Central Government in this behalf,
whichever is lower.

c) Actual Amount of Retrenchment Compensation Received

POINT TO BE NOTED-
I. Computation is based on 15/30 and not 15/26 as held by Supreme Court
II. For computing Average Salary purposes, Wages includes Basic and Dearness Allowance.

SALEEM QURAISHEE Mo: 9175664444 INSPIRE ACADEMY-8888881719


INCOME UNDER HEAD SALARY- Page 4.58

PROBLEM:228 The section which governs retrenchment compensation is ..................


(a) Section 10(10A) (b) Section 10(10B)
(c) Section 10(10C) (d) Section 10(5)

PROBLEM:229 What is the maximum amount of exemption available which is specified by the Central
Government in respect of retrenchment compensation?
(a) Rs. 1,00,000 (b) Rs. 3,00,000 (c) Rs. 5,00,000 (d) Rs. 10,00,000

PROBLEM:230 The amount of exemption available on retrenchment compensation is: Actual amount
received, Rs. 5,00,000 or ..................
(a) 15 days' average salary for every completed years of service
(b) 15 days’ average salary for preceding 10 years
(c) 30 days’ average salary for every completed years of service
(d) 30 days’ average salary for every completed years of service

5) COMPENSATION RECEIVED AT THE TIME OF VRS [SEC. 10(10C)]


A. Meaning: Compensation received under Voluntary Retirement Scheme (called as VRS
Compensation or VRC) falls within the meaning of Profits in Lieu of Salary u/s 17(3), and therefore
is taxable as Salary.

B. Exemption: Exemption u/s 10(10C) is applicable for the Employees of-


I. A Public Sector Company or any other Company.
II. An Authority established under a Central, State or Provincial Act.
III. A Local Authority.
IV. A Co-Operative Society.
V. A University established or incorporated by or under a Central, State or Provincial Act and an
Institution declared to be University u/s 3 of the UGC Act, 1956.
VI. An Indian Institute of Technology u/s 3(g) of the Institutes of Technology Act, 1961.
VII. Any State Government or Central Government.
VIII. Notified Institutes of Management, i.e. IIMs at Ahmedabad, Bangalore, Calcutta, Lucknow, and
Indian Institute of Foreign Trade.
IX. Notified Institutions, having importance throughout India or in any State or States, (e.g.
International Crops Research Institute for Semi-Arid Tropics, Action for Food Production
(AFPRO), Government Tool Room and Training Centre)

C. COMPUTATION OF EXEMPTION: The least of the following shall be available as exemption:


a) Actual voluntary retirement compensation received;
b) Rs 5,00,000;
c) {3 Times (x) retirement benefit salary per month at the time of retirement (x) Number of
completed years of service}
 Any part of year shall be completely ignored:

SALEEM QURAISHEE Mo: 9175664444 INSPIRE ACADEMY-8888881719


INCOME UNDER HEAD SALARY- Page 4.59

PROBLEM:231 The compensation received by the employee on voluntary retirement is exempt under
..................
(a) Section 10(10A) (b) Section 10(10C) (c) Section 10(10D)

PROBLEM:232 The maximum amount of exemption allowed on compensation received on voluntary


retirement is minimum of (a) Actual amount of compensation received or (b) ..................
(a) Rs. 3,00,000 (b) Rs. 5,00,000 (c) Rs. 10,00,000 (d) Rs. 15,00,000

PROBLEM:233 The amount receivable on voluntary retirement should not exceed which of the following?
I. Amount equivalent to 10 months’ salary for each completed year of service; or
II. Amount equivalent to 3 months’ salary for each completed year of service; or
III. Salary at retirement multiplied by the balance months of services left before retirement
(a) (i) & (iii)
(b) (ii) & (iii)
(c) Lower of (i), (ii), (iii)
(d) (i), (ii), & (iii)

6) PROFITS IN LIEU OF SALARY [SEC. 17(3)] –


I. The amount of any compensation due to or received by an assessee from his employer or former
employer at or in connection with the termination of his employment.

II. The amount of any compensation due to or received by an assessee from his employer or former
employer at or in connection with the modification of the terms and conditions of employment.

III. Any payment due to or received by an assessee from his employer or former employer except the
following:
a. payment of gratuity exempted under section 10(10)
b. payment of house rent allowance exempted under section 10(13A)
c. payment of commuted pension exempted under section 10(10A)
d. payment of retrenchment compensation exempted under section 10(10B)
e. payment from an approved Superannuation Fund under section 10(13)
f. payment from statutory provident fund or public provident fund;
g. payment from recognized provident fund to the extent it is exempt under section 10(12)

IV. Any payment from unrecognized provident fund or such other fund to the extent to which it does
not consist of contributions by the assessee or interest or such contributions.

V. Any sum received under a Key man insurance policy including the sum allocated by way of bonus
on such policy.

VI. Any amount received (in lump sum or otherwise) prior to employment or after cessation of
employment.

SALEEM QURAISHEE Mo: 9175664444 INSPIRE ACADEMY-8888881719


INCOME UNDER HEAD SALARY- Page 4.60

7) TAX TREATMENT OF PROVIDENT FUND-


A. CONCEPT OF PROVIDENT FUND-
I. Provident fund scheme is a retirement benefit scheme.
II. Under this scheme, stipulated sum is deducted from the salary of the employee as his
contribution towards the fund. The employer also, generally, contributes simultaneously the
same amount out of his pocket to the fund.
III. The employee's and employer’s contributions are invested in gilt-edged securities interest earned
thereon is also credited to the provident fund account of employees.
IV. Credit balance in a provident fund account of an employee consists of employee's contribution,
interest on employee's contribution, employer's contribution and interest on employer's
contribution.
V. The accumulated sum is paid to the employer at the time of his retirement or resignation.
VI. In the case of death of an employee, accumulated balance is paid to his legal heirs.
VII. Scheme encourages personal savings at micro level and generates funds for investment at macro
level; the Government provides deduction under section 80C.

B. KINDS OF PROVIDENT FUND –


1) STATUTORY PROVIDENT FUND:
I. Statutory provident fund is set up under the provisions of the Provident Funds Act, 1925.
II. This fund is maintained by the Government and the Semi-Government organizations, local
authorities, railways, universities and recognized educational institutions.

2) Recognized provident fund:


I. A provident fund scheme to which the Employee's Provident Fund and Miscellaneous Provisions
Act, 1952 (hereinafter referred to as PF Act, 1952) applies is recognized provident fund.

3) Unrecognized provident fund:


I. If a provident fund is not recognized by the Commissioner of Income-tax, it is known as
unrecognized provident fund.

4) Public provident fund:


I. The Central Government has established the public provident fund for the benefits of general
public to mobilize personal savings.
II. Any member of the public (whether a salaried employee or a self-employed person) can participate
in the fund by opening a provident fund account at the State Bank of India or its subsidiaries or
other nationalized banks.
III. Even a salaried employee can simultaneously become member of employees' provident fund
(whether statutory, recognized or unrecognized) and public provident fund. Any amount (subject
to minimum of ` 500 and maximum of ` 100,000 per annum) may be deposited under this account.
IV. The accumulated sum is repayable after 15 years (it may be extended). This provident fund, at
present, carries compound interest at the rate of 8 per cent per annum.
V. Interest is credited every year by payable only at the time of maturity.
5) APPROVED SUPERANNUATION FUND
It means a superannuation fund which is approved by the Commissioner of Income-tax.

SALEEM QURAISHEE Mo: 9175664444 INSPIRE ACADEMY-8888881719


INCOME UNDER HEAD SALARY- Page 4.61

C. TAX TREATMENT RELATING TO PROVIDENT FUNDS (PF):

Particulars Statutory PF Recognized PF Unrecognized PF PPF


OWN 80C. 80C. NOT ALLOWED 80C.
Contribution
Employer’s Not Taxable. Amount exceeding Not taxable at the time of Not Applicable.
contribution 12% of Salary is contribution.
taxable.
Interest Fully exempt. Exempt upto 9.5% Interest on Employee’s Fully exempt.
credited p.a. Any excess is Contribution is taxable under
taxable. “Other Sources” at the time of
withdrawal and not taxable at
the time of Credit.
Interest on Employer’s
Contribution is not taxable at
the time of credit.
Withdrawal at Exempted Exempt u/s 10(12) I. Employee’s Contribution Exempted u/s
the time of u/s 10(11). subject to conditions. not taxable. 10(11).
retirement / II. Interest thereon is
resignation / taxable under the head
termination, Income from Other
etc. Sources.
III. Employer’s Contribution
and Interest thereon is
taxable as Profits in lieu
of Salary, under
“Salaries”
POINT TO BE NOTED-

1) MEANING OF SALARY= Basic Salary + DA (Considered for Retirement Benefits) + Commission as a


Fixed Percentage of Turnover.

2) WITHDRAWAL FROM RECOGNIZED PROVIDENT FUND: Payment of Accumulated Balance due and
payable to an Employee from a Recognized Provident Fund is exempt, in the following situations -
a) If the Employee has rendered continuous service with his employer for a period of 5 years or
more.

b) If he has not rendered such continuous service of 5 years, then the service has been terminated
I. by reason of such employee’s ill health, or
II. by the contraction or discontinuance of the Employer’s Business, or
III. any other cause beyond the control of the Employee.

c) If, on the cessation of his employment, the employee obtains employment with any other
employer, to the extent, the accumulated balance due and becoming payable to him is
transferred to his individual account in any recognized fund maintained by such other employer.
[Note: The period of service rendered under the previous employer(s) shall also be included in
determining the period of continuous service].

SALEEM QURAISHEE Mo: 9175664444 INSPIRE ACADEMY-8888881719


INCOME UNDER HEAD SALARY- Page 4.62

D. TAX TREATMENT RELATING TO APPROVED SUPERANNUATION FUND:

1) Meaning- A Superannuation Fund which has been and continues to be approved by the CIT / PCIT,
in accordance with the Rules contained in Part B of the VIth Schedule to the Income Tax Act, 1961.

2) Tax Treatment of Contribution and Interest-


a) Employer’s Contribution is exempt from tax in the hands of Employee (upto`1,50,000 per
Employee p. a).
b) Employee’s Contribution qualifies for deduction u/s 80C.
c) Interest on Accumulated Balance is exempt from tax.

3) Payment of Accumulated Balance- Sum received by an Employee under an Approved


Superannuation Fund is exempt from tax u/s 10(13) -
a) on the death of a beneficiary, or
b) to an Employee in lieu of or in commutation of an Annuity on his retirement at or after a specified
age or on his becoming incapacitated prior to such retirement, or
c) by way of refund of contributions on the death of a beneficiary, or
d) by way of refund of contributions to an Employee on his leaving the service in connection with
which the Fund is established otherwise than by retirement at or after a specified age or an his
becoming incapacitated prior to such retirement, to the extent to which such payment does not
exceed the contributions made prior to the commencement of this Act and any interest thereon.

8) AMOUNT TRANSFERRED FROM UNRECOGNIZED PROVIDENT FUND TO


RECOGNIZED PROVIDENT FUND-

PROBLEM:234 Employer's contribution to statutory fund shall be:


a) Fully exempt b) fully taxable c) exempt upto 12% of salary d)exempt up to 10% of salary

PROBLEM:235 Interest credited to statutory provident fund shall be:


a) Fully exempt b) exempt upto 8.5% p.a. c) fully taxable d) exempt upto 9.5%

PROBLEM:236 Employer's contribution to recognized provident fund shall be:


a) Fully exempt b) fully taxable c) exempt upto 12% of salary

PROBLEM:237 Interest credited to recognized provident fund shall be:


a) Fully exempt b)fully taxable c)exempt upto 9.5% p.a d)exempt upto 8.5% p.a.

PROBLEM:238 Employer's contribution to unrecognized provident fund shall be:


a) Fully taxable b) fully exempt c) exempt upto 12% of Salary
d) Neither exempt nor taxable in the year of contribution.
SALEEM QURAISHEE Mo: 9175664444 INSPIRE ACADEMY-8888881719
INCOME UNDER HEAD SALARY- Page 4.63

PROBLEM:239 Interest credited to unrecognized provident fund shall be:


a) Fully taxable b) fully exempt c) exempt upto 9.5% p.a.
d) Neither exempt nor taxable in the year of accrual.

PROBLEM:240 Employee's/assessee's own contribution to statutory provident fund or recognized


provident fund or public provident fund shall be subject to:
a) Deduction under section 80C b) deduction under section 80CCC
c) Deduction under section 16 from gross salary

PROBLEM:241 Employee's contribution to unrecognized provident fund shall be subject to:


a) Deduction u/s 80C b) deduction under section 80CCCc) Nil deduction

PROBLEM:242 Payment from statutory fund and public provident fund shall be:
a) Taxable b) fully exempt
c) Taxable to the extent of employer's contribution and interest thereon

PROBLEM:243 Payment from recognized provident fund after 5 years of service shall be:
a) Taxable b) Fully exempt
c) Taxable to the extent of employer's contribution and interest thereon.

PROBLEM:244 Payment from recognized provident fund before 5 years shall be:
a) Fully taxable b) fully exempt
c) Shall be treated as if the fund was unrecognized right from beginning

PROBLEM:245 Payment from unrecognized provident fund shall be:


a) Fully taxable b) fully exempt
c) Taxable to the extent of employer’s contribution and interest thereon.
d) Same as (c) and the interest employers contribution shall be taxable under the head other sources

PROBLEM:246 The year in which unrecognized provident fund is converted in recognized provident fund:
a) The employer’s contribution till date and interest thereon shall be taxable
b) The employer’s contribution till date shall be taxable
c) it will be assumed as if the provident fund was recognized right from beginning and excess amount
of employers contribution and interest thereon shall be chargeable to tax
d) None of these

PROBLEM:247 Statutory provident fund is set up under the provisions of the Provident Funds Act, 1925 is applicable
(a) Government organizations (b) Semi-Government organizations(c) Local authorities (d) All of the above

PROBLEM:248 Deduction u/s 80C on employee’s contribution is not available in case of :


(a) Statutory Provident Fund (b) Recognized Provident Fund
(c) Unrecognized Provident Fund (d) Public Provident Fund

PROBLEM:249 Employer’s contribution to recognized provident fund is taxable in excess of ____ of salary.
(a) 12% (b) 9.50% (c) 9% (d) 8.50%

SALEEM QURAISHEE Mo: 9175664444 INSPIRE ACADEMY-8888881719


INCOME UNDER HEAD SALARY- Page 4.64

PROBLEM:250 Lump sum payment at the time of retirement or termination of service received from
unrecognized provident fund to the extent it consists of interest on employees contribution is :
(a) Taxable as salary (b) Taxable under Income from other sources
(c) Fully exempt from tax (d) Taxable as business Income

PROBLEM:251 The amount of any contribution to an approved superannuation fund by the employer in respect
of the employee is exempt from tax upto-
(a) Rs. 1, 00,000 (b) Rs. 1, 50,000 (c) Rs. 2,00,000 (d) N

PROBLEM:252 Mr. JATIN retires from a Private service on 1st August 2019. At the time of his retirement after 28
years and 4 months of service, he was getting the salary of Rs. 34,000 per month. He gets Rs. 5,00,000
(including accumulated interest of Rs.1,00,00) from recognized provident fund. The employer and Mr. JATIN
has made a matching contribution. Compute his Income from salary for Assessment Year 2020-21?
ANSWER:252

PROBLEM:253 Mr. JACK retires from a Private service on 1st August 2019. At the time retirement after 28 years and
4 months of service, he was getting the salary of Rs. 34,000 per month. He gets Rs. 5,00,000 (including
accumulated interest of Rs. 1,00,000) from unrecognized provident fund. The employer and Mr. JACK has made
a matching contribution. Compute his Income from salary for Assessment Year 2020-21?
ANSWER:253

25. SECTION SUMMARY


SECTION PARTICULAR

SALEEM QURAISHEE Mo: 9175664444 INSPIRE ACADEMY-8888881719


INCOME UNDER HEAD SALARY- Page 4.65

PROBLEM:254 For the previous year 2019-20 X submits the following information – Basic salary: ` 1,
20,000; dearness allowance: ` 40,000 (46 per cent of which is part of salary for retirement benefits);
commission: ` 6,000 (i.e., 1 per cent of ` 6, 00,000, being turnover achieved by X) and children education
allowance for his 2 children: ` 7,200. The employer contributes ` 20,000 towards provident fund to
which a matching contribution is made by X. Interest credited in the provident fund account on
December 31, 2019 @ 11 per cent comes to ` 93,500. Income of X from other sources is ` 86,000. Find
out the net income of X for the assessment year 2020-21 if the provident fund is (a) statutory provident
fund, (b) recognized provident fund, (c) unrecognized provident fund
ANSWER:254 `

PROBLEM:255 Mr. A retires from service on December 31, 2019, after 25 years of service. Following are
the particulars of his income/investments for the previous year 2019-20:

Particulars `
I. Basic pay @ ` 16,000 per month for 9 months 1,44,000
II. Dearness pay (50% forms part of the retirement benefits) ` 8,000 per 72,000
month for 9 months
III. Lumpsum payment received from the Unrecognized Provident Fund 6,00,000
IV. Deposits in the PPF account 40,000

Out of the amount received from the unrecognized provident fund, the employer’s contribution was
` 2,20,000 and the interest thereon ` 50,000. The employee’s contribution was ` 2,70,000 and the interest
thereon ` 60,000. What is the taxable portion of the amount received from the unrecognized provident
fund in the hands of Mr. A for the assessment year 2020-21? Will your answer be any different if the fund
mentioned above was a recognized provident fund?
ANSWER:255

PROBLEM:256 Mr. Goyal receives the following emoluments during the previous year ending 31.03.2020.

Basic pay `40,000


Dearness Allowance `15,000
Commission `10,000
Entertainment allowance `4,000
Medical expenses reimbursed `25,000
Professional tax paid `2,000 (` 1,000 was paid by his employer)

Mr. Goyal contributes ` 5,000 towards recognized provident fund. He has no other income. Determine
the income from salary for A.Y. 2020-21, if Mr. Goyal is a State Government employee.
ANSWER:256

SALEEM QURAISHEE Mo: 9175664444 INSPIRE ACADEMY-8888881719


INCOME UNDER HEAD SALARY- Page 4.66

PROBLEM:257 Mr. B is working in XYZ Ltd. and has given the details of his income for the P.Y. 2019-20.
You are required to compute his gross salary from the details given below:

Basic Salary ` 10,000 p.m.


D.A. (50% is for retirement benefits) ` 8,000 p.m.
Commission as a percentage of turnover 0.1%
Turnover during the year ` 50,00,000
Bonus ` 40,000
Gratuity ` 25,000
His own contribution in the RPF ` 20,000
Employer’s contribution to RPF 20% of his basic salary
Interest accrued in the RPF @ 13% p.a. ` 13,000
ANSWER:257

PROBLEM:258 Mr. Rahul is employed in Bharat Textiles Ltd. Mumbai on a monthly salary of `20,000. In
addition to this fixed salary, he is entitled to a commission @ 5% on the sales made by him. During the
previous year 2019-20, he had received following allowances and amenities from his employer:
i) Dearness allowance @ ` 2,000 per month which is granted to him under the terms of employment
and counted for retirement benefits.
ii) Bonus equal to two months basic salary..
iii) House rent allowance @ ` 5,000 per month. .
iv) Entertainment allowance @ ` 250 per month.
v) The company paid ` 1,000 as his income-tax penalty.
vi) In September, 2019 during leave he went on a visit to Kashmir with his family. The expenditure
amounting to `16,000 as passage money by air were paid to him be employer as leave travel
assistance.
vii) He had been provided with the amenities of gas, electricity and water, the expenses of which
amounting to ` 12,000 were paid by the company.
viii) Commission on sales of ` 10,00,000 @ 5%.
ix) He was given titan watch worth `9,000 by his employer on the foundation day of the company.
x) He and his employer each contributed 12.5% of his salary to recognized provident fund. The interest
credited to this fund for the previous year at 13.5% rate of interest amounted to `27,000 .
Compute taxable income from salary of Mr. Rahul for the assessment year 2020-21 keeping in mind
that he spent ` 6,000 p.m. as rent of the house hired by him.
ANSWER:258 `

PROBLEM:259 Kapil is employed by SIDBI. He receives the following emoluments during 2019-20

Particulars `

Basic Salary 1,40,000

DA (forming part of salary) 9,000

Sale of computer to him by employer sale price ` 20,000

(Cost of computer ` 20,00,00 used by employee for 3 years 9 months)

Rent free accommodation in Gwalior, the F.R.V. of which is ` 24,000 p.a. Compute his taxable income for
the assessment year 2020-21 assuming the population of Gwalior is 15 lakhs.
ANSWER:259 `

SALEEM QURAISHEE Mo: 9175664444 INSPIRE ACADEMY-8888881719


INCOME UNDER HEAD SALARY- Page 4.67

PROBLEM:260 Ajit submits you the following particulars of his income received from X Ltd. for the year 31-03-20

Particulars `

Salary (after deduction of tax of source and own contribution@ 14% to recognized provident 2,81,000
fund)

Tax deducted at source 20,000

Employer's contribution to provident fund 49,000

Interest credited on 15-6-2019 to provident fund @ 13.5% 32,400

Allowance for holiday of Ajit and his family 4,000

Academic research allowance for training of X (expenditure incurred: ` 10,000) 15,000

House rent allowance (rent paid for a house in Varanasi by Ajit: ` 48,000) 36,000

Ajit pays life insurance premium of ` 15,000 on own life insurance policy (sum assured: `
1,60,000)

On 15-3-2020, Ajit gets a wrist watch (cost: ` 4,500) from the employer as gift. On 20-3-2020 Ajit is transferred from
Varanasi to Indore. While his family remains in Varanasi, he joins his duties at Indore on 21-3-2020. An
accommodation is provided by the employer in a hotel in Indore from 21-3-2020 to 31-3-2020 (tariff being ` 1,500
per day is paid by employer).Determine the taxable income and tax liability of Ajit for the assessment year 2020-21
assuming that income from other sources is ` 118900 Ajit annually contributes ` 4,000 towards the Unit-linked
Insurance Plan and in March, he has received a sum of ` 5,000 from the Income-tax Department (` 4,000 being
income-tax refund and ` 1,000 being interest thereon.)
ANSWER:260 `

PROBLEM:261 Ajay, an Indian citizen is an employee of the Government of India. He was transferred to London
in Indian High Commission on 1-1-2019. During the previous year 2019-20 he did not come to India at all. He
was entitled to the following emoluments in London:
1. Basic Salary ` 17,000 p.m.
2. Overseas Allowance ` 20,000 p.m.
3. Rent-free furnished accommodation at London, whose FRV is ` 1, 20,000 p.a. and cost of furniture provided is `
1, 00,000.His elder son was studying in London School of Economics and he was allowed ` 12,000 p.m. as
education allowance for his education. His younger son was studying in a school run by Indian High Commission
in London. He was provided with a car of 1800 cc engine cubic capacity which he drives himself and uses the
same partly for official purposes and partly for personal purposes. He has also been provided with the services
of a gardener, watchman, sweeper and a cook: the salary of each being `2,500 per month. Compute his
income from salary for the assessment year 2019-20.
ANSWER:261 `

SALEEM QURAISHEE Mo: 9175664444 INSPIRE ACADEMY-8888881719


INCOME UNDER HEAD SALARY- Page 4.68

PROBLEM:262 Rohan, an Indian citizen is an IFS officer of Government of India. On 1-9-2019 he was
transferred to the Indian Embassy in Japan. On 1-11-2019 he was called back to India for an urgent official
work. On 1-2-2020 he was again sent back to Japan. The particulars of his income for the year ended 31-3-
2020 are as under:
1. Salary ` 16,000 per month
2. Dearness Allowance 50% of salary when he is working in India.
3. Rent-free house in Delhi while in India and In Tokyo, while in Japan was provided to him. The rent of the house
in Delhi was ` 1,000 p.m. as per Government rules and the rent of the house in Tokyo was ` 20,000 p.m.
4. He has been provided with a free car of 2000 cc both in Delhi and in Tokyo, which is being used for official
purposes as well as for commuting from residence to the office and back. The car is driven by Rohan himself.
5. He got entertainment allowance of ` 4,000 p.m. in Japan.
6. He was getting overseas allowance of ` 40,000 p.m. in Japan.
7. The Government of India reimbursed ` 80,000 medical expenses incurred on the treatment of himself and his
family members in Tokyo.
Compute the taxable income under the head 'Salaries' for the assessment year 2020-21 Salary is due on the last
day of the month.
ANSWER:262 `

PROBLEM:263 X Ltd. provided the following perquisites to its employee Mr. Y for the P.Y. 2019-20 –
1) Accommodation taken on lease by X Ltd. for ` 15,000 p.m. ` 5,000 p.m. is recovered from the salary of
Mr. Y.
2) Furniture, for which the hire charges paid by X Ltd. is ` 3,000 p.m. No amount is recovered from the
employee in respect of the same.
3) A Santro Car which is owned by X Ltd. and given to Mr. Y to be used both for official and personal
purposes. All running and maintenance expenses are fully met by the employer. He is also provided
with a chauffeur.
4) A gift voucher of ` 10,000 on his birthday.
Compute the value of perquisites chargeable to tax for the A.Y.2020-21, assuming his salary for
perquisite valuation to be ` 10 lakh.
ANSWER:263

SALEEM QURAISHEE Mo: 9175664444 INSPIRE ACADEMY-8888881719


INCOME UNDER HEAD SALARY- Page 4.69

PROBLEM:264 R is employed with X Ltd. He submits the following particulars of his salary income:
i) Basic pay ` 30,000 p.m.
ii) Dearness Allowance ` 15,000 p.m.
iii) A company owned accommodation is provided at Varanasi (Population 16 lakhs)
iv) Free use of computer for his son studying in a school. The computer was purchased by the company
on 1-6-2017 for `1,00,000 and it was sold to R for ` 15,000 on 5-11-2019.
v) The company has given him a loan of ` 6, 00,000 on 1-6-2019 for wedding of his daughter. The entire
loan is still outstanding (assume the interest rate charged by SBI is 12% p.a.).
vi) Free food in office for 300 days. The value per meal is ` 70.
vii) Gift of watch on annual foundation of company valued at ` 16,000.
viii) The company had purchased a motor car on 5-4-2015 for ` 10, 00,000 and it was sold to R for
` 2, 10,000 on 2-4-2019 Car is not being used by R.
ix) The company had purchased video camera for ` 1,20,000 on 1-6-2016 which R has been using for his
personal purposes since then. The camera was sold to R for ` 50,000 on 1-11-2019.
ANSWER:264 `

PROBLEM:265 Shri Bala employed in ABC Co. Ltd. as Finance Manager gives you the list of perquisites
provided by the company to him for the entire financial year 2019-20:
I. Medical facility given to his family in a hospital maintained by the company. The estimated value of
benefit because of such facility is`40,000.
II. Domestic servant was provided at the residence of Bala. Salary of domestic servant is `1,500 per
month. The servant was engaged by him and the salary is reimbursed by the company (employer).
In case the company has employed the domestic servant, what is the value of perquisite?
III. Free education was provided to his two children Arthy and Ashok in a school maintained and
owned by the company. The cost of such education for Arthy is computed at`900 per month and
for Ashok at`1,200 per month. No amount was recovered by the company for such education
facility from Bala.
IV. The employer has provided movable assets such as television, refrigerator and air-conditioner at
the residence of Bala. The actual cost of such assets provided to the employee is`1,10,000.
V. A gift voucher worth`10,000 was given on the occasion of his marriage anniversary. It is given by
the company to all employees above certain grade.
VI. Telephone provided at the residence of Shri Bala and the bill aggregating to`25,000 paid by the
employer. State the taxability or otherwise of the above said perquisites and compute the total
value of taxable perquisites.
ANSWER:265 Rs. 36,400

SALEEM QURAISHEE Mo: 9175664444 INSPIRE ACADEMY-8888881719


INCOME UNDER HEAD SALARY- Page 4.70

PROBLEM:266 Mr. X is employed with AB Ltd. on a monthly salary of ` 25,000 per month and an
entertainment allowance and commission of ` 1,000 p.m. each. The company provides him with the
following benefits:
1) A company owned accommodation is provided to him in Delhi. Furniture costing ` 2, 40,000 were
provided on 1.8.2018.
2) A personal loan of ` 5, 00,000 on 1.7.2019 on which it charges interest @ 6.75% p.a. The entire
loan is still outstanding. (Assume SBI rate of interest to be 12.75% p.a.)
3) His son is allowed to use a motor cycle belonging to the company. The company had purchased
this motor cycle for ` 60,000 on 1.5.2016. The motor cycle was finally sold to him on 1.8.2019 for `
30,000.
4) Professional tax paid by Mr. X is ` 2,000.
Compute the income from salary of Mr. X for the A.Y. 2020-21.
ANSWER:266

PROBLEM:267 R is employed with XY Limited on a basic salary of `5,000 per month. He is also entitled
to dearness allowance @ 100% of basic salary, 50% of which is included in salary for as per terms of
employment. The company gives him HRA of ` 3,000 per month which was increased to ` 3,500
w.e.f. 1-1-2020. He also got an increment of ` 500 in his basic salary w.e.f. 1-2-2020. Rent paid by
him during the previous year 2019-20 is as under:
April and May, 2019-Nil, as he stayed with his parents
June to October, 2019-` 3,000 per month for an accommodation in Ghaziabad
November, 2019 to March, 2020--`4,000 per month for an accommodation in Delhi.
Compute his gross salary for assessment year 2020-21.
ANSWER:271 Taxable HRA ` 10650

SALEEM QURAISHEE Mo: 9175664444 INSPIRE ACADEMY-8888881719


INCOME FROM HOUSE PROPERTY- Page 5.1

SECTION 22-27

1. BASIS OF CHARGE [SEC. 22]


Income is taxable under the head "Income from house property" if the following three conditions are
satisfied:
I. The property should consist of any buildings or lands appurtenant thereto.
II. The assessee should be owner of the property
III. The property should not be used by the owner for the purpose of any business or profession
carried on by him, the profits of which are chargeable to income-tax.

2. PROPERTY CONSISTING OF ANY BUILDINGS OR LANDS APPURTENANT


THERETO
The term "property" is very wide, though under section 22 it is used for a limited purpose, viz., the
property consisting of any buildings or lands appurtenant thereto. All other types of properties are, thus,
excluded from the scope of section 22.
Building-

Land appurtenant thereto –


I. The appurtenant lands in respect of a residential building may be in the form of approach roads
to and from public streets, compounds, courtyards, backyards, playgrounds, kitchen-garden,
motor garage, stable or coach home, cattle-shed, etc., attached to and forming part of building.
II. In respect of non-residential building, the appurtenant lands may be in the form of car-parking
spaces, roads connecting one department with another department, playgrounds for the benefits
of employees, etc.

SALEEM QURAISHEE Mo: 9175664444 INSPIRE ACADEMY-8888881719


INCOME FROM HOUSE PROPERTY- Page 5.2

3. OWNER -
I. Income is taxable under the head, "Income from house property" only if the assessee is the
owner of a house property:
II. The word "owner" includes a legal owner as well as deemed owner.
III. For the purpose of section 22, owner may be an individual, HUF firm, company, co-operative
society or association of persons, etc.
IV. Annual value of property is assessed to tax under section 22 in the hands of owner even if he is
not in receipt of income.
V. It is not necessary that ownership should extend to the site on which building stands as well as
the superstructure.
VI. Income from subletting is not taxable under section 22 but taxable under the head Income from
other sources

 DEEMED OWNER –SECTION 27


Besides the legal owner, section 27 provides that the following persons are to be treated as deemed
owner of house property for the purpose of charging tax on annual value under the head "Income from
house property":

1. TRANSFER TO SPOUSE OR MINOR CHILD - The following conditions should be satisfied:


I. The taxpayer is an individual
II. He or she transfers a house property
III. The property is transferred to his/her spouse (not being a transfer in connection with an
arrangement to live apart) or to his/her minor child (not being a married daughter).
IV. The property is transferred without adequate monetary consideration
If the above conditions are satisfied, then the individual who has transferred the property would be
deemed as "owner" of the property.

CASE1 Mr. JOY has gifted his flat to his wife out of love and affection on 15/8/2019. Now the
Rental Income earned by Mrs. JOY from 15/8/2019 shall be taxable not for Mrs. JOY but shall be
taxable for Mr. JOY

CASE2 Mr. JATIN has gifted Rs. 50 Lakh to Mrs. JATIN on 15/8/2019 and she out of such money
has purchased a flat on 16/8/2019. Now the Rental Income of such house shall not be taxed for Mr.
JATIN but it shall be taxable for Mrs. JATIN . However, the provisions of Section 64(1) would be
applicable which have been discussed in the chapter of Clubbing of Incomes. Section 27 is not
applicable in this case

CASE3 Mr. JACK has gifted a flat to his son on his sixteenth birthday. Any Rental Income from this
flat shall be taxable for Mr. JACK and not for his minor son.

CASE4 Mr. JACK has gifted a flat to his minor daughter on the occasion of her marriage. Now the
Rental Income from such flat shall be taxable for such minor daughter and not for Mr. JACK.

SALEEM QURAISHEE Mo: 9175664444 INSPIRE ACADEMY-8888881719


INCOME FROM HOUSE PROPERTY- Page 5.3

2. HOLDER OF IMPARTIBLE ESTATE -


I. The holder of impartible estate is deemed as owner of the property.

CASE5 Mr. K is a karta of HUF which has many buildings and one temple. Mr. K dissolves HUF and
distributes all assets of HUF within the family members. However, as per the family customs, he
keeps temple building with himself although all the family members have access to such temple
building. Now, in this case, Mr. K is not the owner of the temple building but shall be regarded as a
deemed owner as this temple building and shall be regarded as impartible estate

3. PROPERTY HELD BY A MEMBER OF CO-OPERATIVE SOCIETY/COMPANY/AOP -


I. A member of co-operative society, company or other association of persons to whom a building (or
a part thereof) is allotted or leased under the house building scheme of the society, company or
association, is treated as deemed owner of such property.

4. A PERSON WHO HAS ACQUIRED A PROPERTY UNDER A POWER OF ATTORNEY TRANSACTION -


Section 53A of the Transfer of Property Act requires the following conditions:
I. There is an agreement in writing between the purchaser and the seller
II. The purchaser has paid the consideration or he is ready to pay the consideration (if there is no
consideration as in the case of gift, then section 53A of the Transfer of Property Act is not
applicable
III. The purchaser has taken the possession of the property
If the aforesaid three conditions are satisfied, the purchaser becomes the deemed "owner" of
the property for the purpose of income-tax, even if he is not the registered owner of the
property.

5. A PERSON WHO HAS ACQUIRED A RIGHT IN A BUILDING UNDER SECTION 269UA (f) -
I. Section 269UA (f), covers the case of giving a property on lease for a term of not less than 12
years (whether fixed originally or there is a provision for extension of term and the aggregate
period is not less than 12 years).
II. The above-noted provision of deemed ownership does not cover any right by way of a lease from
month to month or for a period not exceeding one year.

SALEEM QURAISHEE Mo: 9175664444 INSPIRE ACADEMY-8888881719


INCOME FROM HOUSE PROPERTY- Page 5.4

4. PROPERTY SHOULD NOT BE OCCUPIED BY THE OWNER FOR HIS OWN


BUSINESS OR PROFESSION -
CASE6 X owns a property. He uses the property as his office, factory or godown. As the property is used for
the purpose carrying on own business or profession, nothing is taxable under section 22.
CASE7 X Ltd. is a manufacturing company. The factory of the company is situated in Andhra Pradesh. Within
the factory campus, there is a residential colony having 80 quarters for workers. These quarters are given to
the workers for residential purposes. A nominal rent of ` 100 per month is recovered from the workers. As
the purpose of letting out of residential quarters is to run the business smoothly, the residential quarters
will be treated as house property used by the assessee for the purpose of its business. Accordingly, annual
value thereof is not chargeable under section 22. Recovery of rent of ` 100 per month from the workers will
be taken as business receipt.
CASE8 Y Ltd. makes available a few rooms in its factory on nominal rent to the Government for locating a
branch of nationalized bank, post office, police station, central excise office and railway staff quarters for
carrying on its business efficiently and smoothly. Nothing is taxable under section 22. Rent collected, being
incidental to the business of Y Ltd., is assessable as business income under section 28.

5. PROPERTY HELD AS STOCK-IN-TRADE


As a specific head of charge is provided for income from house property, annual value of house property
cannot be brought to tax under any other head of income. It will remain so even if:
I. The property is held by the assessee as stock-in-trade of a business; or.
II. the assessee is engaged in the business of letting out of property on rent,
III. The assessee is a company which is incorporated for the purpose of owning house property.

Exceptions: The rule that income from ownership of house property is taxable under the head "Income
from house property" has the following exceptions:
Exception one: If letting is only incidental and subservient to the main business of the assessee, rental
income is not taxable under the head "Income from house property" but is chargeable as business income
under the head "Profits and gains of business or profession'.
Exception two: In some cases, income is received not only for letting out of property but also for incidental
services or facilities (e.g., a furnished paying guest accommodation, a well-equipped theatre, a safe deposit
vault). In such cases, the subject hired out is a complex one. The income cannot be said to be derived from
mere ownership of house property but because of facilities and services rendered. Income in such case
may be assessable as income from business.

NO NOTIONAL INCOME FOR HOUSE PROPERTY HELD AS STOCK IN TRADE FOR A PERIOD
UPTO TWO YEAR –SECTION 23(5)- W.E.F AY 2018-19
Where the property consisting of any building or land appurtenant thereto is held as stock-in-
trade and the property or any part of the property is not let during the whole or any part of
the previous year, the annual value of such property or part of the property, for the period up
to TWO year from the end of the financial year in which the certificate of completion of
construction of the property is obtained from the competent authority, shall be taken to be
nil.

SALEEM QURAISHEE Mo: 9175664444 INSPIRE ACADEMY-8888881719


INCOME FROM HOUSE PROPERTY- Page 5.5

6. OTHER SPECIAL ASPECTS

CASE TAX TREATMENT

Club In case of Property owned by a Club, the Annual Value of the Clubhouse is not chargeable to
tax because of Principle of Mutuality.

Real Estate For a Company in Real Estate Business, with the object of buying and developing landed
Business properties, the Income from unsold property let-out is taxable under the head “Income from
House Property”.

Rent from Rent received from Employees’ Residential Quarters, Rent received from part of the premises
Residential let-out to Government for locating Branch of a Bank or Post Office or Railways to carry on
business smoothly, is taxable as income under the head Profits and Gains of Business or
Quarters
Profession.

Sub-letting If sub-letting is in the ordinary course of business, such Income is taxable under the head Profits
and Gains of Business or Profession. Otherwise, it will be taxable under the head “Income from
Other Sources.”

Hotel or If the Assessee does not merely let-out a property, but also provides other facilities, it is on par
Lodging House with activity of a Hotel or Lodging House & hence shall be recognized as a Business Activity.

Motive to earn As per MOA of the Company, if the objective of the Company is to acquire property and to earn
Income Income, then such Income is assessable under Profits and Gains from Business or Profession.

7. WHEN PROPERTY INCOME IS NOT CHARGED TO TAX-


In the following cases rental income is not charged to tax:
I. Income from farm house
II. annual value of any one palace of an ex-ruler
III. property income of a local authority
IV. property income of an approved scientific research association
V. property income of an educational institution and hospital
VI. property income of a trade union
VII. house property held for charitable purposes
VIII. property income of a political party
IX. property used for own business or profession
X. one self-occupied property

8. PROPERTY OWNED BY CO-OWNERS [SEC. 26]


I. If a house property is owned by two or more persons, then such persons are known as co-
owners.
II. If respective shares of co-owners are definite and ascertainable, the share of each such person (in
the computed income of property) shall be included in his total income.
III. It may be noted that co-owners are not taxable as an association of persons

SALEEM QURAISHEE Mo: 9175664444 INSPIRE ACADEMY-8888881719


INCOME FROM HOUSE PROPERTY- Page 5.6

9. TREATMENT OF COMPOSITE RENT -


A. Meaning of composite rent: The owner of a property may sometimes receive rent in respect of
building as well as –
1) Other assets like say, furniture, plant and machinery.
2) For different services provided in the building, for eg. –Lifts/Security/Power backup;
The amount so received is known as “composite rent”.

B. Tax treatment of composite rent- Where composite rent includes rent of building and charges for
different services (lifts, security etc.), the composite rent is has to be split up in the following manner-
a) the sum attributable to use of property is to be assessed under section 22 as income from
house property;
b) the sum attributable to use of services is to be charged to tax under the head “Profits and gains
of business or profession” or under the head “Income from other sources”, as the case may be.

C. Manner of splitting up-

 If let out building and other assets are inseparable Where composite rent is received from letting
out of building and other assets (like furniture) and the two lettings are not separable ei.. the other
party does not accept letting out of building without other assets, then the rent is taxable either as
business income or income from other sources, the case may be.

 If let out building and other assets are separable- Where composite rent is received from
letting out of building and other assets and the two lettings are separable i.e. letting out
of one is acceptable to the other party without letting out of the other, then
a) income from letting out of building is taxable under “Income from house property”;
b) Income from letting out of other assets is taxable under “Profits and gains of business or
profession” or “Income from other sources”, as the case may be.
This is applicable even if a composite rent is received by the assessee from his tenant for the
two lettings.

10. COMPUTATION OF INCOME FROM HOUSE PROPERTY-

Particulars LO/DLO SOP


Gross Annual Value (GAV) XXXX
Less: Municipal tax XXXX
Net Annual Value (NAV) XXXX
Less: Deductions u/s XXXX
24(a) Standard deduction [30% of NAV] XXXX
24(b) Interest on borrowed capital XXXX
Income from house property XXXX

SALEEM QURAISHEE Mo: 9175664444 INSPIRE ACADEMY-8888881719


INCOME FROM HOUSE PROPERTY- Page 5.7

SELF-OCCUPIED PROPERTY OR UNOCCUPIED PROPERTY [SECTION 23(2)]


a) Where the property is self-occupied for own residence or unoccupied throughout the previous
year, its Annual Value will be Nil, provided no other benefit is derived by the owner from such
property.
The expression “Unoccupied property” refers to a property which cannot be occupied by the
owner by reason of his employment, business or profession at a different place and he resides at
such other place in a building not belonging to him.

b) The benefit of “Nil” Annual Value is available only for upto two self-occupied or unoccupied house
properties i.e. for either one house property or two-house properties.

c) The benefit of “Nil” Annual Value in respect of upto two self-occupied house properties is available
only to an individual/ HUF.

d) No deduction for municipal taxes is allowed in respect of such property/properties.

SALEEM QURAISHEE Mo: 9175664444 INSPIRE ACADEMY-8888881719


INCOME FROM HOUSE PROPERTY- Page 5.8

11. WHEN UNREALISED RENT SHALL BE EXCLUDED [EXPLN. TO SEC. 23(1)]


Unrealized rent (which the owner could not realize) shall be excluded from rent received/receivable only
if the following conditions are satisfied:
I. The tenancy is bona fide
II. The defaulting tenant has vacated, or steps have been taken to compel him to vacate the
property
III. The defaulting tenant is not in occupation of any other property of the assessee
IV. The assessee has taken all reasonable steps to institute legal proceedings for the recovery of the
unpaid rent or satisfies the Assessing Officer that legal proceedings would be useless

PROBLEM:1 Mr. VARUN own five houses at Cochin, all of which are let out. Compute the gross annual
value of each house from the information given below:

House-I House-ll House-Ill House-I V House-V

Municipal value 1,20,000 2,40,000 1,10,000 90,000 75,000

Fair rent 1,50,000 2,40,000 1,14,000 84,000 80,000

Standard rent 1,08,000 N.A. 1,44,000 N.A. 78,000

Actual rent received 1 receivable 1,80,000 2,10,000 1,20,000 1,08,000 72,000

ANSWER:1

` in Thousands)
PROBLEM:2 X, Y, Z, A and B separately own the following properties (`
Particulars H1 X H2 Y H3 Z H4 A H5 B

Municipal value (MV) 105 105 105 105 105

Fair rent (FR) 107 107 107 107 107

Standard rent under the Rent Control Act (SR) NA 88 88 135 135

Actual rent 103 112 86 114 97

Unrealized rent (conditions satisfied) 1 2 1 2 1

Period of the previous year (in months) 12 12 12 12 12

Period during which the property remains vacant Nil Nil Nil Nil Nil
Find out the gross annual value for the assessment year 2020-21
ANSWER:2

SALEEM QURAISHEE Mo: 9175664444 INSPIRE ACADEMY-8888881719


INCOME FROM HOUSE PROPERTY- Page 5.9

PROBLEM:3 Find out the gross annual value in the case of the following properties let out throughout
` in Thousands)
the previous year for the assessment year 2020-21. (`
Particulars X Y Z A B

Municipal Value (MV) 60 60 60 112 112

Fair Rent (FR) 68 68 68 117 117

Standard rent under the Rent Control Act (SR) 62 62 70 115 115

Annual Rent 67 67 73 121 110

Unrealized rent of the previous year 2 6 5 50 40

Loss due to vacancy 1 1 1 1 -


ANSWER:3

PROBLEM:4 Find out the gross annual values in the case of the following properties for the assessment
year 2020-21 (there is no unrealized rent)
Particulars X Y Z A B C D

Municipal value (per annum)(MV) 60 61 60 80 80 140 140

Fair rent (per annum) (FR) 65 66 64.5 78 78 150 150

Standard rent under the Rent Control Act (per annum) 59.5 59 63 85.5 76 120 120
(SR)

Annual rent 72 57 72 72 NA 96 144

Property remains vacant (in number of month) (1) (11/2) (5) (3) (12) (10) (10)

Loss due to vacancy 6 7.125 30 18 - 80 120


ANSWER:4

12. DEDUCT MUNICIPAL TAXES -

13. DEDUCTION UNDER SECTION 24


The following two deductions are available:
I. Standard deduction u/s 24(a)-
II. Interest on borrowed capital u/s 24(b)-

SALEEM QURAISHEE Mo: 9175664444 INSPIRE ACADEMY-8888881719


INCOME FROM HOUSE PROPERTY- Page 5.10

 STANDARD DEDUCTION [SEC. 24 ( )]


I. 30 per cent of net annual value is deductible irrespective of any expenditure incurred by the
taxpayer.

 INTEREST ON BORROWED CAPITAL [SEC. 24 ( )]

The following points should also be kept in view-


I. If capital is borrowed for the purpose of purchasing a plot of land, interest liability is deductible
even if construction is financed out of own funds.
II. Interest on borrowed capital is deductible on "accrual" basis.
III. Deduction is available even if neither the principal nor the interest is a charge on property.
IV. Interest on unpaid interest is not deductible.
V. No deduction is allowed for any brokerage or commission for arranging the loan.
VI. Interest on a fresh loan, taken to repay the original loan raised for the aforesaid purposes, is
allowable as deduction.
VII. Any interest chargeable under the Act, in the hands of recipient and payable out of India, on
which tax has not been paid or deducted at source, and in respect of which there is no person
who may be treated as an agent, is not deductible, by virtue of section 25, in computing income
chargeable under the head “Income from house property”.(Sec 25)

PRE-CONSTRUCTION PERIOD-
I. Interest of pre-construction period is deductible in five equal installments.
II. The first installment is deductible in the year in which construction of property is completed or in
which property is acquired.
III. Pre-construction period means the period commencing on the date of borrowing and ending on
(a) March 31 immediately prior to the date of completion of construction/ date of acquisition or
(b) date of repayment of loan, whichever is earlier

SALEEM QURAISHEE Mo: 9175664444 INSPIRE ACADEMY-8888881719


INCOME FROM HOUSE PROPERTY- Page 5.11

PROBLEM:5 Mr. X took a loan of Rs. 5,00,000on 01.10.2016 @ 10% p.a. for construction of house which
was completed on 31.03.2020. Compute interest on capital borrowed for the previous year 2019-20.
ANSWER:5

PROBLEM:6 The construction of a house property was completed on 28-2-2020. The assessee had
taken a loan of ` 10,00,000 @ 12% p.a. Compute the deduction of interest for the previous year
2019-20 if the loan was taken on:
(a) 1-4-2017 (b) 1-6-2017 (c) 1-5-2018.
ANSWER:6

PROBLEM:7 R took a loan of ` 6, 00,000 @ 12% p.a. on 1-12-2016. He took additional loan of
`5,00,000 @ 10% p.a. on 1-4-2017. The construction of the house is completed on 15-9-2018.
Compute the deduction allowable on account of interest for the previous year 2019-20 assuming
that nothing has been repaid on account of such loan.
ANSWER:7 `

PROBLEM:8 R took a loan of ` 12, 00,000 @ 12% on 1-9-2015 for construction of a house. He repaid
the loan on 31-12-2018.The house property was completed on 5-10-2018. Compute the deduction
available on account of interest in the previous year 2019-20.
ANSWER:8 `

PROBLEM:9 X owns a house property. It is used by him throughout the previous year 2019-20 for his
(and his family members) residence. Municipal value of the property is ` 1, 66,000, whereas fair
rent is ` 1,76,000 and standard rent is ` 1 ,50,000. The following expenses are incurred by X:
repairs: ` 20,000, municipal tax: ` 16,000, insurance: ` 2,000; interest on capital borrowed to
construct the property: ` 1,36,000; interest on capital borrowed by mortgaging the property for
daughter's marriage: ` 20,000 (in either case capital is borrowed before April 1, 1999). Income of X
from business is ` 7,10,000. Find out the net income of X for the assessment year 2020-21.
ANSWER:9 Net income ` 680000

PROBLEM:10 X owns a residential house property. It has two equal residential units-Unit 1 and Unit 2.
While Unit 1 is self-occupied by X for his residential purpose, Unit 2 is let out (rent being ` 6,000 per
month, rent of 2 months could not be recovered). Municipal value of the property is ` 1,30,000,
standard rent is ` 1,25,000 and fair rent is ` 1,40,000. Municipal tax is imposed @ 12 per cent
which is paid by X. Other expenses for the previous year 2019-20 being repairs: ` 250, insurance: `
600, interest on capital (borrowed during 1998) for constructing the property: ` 63,000.Find the
income of X for the assessment year 2020-21 on the assumption that income of X from other
sources is ` 1,80,000.
ANSWER:10 Net income ` 156790

PROBLEM:11 Mr. Manas owns two house properties one at Bombay, wherein his family resides and the
other at Delhi, which is unoccupied. He lives in Chandigarh for his employment purposes in a rented
house. For acquisition of house property at Bombay, he has taken a loan of ` 30 lakh@10% p.a. on
1.4.2018. He has not repaid any amount so far. In respect of house property at Delhi, he has taken a
loan of ` 5 lakh@11% p.a. on 1.10.2018 towards repairs. Compute the deduction which would be
available to him under section 24(b) for A.Y.2020-21 in respect of interest payable on such loan.
ANSWER:11

SALEEM QURAISHEE Mo: 9175664444 INSPIRE ACADEMY-8888881719


INCOME FROM HOUSE PROPERTY- Page 5.12

14. ARREARS OF RENT OR UNREALIZED RENT [SEC. 25A]


The amount of rent received in arrears or the amount of unrealized rent realized subsequently by an
assessee shall be charged to income tax in the previous year in which rent is received or realized. The
following points should be noted-
i. The recipient of arrears of rent (or recipient of unrealized rent) is chargeable to tax in the year in
which such rent is received regardless of the fact whether in the year of receipt the house property
is owned by him or not.
ii. The recipient can claim 30% of the arrears of rent or unrealized rent realized subsequently as
deduction.

15. SECTION SUMMARY-


SECTION PARTICULAR

SALEEM QURAISHEE Mo: 9175664444 INSPIRE ACADEMY-8888881719


INCOME FROM HOUSE PROPERTY- Page 5.13

PRACTICAL PROBLEMS
PROBLEM:12 Anirudh has a property whose municipal valuation is ` 1,30,000 p.a. The fair rent is`
1,10,000 p.a. and the standard rent fixed by the Rent Control Act is ` 1,20,000 p.a. The property was
let out for a rent of ` 11,000 p.m. throughout the previous year. Unrealized rent was ` 11,000 and all
conditions prescribed by Rule 4 are satisfied. He paid municipal taxes @10% of municipal valuation.
Interest on borrowed capital was ` 40,000 for the year. Compute the income from house property of
Anirudh for A.Y.2020-21.
ANSWER:12

PROBLEM:13 Ganesh has a property whose municipal valuation is ` 2,50,000 p.a. The fair rent is
`2,00,000 p.a. and the standard rent fixed by the Rent Control Act is ` 2,10,000 p.a. The property was let
out for a rent of ` 20,000 p.m. However, the tenant vacated the property on 31.1.2020. Unrealized rent
was ` 20,000 and all conditions prescribed by Rule 4 are satisfied. He paid municipal taxes @8% of
municipal valuation. Interest on borrowed capital was ` 65,000 for the year. Compute the income from
house property of Ganesh for A.Y. 2020-21.
ANSWER:13

PROBLEM:14 P has one house property at Indira Nagar in Bangalore. She stays with her family in the
house. The rent of similar property in the neighborhood is ` 25,000 p.m. The municipal valuation is `
23,000 p.m. Municipal taxes paid is ` 8,000. The house construction began in April 2013 with a loan of
` 20,00,000 taken from SBI Housing Finance Ltd. @9% p.a. on 1.4.2013. The construction was
completed on 30.11.2015. The accumulated interest up to 31.3.2015 is ` 3,60,000. On 31.3.2020, P
paid ` 2,40,000 which included ` 1,80,000 as interest. There was no principal repayment prior to this
date. Compute P ’s income from house property for A.Y. 2020-21.
ANSWER:14

PROBLEM:15 Mr. Sehgal owns a house in Bangalore construction of which was completed on 1-7-1999.
Half portion is let out for residential purposes on a monthly rent of `8,000. However, this portion
remained vacant for three months (from 1-1-2020 to 31-3-2020) during the previous year 2019-20.
1/4th portion is used by Mr. Sehgal for the purpose of his profession while the remaining 1/4th
portion is use for his own residence for the full year. The other expenses regarding the house were:

No. Particulars `

(i) Municipal taxes 10,000

(ii) Repairs 5,000

(iii) Interest on loan for renovation of house 40,000

(iv) Ground rent 2,000

(v) Annual charge 10,000

(vi) Fire insurance premium 4,000

Compute the taxable "Income from house property" for the assessment year 2020-21.
ANSWER:15 ` 16900

SALEEM QURAISHEE Mo: 9175664444 INSPIRE ACADEMY-8888881719


INCOME FROM HOUSE PROPERTY- Page 5.14

PROBLEM:16 X, a chartered accountant, has a house property situated at Delhi, which has 4 identical
units, unit I was used by him for his professional purposes, unit II was let out for residential
purposes at ` 5,0000 p.m., unit III and unit IV were self occupied. Other particulars of the property
are as under:

Date of completion 31-1-1996

Municipal Taxes paid ` 20,0000

Interest on money borrowed for construction of house prop. ` 60,0000

Compute his income under the head house property for the assessment year 2020-21.
ANSWER:16 `

PROBLEM:17 Arjun own a house, construction of which was completed on 1-10-1996, half of the house
` 10,000 p.m. This portion was, however, vacant for 3 months. 25% of the house was
was let out @`
used by the owner for residence and 25% for business for the full year. Other expenses of the house
were:

Municipal taxes 20,000

Repairs 16,000

Interest on loan taken for construction 10,000

Fire insurance premium 6,000

Calculate the income from house property of Mr. Arjun for the assessment year 2020-21.
ANSWER:17 ` 48500

PROBLEM:18 Mr. Goel owns a house property at Allahabad which is let out for residential purposes.

No. Particulars `

(i) Annual Rent 60,000

(ii) Municipal Valuation 48,000

(iii) Municipal taxes (paid by tenant) 4,000

(iv) Expenses incurred

a) Repairs met by tenant 3,000

b) Fire Insurance premium paid 1,500

c) Electricity and water charges paid by Mr. Goel 5,400

d) Lift maintenance charges paid by Mr. Goel 2,400

SALEEM QURAISHEE Mo: 9175664444 INSPIRE ACADEMY-8888881719


INCOME FROM HOUSE PROPERTY- Page 5.15

e) Collection charges (including ` 300 of litigation expenses 750

f) During the previous year 2006-07, he had claimed a deduction of


unrealized rent of ` 22,500 out of which ` 16,500 were allowed as
deduction for that year. On 10-8-2019, however, he recovered `
10,500 from the defaulting tenant.

The construction of the property was completed on 31-8-1996. ` 60,000 is a composite rent of property,
as well as amenities provided to the tenant. Compute the taxable "Income from house property" for the
assessment year 2020-21.
ANSWER:18 `

PROBLEM:19 Mr. X is the owner of a residential house, whose construction was completed on 31-8-
1994. It has been let out from 1-12-1994 for residential purposes. Its particulars for the financial
year 2019-20 are given below:

No. Particulars `

(i) Municipal valuation 55,000

(ii) Expected fair rent per annum 60,000

(iii) Standard rent under the Rent Control Act 6.000 p.m.

(iv) Actual monthly rent 6,000 p.m.

(v) Municipal taxes (including ` 5,000 paid by tenant) paid 15,000

(vi) water/sewage benefit tax levied by State Government but disputed in Court 6,000

(vii) Fire Insurance payable 800

(viii) Interest on loan taken for the construction of the house. The interest has
been paid outside India to a non-resident without deduction of tax at source,
as the non-resident agreed to pay income tax on such interest directly to the
Government. 10,000

(ix) Legal charges for the recovery of rent 4,000

(x) Stamp duty and registration charges incurred in respect of the lease
agreement of the house. 2,000

(xi) The unrealized rent of the earlier years amounted to ` 10,000 but a deduction claimed so far
was only for ` 7,000. Now, there is a recovery of ` 8,000 from the defaulting tenants.

Compute his income from house property for the assessment year 2020-21.
ANSWER:19 ` 46900

SALEEM QURAISHEE Mo: 9175664444 INSPIRE ACADEMY-8888881719


INCOME FROM HOUSE PROPERTY- Page 5.16

PROBLEM:20 A house was completed on 1-4-2019 and following information is available about this
house:

Municipal value of the house 84,000 p.a

Fair rental value of the house 90,000 p.a.

Actual Rent 7,200 p.m.

Municipal taxes paid 15,000

Let out for the period 1-4-2019 to 30-6-2019 and self-occupied from 1-7-2019 onwards:

Fire Insurance Premium 1,600

Land Revenue paid 3,000

Interest on Loan for the period

(a) 1-4-2016 to 31-3-2019 51,000

(b) 1-4-2019 to 31-3-2020 17,000

Calculate income from the house property for the assessment year 2020-21-
ANSWER:20 ` 25300

PROBLEM:21 Mr. X has 3 houses which are self-occupied and the details of these houses is as under.
Particulars House I House II House III
(In Rs.) (In Rs.) (In Rs.)
Fair rent 11,00,000 12,00,000 11,50.000
Municipal valuation 11,24,000 11,78,000 11,25,000
Standard rent 13,00,000 12,50,000 11,40,000
Municipal taxes paid 1,00,000 80,000 90,000
Interest on capital borrowed on 3,20,000 2,90,000 1,90,000
01.04.2011 and all the necessary
conditions are complied with to avail
higher amount of interest.
Repair charges 10,000 3,000 8,000
Date of completion of house 01.10.2016 01.10.2016 01 10.2016
Compute income under the head house property
ANSWER:21

SALEEM QURAISHEE Mo: 9175664444 INSPIRE ACADEMY-8888881719


INCOME FROM HOUSE PROPERTY- Page 5.17

PROBLEM:22 Mr. X occupied two flats for his residential purposes, particulars of which are as follows:
Particulars Flat I (in Rs.) Flat II (in Rs.)
Municipal Valuation 90,000 45,000
Fair Rent 1,20,000 40,000
Fair rent under Rent Control Act
(i.e. Standard Rent) 80,000 Not available
Municipal taxes paid 10% of municipal valuation 10% of municipal valuation
Fire insurance paid 1,000 600
Interest payable on capital borrowed
for purchase of flat 40,000 Nil
Income of Mr. X from his Proprietary business- XYZ Warehousing Corporation is Rs.6,50,000.
Determine the taxable income and tax liability for the assessment year 2020-21. You are informed that
Mr. X could not occupy flat for 2 months commencing from December 1st, 2019 and that he has attained
the age of 60 on 23.08.2019.
ANSWER:22

PROBLEM:23 Mrs. X has two houses, both of which are self-occupied. The particulars of these are given
below
Particulars (Value in Rs.)
House -I House - II
Municipal Valuation per annum 1,20,000 1,15,000
Fair Rent per annum 1,50,000 1,75,000
Standard rent per annum 1,00,000 1,65,000
Date of completion 31-03-1999 31-03-2001
Municipal taxes payable during the year (paid for House II only) 12% 8%
Interest on money borrowed for repair of property during current year - 55,000
Compute Mrs. X’s income from the House Property for the Assessment Year 2020-21.
ANSWER:23

SALEEM QURAISHEE Mo: 9175664444 INSPIRE ACADEMY-8888881719


INCOME FROM HOUSE PROPERTY- Page 5.18

PROBLEM:24 X (age: 24 years) has occupied two houses for his residential purposes, particulars of
which are as follows:

Particulars House I House II


` `

Municipal valuation (MV) 60,000 30,000

Fair rent (FR) 85,000 32,000

Standard rent under Rent Control Act (SR) 65,000 36,000

Municipal taxes paid 10% 10 %

Fire insurance 600 360

Interest on capital borrowed for construction of houses (` 9,44,000 1,41,600 Nil


is borrowed @ 15 percent per annum on April 10, 2019, construction
is completed on March 10, 2020 and loan is yet to be repaid)

Income of X from business is ` 6, 30,000. Determine the taxable income and tax liability for the
assessment year 2020-21 on the assumption that he contributes ` 140,000 towards the public
provident fund. X could not occupy House II for two months commencing from December 1, 2019.
ANSWER:24 T.I ` Tax Liability `

PROBLEM:25 A owns a house property at Delhi. 60% of the house property is self-occupied for
residence and 40% is let out on a monthly rent of ` 5,000. The let-out portion was also self-
occupied from 1-10-2019 to 31-12-2019. However, w.e.f. 1-1-2020 the entire house was let out for
` 12,500 pm. The construction of the house property was completed on 31-12-1998.The following
expenses were incurred for the above house property during the year ending on 31-3-2020.

Municipal tax paid: `

for financial year 2017-18 5,000

for financial year 2018-19 10,000

for financial year 2019-20 15,000

Insurance premium paid 3,000

Land revenue due 6,000

Interest on money borrowed for construction of house property 18,000

Calculate income under the head house property of A for the assessment year 2020-21.
ANSWER:25 ` 66000

SALEEM QURAISHEE Mo: 9175664444 INSPIRE ACADEMY-8888881719


INCOME FROM HOUSE PROPERTY- Page 5.19

PROBLEM:26 For the assessment year 2020-21 X (age : 64 years) submits the following information:
` 9, 82,000
Income from business`
Property income

Particulars House I House II


` `

Fair rent (FR) 3,50,000 3,20,000

Municipal valuation (MV) 3,60,000 3,50,000

Standard rent (SR) 3,00,000 5,00,000

Annual rent 6,00,000 4,20,000

Unrealized rent of the previous year 2019-20 10,000 80,000

Unrealized rent of the previous year 2018-19 - 3,00,000

Vacant period (number of months) (2) (4)

Loss on account of vacancy 1 ,00,000 1 ,40,000

Municipal taxes paid 40,000 50,000

Repairs 5,000 7,000

Insurance 20,000 30,000

Land revenue 25,000 40,000

Ground rent 66,000 82,000

Interest on capital borrowed by mortgaging House I (funds are used


for construction of House II) 1 ,40,000 -

Nature of occupation Let out for Let out for


residence business

Determine the taxable income and tax liability of X forth assessment year 2020-21 assuming that X
deposits ` 1,50,000 in the public provident fund account.
ANSWER:26 1119000. Tax `

SALEEM QURAISHEE Mo: 9175664444 INSPIRE ACADEMY-8888881719


INCOME FROM HOUSE PROPERTY- Page 5.20

PROBLEM:27 Two brothers Arun and Bimal are co-owners of a house property with equal share. The
property was constructed during the financial year 1998-1999. The property consists of eight identical
units and is situated at Cochin. During the financial year 2019-20, each co-owner occupied one unit
for residence and the balance of six units were let out at a rent of`12,000 per month per unit. The
municipal value of the house property is`9, 00,000 and the municipal taxes are 20% of municipal
value, which were paid during the year. The other expenses were as follows:
(i) Repairs 40,000
(ii) Insurance premium (paid) 15,000
(iii) Interest payable on loan taken for construction of house 3, 00,000
One of the let-out units remained vacant for four months during the year.
Arun could not occupy his unit for six months as he was transferred to Chennai. He does not own any
other house. The other income of Mr. Arun and Mr. Bimal are `2,90,000 and`1,80,000, respectively, for
the financial year 2019-20. Compute the income under the head Income from House Property' and the
total income of two brothers for the assessment year 2020-21.
ANSWER:27

PROBLEM:28 Mr. Raman is a co-owner of a house property along with his brother holding equal share
Particulars `

Municipal value of the property 1,60,000


Fair rent 1,50,000
Standard rent under the Rent Control Act 1,70,000
Rent received 15,000 p.m.

The loan for the construction of this property is jointly taken and the interest charged by the bank is
`25,000, out of which `21,000 has been paid. Interest on the unpaid interest is`450. To repay this loan,
Raman and his brother have taken a fresh loan and interest charged on this loan is`5,000. The municipal
taxes of`5,100 have been paid by the tenant. Compute the income from this property chargeable in the
hands of Mr. Raman for the A.Y. 2020-21.
ANSWER:28 `48000.

SALEEM QURAISHEE Mo: 9175664444 INSPIRE ACADEMY-8888881719


INCOME FROM HOUSE PROPERTY- Page 5.21

16. MULTIPLE CHOICE QUESTIONS-


PROBLEM:1 Which is the charging section of Income from house property?
(a) Section 15 (b) Section 22 (c) Section 24 (d) Section 10(10D)
PROBLEM:2 Income from vacant plot is taxable under the head _________.
(a) Income from House Property (b) Income from other sources
(c) Profits & Gains of Business or Profession (d) Capital Gains

PROBLEM:3 Which of the following conditions need to be satisfied in order to tax any income under
the head Income from house property?
(a) The property must consist of building or land appurtenant thereto
(b) The assessee must be the owner of such house property
(c) The property must not be used for business or profession carried on by assessee.
(d) All of the above

PROBLEM:4 Income from subletting of house property is taxable under the head _______.
(a) Income from House Property (b) Income from other sources
(c) Profits & Gains of Business or Profession (d) Capital Gains

PROBLEM:5 In case the letting out of property is incidental to the main business, then income from
such property shall be taxable as
(a) Income from House Property (b) Income from other sources
(c) Profits & Gains of Business or Profession (d) Capital Gains

PROBLEM:6 Annual value of property of a social club will be :


(a) Taxable as Income from House Property (b) Taxable as Income from other sources
(c) Exempt from tax (d) Taxable as profits and gains of business

PROBLEM:7 In case any property is owned by an assessee and the same is given by him to the
partnership firm, in which he is a partner, for carrying on the business of such firm, then the income
from such property will :
(a) Not be taxable (b) be taxable as Income from other sources
(c) Be taxable as Income from house Property (d) taxable as Profits and gains of business

SALEEM QURAISHEE Mo: 9175664444 INSPIRE ACADEMY-8888881719


INCOME FROM HOUSE PROPERTY- Page 5.22

PROBLEM:8 Income from building constructed on leasehold is taxable as :


(a) Income from House Property (b) Income from other sources
(c) Profits & Gains of Business or Profession (d) Capital Gains

PROBLEM:9 The assessee, who was deriving income from “House property” realized a sum of Rs.
52,000 on account of display of advertisement hoarding of various concerns on the roof of the
building. The same will be taxable under:
(a) Income from House Property (b) Income from other sources
(c) Profits & Gains of Business or Profession (d) Capital Gains

PROBLEM:10 Expected Rent is equal to ______.


(a) Fair Rent (b) Municipal Valuation
(c) Lower of Fair Rent of Municipal valuation
(d) Higher of Fair rent of municipal valuation subject to the maximum of Standard Rent

PROBLEM:11 If Actual Rent received or receivable exceeds Expected Rent, the Gross Annual Value
equals to –
(a) Actual Rent received or receivable (b) Expected Rent
(c) Actual Rent –Expected Rent (d) None of these

PROBLEM:12 The sum for which the property might reasonably be expected to let year to year is known
as (a) Expected Rent (b) Standard Rent (c) Annual value (d) Municipal Valuation

PROBLEM:13 In which of the following cases the annual value of the house is taken to be NIL.
(a) Self occupied house (b) Vacancy for the whole period
(c) If the assessee holds two house properties (d) Both (a) & (b) but not (c)

PROBLEM:14 Find out the expected rent of house property A, if the following is given:
Municipal value = Rs. 1,00,000; Fair Rent = Rs. 88,000;
Standard Rent = Rs. 1,12,000. Actual Rent = Rs. 1,25,000

PROBLEM:15 Find out the expected rent of house property H, if the following is given :
Municipal value = Rs. 70,000; Fair Rent = Rs. 88,000;
Standard Rent = Rs. 1,12,000. Actual Rent = Rs. 1,25,000

SALEEM QURAISHEE Mo: 9175664444 INSPIRE ACADEMY-8888881719


INCOME FROM HOUSE PROPERTY- Page 5.23

PROBLEM:16 Find out the expected rent of house property P, if the following is given :
Municipal value = Rs. 70,000; Fair Rent = Rs. 88,000;
Standard Rent = Rs. 60,000. Actual Rent = Rs. 1,25,000

PROBLEM:17 Find out the expected rent of house property Q, if the following is given
Municipal value = Rs. 65,000; Fair Rent = Rs. 88,000;
Standard Rent = Rs. 60,000 Actual Rent = Rs. 1,25,000

PROBLEM:18 Find the Gross Annual Value of house property A if the following is given:
Municipal value = Rs. 10,000; Fair Rent = Rs. 88,000;
Standard Rent = Rs. 92,000. Actual Rent = Rs. 89,000

PROBLEM:19 Find the Gross Annual Value of house property A if the following is given :
Municipal value = Rs. 1,00,000; Fair Rent = Rs. 88,000;
Standard Rent = Rs. 92,000. Actual Rent = Rs. 89,000

PROBLEM:20 Find the Gross Annual Value of house property A if the following is given :
Municipal value = Rs. 1,00,000; Fair Rent = Rs. 1,20,000;
Standard Rent = Rs. 1,50,000. Actual Rent = Rs. 1,30,000
ANSWER;36

PROBLEM:21 Find the Gross Annual Value of house property A if the following is given :
Municipal value = Rs. 1,60,000; Fair Rent = Rs. 1,20,000;
Standard Rent = Rs. 1,50,000. Actual Rent = Rs. 1,55,000.

PROBLEM:22 Find the Gross Annual Value of house property A if the following is given :
Municipal value = Rs. 1,40,000; Fair Rent = Rs. 1,20,000;
Standard Rent = Rs. 1,50,000. Actual Rent = Rs. 1,55,000.

PROBLEM:23 Calculate the Gross Annual value from the following details :
Municipal value = Rs. 45,000; Fair Rental = Rs. 50,000;
Standard Rent = Rs. 48,000. Actual Rent = Rs. 42,000.

SALEEM QURAISHEE Mo: 9175664444 INSPIRE ACADEMY-8888881719


INCOME FROM HOUSE PROPERTY- Page 5.24

PROBLEM:24 Calculate the Gross Annual value from the following details:
Municipal value = Rs. 45,000; Fair Rental Value = Rs. 50,000;
Standard Rent = Rs. 48,000. Actual Rent = Rs. 42,000.

PROBLEM:25 Calculate the Gross Annual value from the following details :
Municipal value = Rs. 45,000; Fair Rental value = Rs. 50,000;
Standard Rent = Rs. 48,000. Actual Rent Receivable – Rs. 75,000;
Unrealized rent = Rs. 2000

PROBLEM:26 X is the owner of a house, the details of which are given below:
(a) Municipal value: Rs. 30,000 (b) Actual rent : Rs. 32,000
(c) Fair rent : Rs. 36,000 (d) Standard rent : Rs. 40,000
The gross annual value would be –

PROBLEM:27 Municipal value : Rs. 14,000; Fair rent : Rs. 14,500; Standard rent : Rs. 14,200. Actual rent as
property let-out throughout the previous year : Rs. 16,800. Unrealized rent of the previous year : Rs.
7,000. The annual value of the house property shall be

PROBLEM:28 X is owner of house which has been let out at a monthly rent of Rs. 25,000. The fair rent of
the house is Rs. 2, 90,000 and standard rent is Rs. 2,60,000. The municipal value of house is Rs. 2,
80,000 and municipal taxes are levied @ 10% of municipal value. The entire amount of municipal
taxes are outstanding for the year ended 31-03-2020. The amount of municipal taxes to be allowed as
deduction for computing the annual value will be :

PROBLEM:29 Z is owner of house which has been let out at a monthly rent of Rs. 25,000. The fair rent of
the house is Rs. 2,90,000 and standard rent is Rs. 2,60,000. The municipal value of house is Rs.
2,80,000 and municipal taxes are levied @ 10% of municipal value. The entire amount of municipal
taxes are outstanding for the year ended 31-03-2020 are paid by the owner on 31-03-2020. The
amount of municipal taxes to be allowed as deduction for computing the annual value will be
ANSWER;37

SALEEM QURAISHEE Mo: 9175664444 INSPIRE ACADEMY-8888881719


INCOME FROM HOUSE PROPERTY- Page 5.25

PROBLEM:30 T is owner of house which has been let out at a monthly rent of Rs. 25,000. The fair rent of
the house is Rs. 2,90,000 and standard rent is Rs. 2,60,000. The municipal value of house is Rs.
2,80,000 and municipal taxes are levied @ 10% of municipal value. The entire amount of municipal
taxes are outstanding for the year ended 31-03-2020 out of which half of the municipal taxes are paid
by the tenant. The amount of municipal taxes to be allowed as deduction for computing the annual
value will be :

PROBLEM:31 Which of the following deduction are to be made from income house property?
(a) Statutory deduction (b) Interest on borrowed loan (c) Both (a) and (b) (d) Option (a) but not (b)

PROBLEM:32 When did pre-acquisition or pre-construction period commences –


(a) On the 1st year when loan is borrowed (b) On the date of borrowing
(c) On the 1st April of the year when construction is completed
(d) On the 31st March of the year when loan is borrowed

PROBLEM:33 When did pre-acquisition or pre-construction period ends?


(a) 31st march immediately prior to date of completion of construction or acquisition of property
(b) Date of repayment (c) (a) or (b) whichever is earlier (d) Any of these

PROBLEM:34 Which of the following amount is not allowed for deduction from income from house
property?
(a) Interest on loan borrowed for construction of house property.
(b) Interest on fresh loan taken to repay original loan
(c) Interest on unpaid interest (d) Interest on unpaid purchase price

PROBLEM:35 The maximum limit of deduction under section 24(b) for interest on borrowed capital on
or after 1-4-1999 for repairs of house property used for self-occupation is : ……………………

PROBLEM:36 The maximum limit of deduction under section 24 (b) for interest on borrowed capital
before 1-4-1999 for construction of house property used for self-occupation is : …………………………..

SALEEM QURAISHEE Mo: 9175664444 INSPIRE ACADEMY-8888881719


INCOME FROM HOUSE PROPERTY- Page 5.26

PROBLEM:37 The maximum limit of deduction under section 24(b) for interest on borrowed capital on
or after 1-4-1999 for construction of house property used for self-occupation if the house is
completed within 5 years from the end of previous year in which loan is taken is : ………………………..

PROBLEM:38 The maximum limit of deduction under section 24(b) for interest on borrowed capital on
or after 1-4-1999 for construction of house property used for self-occupation shall be Rs. 2,00,000 if
(a) The house is completed within 5 years from the end of previous year in which loan is taken.
(b) Taken house is completed within 5 years from the end of previous year in which construction is
started. (c) The house is completed within 5 years from the date when the loan is taken
(d) The house is completed within 5 years from the date when construction is started.

PROBLEM:39 M took a loan or Rs. 6,00,000 on 1-4-2018 from a bank for construction of a house. The loan
carries an interest @ 12% p.a. The construction is completed on 30-6-2019. The entire loan is still
outstanding. The pre-construction period interest will be Rs. __............................................

PROBLEM:40 R took a loan or Rs. 8, 00,000 on 1-4-2018 from a bank for construction of a house. The loan
carries an interest @ 12% p.a. The construction is completed on 31-03-2019. The entire loan is still
outstanding on 31-03-2020. The pre-construction period interest will be Rs. _________.

PROBLEM:41 K took a loan or Rs. 8, 00,000 on 1-4-2017 from a bank for construction of a house. The loan
carries an interest @ 12% p.a. The construction is completed on 31-03-2019. The entire loan is still
outstanding on 31-03-2020. The pre-construction period interest allowable in Assessment Year 2020-
21 will be……………………………………………..

PROBLEM:42 K took a loan or Rs. 8,00,000 on 1-4-2016 from a bank for construction of a house. The loan
carries an interest @ 12% p.a. The construction is completed on 31-03-2017. The entire loan is still
outstanding on 31-03-2020. The total interest allowable in Assessment Year 2020-21 will
be……………………………………….

SALEEM QURAISHEE Mo: 9175664444 INSPIRE ACADEMY-8888881719


INCOME FROM HOUSE PROPERTY- Page 5.27

PROBLEM:43 T is owner of house which has been let out at a monthly rent of Rs. 25,000. The fair rent of
the house is Rs. 2, 90,000 and standard rent is Rs. 2,60,000. The municipal value of house is Rs.
2,80,000 and municipal taxes are levied @ 10% of municipal value. The entire amount of municipal
taxes for the year ended 31-03-2020 are paid by the owner. The income from house property will
be…………………………………….

PROBLEM:44 T is owner of house which has been let out at a monthly rent of Rs. 30,000. The fair rent of
the house is Rs. 2,90,000 and standard rent is Rs. 2,60,000. The municipal value of house is Rs.
2,80,000 and municipal taxes are levied @10% of municipal value. The entire amount of municipal
taxes for the year ended 31-03-2020 are paid by the owner. Interest on borrowed capital is Rs.
2,10,000 (outstanding). The income from house property will be :………………………………..

PROBLEM:45 T is owner of house which has been let out at a monthly rent of Rs. 30,000. The fair rent of
the house is Rs. 2,90,000 and standard rent is Rs. 2,60,000. The municipal value of house is Rs.
2,80,000 and municipal taxes are levied @10% of municipal value. The entire amount of municipal
taxes for the year ended 31-03-2020 are paid by the owner. Interest on borrowed capital is Rs.
60,000 (outstanding). The income from house property will be ………………………………..

PROBLEM:46 Under which of the following circumstances the income from house property is exempt
from tax –
(a) Farm house (b) Trade Union (c) One self occupied property (d) All of these

PROBLEM:47 If the respective shares of income of co-owners are not definite and ascertainable, the co-

owners shall be assessed as : …………………………………….

PROBLEM:48 The maximum limit of deduction under section 24(b) for interest on borrowed capital on or
after 1-4-1999 for acquisition or construction of such house property is :

SALEEM QURAISHEE Mo: 9175664444 INSPIRE ACADEMY-8888881719


INCOME FROM HOUSE PROPERTY- Page 5.28

PROBLEM:49 Who amongst the following is not a deemed owner?


(a) An individual who transfers his house property otherwise then for adequate consideration to his
or her spouse.
(b) A member of a co-operative society, company or an AOP to whom a building or part thereof is
allotted. (c) The holder of impartible estate of an HUF (d) None of the above

PROBLEM:50 Mr. Kamal had two children Sumit and Sushmita (married with Aman) of age 15 & 17
respectively and wife named Anu. In which of the following case he will not be considered as deemed
owner?
(a) Transfer of property (b) Transfer of property to Sushmita
(c) Transfer of property to Sumit (d) None of the above

PROBLEM:51 What are the conditions to be fulfilled in order to claim exemption of unrealized rent?
(a) The defaulting tenant is in occupation of any other property of the assessee.
(b) Steps have been taken to compel him to vacate the property
(c) The tenancy is bona fide (d) Both (a) and (c)

PROBLEM:52 The net annual value of house let-out is Rs. 1,00,000 and actual amount spent by the
assessee on repairs and insurance premium is Rs. 20,000, the amount of deduction allowed under
section 24(a) shall be Rs. ________.

PROBLEM:53 Which out of the following is not a case of deemed ownership of house property?
(a) Transfer to a spouse for inadequate consideration
(b) Transfer to a minor child for inadequate consideration
(c) Holder of an impartible estate (d) Co-owner of a property

PROBLEM:54 Jagdish, after sale of his house property during August, 2018, received arrears of rent
amounting to Rs. 40,000 on 2nd February, 2020. The said income is chargeable to tax under the head
______ and the taxable income would be Rs. _______.

PROBLEM:55 Megha received Rs. 30,000 as arrears of rent during the previous year 2019-20 The amount
taxable under section 25A would be______.

SALEEM QURAISHEE Mo: 9175664444 INSPIRE ACADEMY-8888881719


INCOME FROM HOUSE PROPERTY- Page 5.29

PROBLEM:56 In case assessee is owner of more that one house which are self-occupied by him, then at
the option of the assessee :
(a) One house shall be treated as self occupied and the other house shall be deemed to let out
(b) One house shall be treated as self occupied and the other house shall be deemed to be vacant
(c) Both the houses shall be treated as deemed to be let out
(d) Both the houses shall be treated as self occupied

PROBLEM:57 Mrs. Preeti owns a house property which is let out @ Rs. 10,000 p.m. During the previous
year ending 31st March 2020, she received –
(i) arrears of rent of Rs. 30,000; and
(ii) unrealized rent of Rs. 20,000.

Compute her income chargeable to tax under the head ‘Income from House Property’.

PROBLEM:58 Sanjeev owns a house property. Following are the details about the property :
Municipal value of house : Rs. 72,000 per annum.
Fair rent of house : Rs. 66,000 per annum.
Standard rent of house : Rs. 60,000 per annum.
The house was let out at Rs. 6,000 per month but was sold on 1st January, 2020. Find out income from
house property for the assessment year 2020-21.

PROBLEM:59 X is the owner of a commercial property let out at Rs. 20,000 p.m. The municipal tax on the property
is Rs. 25,000 annually, 50% of which is payable by the tenant. This tax was actually paid on 15-04-2020. He had
borrowed a sum of Rs. 10 lacs from his cousin, resident in U.S.A. (in dollars) for the construction of the
property on which interest @ 10% is payable. He has also received arrears of rent of Rs. 20,000 during the year,
which was not charged to tax in the earlier years. What is the property income of X for assessment year 2020-
21?

PROBLEM:60 During the financial year 2019-20, Mr. A received a sum of Rs. 1,80,000 (Rs. 60,000 p.a.) by
way of enhancement for the last three years as the Government department (tenant) enhanced the
rate of rent with retrospective effect. The sum of Rs. ____ be taxable in the assessment year 2020-21?

SALEEM QURAISHEE Mo: 9175664444 INSPIRE ACADEMY-8888881719


INCOME FROM HOUSE PROPERTY- Page 5.30

PROBLEM:61 X, an American national, is resident in India during the PY ending on 31-03-2020. He was the
owner of a building located in New York. The same was on rent @ US $ 12,500 p.m. The Municipal
Corporation of New York was paid taxes on such building of US $ 10,000 on 12-2-2020. The value of
one US $ in Indian rupee remained at Rs. 60 throughout the year. X wants to know his taxable income
for house property for assessment year 2020-21.

PROBLEM:62 Where an assessee has two house properties for self-occupation, the benefit of NIL annual
value will be available in respect of
(a) Both the properties
(b) The property which has been acquired/ constructed first
(c) Any one of the properties, at the option of the assessee
(d) Any one of the properties and once option is exercised cannot be changed in subsequent years

PROBLEM:63 R is a member of house building Cooperative Society who is the owner of flats constructed
by it. One of the flats is allotted to R. The income from such house property shall be taxable in the
hands of:
a) Co-operative society b) R as deemed owner c) PGBP d) None of these

PROBLEM:64 R gifted his house property to his wife in 2017. Mrs. R has let out the house property @ Rs
5,000 p.m. The income from such house property will be taxable in the hands of:
a) Mrs. R
b) R. However, income will be first computed as Mrs. R's income and thereafter clubbed in the income
of R
c) R as he will be treated as deemed owner of the house property and liable to tax
d) None of these

PROBLEM:65 R gifted the house property to his minor son which was let out @ Rs5,000 p.m. Income
from such house property shall be taxable in the hands of:
a) Minor son
b) R. However, it will be first computed as minor's income & thereafter clubbed in the income of R
c) R as he will be deemed owner of such house property & liable to tax
d) None of these

PROBLEM:66 R transferred his house property to his wife under an agreement to live apart. Income
from such house property shall be taxable in the hands of:
a) R as deemed owner
b) R. However, it will be first computed as Mrs. R income & thereafter clubbed in the hands of R
provided the income of the father is higher than the income of her mother
c) Mrs. R. d) None of these

SALEEM QURAISHEE Mo: 9175664444 INSPIRE ACADEMY-8888881719


INCOME FROM HOUSE PROPERTY- Page 5.31

PROBLEM:67 R has taken a house property on lease for 15 years from G and let out the same to S.
Income from such house to R shall be taxable as
a) Income under the head other sources b) Income from house property as R is the deemed
owner. c) Income under house property d) none of these
(ii) What shall be the answer if R had taken it on lease for 10 years?

PROBLEM:68 R gifted his house property to his married minor daughter. The income from such house
property shall be taxable in the hands of:
a) R as deemed owner
b) R. However, it will be first computed as minor daughter's income & clubbed in the income of R
provided the income of the father is higher than the income of her mother
c) Income of married minor daughter d) None of these

PROBLEM:69 R is owner of superstructure although the land was taken by him on lease. The income
from such house property shall be taxable under the head:
a) Income from other sources b) income from house property c) PGBP d) Capital
Gains

PROBLEM:70 R has taken a house on rent and sublets the same to G. Income from such house property
shall be taxable under the head:
a) Income from house property b) income from other sources
c) PGBP d) Capital Gains

PROBLEM:71 Tick, from the under mentioned, the cases where annual value can be negative:
a) let out property b) one self occupied property
c) deemed let property
d) one property which could not be occupied due to employment elsewhere
e) partly let out & partly self occupied property

PROBLEM:72 If Annual Value of the house property let is negative then tick the deductions which shall
be allowed u/s 24.
a) All deductions b) No deduction
c) Deduction on account of interest on money borrowed d) None of these

PROBLEM:73 In case of deemed let out property, tick the deductions which shall be allowed under
section 24.
a) Repairs and collection charges b) Insurance premium c) Ground rent/lease rent
d) Annual charge e) Interest on money borrowed
f) Vacancy allowance g) Standard deduction of 30%

PROBLEM:74 A, owned certain vacant land which he had let out to Ryder sports Academy for carrying
out, certain sports activity. He earned Rs. 50,000 per month from such letting out. The income,
which would be taxable under the head “Income from House Property” would be ................
(a) Rs. 6,00,000 (b) Nil (c) Rs. 4,20,000 (d) Rs. 3,00,000

SALEEM QURAISHEE Mo: 9175664444 INSPIRE ACADEMY-8888881719


INCOME FROM HOUSE PROPERTY- Page 5.32

PROBLEM:75 Arun, who has taken a property in Delhi on lease (paying monthly lease Rs. 50,000)
sublets it to Varun, who pays him a rent of Rs. 30,000 p.m. Calculate the amount taxable in the
hands of Arun as “Income from House Property”?
(a) Rs. 5,20,000 (b) Rs. 3,60,000 (c) Rs. 1,20,000 (d) Nil

PROBLEM:76 In which of the following cases, the transferor even though not legal owner of a property,
is deemed to be the owner for the purpose of sections 22 to 26?
(a) Transfer of property to spouse for inadequate consideration
(b) Transfer of property to minor child, except minor married daughter
(c) Both (a) & (b) (d) Neither (a) nor (b)

PROBLEM:77 Ajay’s minor daughter was married to a renowned professional. When she came after
marriage, he transferred his house property to her for Rs. 5,00,000 when the fair value of such
property was Rs. 50,00,000. Who will be considered as the owner of such house property for tax
purpose?
(a) Ajay (b) Minor married daughter (c) Both (a) &(b) equally (d) None of the above

PROBLEM:78 Sanjay, did not had a very understanding wife, and due to prolonged differences of
opinion and fights on a regular basis, they decided to live apart. As a precondition for such an
agreement to live apart, Sanjay transferred his house property to his spouse, which had a fair value
of Rs. 50,00,000. The property yielded a rent of Rs. 3,00,000 after the date of transfer. The income
taxable in the hands of Sanjay on account of such property before any deductions would be
................
(a) Rs. 3,00,000 ( b) Nil (c) Rs. 1,50,000 (d) Rs. 2,10,000

PROBLEM:79 MR C had a very understanding wife and due to love and affection he transferred his
house property to his spouse, for Rs. 15,00,000 when the fair value of such property was Rs.
50,00,000. The property yielded a rent of Rs. 6,00,000 after the date of transfer. The income
taxable in the hands of C’s wife on account of such property before any deductions would be
................
(a) Rs. 3,00,000 (b) Nil (c) Rs. 1,50,000 (d) Rs. 6,00,000

PROBLEM:80 R owns a flat in a co-operative society, which she has let out to X for a monthly rental of
Rs. 30,000 for the entire year. In such a case the amount of income taxable in the hands of R before
any deductions would be ................
(a) Rs. 2,40,000
(b) Nil, since she is not the owner of the flat and is in the name of the society itself
(c) Rs. 1,20,000 (d) Rs. 3,60,000

PROBLEM:81 A, has leasing rights in respect of a property for a period of 99 years. He derives certain
rental income from such property amounting Rs. 6,00,000. Income which is chargeable to tax under
the head “Income from House Property” in the case of A before any deductions would be ................
(a) Rs. 6,00,000 (b) Nil (c) Rs. 4,20,000 (d) Rs. 3,00,000

SALEEM QURAISHEE Mo: 9175664444 INSPIRE ACADEMY-8888881719


INCOME FROM HOUSE PROPERTY- Page 5.33

PROBLEM:82 N, has leasing rights in respect of a property for a total period of 11 years, but renewed
every time after a period 11 months. She derives certain rental income from such property
amounting Rs. 3,00,000. Income which is chargeable to tax under the head "Income from House
Property” in the case of N before any deductions would be ................
(a) Rs. 6,00,000 (b) Nil (c) Rs. 4,20,000 (d) Rs. 3,00,000

PROBLEM:83 Ankit , who is carrying on business has a property situated in PUNE. He lets out such
property to PUNE police department to locate their branch in order to run his business more
efficiently. The rental income from such let out property would be taxed in the hands of Aman as
................
(a) Income from House Property (b) Income from PGBP
(c) Income from Capital Gain (d) Income from Other Sources

PROBLEM:84 Shyam is engaged in the business of letting out residential flats to various tenants on a
monthly basis. He earns a monthly income of Rs. 6,00,000 from such flats. Income which is
chargeable to tax under the head “Income from House Property” in the case of Shyam before any
deductions would be ................
(a) Rs. Rs. 2,00,000
(b) Nil, as it is income from profits and gains of Business and profession
(c) Rs. 60,00,000 (d) Rs. 65,00,000

PROBLEM:85 Shyam owns a hotel building, which he has let out to XY Marriott on a monthly basis. He
earns a monthly income of Rs. 15,00,000 from such letting out. Income which is chargeable to tax
under the head “Income from House Property” in the case of Shyam before any deductions would be
................
(a) Rs. 1,80,00,000
(b) Nil, as it is income from profits and gains of business and Profession.
(c) Rs. 1,50,00,000 (d) Rs. 1,65,00,000

PROBLEM:86 Mohan owns a hotel building, which he is used to run a hotel in which various people
come from time to time and stay. He earns a total receipt of Rs. 1,50,00,000 from such letting out
and a net income of Rs. 15,00,000. Income which is chargeable to tax under the head “Income from
House Property” in the case of Mohan before any deductions would be ................
(a) Rs. 15,00,000
(b) Nil, as it is income from Profits and Gains of Business and Profession
(c) Rs. 1,50,00,000 (d) Rs. 1,65,00,000

PROBLEM:87 SANDEEP, who is carrying on business has a property situated in PUNE. He provides such
property to his employees as a Guest House in order to run his business more efficiently for which
he earns a discounted rent of Rs. 1,80,000 per year. The rental income from such let out property
would be taxed in the hands of SANDEEP as
(a) Income from House Property (b) Income from PGBP
(c) Income from Capital Gain (d) Income from Other Sources

SALEEM QURAISHEE Mo: 9175664444 INSPIRE ACADEMY-8888881719


INCOME FROM HOUSE PROPERTY- Page 5.34

PROBLEM:88 SHIVA has an office space which was furnished with Furniture. He had let out the entire
office to MAYUR and Co. a firm of Chartered Accountants for a rent of Rs. 20,000 per month. The
letting of the office space without furniture is not possible. In such a case, income taxable under the
head “Income from House Property” in the case of SHIVA before any deductions would be
................
(a) Rs. 1,80,000
(b) Nil, as it is income from Profits and Gains of Business and profession or Income from Other
Sources.
(c) Rs. 1,20,000 (d) Rs. 3,00,000

PROBLEM:89 When the income from letting of the property is inseparable from letting of assets,
................
(a) Entire income will be taxable as Income from House Property
(b) Entire income will be taxable as income from PGBP
(c) Part of the income will be taxable as Income from House Property and part of the income will be
taxable as income from PGBP
(d) Entire income will be taxable as income from PGBP or Income from Other Sources, as the case
may be

PROBLEM:90 X Ltd. has let out house property to a tenant and also charged for certain other services
provided along with the property. The entire rent received by X Ltd. is collectively known as
................
(a) Combined Rent (b) Collective Rent (c) Total Rent (d) Composite Rent

PROBLEM:91 In case of composite rent, the portion of rent attributable to rent of property is chargeable
under the head ................ and the portion of rent attributable to rent of other facilities is
chargeable under the head ................
(a) Income from House Property, Income from PGBP
(b) Income from PGBP, Income from House Property
(c) Income from House Property, Income from Other Sources
(d) Income from Other Sources, Income from House Property

PROBLEM:92 G, owns an agricultural land which also has a small dwelling unit like a Farm House, which
he has let out. In such a case rent of Rs. 1,20,000 received for such let out is ................
(a) Exempt from tax (b) Chargeable to tax as Income from House Property
(c) Chargeable to tax as Income from PGBP
(d) Chargeable to tax as Income from Other Sources

PROBLEM:93 Annual Value of one palace of an ex-Ruler will be taken as ................


(a) The actual value of rent received
(b) The value of similar property in the same locality
(c) The value of similar property in the same locality less taxes paid, if any (d) Nil

SALEEM QURAISHEE Mo: 9175664444 INSPIRE ACADEMY-8888881719


INCOME FROM HOUSE PROPERTY- Page 5.35

PROBLEM:94 Manjeet lets out an owned house to Amaan , for a rent of Rs. 25,000 per month. The
municipal taxes due on the property amounted to Rs. 10,000 out of which taxes amounting Rs.
5,000 were actually paid by Manjeet during the financial year. He also paid municipal taxes of Rs.
15,000 of earlier years during the current previous year. The deduction for Municipal taxes in such a
case would be ................
(a) Rs. 5,000 (b) Rs. 10,000 (c) Rs. 20,000 (d) Rs. 18,000

PROBLEM:95 S owns a house property in Australia on which she earns a rent of Rs. 25,000 per month.
The Municipal taxes due on the property amounted to Rs. 10,000. The deduction for Municipal
taxes paid in foreign country for Indian tax purpose would be ................
(a) Allowed as a deduction in full (b) Not allowed as a deduction
(c) 50% would be allowed as a deduction (d) None of the above

PROBLEM:96 Anil, took a loan of Rs. 20,00,000 at an interest rate of 10% p.a. He paid half of the
interest due during the current year while the remaining half remained unpaid. The deduction for
interest on loan in case of self-occupied property would be ................
(a) Rs. 1,00,000 (b) Rs. 2,00,000 (c) Rs. 2,50,000 (d) Rs. 2,40,000

PROBLEM:97 Shyam received Rs. 45,000 as arrears of rent from his tenant in August 2019, for the
property which he had sold to Ram in January 2019. Compute the amount of rent chargeable to tax
in the hands of Shyam during A.Y. 2020-21, as per section 25A, under the head “Income from House
Property”?
(a) Rs. 45,000 (b) Rs. 31,500 (c) Rs. 22,500

PROBLEM:98 Any amount paid for brokerage or commission for arrangement of loan ................ be
allowed as deduction in computing Income from House Property?
(a) Will (b) Will not (c) Either (a) or (b) (d) None of the above

PROBLEM:99 The Fair Rent & Municipal Value of a house property is Rs. 2,00,000 & Rs. 2,20,000
respectively. The Standard Rent of the same house property is Rs. 2,10,000. What will the Expected
Rent?
(a) Rs. 2,00,000 (b) Rs. 2,10,000 (c) Rs. 2,20,000 (d) None of the above

PROBLEM:100 Treatment of unrealized rent for determining income from house property
(a) To be deducted from expected rent (b) To be deducted from actual rent
(c) To be deducted under section 24 from annual value
(d) To be deducted from both expected rent and actual rent

PROBLEM:101 Mr. J received Rs. 30,000 as arrears of rent during the AY 2020-2021 i.e. PY 2019-
2020. The amount taxable under section 25A would be—

SALEEM QURAISHEE Mo: 9175664444 INSPIRE ACADEMY-8888881719


INCOME FROM HOUSE PROPERTY- Page 5.36

PROBLEM:102 Mr. Jay received Rs. 90,000 in May, 2019 towards recovery of unrealized rent, which
was deducted from actual rent during the PY 2017-2018 for determining annual value. Legal expense
incurred in relation to unrealized rent is Rs. 20,000. The amount taxable under section 25A for AY
2020-2021 i.e. PY 2019-2020 would be

PROBLEM:103 Mr. J and Mr. S are co-owners of a self- occupied property. They own 50% share
each. The interest paid by each co-owner during the previous year on loan (taken for acquisition of
property during the year 2005) is Rs. 2,05,000 each. The amount of allowable deduction in respect of
each co-owner is—

PROBLEM:104 Mr. J was allowed deduction of unrealized rent to the extent of Rs. 40,000 in the past
when unrealized rent was also Rs. 40,000. He is able to recover from the tenant Rs. 35,000 as full and
final settlement during the AY 2020-2021 i.e. PY 2019-2020. He is liable for tax on:

PROBLEM:105 Mr. KARAN borrowed Rs. 5,00,000 @ 12% p.a. on 1/4/2015 for construction of
house property which was completed on 2/4/2019. The amount of loan is still unpaid. What will be
the deduction of interest for AY 2020-2021 i.e. PY 2019-2020 if the house property is let out:

PROBLEM:106 Mr. ANIL borrowed Rs. 5,00,000 @ 12% p.a. on 1/4/2015 for construction of house
property which was completed on 2/4/2019. The amount of loan is still unpaid. What will be the
deduction of interest for AY 2020-2021 i.e. PY 2019-2020 if the house property is self-occupied:

PROBLEM:107 Interest of borrowed capital from outside India is deductible while calculating
Income from HP if condition given under section.
(a) 25B is satisfied (b) 22 is satisfied (c) 25 is satisfied (d) 25A is satisfied

PROBLEM:108 Mr. J took loan from a bank for Rs. 10,00,000 on 1/11/2016 @ 8% pa for the
construction of the house which is self occupied. Construction of the house got completed on
15/3/2020. Compute interest allowed as deduction u/s 24(b) for AY 2020-2021 i.e. PY 2019-2020.
ANSWER:108

PROBLEM:109 Mr. SANTOSH took loan from a bank for Rs. 1,00,000 on 1/1/2016 @ 12% pa for the
construction of the house which is self occupied. Construction of the house got completed on
1/1/2020 and full amount of loan was paid back on 31/1/2019. Compute interest allowed as
deduction under section 24(b) for AY 2020-2021 i.e. PY 2019-2020.
ANSWER:109

SALEEM QURAISHEE Mo: 9175664444 INSPIRE ACADEMY-8888881719


INCOME FROM HOUSE PROPERTY- Page 5.37

PROBLEM:110 Mr. MANOJ is owner of one self-occupied house property in Mumbai for his
residence. Fair rent of that property is Rs. 5,06,000 per annum. Municipal valuation is Rs. 5,88,000.
Municipal taxes paid are Rs. 50,000 including Rs. 1,000 for an earlier year. The house was constructed
in December, 2011 with a loan of Rs. 12,00,000 from SBI taken in November. 2010. During the AY
2020-2021 i.e. PY 2019-2020 Mr. MANOJ paid back Rs. 2,30,000 which includes Rs. 1,68,000 as
current year interest. Compute the income from house property income for the AY 2020-2021 i.e. PY
2019-2020
ANSWER:110

PROBLEM:111 Mr. ANKUSH owns two house properties. First property was used half for running his
business and the other half was let-out at Rs. 40,000 per month. The second property was wholly
used as a residence by Mr. ANKUSH Municipal value of the two properties was the same at Rs.
7,20,000 each per annum and municipal taxes paid are 10%. Mr. ANKUSH ’s income from house
property for the AY 2020-2021 i.e. PY 2019-2020 will be………………………

SALEEM QURAISHEE Mo: 9175664444 INSPIRE ACADEMY-8888881719


PROFITS AND GAINS OF BUSINESS OR PROFESSION- Page 6.1

SECTION 28-44DA
1. CHARGING SECTION [SEC. 28]
Under section 28, the following income is chargeable to tax under the head "Profits and gains of
business or profession":
I. Profits and gains of any business or profession;

II. Compensation due or received by:


a) For termination/modification of agreement for managing a company.
b) For termination/modification of terms of agency.
c) For vesting of management of any property or Business in government under any law.
d) Any person in connection with the termination or the modification of the terms and conditions, of
any contract relating to his business

III. Income derived by a trade, professional or similar association from specific services performed
for its members;

IV. The value of any benefit or perquisite, whether convertible into money or not, arising from
business or the exercise of a profession;

V. Export Incentives which includes-


 Any profit on transfer of the Duty Entitlement Pass Book Scheme;
 any profit on the transfer of the duty free replenishment certificate;
 export incentive
 Duty drawback of Customs and central excise

VI. Any interest, salary, bonus, commission or remuneration received by a partner from firm

VII. Any sum received for not carrying out any activity in relation to any business or not to share any
know-how, patent, copyright, trademark, etc.

VIII. Non-compete fee received/ receivable (which is of recurring nature) in relation to not carrying
out any profession is brought within the scope of section 28 [Amendment vide Finance Act,
2016 w.e.f. AY 2017-18].

IX. Any sum received under a Keyman insurance policy including bonus;

SALEEM QURAISHEE Mo: 9175664444 INSPIRE ACADEMY-8888881719


PROFITS AND GAINS OF BUSINESS OR PROFESSION- Page 6.2

X. Profits or gains arising from conversion of inventory into capital asset or its treatment as
capital asset shall be charged as business income. W.e.f. 01.04.2019
a) Any profit or gains arising from conversion of inventory into capital asset or its treatment
as capital asset shall be charged to tax as business income.
b) FMV of the inventory on the date of conversion/ treatment determined in the prescribed
manner, shall be deemed to be the full value of the consideration received or accruing as
a result of such conversion or treatment.

XI. Any sum received (or receivable) in cash or kind, on account of any capital asset (other than land
or goodwill or financial instrument) being demolished, destroyed, discarded or transferred, if the
whole of the expenditure on such capital asset has been allowed as a deduction under section
35AD; and

XII. Income from speculative transaction

XIII. Income from the aforesaid activities is computed in accordance with the provisions laid down in
sections 29 to 44DB.

2. BUSINESS INCLUDES-

3. BUSINESS INCOME NOT TAXABLE UNDER THE HEAD "PGBP"


I. Rental income in case of a dealer in house property.
II. Dividend on shares in case of a dealer in shares.
III. Winning from lotteries.
IV. Interest received on compensation or enhanced compensation.

4. BASIC PRINCIPLES FOR ARRIVING AT BUSINESS INCOME-


I. Business or profession should be carried on by the assessee.
II. Business or profession should be carried on during the previous year -
III. Income of previous year is taxable during the following assessment year -
IV. Tax incidence arises in respect of all businesses or professions -
V. Legal ownership vs. beneficial ownership -
VI. Real profit vs. Anticipated profit –
VII. Real profit vs. Notional profit –
VIII. Recovery of sum already allowed as deduction.
IX. Mode of book entries not relevant.

SALEEM QURAISHEE Mo: 9175664444 INSPIRE ACADEMY-8888881719


PROFITS AND GAINS OF BUSINESS OR PROFESSION- Page 6.3

5. TRADING LOSS-
I. Losses should be revenue in nature
II. Losses should be incurred during the previous year.
III. Losses should be incidental to the business or profession carried on the assessee.
IV. It should not be notional or fictitious
V. It should have been actually incurred and not merely anticipated to incur in future.
VI. There should not be any, direct or indirect, restriction under the Act against the deductibility of such
loss

6. WHAT IS THE SCHEME OF BUSINESS DEDUCTIONS/ALLOWANCES?


I. It is the responsibility of the assessee to prove that a particular deduction is admissible in his
case.
II. Allowances are cumulative.
III. Expenditure should relate to the previous year.
IV. Business should be carried on during the previous year.
V. Expenditure should have been incurred in connection with assessee business.
VI. Benefit of expenditure may extend to somebody else –
VII. Benefit of expenditure may extend beyond the relevant previous year-
VIII. No allowance in respect of exhaustion of wasting assets -
IX. No allowance in respect of expenditure Incurred before the setting up of a business -
X. No allowance in respect of non-assessable business.
XI. No allowance in respect of anticipated losses.
XII. No deduction in respect of depreciation of investment -
XIII. Relevance of distinction between capital and revenue expenditure –

7. HOW TO COMPUTE ASSESSABLE PROFIT OR LOSS FROM BUSINES OR


PROFESSION.

SALEEM QURAISHEE Mo: 9175664444 INSPIRE ACADEMY-8888881719


PROFITS AND GAINS OF BUSINESS OR PROFESSION- Page 6.4

8. METHOD OF ACCOUNTING [SECTION 145]

I. The profits from business and profession and income under the head 'Income from Other Sources'
are to be computed in accordance with the method of accounting regularly employed by the
assessee.
II. As per section 145 of the Income tax Act only one of the following two methods of accounting can
be followed:
(a) Mercantile system
(b) Cash system.
III. The income chargeable under the head PGBP/OS shall be computed in accordance with INCOME
COMPUTATION AND DISCLOSURE STANDARDS to be notified by the Central Government from
time to time
IV. If income has not been computed as per above said ICDS the AO may make assessment in the manner
provided under section 144.
V. These standards will not be applicable for maintenance of books of account but shall be relevant for
computation of total income and disclosure of information in the return.

INCOME COMPUTATION AND DISCLOSURE STANDARDS-


1) Section 145(2) empowers the Central Government to notify in the Official Gazette from time to time
income computation and disclosure standards to be followed by any class of assessee or in
respect of any class of income.

2) APPLICABILITY-
i. All assessee
ii. Following mercantile system of accounting.
iii. Having income under the Head PGBP/OS
NOT APPLICABLE-
i. I/HUF (Tax audit not compulsory)
ii. Persons following cash system

3) The ten notified ICDSs are:


ICDS I : Accounting Policies
ICDS II : Valuation of Inventories
ICDS III : Construction Contracts
ICDS IV : Revenue Recognition
ICDS V : Tangible Fixed Assets
ICDS VI : The Effects of Changes in Foreign Exchange Rates
ICDS VII : Government Grants
ICDS VIII : Securities
ICDS IX : Borrowing Costs
ICDS X : Provisions, Contingent Liabilities and Contingent Assets

SALEEM QURAISHEE Mo: 9175664444 INSPIRE ACADEMY-8888881719


PROFITS AND GAINS OF BUSINESS OR PROFESSION- Page 6.5

 Cardinal features of Notified ICDSs


1) Applicability: All the notified ICDSs are applicable for computation of income chargeable under the
head “Profits and gains of business or profession” or “Income from other sources” and not for the
purpose of maintenance of books of accounts. This is stated in the Preamble at the beginning of
each ICDS.
2) Position in case of conflict with the Income--tax Act, 1961: In the case of conflict between the
provisions of the Income tax Act, 1961 and the notified ICDSs, the provisions of the Act shall prevail
to that extent. This is also stated in the Preamble at the beginning of each ICDS.

3) Scope Paragraph: Each of the ten notified ICDSs has a scope paragraph explaining what exactly the
ICDS deals with. In some standards, the scope paragraph also specifies what the ICDS does not deal
with.
4) Transitional Provisions: All ICDSs (except ICDS VIII on Securities) contain transitional provisions to
facilitate first time adoption and prevent any tax leakage or any double taxation.

5) Disclosure Requirements: All ICDSs (except ICDS VI on Effects of changes in foreign exchange rates
and ICDS VIII on Securities) contain specific disclosure requirements. The last paragraph(s) of these
ICDSs is on disclosure.

Method of Accounting in certain cases: [Sec. 145A]


a) Applicability: For determining the income chargeable under the head “Profits and gains of business
or profession
b) Valuation as prescribed under ICDS notified u/s 145(2):
Valuation of Treatment
I. Inventory Lower of Actual Cost or NRV, computed as
per ICDS
II. Inventory being- Actual Cost initially recognized as per ICDS
a) Unlisted Securities, or
b) Listed Securities , but not quoted on a recognized
stock exchange with regularity from time to time
III. Inventory being Securities other than those Lower of Actual Cost or NRV, computed as
referred above per ICDS
c) Valuation – Inclusive of Taxes: Valuation of Purchase and Sale of Goods or Services, and of
Inventory, shall be adjusted to include the amount of any tax, duty, cess or fee (by whatever name
called) actually paid or incurred by the assessee to bring the goods or services to the place of its
location and condition as on the date of valuation.

d) For the purposes of this section, any tax, duty, cess or fee (by whatever name called) under any law
for the time being in force, shall include all such payment notwithstanding any right arising as a
consequence to such payment.

SALEEM QURAISHEE Mo: 9175664444 INSPIRE ACADEMY-8888881719


PROFITS AND GAINS OF BUSINESS OR PROFESSION- Page 6.6

4. Taxability of Certain Income-Sec 145B


Nature of Income Deemed to be income in -
I. Interest received on any Compensation or on The PY in which it is received.
Enhanced compensation
II. Claim for escalation of price in a contract or Export The PY in which reasonable certainty of
Incentives its realization is achieved.
III. Income being Subsidy or grant or Cash Incentive or The PY in which it is received, if not charged to
Duty Drawback or waiver or concession or income-tax in any earlier PY.
reimbursement as referred u/s 2(24) (xviii)

PROBLEM:1 “Business" under Income-tax Act, does not include ................


(a) Any trade (b) Commerce
(c) Manufacture (d) Adventure or concern in the nature of leisure or sport

PROBLEM:2 Income from adventure or concern in the nature of trade, commerce or manufacture is
taxable under which of the following heads of Income?
(a) Profits and gains of business or profession (b) Capital gains
(c) Salaries (d) Income from other sources

PROBLEM:3 Profession involves an occupation requiring ................


(a) Purely intellectual skill (b) Manual skill based on special learning
(c) Both (a) and (b) (d) None of the above

PROBLEM:4 Vocation refers to an activity on which a person spends major part of his time to earn
................
(a) Respect (b) Esteem (c) Professional satisfaction (d) Livelihood

PROBLEM:5 Which of the following activities are taxable under the head ‘Profits and gains of business
or profession’?
(a) Business (b) Profession (c) Vocation (d) All of the above

PROBLEM:6 Ranjan started his business on October 1,2018 For FY 2018-19, he earned Rs. 3,00,000.
He is under impression that this income would be taxable under Income from other sources, since
the business has been in existence for only part of the year. You are required to suggest which of the
following is correct?
(a) "Profits and gains of business or profession”, since income from business, even if it is carried on
during part of the year is also taxable under this Head
(b) Income from other sources, since it is carried on for only part of the year
(c) Either of the above (d) None of the above

SALEEM QURAISHEE Mo: 9175664444 INSPIRE ACADEMY-8888881719


PROFITS AND GAINS OF BUSINESS OR PROFESSION- Page 6.7

PROBLEM:7 ANIL was employed as managing director of Indian company. In FY 2019-20 his services
are terminated and he received Rs. 5 lakhs as compensation. Such amount would be ................
(a) Taxable under head Salaries (b) Taxable as Income from other sources
(c) Taxable under head PGBP (d) None of the above

PROBLEM:8 Aman is the distributor of XYZ Co. for entire Northern Region. During FY 2019-20 company
has modified the contract of Aman’s agency and in future Aman will be the distributor only for Delhi,
the company paid Rs. 10 lakhs as compensation. Such compensation would be ................
(a) Taxable under head IFOS (b) Taxable under head PGBP
(c) Taxable under head Capital Gain (d) None of the above

PROBLEM:9 PUNE club is a trade organization which rendered certain services to its members. Under
which head would such income be taxable?
(a) Under head PGBP (a) Income from other sources
(b) Capital gains (c) Exempt from tax

PROBLEM:10 Which of the following compensation is not taxable under “Profits and Gains of Business
or Profession”?
(a) Modification of managing agreement for whole of affairs of Indian company
(b) Termination of terms of agency agreement in India, for any part of the activities relating to
business of any other person
(c) Vesting with the Government, management of any property under any law for the time being in
force.
(d) Modification of terms of employment of a key employee in the company

PROBLEM:11 Export incentives taxable under PGBP excludes ................


(a) Profit on sale of import licenses granted under Imports (Control) Order on account of exports
(b) Cash assistance receivable against exports
(c) Subsidy on purchase of new plant and machinery for setting up new plant
(d) Profit on transfer of Duty Entitlement Pass book

PROBLEM:12 X is a lawyer providing services to ABC Co. He went to Chandigarh to argue the case for
the company and was reimbursed by ABC Co. for stay and travel expenses. What would be the tax
treatment for reimbursement received by Mr. X?
(a) It is taxed as income from other sources
(b) Its taxed as a perquisite arising during carrying on of any business or profession
(c) Not taxable under any Head
(d) Should be taxed as income from profession under other sources

PROBLEM:13 Which of the following are taxable under the head PGBP?
(a) Interest received by a partner from a firm (b) Salary & Bonus received by a partner from a firm
(c) Commission received by a partner from a firm (d) All of the above

SALEEM QURAISHEE Mo: 9175664444 INSPIRE ACADEMY-8888881719


PROFITS AND GAINS OF BUSINESS OR PROFESSION- Page 6.8

PROBLEM:14 A Inc. purchased the India rice business from company C and in the process, agreed with
the promoters of C that they would not carry out rice business. Such payment for not carrying rice
business would be taxable under which head?
(a) Income from Capital Gains (b) Profits and Gains of Business and Profession
(c) Salaries (d) Income from Other Sources

PROBLEM:15 S Ltd. had taken a keyman insurance policy on its MD, Mr. M who left the company on
April 1, 2019. S Ltd. received a sum of R. 40, 00,000 from the insurance company on such policy and
Rs. 10,00,000 as bonus thereon. Such receipt would be taxable as ................
(a) Rs. 40,00,000 under PGBP (b) Rs. 50,00,000 under PGBP
(c) Rs. 40,00,000 under PGBP and Rs. 10,00,000 under Income from Other Sources
(d) Rs. 10,00,000 under Income from Other Sources

PROBLEM:16 Samrat estimates that his business is going to incur loss in next year due to adverse
market condition. Therefore, he creates a provision for anticipated loss and debited Rs. 10 lakhs in
P&L Account such provision would be ................
(a) Disallowed as it is anticipated loss (b) Allowed as an expense
(c) Allowed as an expense in computation of IFOS (d) None of the above

PROBLEM:17 Which of the following are allowed as expense in computation of income under head
PGBP?
(a) Loss on embezzlement made by employee
(b) Non-recovery of advance money paid to supplier who does not supply raw material
(c) Loss by failure of bank in which money is deposited (d) All of the above

PROBLEM:18 Which of the following losses are not allowed as a deduction, in computing income from
Business and Profession?
(a) Pre-incorporation losses
(b) Capital losses, which are dealt with under the head Capital Gains
(c) Losses incurred after closure of business (d) All of the above

PROBLEM:19 Which of the following are not allowed as expenses in computation of income under head
PGBP?
(a) Loss of stock in trade due to enemy action
(b) Loss on account of robbery in the course of business
(c) Loss on account of fluctuation in exchange rate at time of remitting money for purchase of raw
material (d) Loss on sale of security held as investment

PROBLEM:20 In computing income under the head “Profits and Gains of Business or Profession”,
income from which of the following business is taxable?
(a) Legal business (b) Illegal business (c) Both legal and illegal business (d) None of the above

SALEEM QURAISHEE Mo: 9175664444 INSPIRE ACADEMY-8888881719


PROFITS AND GAINS OF BUSINESS OR PROFESSION- Page 6.9

PROBLEM:21 Under which provision of Income-tax Act, income under the head PGBP are computed?
(a) Sections 30 to 43D (b) Sections 44 to 44D (c) Sections 56 to 59 (d) Both (a) and (b)

PROBLEM:22 Which principle is incorrect for allowability of an expenditure in computation of income


under head PGBP?
(a) Expenditure should be incurred during the previous year
(b) Expenditure should be incurred for business purpose
(c) Expenses incurred before the setting up of a business are allowed
(d) No deduction is allowable in respect of a discontinued business

PROBLEM:23 A business foresees, certain losses which are likely to be incurred in future. Which of the
following is allowed as a deduction in computing business income?
(a) Provision for contingencies (b) Anticipated losses
(c) Reserves for contingencies (d) None of the above

PROBLEM:24 For tax purpose, which are the prescribed method of Accounting?
(a) Cash (b) Mercantile
(c) Either cash or mercantile, regularly employed by assessee (d) Both cash and mercantile

PROBLEM:25 Under which provision of Income-tax Act, the Central Government is empowered to notify
Income Computation and Disclosure Standards?
(a) Section 145(2) (b) Section 145(3) (c) Section 144 (d) None of the above

PROBLEM:26 X Limited filed its return for AY 2019-20. In which of the following cases AO is empowered
to make assessment u/s 144?
(a) When AO is not satisfied about correctness of accounts
(b) Where accounting method provided in income-tax are not regularly followed
(c) Where income has not been computed in accordance with ICDS (d) All of the above

PROBLEM:27 In which of following cases, the provision of ICDS do not apply?


(a) Assessee has no income under the head PGBP and IFOS
(b) Assessee follows cash system of accounting
(c) Assessee, being an individual or HUF, is not required to get his account audited u/s 44AB
(d) All of the above

PROBLEM:28 Income Computation and Disclosure Standards are not meant for ................
(a) Maintenance of books of accounts (b) Computing income under PGBP
(c) Computing income under IFOS (d) All of the above

PROBLEM:29 When a conflict arises between provision contained in ICDS and Income-tax Rules, 1962,
which provisions shall prevail?
(a) Income-tax Rules, 1962 (b) ICDS (c) Judiciary Case Laws (d) None of the above

SALEEM QURAISHEE Mo: 9175664444 INSPIRE ACADEMY-8888881719


PROFITS AND GAINS OF BUSINESS OR PROFESSION- Page 6.10

PROBLEM:30 Which of the following statement is correct?


(a) ICDS shall apply for computation of income under head PGBP or IFOS, irrespective of whether
assessee follows Accounting Standard or Ind-AS
(b) ICDS apply when individual is not required to get his accounts audited
(c) ICDS are used for maintenance of books of accounts (d) None of the above

PROBLEM:31 Income under the head “Profits and gains of business or profession” would tax ................
(a) Income from only one business of assessee (b) Income from all the business of assessee
(c) Only income from one of the profitable business of the assessee (d) None of the above

PROBLEM:32 In making a computation of taxable income, if the profit and loss account of assessee
shows a profit, any expenses which has not been debited to the profit and loss account but is
allowable as a deduction for income tax purpose would be ................
(a) Added to the profit and loss credit balance (b) Reduced from the profit and loss balance
(c) No impact since its not debited to the profit and loss account (d) Allowed 50%

PROBLEM:33 In making a computation of taxable income, if the profit and Loss account of assessee
shows a loss, any expenses which has not been debited to the profit and loss account but is
allowable as a deduction for income tax purpose would ................
(a) Increase the loss (b) Reduce the profit
(c) No impact since its not debited to the profit and loss account (d) Allowed 50%

PROBLEM:34 Incomes or receipts which are taxable under PGBP, but have not been credited to P&L
Account, are to be ................
(a) Added to the profit and loss balance, when P&L shows a credit balance
(b) Reduced from the profit and loss balance, when P&L shows a debit balance
(c) No impact (d) Both (a) and (b)

PROBLEM:35 While computing income under the head “Profits and gains of business or profession”,
expenses, which are excessive/losses not allowable under PGBP, debited to P&L Account are to be
................
(a) Reduced from Profit and Loss Account balance, when P&L shows a credit balance
(b) Reduced from the profit and Loss Account balance when P&L shows a debit balance
(c) Added to the Profit and Loss Account balance when P&L shows a debit balance
(d) No impact

PROBLEM:36 While computing income under the head “Profits and gains of business or profession”,
expenses, which are not allowable, but are debited to P&L Account are ................
(a) Added to the profit and loss Account balance when P&L shows a credit balance
(b) Reduced from the profit and loss account balance, when P&L shows a Credit Balance
(c) No impact (d) Both (a) and (b)

SALEEM QURAISHEE Mo: 9175664444 INSPIRE ACADEMY-8888881719


PROFITS AND GAINS OF BUSINESS OR PROFESSION- Page 6.11

9. SPECIFIC DEDUCTIONS UNDER THE ACT-

10. RENT, RATES, TAXES, REPAIRS AND INSURANCE FOR BUILDING [SEC.
30]-
The following expenses in respect of premises are allowed as deduction u/s 30. Such expenses are
allowed even if the assessee is not the owner of the premises provided such premises are used for the
purposes of business/profession:
I. Rent charges paid by the tenant (where the building is owned by the assessee, notional rent in
respect of such building is not allowed to be deducted)
II. Revenue expenses on repairs
III. Insurance premium relating to the premises
IV. Municipal taxes, land revenues, local rate, etc. (subject to provisions of Section 43B)
Point to be noted-
I. Capital expenditure on repairs incurred by the owner shall be added to the cost of building.
II. Capital expenditure on repairs incurred by the tenant is deemed as building in the hands of the
tenant on which depreciation is allowed to the tenant.

11. REPAIRS AND INSURANCE OF MACHINERY, PLANT AND FURNITURE


[SEC. 31] –
The following expenses in respect of plant & machinery and furniture & fixtures are allowed as
deduction u/s 31 provided these assets are used for the purposes of business/profession. The assessee
need not be the owner of the assets:
I. Revenue expenses on repairs
II. Insurance premium relating to the asset
Point to be noted-
I. Capital expenditure on repairs incurred by the owner shall be added to the cost of the asset.

SALEEM QURAISHEE Mo: 9175664444 INSPIRE ACADEMY-8888881719


PROFITS AND GAINS OF BUSINESS OR PROFESSION- Page 6.12

12. DEPRECIATION ALLOWANCE [SEC. 32]-


A. CONDITIONS FOR CLAIMING DEPRECIATION
I. Asset must be owned by the assessee
II. It must be used for the purpose of business or profession
III. It should be used during the relevant previous year.
IV. Depreciation is available on tangible as well as intangible assets

POINT TO BE NOTED-
1) The asset should be owned by the assessee or the assessee should be the co-owner of the asset.
2) It is not necessary that the assessee should be the registered owner of the asset.
3) Where an assessee carries on a business or profession in a building not owned by him but in respect of
which he holds a lease or right of occupancy, he is entitled to depreciation, in respect of capital
expenditure incurred by him on construction of any structure or any work in relation to the building by
way of improvement, renovation or extension.
4) In the case of financial lease, the lessee can claim depreciation.
5) Depreciation in the case of hire purchase - depreciation will be available to the hirer (and not seller)
from the year in which the asset is taken on hire.

B. ASSET

C. BLOCK OFASSETS [SEC. 2(11)] - The term "Block of Assets" means a group of assets falling within a
class of assets comprising:
I. tangible assets, being buildings, machinery, plant or furniture;
II. intangible assets, being know-how, patents, copyrights, trademarks, licenses, franchises or any
other business or commercial rights of similar nature, in respect of which the same percentage of
depreciation is prescribed.

SALEEM QURAISHEE Mo: 9175664444 INSPIRE ACADEMY-8888881719


PROFITS AND GAINS OF BUSINESS OR PROFESSION- Page 6.13

D. RATES OF DEPRECIATION-
S.NO ASSET RATES
1) BUILDING-
a. Building residential 5%
b. Building Commercial/factory/shop/Road 10%
c. Temporary structure 40%

2) PLANT AND MACHINERY


a. General 15%

b. Car/Buses/lorries used in the business of running them on hire 30%

c. Motor buses, motor lorries and motor taxis used in a business of running
them on hire, acquired on or after the 23/8/2019 but before the 1/4/2020 45%
and is put to use before 1/4/2020 (amended from AY 2020-2021)
15%
d. Other cars (both Indian or imported)

e. Motor cars, other than those used in a business of running them on hire,
acquired on or after the 23rd day of August, 2019 but before the 1st day of
April, 2020 and is put to use before the 1st day of April, 2020 (amended from 30%
AY 2020-2021)

f. Books owned by assessee carrying on a profession/Books owned by a


library/Books being annual publication/ All Other Books 40%

g. Computers and computer software (including computer peripherals) 40%

h. Pollution control equipment/Energy saving devices, energy renewal devices,


40%
including Windmill

i. Aero planes/lifesaving medical equipment 40%

j. Ships 40%

3) Furniture & Fixtures 10%


4) Intangible assets 25%

E. TYPES OF DEPRECIATION-

SALEEM QURAISHEE Mo: 9175664444 INSPIRE ACADEMY-8888881719


PROFITS AND GAINS OF BUSINESS OR PROFESSION- Page 6.14

F. WRITTEN DOWN VALUE – SEC 43(6)

G. Depreciation in case of succession, amalgamation, business re-organization or demerger - [Fifth


proviso to sec. 32(1) and sec. 44DB]
These provisions are applicable while determining depreciation if there is a change of ownership
of assets because of the following:
I. Conversion of firm or sole proprietary concern into company.
II. Succession to business other than on death - business of HUF taken over by a member, business
of a firm taken over by a partner, conversion of HUF concern into company;
III. Amalgamation of a company. /Demerger of a company.
IV. Conversion of private company/ unlisted public company into limited liability partnership.
V. Amalgamation or demerger of co-operative banks.
In the year in which change of ownership takes place because of the aforesaid reasons, depreciation
shall be calculated as under:
1. Find out the amount of depreciation of the previous year in which ownership of assets changes
(because of the aforesaid reasons) on the assumption that the succession, amalgamation or
demerger has not taken place.
2. The amount of depreciation so determined shall be apportioned between the predecessor and
successor, in the ratio of number of days for which the assets are used by them during the
previous year in which ownership changes.

SALEEM QURAISHEE Mo: 9175664444 INSPIRE ACADEMY-8888881719


PROFITS AND GAINS OF BUSINESS OR PROFESSION- Page 6.15

H. ADDITIONAL DEPRECIATION – The provisions of additional depreciation are given below-


1) Assessee eligible for additional depreciation-
I. The assessee should be engaged in the manufacture or production of any article or thing
II. An assessee who is in the business of generation, transmission or distribution of power.(FA-2016)
III. An Industrial undertaking established in the notified backward area in the state of
AP/Bihar/Telangana/WB shall be eligible for additional depreciation if eligible assets are acquired
and installed on or after 1-4-2015 but before 1-4-2020.

2) ASSETS FOR WHICH ADDITIONAL DEPRECIATION IS ALLOWED


I. Additional depreciation is available only in respect of new plant and machinery acquired and
installed after March 31, 2005
II. Additional depreciation is not available in respect of building/ furniture/ old plant and machinery.
III. Eligible plant and machinery -Any plant and machinery which has been acquired and installed after
March 31, 2005 by an assessee is qualified for additional depreciation.

3) The following assets are not eligible for additional depreciation-


 ships and aircrafts;
 any machinery or plant which, before its installation by the assessee, was used either within
or outside India by any other person;
 any machinery or plant which is installed in any office premises or any residential
accommodation, or accommodation in the nature of a guest house;
 any office appliances or road transport vehicles;
 Any machinery or plant, the whole of the actual cost of which is allowed as a deduction
(whether by way of depreciation or otherwise) in computing the income chargeable under
the head "Profits and gains of business or profession" of any one previous year.

4) RATE OF ADDITIONAL DEPRECIATION:


I. Rate of Additional deprecation-20%.
II. Rate of additional depreciation -35% (Notified backward areas).
III. Additional Depreciation 35% not available for P&M used in business of generation,
transmission and distribution of power even if they are set up in notified backward areas.
(only 20% shall be allowed.)
IV. If asset acquired and put to use less than 180 days the rate of depreciation is 50% and the
remaining 10/17.5 % shall be allowed as deduction in the next year.

I. UNABSORBED DEPRECIATION
1) Depreciation which could not be fully deducted from profits and gains of current year of business or
profession (due to insufficient profit), is termed as unabsorbed depreciation.
2) The unabsorbed depreciation can be deducted from income under any other head (except with
Casual income and Salaries) of the same assessment year.
3) If depreciation still remains unabsorbed, it can be carried forward for indefinite period and can be
set off against any income (except with Casual income and Salaries) of the assessee.

SALEEM QURAISHEE Mo: 9175664444 INSPIRE ACADEMY-8888881719


PROFITS AND GAINS OF BUSINESS OR PROFESSION- Page 6.16

J. DEPRECIATION IN THE CASE OF POWER UNITS:

 However, such option shall be exercised before the due date of furnishing return of income. Further, it may be
noted that once the option is exercised, it shall be applicable for all subsequent assessment years.
 Rate of depreciation would be percentage specified in IA to the income tax rules

SALE OF ASSETS UNDER SLM IN CASE OF POWER UNITS-

K. MEANING OF "ACTUAL COST" [SEC. 43(1)] – As per section 43(1). "Actual cost" means the actual
cost of the assets to the assessee, reduced by that portion of the cost thereof, if any, as has been
met directly or indirectly by any other person or authority.
Following second proviso shall be inserted after the first proviso to clause (1) of section 43 by the
Finance Act, 2017, w.e.f. 1-4-2018 :

 Where the assessee incurs any expenditure for acquisition of any asset or part thereof in respect of
which a payment or aggregate of payments made to a person in a day, otherwise than by an account
payee cheque drawn on a bank or an account payee bank draft or use of electronic clearing system
through a bank account or any other mode as may be prescribed exceeds ten thousand rupees, such
expenditure shall be ignored for the purposes of determination of actual cost.

 Normal depreciation/Additional Depreciation/32AC/32AD-NA pertaining to a payment exceeding


`10,000 which is made by account payee cheque /draft/use of ECS any other mode as may be
prescribed

The above definition contains 3 elements:


a) It should be the actual cost of the asset;
b) It should be the actual cost of the asset to the assessee;
c) It should be exclusive of any portion of the cost which has been met directly or indirectly by any
other person or authority.

SALEEM QURAISHEE Mo: 9175664444 INSPIRE ACADEMY-8888881719


PROFITS AND GAINS OF BUSINESS OR PROFESSION- Page 6.17

POINT TO BE NOTED-
I. Depreciation is allowed only on the actual cost of the asset.
II. Revaluation of asset should be completely ignored for calculating depreciation under the
Income tax Act.
III. Actual cost to include all expenditure necessary to bring such asset into existence and to put
that in working condition:
IV. Receipts directly connected or incidental to the work of construction before commencement of
business would reduce the cost.
V. Interest received on surplus funds before commencement of business would not reduce cost
but such interest will be taxable under the head income from other sources
What is included/not included in cost of the asset-
1) Interest
a) In case new business is being set up: Interest incurred before the commencements of the
production can only be capitalized as a cost of the fixed assets created out of the money borrowed.
It cannot be claimed as a deduction u/s 36(1)(iii);

b) In case a business is existing and asset is purchased for extension of such existing business
[Proviso to section 36(1)(iii)- Interest for the period beginning from the date on which capital is
borrowed for acquisition of asset and till the date on which such asset was first put to use shall not
be allowable as deduction u/s 36 but such interest will be added to the cost of asset.

c) Interest on money borrowed for the purpose of acquiring a capital asset- pertaining to the period
after the asset is put to use in all the above cases, has to be claimed as a deduction u/s 36(1)(iii).
The interest of the period after the asset has been put to use cannot be capitalized as the cost of
the asset

NOTIONAL ACTUAL COST-


Situation Notional Actual Cost

1. Asset used in business after it ceases to be used for Actual cost of the asset minus the amount of deduction
Scientific Research allowed u/s 35 i.e. it will be 'Nil'
2. Asset acquired by way of gift or inheritance Actual cost to the previous owner minus the
depreciation ALLOWABLE ON THAT ASSET ASSUMMING
it is only asset in relevant block.
3. Depreciable asset transferred to reduce tax liability Actual cost as determined by Assessing Officer with
by claiming depreciation at enhanced cost approval of Joint Commissioner. Genuine cases not
covered.
4. Depreciable assets earlier transferred reacquired by (a) Original actual cost minus the depreciation actually
the assessee allowed to him assuming it is only asset in the
relevant block. Or
(b) actual price for which reacquired, whichever is less
5. Asset previously used by any person and on which Actual cost in the hands of the person who has leased
depreciation allowed to him, acquired by another back the asset shall be same as the W.D.V. of the said
person but lease back to the seller asset to the seller at the time of transfer thereof.

SALEEM QURAISHEE Mo: 9175664444 INSPIRE ACADEMY-8888881719


PROFITS AND GAINS OF BUSINESS OR PROFESSION- Page 6.18

Situation Notional Actual Cost

6. Buildings brought into use for business purpose Depreciation that would have been Actual cost of the
subsequent to its acquisition building minus all allowable had the building been used
for business since its acquisition.
7. Assets transferred by holding company of 100% Actual cost to the transferee company shall be same as
subsidiary or vice versa where transferee company would have been to transferor company, if it continued
is an Indian company to hold it.
8. Asset transferred under a scheme of amalgamation Actual cost to the amalgamated company shall be same
as would have been to the amalgamating company, if it
continued to hold it
9. Asset transferred to the resulting company in case Same as would have been to demerged company, if it
of demerger continued to hold the asset
10. Interest pertaining to post acquisition period Interest on money borrowed for the purpose of
acquiring a capital asset, pertaining to the period after
the asset is put to use is to be claimed as a revenue
expenditure u/s 36(1)(iii)
11. Actual cost of Cenvatable asset Actual cost minus duty of excise/ customs for which
credit of cenvat has been taken
12. Asset acquired where portion of cost met by some Actual cost minus cost met by some other person
other person
13. Asset acquired by non- resident outside India but Actual cost minus depreciation that would have been
brought by him to India for the purpose of business allowable in India since the date of its acquisition
and profession

14. Asset acquired by a company under a scheme for Actual cost to the company shall be the amount which
corporatization of a recognized stock exchange in would have been regarded as actual cost had there
India. been no such corporatization.

15. Actual cost of capital asset has been allowed as 'Nil'—


deduction u/s 35AD and capital asset is transferred
by way of transactions referred to in section 47
(like gift ,will etc)
16 Capital asset on which 100% deduction has been Actual cost less Depreciation @ the rate in force that
allowed or is allowable to the assessee under would have been allowable had the asset used since the
section 35AD and used for other business purpose. date of its acquisition.
FA 2017

SALEEM QURAISHEE Mo: 9175664444 INSPIRE ACADEMY-8888881719


PROFITS AND GAINS OF BUSINESS OR PROFESSION- Page 6.19

PROBLEM:37 R is owner of two buildings which are used by him for his own business. The written down
value of these buildings is ` 5, 00,000 on 1-4-2019 and the rate of depreciation allowable on this block
is 10%. He purchased another building on 1-12-2019 for ` 2, 50,000 and sold one of the two old
buildings on 1-3-2020 for ` 8, 50,000. Transfer expenses amounted to ` 20,000. Compute the
allowable depreciation/taxable capital gain for the assessment year 2020-21. What would have been
the difference had the building been sold for ` 3, 50,000?
ANSWER:37 Rs. 27,500

PROBLEM:38 R purchases on 5-5-2019 a new machine at a cost of ` 2,50,000. In bringing it to factory


site, he incurs expenses on clearing, freight and loading amounting to ` 10,000, ` 4,000 and ` 1,500
respectively. In installing the machine, a further cost of ` 30,000 is incurred. It is then found that a
part is broken and in repairing such broken part an additional expenditure of ` 24,500 becomes
necessary. Ultimately on 2-2-2020 the machinery is ready for use and is actually brought into use on
that day. If the rate of depreciation is 15% determine the actual cost of the machinery for purposes of
depreciation and also the amount of depreciation admissible for the assessment year 2020-21
assuming the W.D.V. of the same block as on 1-4-2019 was ` 2,40,000. Give reason for your answer.
ANSWER:38 Depreciation Rs. 60,000

PROBLEM:39 R is engaged in the business of transportation of goods. On 1-4-2019 the W.D.V. of his
various assets was as given below:
No. Particulars `

A. Office building (Rate of depreciation 10%) 9,20,000

B. Commercial vehicles (Rate of depreciation 30%) 30,90,000

C. Car (Rate of depreciation 15%) 2,10,000


During the year, he sold one of the commercial vehicles which was over the age of 15 years for `60,000.
It was replaced by another vehicle at the cost of ` 9, 00,000 on 1-11-2019. A new vehicle was purchased
on 1-1-2020 for ` 15,00,000. Calculate depreciation admissible for the previous year 2019-20.
ANSWER:39 Rs 13,92,500

PROBLEM:40 Written down value of 4 machines at the beginning of the PY 2019-20,forming part of a
block of assets carrying 15% rate of depreciation was ` 5,00,000.The following 4 machines of the
same block were bought.
MACHINES DATE OF PURCHASE DATE WHEN PUT TO USE COST
P 05-01-2019 14-01-2020 50,000
Q 05-04-2019 15-05-2019 1,00,000
R 15-05-2019 31-01-2020 2,00,000
S 15-11-2019 27-03-2020 1,50,000
Four machines of this block (other than those which were acquired and put to use less than 180 days)
were sold for ` 4, 00,000.
A. Calculate the depreciation for AY 2020-21.
B. What will be the answer if four machines were sold for 7, 00,000 instead of 4, 00,000.
ANSWER:40

SALEEM QURAISHEE Mo: 9175664444 INSPIRE ACADEMY-8888881719


PROFITS AND GAINS OF BUSINESS OR PROFESSION- Page 6.20

PROBLEM:41 WDV of the block having two machines namely X & Y as on 01-04-2019 is
6,00,000.Machine Z purchased on 5-11-2019 for 3,00,000 and put to use on the same day. Machine Z
is sold on 28-03-2020 for 4, 00,000.
A. Compute depreciation allowable for AY 2020-21.
B. What will be amount of depreciation allowed if machine X is sold instead of Machine Z.
C. What will be amount of depreciation allowed if both machine X and Y are sold instead of Z.
ANSWER:41

PROBLEM:42 The WDV of a block of asset as on 01-04-2019 is ` 8, 00,000. An asset of the same block was
acquired during the PY for `3, 00,000. Thereafter, all the assets of block were sold for `12, 00,000.
Compute Depreciation for AY 2020-21 and also indicate if there is any STCG/STCL?
A. Would your answer change if sale consideration is ` 9, 00,000?
ANSWER:42

PROBLEM:43 The WDV of the machines of 15% blocks as on 01-04-2019 is `2, 00,000. A new machine is
purchased on 14-08-2019 for `80,000 which is also eligible for 15% depreciation. All the assets in the
blocks including the new machinery are sold on 08-02-2020. Compute the depreciation and STCG/STCL
for AY 2020-21, if the sale consideration is
I. `2,00,000
II. `3, 80,000.
ANSWER:43

PROBLEM:44 XYZ Ltd, a manufacturing concern, furnishes you the following information-
PARTICULAR `
1 Opening WDV of Plant and machine as on 1-04-2019 30,00,000
2 New Plant and machine purchased and put to use on 08-06-2019 20,00,000
3 New plant and machine acquired and put to use on 15-12-2019 8,00,000
4 Computer acquired and installed in office on 2-1-2020 3,00,000
Compute the amount of Depreciation and additional depreciation for AY 2020-21.
ANSWER:44 Depreciation `12,90,000 ` 90000

PROBLEM:45 Mr. Z has the following assets which are eligible for depreciation at 15% on WDV basis:
Date Particulars Amount (Rs)
01.04.2016 WDV of Plant X ‘and Plant Y 2,00,000
10.12.2019 Acquired a new Plant Z' for 2,00,000
22.01.2020 Sold Plant Y for 4,00,000
Expenditure incurred in connection with transfer 10,000
Compute eligible depreciation claim/chargeable capital gain, if any, for AY 2020-21.
ANSWER:45

PROBLEM:46 A car purchased by Dr. Abhay on 10.08.2016 for ` 5,25,000 for personal use is brought into
professional use on 1.07.2019 by him, when its market value was ` 2,50,000. Compute the actual cost
of the car and the amount of depreciation for the assessment year 2020-21 assuming the rate of
depreciation to be 15%.
ANSWER:46

SALEEM QURAISHEE Mo: 9175664444 INSPIRE ACADEMY-8888881719


PROFITS AND GAINS OF BUSINESS OR PROFESSION- Page 6.21

PROBLEM:47 A newly qualified ca, Mr. Kunal , has acquired the following assets in his office during fy
2019-20 at the cost shown below. calculate the amount of depreciation to be claimed from his
professional income for ay 2020-21:
S No Description Date of Date when Amount
acquisition put to use (Rs)
1 Computer 27.09.19 02.10.19 35,000
2 Computer software 01.10.19 01.10.19 8,500
3 Computer printer 01.10.19 03.10.19 12,500
4 Books (of which books being annual publications of Rs 01.04.19 01.04.19 13,000
12,000)
5 Office furniture (acquired from a practicing CA) 01.04.19 01.04.19 3,00,000
6 Laptop 26.09.19 01.10.19 43,000
ANSWER:47

PROBLEM:48 Calculate depreciation for the AY 2020-2021 i.e. PY 2019-2020.


I. Factory building (10%) WDV Rs. 5,00,000
II. Plant & Machinery (15%) WDV Rs. 8,00,000
III. Additions to Plant & machinery on
 30/6/2019 Rs. 1,00,000
 31/12/2019 Rs. 1,00,000
IV. Sale of Plant & machinery on 1/12/2019 Rs. 6,00,000
V. Computer (40%) Rs. 6,00,000
VI. Addition to computer on 1/1/2020 Rs. 60,000
VII. Furniture and Fixtures (10%) WDV Rs. 1,00.000
VIII. Motor Car (15%) WDV Rs. 60,000
ANSWER:48

PROBLEM:49 Mr. A is engaged in the business of generation and distribution of electric power. He
always opts to claim depreciation on WDV for income tax purposes. From the following details,
compute the depreciation allowable as per the provisions of the Income Tax Act, 1961 for AY 2020-21
Particulars Rs (in lacs)
Opening WDV of block (15% rate) 42
New machinery purchased on 12.10.2019 10
Machinery imported from Colombo on 12.04.2019. This machine had been used only in 9
Colombo earlier and the assessee is the first user in India.
New computer installed in generation wing of the unit on 15.07.2019 2
ANSWER:49

SALEEM QURAISHEE Mo: 9175664444 INSPIRE ACADEMY-8888881719


PROFITS AND GAINS OF BUSINESS OR PROFESSION- Page 6.22

PROBLEM:50 Mr. Gopi carrying on business as proprietor converted the same into a limited company
W.e.f 01.07.2019. The name of the company is Gopi Pipes (P) Ltd. The details of the assets are given
below:
Block I WDV of plant & machinery (rate of depreciation @ 15%) Rs 12,00,000
Block II WDV of building (rate of depreciation @ 10%) Rs 25,00,000
The company Gopi Pipes (P) Ltd acquired new plant and machinery in December 2019 for 10,00,000. It
has been doing the business from 01.07.2019. Compute the quantum of depreciation to be claimed by
Mr. Gopi and successor Gopi Pipes (P) Ltd for AY 2020-21. Ignore additional depreciation.
ANSWER:50

PROBLEM:51 Dr. Sanjeev purchased a house property on 01.12.2017 for` `10,00,000. Till 01.05.2019, the
same was self-occupied as a residence. On this date, the said building was brought into use for the
purpose of his medical profession.
1) What would be the depreciation allowable for the Assessment Year 2020-21 assuming that he owns
no other building and the rate of depreciation is 10%?
2) Will the answer be different if the House Property had been gifted to him by his father, who had
purchased the same on 01.05.2016 for`9,00,000?
3) Will the treatment be the same if the item under consideration was not a Building, but a Car?
ANSWER:51

PROBLEM:52 Joy Ltd. has a block of assets carrying 15% rate of depreciation, whose WDV on 1/4/2019
was Rs. 40,00,000. It purchased another asset of the same block on 1/11/2019 for Rs. 14,40,000 and
put to use on the same date. Joy Ltd. was amalgamated with Dilip Ltd w.e.f. 1/1/2020. You are required
to compute the depreciation allowable to Joy Ltd and Dilip Ltd. for the AY 2020-2021 i.e. PY 2019-2020.
ANSWER:52

PROBLEM:53 Sai Ltd. has a block of assets carrying 15% rate of depreciation, whose written down value
on 01.04.2019 was ` 40 lacs. It purchased another asset (second-hand plant and machinery) of the
same block on 01.11.2019 for ` 14.40 lacs and put to use on the same day. Sai Ltd. was amalgamated
with Shirdi Ltd. with effect from 01.01.2020. You are required to compute the depreciation allowable
to Sai Ltd. & Shirdi Ltd. For the previous year ended on 31.03.2020 assuming that the assets were
transferred to Shirdi Ltd. at ` 60 lacs. Also assume that the plant and machinery were purchased by
way of account payee cheque.
ANSWER:53

PROBLEM:54 An electricity company which was charging depreciation on SLM and whose actual cost of the
asset was 5,00,000 and WDV 4,50,000 sold the asset during 2019-20 after 2 years. What will be tax treatment
if asset is sold for
I. 350000
II. 480000
III. 600000
ANSWER:54

SALEEM QURAISHEE Mo: 9175664444 INSPIRE ACADEMY-8888881719


PROFITS AND GAINS OF BUSINESS OR PROFESSION- Page 6.23

PROBLEM:55 A is the owner of the building and runs a business in it. A cannot claim deduction for
which of the following expenses?
(a) Rent of the building (b) Rates of the building
(c) Taxes of the building (d) Current repairs of the building

PROBLEM:56 Sunil has leased building from Anuj. He cannot claim deduction for which of the following?
(a) Rent of the building (b) Rates of the building
(c) Taxes of the building (d) Capital repairs of the building of a permanent nature

PROBLEM:57 Karan & Co. made an expenditure of Rs. 5 lakhs towards sound proofing of a rented
building, which is used exclusively for business purpose. The amount allowed as deduction in
computation of taxable income under head PGBP shall be Rs. ................
(a) 5,00,000 (b) 50,000 (c) 25,000 (d) None of the above

PROBLEM:58 In respect of Plant and Machinery or Furniture used for business purpose, deduction for
which of the following expenses are allowed?
(a) Current repairs
(b) Insurance premium paid against risk of damage of Plant and Machinery or Furniture
(c) Capital repairs of permanent nature (d) Both (a) and ( b)

PROBLEM:59 Depreciation under the Income-tax Act, is allowed as per ................


(a) Written Down Value Method
(b) Straight Line Method of depreciation for companies engaged in power generation
(c) Both (a) and (b) (d) SLM to all the companies

PROBLEM:60 Depreciation under the Income-tax Act, is allowed for ................


(a) Tangible assets like Building, Furniture
(b) Intangible assets like Goodwill, Brand (c) Both (a) and (b) (d) None of the above

PROBLEM:61 Depreciation under the Income-tax Act, is allowed only when ................
(a) Asset is owned wholly or partly by the assessee (b) Asset is not owned wholly by the assessee
(c) Asset is leased by the assessee (d) None of the above

PROBLEM:62 Alpha India Limited, engaged in manufacturing yarn can claim depreciation based on ......
(a) Actual cost of the asset, on year to year basis
(b) WDV of individual asset (c) WDV of the “Block of Assets”
(d) Either WDV or Straight-Line Method at the option of the assessee

SALEEM QURAISHEE Mo: 9175664444 INSPIRE ACADEMY-8888881719


PROFITS AND GAINS OF BUSINESS OR PROFESSION- Page 6.24

PROBLEM:63 An assessee engaged in the business of providing courier service intends to claim
“Additional depreciation” on Building purchased from seller who used it for his business, but his
advisor has informed otherwise. Which of the reasons given by the advisor are true?
(a) Assessee is not engaged in the business of manufacture or production of any article or thing
(b) Depreciation is available only on new Plant and Machinery (c) None of the above
(d) Both (a) and (b)

PROBLEM:64 An assessee engaged in manufacture of specified article or thing is wondering on which of


the following can he claim additional depreciation. Can you help him?
(a) Cases, where cost of P&M has been already allowed as a deduction in computing PGBP income
for earlier years (b) Used Plant and Machinery acquired by him during the year
(c) Office appliances (d) None of the above

PROBLEM:65 Aman has leased certain machinery to Chaman. In such a case, depreciation is allowed to .
(a) Aman assuming, he is the owner of the asset (b) Chaman as per the Accounting standard
(c) Either of the two as per commercial agreement
(d) Equally by both Aman and Chaman

PROBLEM:66 Sanjay, who runs business from a leasehold building, incurred certain capital expenditure
for construction of a structure on leased premises. In respect of such expenditure, Suraj would be
entitled to ................
(a) Depreciation under Income-tax (b) Nil deduction
(c) Claim entire amount as revenue expenses
(d) Claim it as revenue deduction or claim depreciation at his option

PROBLEM:67 Which of the following are excluded from definition of Plant under Income-tax Act, 1961?
(a) Tea bushes (b) Livestock (c) Buildings (d) All of the above

PROBLEM:68 Block of assets means a group of assets ................


(a) Falling within same class of assets which has same rate of depreciation
(b) Falling within same class of assets or same rate of depreciation
(c) Both (a) and (b) (d) None of the above

PROBLEM:69 Which of the following categories can form “Block of Assets “under Income-tax Act, 1961?
(a) Building, Plant & Machinery (b) Intangible assets (c) Furniture (d) All of the above

PROBLEM:70 AMIT purchased a building for Rs. 5 crores on June 30, 2019, which would be used
exclusively for residential purpose. What would be the amount of depreciation, which he can claim in
respect of such property while computing his business income?
(a) Rs. 25 lakhs (b) Rs. 12.5 lakhs c) Rs. 50 lakhs (d) None of the above

SALEEM QURAISHEE Mo: 9175664444 INSPIRE ACADEMY-8888881719


PROFITS AND GAINS OF BUSINESS OR PROFESSION- Page 6.25

PROBLEM:71 Amit purchased a building for Rs. 5 crores on June 30, 2019 which shall be used
exclusively for business purpose. The depreciation available thereon shall be?
(a) RS 25 lakhs (b) Rs. 12.5 lakhs (c) Rs. 50 lakhs (d) None of the above

PROBLEM:72 NARESH purchased a building for Rs. 5 crores on June 30, 2019 which would be used for
installing machinery and plant forming part of water supply project, which is put to use to provide
infrastructure facilities u/s 80-IA (4) (1). The depreciation available thereon shall be Rs. ................
(a) 25 lakhs (b) 12.5 lakhs (c) 50 lakhs (d) 200 lakhs

PROBLEM:73 Opening WDV of the block of assets was Rs. 15, 00,000. During the year, asset was acquired under
this block on 15th June 2019 amounting to Rs. 10, 00,000. Rate of depreciation of the block is 15%. Calculate
the amount of depreciation available during the previous year for the block.
(a) Rs. 3, 25,000 (b) Rs. 3, 75,000 (c) Rs. 3, 00,000 (d) Rs. 2, 25,000

PROBLEM:74 Opening WDV of the block of assets was Rs. 15, 00,000. During the year, asset was acquired under
this block on 15th June 2019 amounting to Rs. 10, 00,000. One of the assets falling within the block was sold for
Rs. 5, 50,000 on 14-01-2020. Rate of depreciation of the block is 10%. Calculate the amount of depreciation
available during the previous year for the block.
(a) Rs. 1, 95,000 (b) Rs. 2, 50,000 (c) Rs. 1, 45,000 (d) Rs. 2, 22,500

PROBLEM:75 Opening WDV of the block of assets was Rs. 15, 00,000. During the year, asset was acquired under
this block on 15th Jan 2020 amounting to Rs. 10, 00,000. One of the assets falling within the block was sold for
Rs. 5, 50,000 on 14-01-2020. Rate of depreciation of the block is 10%. Calculate the amount of depreciation
available during the previous year for the block.
(a) Rs. 1, 95,000 (b) Rs. 2, 50,000 (c) Rs. 1, 45,000 (d) Rs. 2, 22,500

PROBLEM:76 Opening WDV of the block of assets was Rs. 25, 00,000. During the year, asset was acquired under
this block on 11th October 2019 amounting to Rs. 15, 00,000. Also, moneys payable in respect of asset falling
within this block was Rs. 38,00,000. Rate of depreciation of the block is 10% calculate the amount of
depreciation available during the previous year for the block.
(a) Rs. 20,000 (b) Rs. 10,000 (c) Rs. 1, 50,000 (d) Rs. 15,000

PROBLEM:77 XYZ Ltd is engaged in production of textile articles. Opening WDV of the lock of assets was Rs. 15,
00,000. During the year, plant was acquired under this block on 15th June 2019 amounting to Rs. 10,00,000.
One of the asset falling within the block was sold for Rs. 5,50,000 on 14-01-2020. Rate of depreciation of the
block is 15%. Calculate the total amount of depreciation including additional depreciation available during the
previous year for the block.
(a) Rs. 2, 92,500 (b) Rs. 4,92,500 (c) Rs. 3, 92,500 (d) Rs. 3,52,500

SALEEM QURAISHEE Mo: 9175664444 INSPIRE ACADEMY-8888881719


PROFITS AND GAINS OF BUSINESS OR PROFESSION- Page 6.26

PROBLEM:78 XYZ Ltd is engaged in production of textile articles. Opening WDV of the lock of assets was Rs. 15,
00,000. During the year, plant was acquired under this block on 15th December 2019 amounting to Rs. 10,
00,000. One of the assets falling within the block was sold for Rs. 5, 50,000 on 14-01-2020. Rate of
depreciation of the block is 15% calculate the total amount of depreciation including additional depreciation
available during the previous year for the block.
(a) Rs. 2, 92,500 (b) Rs. 3, 17,500 (c) Rs. 4, 92,000 (d) Rs. 3, 52,500

PROBLEM:79 R acquired an asset for Rs 5, 22,000 which includes Rs72, 000 as GST for which the assessee has
claimed ITC. The actual cost of acquisition to be included in the block of asset shall be:
a) Rs 5, 22,000 b) Rs 4,50,000 c) none of these two

PROBLEM:80 An electricity company charging depreciation on SLM on each asset separately, sells one
of its machinery in April, 2019 at Rs. 1,20,000. The WDV of the machinery at the beginning of the
year i.e. on 1st April, 2019 is Rs. 1,35,000. No new machinery was purchased during the year. The
shortfall of Rs. 15,000 is treated as—
(a) Terminal depreciation (b) Short-term capital loss
(c) Normal depreciation (d) Any of the above, at the option of the assessee

PROBLEM:81 Which of the following assets are eligible for depreciation @ 40% under the Income-tax?
(a) Computers including computer software (b) Books (not used in library)
(c) Specified lifesaving medical equipment (d) All of the above

PROBLEM:82 What would be the rate of depreciation in case of know-how, patents copyrights,
trademarks, licenses, franchises or any other business or commercial right of similar nature?
(a) 100% (b) 25% (c) 40% (d) 5%

PROBLEM:83 In computing written down value for Income Tax purpose, which of the following are
true?
(a) Cost of asset acquired is added to the opening block of assets
(b) Cost of asset acquired is reduced from the opening block of assets
(c) (Sales consideration - Expenses) for assets sold is reduced from block of assets
(d) Both (a) and (c)

PROBLEM:84 Capital gains on sale of asset forming part of block of depreciable asset can be ................
(a) Long term capital gain (b) Long term capital gain or short-term capital gains
(c) Short term capital gain (d) Income from PGBP

SALEEM QURAISHEE Mo: 9175664444 INSPIRE ACADEMY-8888881719


PROFITS AND GAINS OF BUSINESS OR PROFESSION- Page 6.27

PROBLEM:85 In case of a new business interest up to the date of commencement of commercial


production is ................
(a) Added to the actual cost of the asset (b) Reduced from the actual cost of the asset
(c) Claimed as a revenue deduction (d) None of the above

PROBLEM:86 ANUJ had a running business. He purchased an asset on 1-4-2019 which was put to use on
30-9-2019. Interest up to the date the asset is first put to use would be ................
(a) Added to the actual cost of the asset (b) Reduced from the actual cost of the asset
(c) Claimed as a revenue deduction (d) None of the above

PROBLEM:87 Vikrant borrowed Rs. 20 lakhs on 1-4-2018 @ 12% p.a. to construct the building for the
purpose of business. The construction was completed on 30-6-2019 and building was immediately
put to use. Cost of construction of building (excluding interest) is Rs. 50 lakhs, what would be the
interest amount that will be added to cost of building?
(a) Rs. 3,00,000 (b) Rs. 2,40,000 (c) Rs. 4,80,000 (d) Nil

PROBLEM:88 Sushant borrowed Rs. 20 lakhs @ 12% p.a. to construct the building for business.
Pending construction, such borrowing was deposited in bank and yielded Rs. 50,000 as interest,
such interest income is ................
(a) Taxable as business income (b) Taxable as IFOS
(c) To be reduced from the cost of construction of building (d) None of the above

PROBLEM:89 Manish purchased Furniture for Rs. 80,000 on 05-10-2019, which was paid as under:
Account payee cheque: Rs. 35,000
Cash on 05-10-2019: Rs. 20,000
Bearer cheque Of RS 25,000 06-10-2019
The cost of Furniture, for the purpose of depreciation shall be Rs. ................
(a) 35,000 (b) 80,000 (c) 60,000 (d) 45,000

PROBLEM:90 Sameer purchased Machinery worth Rs. 80,000 on 15.10.2019, and paid consideration as
under:
Account payee cheque : Rs. 35,000
Cash on 05.10.2019 : Rs. 35,000
Bearer cheque on 10,000 06.10.2019. What would be the cost of machinery, for the purpose of
depreciation?
(a) Rs. 45,000 (b) Rs. 80,000 (C ) Rs. 35,000

SALEEM QURAISHEE Mo: 9175664444 INSPIRE ACADEMY-8888881719


PROFITS AND GAINS OF BUSINESS OR PROFESSION- Page 6.28

PROBLEM:91 RAJESH purchased Building worth Rs. 80,000 on 08.10.2019 and paid consideration as
under:
Account payee cheque: Rs. 60,000
Cash on 08.10.2019 Rs. 10,000
Bearer cheque on 10,000 09.10.2019
What would be the cost of Building for the purpose of depreciation?
(a) Rs. 80,000 (b) Rs. 5,000 (c) Rs. 60,000 (d) Rs. 40,000

PROBLEM:92 Ankush purchased Furniture worth for 80,000 on 25.10.2019 and paid consideration as
under:
Account payee cheque : Rs. 65,000
Cash on 25.10.2019 Rs. 5,000
Bearer cheque on : Rs. 10,000 28.10.2019
In that case, what would be the cost of Furniture for the purpose of depreciation?
(a) Rs. 65,000 (b ) Rs. 80,000 (d) Rs. 32,500

PROBLEM:93 MNP Ltd. purchased an asset of Rs. 5 lakhs exclusively for scientific research. Later, it
was transferred for being used in another business, the cost of acquisition added to block of assets in
the non-scientific business would be Rs. ................
(a) Nil (b) 5,00,000 (c) 1,00,000 (d) None of the above

PROBLEM:94 Sunil had a residential Building where he lived, but later moved to a larger apartment.
What would be the actual cost of such Building, if it is brought into use for the purpose of the
business or profession by Sunil?
(a) Purchase Cost (b) Actual cost
(c) Actual cost of the building to the Sunil
Less: Depreciation that would have been allowable, if Building were used for business purposes
since date of acquisition. (d) Either of the above

PROBLEM:95 Aman has received a building as a gift from his brother Sumit, whose cost of acquisition
was Rs. 10 lakhs. Sumit claimed depreciation of Rs. 2 lakhs on such building before transfer. The
cost which would be added to block of asset for Aman, if he uses it in business would be Rs.
................
(a) 10 lakhs (b) 8 lakhs (c) 2 lakhs (d) None of the above

PROBLEM:96 X Limited had an asset with tax WDV of Rs. 5,00,000. It transferred it to its related
company at Rs. 10 lakhs, to enable it to claim higher depreciation. The actual cost of assets in hands
of transferee, would be ................
(a) Actual cost as determined by Assessing Officer
(b) Actual cost will be equal to price at which assets are transferred
(c) Actual cost will be FMV of assets being transferred on transfer date (d) None of the above

SALEEM QURAISHEE Mo: 9175664444 INSPIRE ACADEMY-8888881719


PROFITS AND GAINS OF BUSINESS OR PROFESSION- Page 6.29

PROBLEM:97 In case the assessee acquires a second hand asset, actual cost to the assessee shall be an
amount as the AO may determine having regard to all circumstances of the case with the previous
approval of Joint Commissioner, if ................
(a) AO believes asset is not relevant for assessee’s business
(b) AO is satisfied that the main purpose of transfer was the reduction of tax liability through
enhanced depreciation (c) Both the above (d) None of the above

PROBLEM:98 A owned a building, which he sold to B in FY 2017-18. In FY 2019-20, he reacquires the


building from B. In such a case, depreciation shall be allowed on ................
(a) Purchase cost of A at time of reacquisition
(b) Actual cost when asset was acquired by A less depreciation claimed by A before first transfer
(c) Actual cost when asset was acquired, less depreciation claimed by A before first transfer, less
depreciation he would have been allowed if he had continued to own the asset
(d) (a) or (c), whichever is lower

PROBLEM:99 Aman had transferred a building on 01.04.2018 with a W.D.V Rs. 10 lakhs. On
01.04.2019, he reacquired such building at Rs. 15 lakhs. What would be the cost of building that
should be added to block of assets, if such building is entitled to depreciation @ 10%?
(a) Rs. 9 lakhs (b) Rs. 15 lakhs (c) Rs. 10 lakhs (d) None of the above

PROBLEM:100 Anuj transferred a building to Ankush on 01.04.2018 having W.D.V Rs. 10 lakhs and
transferred at Rs. 15 lakhs, on same date leased back such building to Anuj. What would be the cost
of building for depreciation purpose for Ankush?
(a) Rs. 10 lakhs (b) Rs. 15 lakhs (c) Rs. 9 lakhs (d) None of the above

PROBLEM:101 SACHIN purchased building on 01.04.2018 for Rs. 10 lakhs for residential purpose and
later on 01.04.2019 he used it for business purpose. The cost of building eligible for depreciation for
FY 2019-20 would be? (assume 10% depreciation rate) ................
(a) 10 lakhs (b) 15 lakhs (c) 9 lakhs (d) None of the above

PROBLEM:102 A Ltd., owned 100% shares in B Ltd. What would be the actual cost of capital asset
transferred by A Ltd. to B Ltd. or vice versa?
(a) Cost to transferor, if he had continued to own the capital assets.
(b) Price agreed between parties
(c) Book value in transferee’s books (d) Either of the above

PROBLEM:103 SSG Ltd. an Indian company is a 100 % subsidiary of RSG Ltd. another Indian company. It
transferred a building having WDV of Rs. 10 lakhs on 01.04.2019 to RSG for Rs. 15 lakhs. The
actual cost for RSG Ltd. would be Rs. ................
(a) 10 lakhs (b) 15 lakhs (c) 9 lakhs (d) None of the above

SALEEM QURAISHEE Mo: 9175664444 INSPIRE ACADEMY-8888881719


PROFITS AND GAINS OF BUSINESS OR PROFESSION- Page 6.30

PROBLEM:104 RSG Ltd. an Indian company has a 100% subsidiary, SSG Ltd. a foreign company,
transferred a building having WDV of Rs. 10 lakhs for Rs. 15 lakhs on 30.06.2019 to SSG Ltd. In such
a case, what would be the actual cost for SSG Ltd., that will be added to block of assets?
(a) Rs. 15 lakhs (b) Rs. 10 lakhs (c) Rs. 9 lakhs (d) None of the above

PROBLEM:105 Andy Ltd., an Indian company, amalgamated with another Indian company, Candy Ltd.
w.e.f. 01.04.2019. What would be the actual cost of assets transferred by Andy Ltd., that will be
added to block of assets of Candy Ltd. for depreciation purpose?
(a) W.D.V of assets of Andy Ltd. on 31.03.2019 (b) Book Value of assets transferred to Candy Ltd.
(c) Fair market value of assets transferred (d) None of the above

PROBLEM:106 Joy Purchased Plant and Machinery worth of Rs. 11 lakhs, which included GST Rs. 1
lakh, for which he claimed credit under Indirect Tax Laws. The actual cost of assets eligible for
depreciation would be Rs. ................
(a) 11 lakhs (b) 10 lakhs (c) 1 lakh (d) None of the above

PROBLEM:107 Subsidy granted by the Central/State Government for acquisition of asset shall be reduced
to the extent of ................
(a) 50% of the subsidy (b) Entire subsidy (c) Nil subsidy (d) Either of the above

PROBLEM:108 MOHSIN purchased Furniture worth Rs. 15 lakhs for the purpose of his business.
However, a charitable institution had borne part of cost of such assets amounting Rs. 5 lakhs. The
actual cost of assets eligible for depreciation would be Rs. ................
(a) 10 lakhs (b) 15 lakhs (c) 5 lakhs (d) None of the above

PROBLEM:109 Alex, a non-resident, acquired certain Plant and Machinery in Singapore. He moved to
India for the first time in April, 2019, and used such Machinery for the purposes of his business or
profession. In such a case, actual cost of Plant and Machinery shall be reduced by ................
(a) Nil
(b) Depreciation that would have been allowable had the asset been used in India for the said
purposes since the date of its acquisition by the Alex
(c) 50% of the cost of asset
(d) Depreciation under Straight Line Method since the date of its acquisition

PROBLEM:110 X claimed deduction u/s 35AD on a Building worth Rs. 10 lakhs. During FY 2019-20 he
transferred such Building into his other non-specified business. In such a case, actual cost of Plant
and Machinery shall be reduced by Rs. ................
(a) 10 lakhs (b) Nil (c) 9 lakhs d) None of the above

PROBLEM:111 Compute depreciation in hands of Money Ltd. for AY 2020-21, which owns Building A,
WDV on 01.04.2019 Rs. 50 lakhs, purchased Building B on 30.06.2019 for Rs. 30 lakhs and on
same date sold part of Building A for Rs. 40 lakhs (rate of depreciation 10%)?
(a) Rs. 4 lakhs (b) Rs. 8 lakhs (c) Rs. 3 lakhs (d) None of the above

SALEEM QURAISHEE Mo: 9175664444 INSPIRE ACADEMY-8888881719


PROFITS AND GAINS OF BUSINESS OR PROFESSION- Page 6.31

PROBLEM:112 Compute WDV of Building as on 31.03.2020 in hands of Money Ltd. for AY 2020-21, which
owns Building A, WDV on 01.04.2019 Rs. 50 lakhs, purchased Building B on 30.06.2019 for Rs. 30
lakhs, and on same date sold part of Building A for Rs. 40 lakhs (rate of depreciation 10%)?
(a) Rs. 40 lakhs (b) Rs. 80 lakhs (c) Rs. 36 lakh (d) None of the above

PROBLEM:113 Money Ltd. owns a building A & B having WDV on 01.04.2019 of Rs. 50 lakhs. Further it
purchased Building C on 30.06.2019 for Rs. 30 lakhs and on same date sold Building B for Rs. 40
lakhs. All these building are used exclusively for business purpose. In such a case what would be the
WDV as on 01.04.2020?
(a) Rs. 36 lakhs (b) Rs. 80 lakhs (c) Rs. 40 lakhs (d) None of the above

PROBLEM:114 Which of the following is true for “Block of Assets” in the context of computing
depreciation under the head PGBP?
(a) It's a group of assets
(b) Same rate of depreciation is prescribed for all asset within a group
(c) It may be for tangible or intangible asset (d) All of the above

PROBLEM:115 Which of the following are rate of depreciation for block of Building: ...............?
(a) 5, 10 and 100 percent (b) 5, 10 and 40 percent
(c) 5, 20 and 100 percent (d) None of the above

PROBLEM:116 In computing written down value of an assets for depreciation, money payable for sale of
assets is reduced. In this context, sale includes ................
(a) Exchange (b) Compulsory acquisition of an asset under any law
(c) Transfer in amalgamation, where amalgamated company is an Indian company
(d) (a) and (b)

PROBLEM:117 Assets acquired by assessee during the PY and put to use for business for a period of less
than 180 days, shall be entitled to depreciation @ ................
(a) 50% of the normal depreciation (b) 100% of normal depreciation
(c) Nil depreciation (d) Assessee can claim depreciation, if there are profits

PROBLEM:118 What are the cases, where WDV of a block at the end of year shall be reduced to Nil and
hence no depreciation is charged?
(a) Where all the assets of block are transferred
(b) Where some assets of block are transferred but money payable for such sale exceeds the
opening WDV together with value of assets acquired during the year
(c) Both (a) & (b) (d) None of the above

PROBLEM:119 Solar heights is engaged in the business of generation and generation and distribution of
power. It has the option to claim depreciation on ................
(a) Straight Line Method (b) Written Down Value Method
(c) Neither of the above
(d) Either Straight Line Method or Written Down Value Method
SALEEM QURAISHEE Mo: 9175664444 INSPIRE ACADEMY-8888881719
PROFITS AND GAINS OF BUSINESS OR PROFESSION- Page 6.32

PROBLEM:120 Asset acquired by an undertaking engaged in the business of generation or generation and
distribution of power, which have been used for less than 180 days, shall be eligible for depreciation
@ ................
(a) 50% of normal depreciation (b) 100% of normal depreciation
(c) 25% of normal depreciation (d) Nil

PROBLEM:121 Terminal depreciation for power generation undertaking can be computed by reducing
from sales consideration ................
(a) Actual cost of the capital Asset - depreciation allowed (b) Actual cost of the capital Asset
(c) Either of the above (d) None of the above

PROBLEM:122 In which of the following case, depreciation is proportioned on the basis of number of
days, for which the assets are used in business?
(a) Amalgamation of Companies (b) Demerger of any company
(c) Succession of any business otherwise than on death as per section 170
(d) All of the above

PROBLEM:123 Which of the following assessee are eligible for additional depreciation?
(a) Succession of partnership firm by a company(b) Conversion of a private company to LLP
(c) Succession of a proprietary concern by a company (d) All of the above

PROBLEM:124 Additional depreciation is available at the rate of ................ for units which are not
engaged in notified backward areas.
(a) 10% (b) 20% (c) 40% (d) None of the above

PROBLEM:125 Which of the following assessee are eligible for additional depreciation?
(a) Assessee engaged in business of manufacture or production of Article
(b) Assessee engaged in business of generation, transmission or distribution of power
(c) Assessee established an industrial undertaking in notified backward area in the state of Andhra
Pradesh after 01.04.2015 but before 01.04.2020 (d) All of the above

PROBLEM:126 Which of the assets are not eligible for additional depreciation?
(a) Ships & Aircraft
(b) Second hand Machinery used outside India before acquisition by assessee
(c) Any office appliance or road transport vehicle (d) All of the above

PROBLEM:127 Which of the assets are not eligible for additional depreciation?
(a) Machinery installed in any office premises
(b) Where 100% machinery cost is allowed as deduction under PGBP
(c) New machinery used in manufacturing plant (d) Both (a) & (b)

SALEEM QURAISHEE Mo: 9175664444 INSPIRE ACADEMY-8888881719


PROFITS AND GAINS OF BUSINESS OR PROFESSION- Page 6.33

PROBLEM:128 In respect of additional depreciation, which of the following statements are correct?
(a) Rate of additional depreciation is 20%
(b) Assets purchased and put to use for less than 180 days during the previous year are
depreciated@ 10%
(c) Additional depreciation rate will be 35%, in case of Industrial undertaking in notified backward
areas (d) All of the above

PROBLEM:129 Sunil Limited, incurred business losses during FY 2019-20 and is wondering whether the
claim of depreciation is mandatory for them. Under Income-tax Act, claim of depreciation is
................
(a) Mandatory whether or not the assessee claims it (b) Optional
(c) Applicable only when assessee has profit (d) None of the above is true

PROBLEM:130 Depreciation on assets, used non exclusively for business is ................


(a) Allowed proportionately, as AO may determine (b) Not allowed
(c) Entirely allowed (d) None of the above

PROBLEM:131 Depreciation on asset acquired under instalment basis is allowed ................


(a) On instalments paid (b) On entire purchase price
(c) Not allowed until last instalment is paid (d) Either (a) or (b) at the option of the assessee

PROBLEM:132 Depreciation on asset acquired under hire purchase is allowed ................


(a) On hire charges paid (b) On entire purchase price, including interest
(c) Only cost of the asset
(d) Only cost of the asset and interest paid is allowed as a revenue deduction

PROBLEM:133 Government subsidy, not directly related to the cost of any particular asset is excluded in
which proportion ratio from various assets?
(a) Value of asset: All the assets in respect of which subsidy received
(b) All the assets in respect of which subsidy received: Value of asset
(c) Value of asset: Total assets of the company (d) None of the above

SALEEM QURAISHEE Mo: 9175664444 INSPIRE ACADEMY-8888881719


PROFITS AND GAINS OF BUSINESS OR PROFESSION- Page 6.34

13. INVESTMENT IN NEW PLANT AND MACHINERY IN NOTIFIED BACKWARD


AREAS IN CERTAIN STATES 32AD
A. AVAILABLE TO- Any assessee (Company and Non Corporate)
B. Conditions-
I. Assessee should set up an undertaking or enterprises for manufacture or production of
any article or thing on or after 1-4-2015
II. Undertaking or enterprises should be set up in any notified backward area in the state of
AP/Bihar/Telangana/WB
III. New asset should be acquired and installed during the period 1-4-2015 to 31-03-2020.

C. ELIGIBLE PLANT & MACHIENERY-


I. New asset means any new plant and machinery.

D. INELIGIBLE PLANT AND MACHINERY-


I. ships and aircrafts;
II. any machinery or plant which, before its installation by the assessee, was used either within
or outside India by any other person;
III. any machinery or plant which is installed in any office premises or any residential
accommodation, or accommodation in the nature of a guest house;
IV. any office appliances or road transport vehicles;
V. Any machinery or plant, the whole of the actual cost of which is allowed as a deduction
(whether by way of depreciation or otherwise) in computing the income chargeable under
the head "Profits and gains of business or profession" of any one previous year.

E. QUANTUM OF INVESTMENT ALLOWNACE-


I. The deduction will be available for the AY relevant to the PY in which new asset is installed.
II. Rate-15%

POINT TO BE NOTED-
I. The deduction is over and above the deduction available under section 32AC.
II. If an undertaking is set up in notified backward area it shall be eligible to claim 32AC as well as
32AD.

PROBLEM:134 Mr. X, set up a manufacturing unit in Warangal in the state of Telangana on 01.06.2019. It
invested ` 30 crore in new plant and machinery on 1.6.2019. Further, he invested ` 25 crore in the
plant and machinery on 01.11.2019, out of which ` 5 crore was second hand plant and machinery.
Compute the depreciation allowable under section 32. Is Mr. X entitled for any other benefit in respect
of such investment? If so, what is the benefit available?
ANSWER:134

SALEEM QURAISHEE Mo: 9175664444 INSPIRE ACADEMY-8888881719


PROFITS AND GAINS OF BUSINESS OR PROFESSION- Page 6.35

14. TEA/COFFEE/RUBBER DEVELOPMENT ACCOUNT [SEC. 33AB]-


A. Conditions:
I. It must be engaged in the business of growing and manufacturing tea or coffee or rubber in India.
II. The assessee must make a deposit in a "special account" (i.e., deposit with NABARD) or deposit
under a scheme approved by the Tea Board or Coffee Board or Rubber Board.
III. Deposit should be made within 6 months from the end of the previous year or before the due
date of furnishing return of income, whichever is earlier.
IV. The accounts of the taxpayer should be audited by a chartered accountant.

B. Amount of deduction -
I. a sum equal to amount deposited in the "special account"; or
II. 40 per cent of the profit of such business computed under the head "Profits and gains of business
or profession" before making any deduction under section 33AB and before adjusting brought
forward business loss under section 72,(whichever is less)

C. OTHER POINTS-
I. Where any deduction is claimed under this section, no deduction shall be allowed in respect of
such amount in any other previous year.
II. The amount standing to the credit of the special account may be withdrawn only for the purpose
specified in the approved scheme.
III. If the amount released from the "special account" in a year is not utilized in the same previous
year for the purpose for which it is released, the amount not so utilized will be treated as taxable
profits of that year and taxed accordingly.

D. TAX TREATMENT OF AMOUNT WITHDRAWN -


TAXABLE AS PROFIT NOT TAXABLE AS PROFIT
1 Closure of business Death of tax payer
2 Dissolution of firm Partition of HUF
Liquidation of company

E. Amount cannot be utilized for certain purpose.


a. For acquiring plant and machinery which is to be installed in any office premises or residential
accommodation or guest houses.
b. Any office appliances (other than computers)
c. Any machinery which is to be installed in an undertaking producing low priority items
(specified in the Eleventh Schedule)
d. Plant or machinery entitled to 100 per cent deduction in any one year, cannot be acquired.

F. Consequence if new asset is transferred within 8 years.


a. Machinery so acquired should not be transferred within 8 years from the end of the year in
which it is acquired.
b. If transferred within 8 years deduction allowed earlier will be withdrawn and taxable as
business profit
c. This rule of 8 years is not applicable in the case of transfer to the Central Government, State
Government, statutory corporation, Government Company or transfer in a scheme of
conversion of firm into company.

SALEEM QURAISHEE Mo: 9175664444 INSPIRE ACADEMY-8888881719


PROFITS AND GAINS OF BUSINESS OR PROFESSION- Page 6.36

PROBLEM:135 X Ltd. is engaged in the business of growing and manufacturing tea in India. During the
previous year 2019-20, its deposits ` 100 lakh in the "special account" and claims the same as
deduction under section 33AB (i.e., 40 percent of the business profit: ` 250 lakh). During 2019-20,
the company withdraws ` 35 lakh from the "special account" which is utilized as follows -
a. ` 25 lakh on December 31, 2019 for the purpose of the scheme framed by the Tea Board; and
b. ` 4 lakh for other purpose on January 27, 2020.
` 6 Iakh is not utilized up to March 31, 2020.
Find out the amount chargeable to tax for the assessment year 2020-21.
ANSWER:135

15. SITE RESTORATION FUND [SEC. 33ABA] -


A. Conditions-
I. The taxpayer is engaged in the business of the prospecting for, or extraction or production of,
petroleum or natural gas or both in India.
II. The Central Government has entered into an agreement with the taxpayer for such business.
III. It must make a deposit with SBI in a "special account" in accordance with a scheme approved by
the Ministry of Petroleum and Natural Gas or deposit any amount in a "site restoration account"
under a scheme framed by the Ministry of Petroleum and Natural Gas.
IV. The deposit should be made before the end of the previous year.
V. Books of account of the taxpayer should be audited.

B. Amount of deduction-
I. a sum equal to amounts deposited as given above; or
II. 20 per cent of the profit of such business computed under the head "Profits and gains of business
or profession" before making any deduction under section 33ABA and before adjusting brought
forward business loss under section 72,(whichever is less)

PROBLEM:136 For claiming deduction u/s 33AB, which of the following conditions should be satisfied?
(a) Assessee must engaged in business of growing and manufacturing tea or coffee or rubber in
India
(b) Assessee must deposit with NABARD specified amount for the purpose of scheme notified by
the Tea Board
(c) Get its Account audited by a Chartered Accountant and furnish audit report in Form No. 3 AC
(d) All of the above

PROBLEM:137 Deduction u/s 33AB requires deposit of amount with NABARD, within earlier of which of
the following periods?
(a) Within six months from end of previous year or before the due date of furnishing the ROI
(b) Within 3 months from end of previous year or before the due date of furnishing the ROI
(c) Within six months from end of previous year or before the date of furnishing the ROI
(d) None of the above

SALEEM QURAISHEE Mo: 9175664444 INSPIRE ACADEMY-8888881719


PROFITS AND GAINS OF BUSINESS OR PROFESSION- Page 6.37

PROBLEM:138 Quantum of deduction u/s 33AB to assessee engaged in growing and manufacturing tea
and satisfying specified conditions is ................
(a) Lower of amount deposited in special scheme or 40% of profits of such business
(b) Lower of amount deposited or 60% of profits of such business
(c) Lower of amount deposited or 50% of profits of such business (d) None of the above

PROBLEM:139 Amount deposited with NABARD or the Deposit Account in context of section 33AB should
not be utilized ................
(a) For any Plant installed in any office premises
(b) For any office appliance, including computer
(c) Machinery whose entire actual cost is allowed as deduction (d) Both (a) and (c)

PROBLEM:140 Any amount deposited with NABARD in context of section 33AB cannot be withdrawn
until there is
(a) Closure of business (b) Death of assessee or partition of HUF
(c) Dissolution of firm or liquidation of company (d) All of the above

PROBLEM:141 Deduction u/s 33 ABA is available, provided ................


(a) Assessee is engaged in business of prospecting for or extraction or production of petroleum or
natural gas and has an agreement with Central Government
(b) Assessee must make prescribed deposit in Site Restoration Account approved by Ministry of
Petroleum
(c) Get accounts audited by a CA and furnish audit report in Form No. 3 AD (d) All of the above

PROBLEM:142 The due date to deposit amount with Site Restoration Account for the purpose of claiming
deduction u/s 33ABA is ................
(a) Within 6 months from end of previous year (b) Before the due date of furnishing the ROI
(c) Within six months from end of previous year or before the date of furnishing the ROI
(d) Before end of previous year

PROBLEM:143 Quantum of deduction u/s 33 ABA shall be ................


(a) Lower of amount deposited in the scheme or 20% of profits of such business
(b) Lower of amount deposited in the scheme or 60% of profits of such business
(c) Lower of amount deposited in the scheme or 50% of profits of such business
(d) None of the above

PROBLEM:144 The amount deposited with Site Restoration Account or Special Account with SBI in
context of section 33 ABA should not be utilized for ................
(a) Plant or Machinery to be installed in any office premises
(b) For any office appliance (excluding computer)
(c) Machinery whose entire actual cost is allowed as deduction (d) All of the above

SALEEM QURAISHEE Mo: 9175664444 INSPIRE ACADEMY-8888881719


PROFITS AND GAINS OF BUSINESS OR PROFESSION- Page 6.38

16. EXPENDITURE ON SCIENTIFIC RESEARCH [SEC. 35]-


The term "scientific research" means "any activity for the extension of knowledge in the fields of natural or
applied sciences including agriculture, animal husbandry or fisheries". With a view to accelerating scientific
research, section 35 provides tax incentives. Under this section amount deductible in respect of scientific
research may be classified as under:

SALEEM QURAISHEE Mo: 9175664444 INSPIRE ACADEMY-8888881719


PROFITS AND GAINS OF BUSINESS OR PROFESSION- Page 6.39

PROBLEM:145 X Ltd. commenced production of paper on December 1, 2019. The company has made the
following expenditure on scientific research up to the year ending on March 31, 2020:
1. On December 13, 2019, the company pays ` 80,000 to the Indian Agricultural Research Institute, New
Delhi, being an approved research institution under section 35(1)(ii), for the purpose of carrying out
scientific research in natural science.
2. On December 21, 2019, the company pays ` 70,000 to the Indian Institute of Management,
Ahmadabad, being an approved institute under section 35(1)(iii), for the purpose of carrying out
research in social or statistical science.
3. On January 10, 2020, the company pays ` 40,250 to an approved National Laboratory for carrying out
programmes of scientific research.
4. On December 23, 2019, the company purchases a plot of land for ` 6,00,000. Later on a laboratory
building is constructed (cost of construction ` 4,70,000, date of completion of construction March 1,
2020) to start an in-house research.
5. Before the commencement of the production, the company had made the following revenue
expenditure for its research laboratory-
Expenditure on salary and perquisite to research personnel and research material during the 12 months
ending on November 30, 2016: ` 30,000.
Expenditure on salary of research professional from December 1, 2016 to November 30, 2019 :` 91,000
(out of which amount certified by the prescribed authority is ` 32,000).
Expenditure on providing rent-free flat and club-facility to research personnel from December 1, 2016 to
November 30, 2019 :` 18,000.
Expenditure on research material from December 1, 2016 to November 30, 2019 :` 76,800 (out of which
amount certified by the prescribed authority is ` 44,800).

Capital expenditure on scientific research (not certified by the prescribed authority).


Particulars Expenditure incurred up Expenditure incurred
to November 30, 2016 between December 1,
2017 and November 30,
`
2019 `

Purchase of land for growing for research 2,50,000 3,60,000

Purchase of equipment’s for research 2,30,000 1,40,000

Cost of cultivation of herbals 22,000 44,600


Determine the amount of deduction available to X Ltd. under section 35(1) for the assessment year 2020-21 , if the
scientific research is related to the business of the assessee-company.
ANSWER:145 Rs. ………

SALEEM QURAISHEE Mo: 9175664444 INSPIRE ACADEMY-8888881719


PROFITS AND GAINS OF BUSINESS OR PROFESSION- Page 6.40

PROBLEM:146 Calculate the taxable profit of the assessee for the assessment year 2020-21 from the particulars -
`
Particulars

Profit for the previous year 2019-20 (Before allowing the following amounts) 13,70,000

(i) Amount given to approved and notified Research Institute for conducting scientific research
(research not related to the business of the assessee) 80,000

(ii) Cost of land acquired for constructing research laboratory 2,00,000

(iii) Cost of building and plant & machinery required for research 12,00,000

(iv) Amount given as salary to staff engaged in research (relating to a field related to assessee
own business during 2019-20 (Business started on 1-4-2019). The amount is certified by the
prescribed authority. 90,000

(v) Salary given to staff engaged in research within the premises during 2019-20. 1,20,000

ANSWER:146

PROBLEM:147 Mr. Praveen Kumar has furnished the following particulars relating to payments made towards
scientific research for the year ended 31.3.2020.

SI. No. Particulars (in lacs)

I. Payments made to K Research Ltd. 20

II. Payment made to LMN College 15

III. Payment made to OPQ College 10

IV. Payment made to National Laboratory 8

V. Machinery purchased for in-house scientific research 25

VI. Salaries to research staff engaged in in-house scientific research 12

Note: K Research Ltd. and LMN College are approved research institutions and these payments are to be used for
the purposes of scientific research. Compute the amount of deduction available under section 35 of the Income-
tax Act, 1961 while arriving at the business income of the assessee.
ANSWER:147

PROBLEM:148 o deduction u/s 35 shall be admissible in respect of any expenditure incurred on the ......
(a) Acquisition of Land(b) Acquisition of Building(c) Revenue expenditure(d) None of the above

SALEEM QURAISHEE Mo: 9175664444 INSPIRE ACADEMY-8888881719


PROFITS AND GAINS OF BUSINESS OR PROFESSION- Page 6.41

PROBLEM:149 Least of which of the following is charged to tax under PGBP, where capital gains arise on
sale of asset used for scientific research, not used for any other purpose?
(a) Sale proceeds or actual cost
(b) Deductions u/s 35 or sale proceeds of the asset, whichever is less
(c) Either of the above (d) None of the above

PROBLEM:150 A Ltd. claimed deduction of Rs. 2 lakh u/s 35 for Plant and Machinery used for scientific
research in F.Y. 2018-19. Subsequently it sold such Plant and Machinery for Rs. 1 lakh in F.Y. 2019-
20 . What amount would be taxable in F.Y. 2019-20 as business income on such account?
(a) Rs. 5 lakhs (b) Rs. 1 lakh (c) Rs. 4 lakhs (d) Nil

PROBLEM:151 A Ltd. claimed deduction of Rs. 2 lakhs u/s 35 for Plant and Machinery used for scientific
research in F.Y. 2018-19. Subsequently it sold such Plant and Machinery for Rs. 1 lakh in F.Y. 2019-
20. What amount would be taxable in F.Y. 2019-20 as capital gains income on such account?
(a) Rs. 5 lakhs (b) Rs. 1 lakh (c) Rs. 4 lakhs (d) Nil

PROBLEM:152 A Ltd. purchased Machinery for Rs. 5 lakhs, which was exclusively used for scientific
research, but later transferred to other business of A Ltd. The cost of acquisition added to block of
assets would be ................
(a) Nil (b) 5 lakh (c) 1 lakh (d) None of the above

PROBLEM:153 Unabsorbed capital expenditure on scientific research shall be carried forward for
................
(a) Indefinite period of time (b) 8 years (c) 12 years (d) Next year

PROBLEM:154 Capital expenditure on scientific research can be set off ................


(a) Against PGBP income for other business
(b) Balance after PGBP against income under any head for same year
(c) Balance carried forward indefinitely (d) All of the above

PROBLEM:155 Rohit Ltd. has business income of Rs. 50 lakhs before section 35 deduction and incurred
revenue and capital expenditure of Rs. 30 lakhs and 40 lakhs, respectively for scientific research.
The deduction available in respect of expenditure on scientific research shall be Rs. ................
(a) 50 lakhs (b) 30 lakhs (c) 40 lakhs (d) None of the above

PROBLEM:156 Rohit Ltd., engaged in manufacturing , paid Rs. 10 lakhs to an eligible Scientific Research
Association, which is notified by Central Government, for space aviation purpose during FY 2019-20.
Deduction available u/s 35 shall be ? ................
(a) 10 lakhs (b) 12.5 lakhs (c) 15 lakhs (d) None of the above

SALEEM QURAISHEE Mo: 9175664444 INSPIRE ACADEMY-8888881719


PROFITS AND GAINS OF BUSINESS OR PROFESSION- Page 6.42

PROBLEM:157 Aman Ltd. paid Rs. 10 lakhs to a research association engaged in social sciences, which
used such amount for undertaking scientific research during FY 2019-20. Deduction available u/s 35
shall be ................
(a) 10 lakhs (b) 12.5 lakhs (c) 15 lakhs (d) None of the above

PROBLEM:158 ANIL Ltd. paid Rs. 10 lakhs to a University which used such amount to undertake
research in statistical research, which is not related to the business of Chaman Ltd. (during FY 2019-
20). Deduction available u/s 35 shall be Rs. ................
(a) 10 lakhs (b) 12.5 lakhs (c) 15 lakhs (d) None of the above

PROBLEM:159 Anuj Ltd. paid Rs. 20 lakhs to IIT Bombay, with a special direction to use such amount
exclusively for scientific research under a program approved by prescribed authority. Deduction
available u/s 35(2AA) would be Rs. ................
(a) 20 lakhs (b) 25 lakhs (c) 30 lakhs (d) None of the above

PROBLEM:160 ATUL Ltd. entered into an agreement with prescribed authority u/s 35(2AB), to undertake
in-house scientific research and development, and incurred revenue and capital expenditure
(excluding land) of Rs. 30 lakhs and Rs. 40 lakhs respectively. Deduction available u/s 35(2AB)
would be ................
(a) Rs. 50 lakhs (b) Rs. 30 lakhs (c) Rs. lakhs (d) Rs. 105 lakhs

PROBLEM:161 Compute deduction u/s 35 for ABC Ltd., if it incurred revenue expenses of Rs. 10 lakhs,
Building cost of Rs. 20 lakhs and Land of Rs. 15 lakhs in respect of in-house scientific research on
related business?
(a) Rs. 30 lakhs (b) Rs. 10 lakhs (c) Rs. 25 lakhs (d) Rs. 45 lakhs

PROBLEM:162 Compute deduction u/s 35 for PQR Ltd., if it incurred revenue expenses of Rs. 10 lakhs,
Building cost of Rs. 20 lakhs and Land of Rs. 15 lakhs in respect of in-house Scientific Research on
related business. It had further spent revenue expenses of Rs. 25 lakhs during 30.06.2016 to
29.06.2019 for such work?
(a) Rs. 30 lakhs (b) Rs. 55 lakhs (c) `35 lakhs (d) Rs. 60 lakh

SALEEM QURAISHEE Mo: 9175664444 INSPIRE ACADEMY-8888881719


PROFITS AND GAINS OF BUSINESS OR PROFESSION- Page 6.43

17. AMORTIZATION OF SPECTRUM FEES FOR PURCHASE OF SPECTRUM-


NEW SECTION 35ABA
New section 35ABA is inserted to provide amortization of amount paid on the acquisition of any right to
use spectrum for telecommunication services by paying spectrum fees.
I. Any capital expenditure incurred and actually paid by the assessee on acquisition of any right ot use
spectrum for Telecom services by paying spectrum fee will be allowed as deduction in equal
installments.
II. Deduction will start from the year in which payment is made (or the year of commencement of
business, whichever is later) and ending with the year in which spectrum comes to an end,
irrespective of the previous year in which liability for expenditure was incurred according to the
method of accounting regularly followed by assessee or payable in such manner as may be
prescribed.
III. Where the spectrum is transferred and the proceeds of transfer are less than the expenditure
remaining unallowed, a deduction equal to the expenditure remaining unallowed as reduced by the
proceeds of transfer, shall be allowed in the year of transfer of spectrum.
IV. If spectrum is transferred and proceeds of transfer exceed the amount of expenditure remaining
unallowed, the excess amount (to the extent it does not exceed deduction claimed u/s 35ABA) shall
be chargeable to tax as profits and gains of business in the previous year in which such spectrum has
been transferred.
V. Unallowed expenditure in a case where a part of spectrum is transferred would be amortized.
VI. Where the deduction has been claimed and granted to the assessee in accordance with the
provisions of this section and, subsequently, there is failure to comply with any of the provisions of
this section, then the deduction shall be deemed to have been wrongly allowed. In such case, the
assessing officer can re-compute the total income of the assessee for the previous year (in which
deduction was claimed) and make necessary rectification. Such rectification can be made within 4
years from the end of the previous year in which failure takes place.
VII. Under the scheme of amalgamation, if the amalgamating company sells or transfers the spectrum to
an amalgamated company, being an Indian company, then the provisions of this section shall apply
the amalgamated company as they would have applied to the amalgamating company if later had
not transferred the spectrum. Similar rule will be applicable in case of demerger

SALEEM QURAISHEE Mo: 9175664444 INSPIRE ACADEMY-8888881719


PROFITS AND GAINS OF BUSINESS OR PROFESSION- Page 6.44

18. AMORTIZATION OF TELECOM LICENSE FEES [SEC. 35ABB] –


A. CONDITIONS:
I. The expenditure is capital in nature.
II. It is incurred for acquiring any right to operate telecommunication services
III. The expenditure is incurred either before the commencement of business or thereafter at any
time during any previous
IV. The payment for the above has been actually made to obtain license year
If all the above conditions are satisfied, then one can claim deduction under section 35ABB. If,
however, these conditions are not satisfied, then deduction under section 35ABB is not available
[one may claim deduction under section 37(l).

B. AMOUNT OF DEDUCTION:

C. PROFIT OR LOSS ON SALE OF TELECOM LICENCE:


I. Any profit or loss on sale of telecom license is taken into consideration while computing business
income.
II. Where under a scheme of amalgamation/demerger, a telecom license is transferred to an Indian
company, and then the provisions of section 35ABB shall continue to apply to the transferee-
company.

PROBLEM:163 X Ltd., a company providing telecommunication service, obtains a telecom license on April
20, 2019 for a period of 10 years which ends on March 31, 2029 (license fee being ` 18 lakhs). Find
out the amount of deduction under section 35ABB if-
a. The entire amount is paid on May 6, 2019; or
b. the entire amount is paid on April 1, 2020; or
c. the entire amount is paid in three equal installments on April 30, 2019, April 30, 2020 and April 30,
2021.
ANSWER:163

PROBLEM:164 Ankur Limited paid Rs. 13,20,000 as spectrum fees for use of telecommunication services
on 30-6-2018 for 10 years. If business commenced on 30-6- 2019, deduction allowable u/s 35 ABA in
FY 2019-20 shall be ................
(a) Rs. 1,32,000 (b) Rs. 2,64,000 (c) Rs. 1,20,000 (d) None of the above

PROBLEM:165 AIRCEL paid Rs. 13,20,000 to obtain license to operate telecommunication services on
30-6-2018 for 10 years, whereas the business commenced on 30-6-2019. The deduction allowable u/s
35ABB in previous year 2019-20 shall be ................
(a) Rs. 1,32,000 (b) Rs. 2,64,000 (c) Rs. 1,20,000 (d) None of the above

PROBLEM:166 Aircel Ltd. acquired a license to operate telecommunication services for Rs. 30 lakhs on
1-4-2017 for 12 years, WDV of such license as on 1-4-2019 is Rs. 25 lakhs and part of such license
was sold for Rs. 18 lakhs. The deduction allowable u/s 35ABB in previous year 2019-20 shall be ......
(a) Rs. 70,000 (b) Rs. 2,50,000 (c) Rs. 1,30,000 (d) None of the above

SALEEM QURAISHEE Mo: 9175664444 INSPIRE ACADEMY-8888881719


PROFITS AND GAINS OF BUSINESS OR PROFESSION- Page 6.45

PROBLEM:167 IDEA Ltd. acquired a license to operate telecommunication services for Rs. 30 lakhs on
1-4-2017 for 12 years, WDV of such license as on 1-4-2019 is Rs. 25 lakhs and part of such license
was sold for Rs. 28 lakhs. The deduction allowable u/s 35ABB in previous year 2019-20 shall be Rs.
................
(a) 60,000 (b) 2,40,000 (c) 1,30,000 (d) Nil

PROBLEM:168 NEW Ltd. acquired a license to operate telecommunication services for Rs. 30 lakhs on 1-
4-2017 for 12 years, WDV of such license as on 1-4-2019 is Rs. 25 lakhs and entire license was sold
for Rs. 28 lakhs. The deduction allowable u/s 35ABB in FY 2019-20 shall be Rs. ................
(a) 80,000 (b) 2,00,000 (c) 1,32,000 (d) Nil

PROBLEM:169 IDEA Ltd. acquired a license to operate telecommunication services for Rs. 30 lakhs on
1-4-2017 for 12 years WDV of such license as on 1-4-2019 is Rs. 25 lakhs and entire license was sold
for Rs. 27 lakhs. The business income for previous year FY 2019-20 in such a case shall be Rs. .........
(a) 3,00,000 (b) 2,00,000 (c) 4,00,000 (d) None of the above

PROBLEM:170 VODAPHONE Ltd. acquired a license to operate telecommunication services for Rs. 30
lakhs on 1-4-2017 for 12 years, WDV of such license as on 1-4-2019 is Rs. 25 lakhs. It amalgamated
with B Ltd. on 30-6-2019. The deduction allowable u/s 35ABB for FY 2019-20 in hands of
VODAPHONE Ltd. shall be Rs. ................
(a) 80,000 (b) 2,00,000 (c) 1,32,000 (d) Nil

SALEEM QURAISHEE Mo: 9175664444 INSPIRE ACADEMY-8888881719


PROFITS AND GAINS OF BUSINESS OR PROFESSION- Page 6.46

19. SPECIFIED BUSINESS [SEC. 35AD]-


Section 35AD provides for an investment linked tax incentive.
A. 100% weighted deduction available to assessee carrying on specified business
(i) Setting up and operating a cold chain facility.
(ii) Setting up and operating a warehousing facility for storage of agricultural produce.
(iii) building and operating anywhere in India, a hospital with at least 100 beds for patients;
(iv) developing and building a housing project under a scheme for affordable housing framed by the
Central Government or a State Government, as the case may be, and notified by the Board in
this behalf in accordance with the guidelines as may be prescribed;
(v) Production of fertilizer in India
(vi) Laying and operating a cross-country natural gas or crude or petroleum oil pipeline network for
distribution, including storage facilities being an integral part of such network.
(vii) Business of building and operating anywhere in India, a hotel of two-star or above category,
as classified by the Central Government;
(viii) Developing and building a housing project under a scheme for slum redevelopment or
rehabilitation framed by the Central Government or A State Government, as the case may be,
and notified by the Board in this behalf in accordance with the guidelines as may be prescribed.
(ix) setting up and operating an inland container depot or a container freight station notified or
approved under the Customs Act, 1962
(x) Bee-keeping and production of honey and beeswax.
(xi) Setting up and operating a warehousing facility for storage of sugar.
(xii) Laying and operating a slurry pipeline for transportation of iron ore.
(xiii) Setting up and operating a semiconductor wafer fabrication manufacturing unit, if such unit
is notified by the board in accordance with prescribed guidelines.
(xiv) Developing or maintaining and operating or developing ,maintaining and operating a new
Infrastructure facility on or after 01-04-2017 (FA-2016)

B. CONDITIONS-
I. The specified business should not be set up by splitting up, or the reconstruction, of a business
already in existence.
II. It should not be set up by the transfer of old plant and machinery.
 20 per cent old machinery is permitted- If the value of the transferred assets does not exceed
20 per cent of the total value of the machinery or plant used in the business, this condition is
deemed to have been satisfied.
 Second-hand imported machinery is treated as new
III. Books of account of the assessee should be audited.
IV. Expenditure incurred on acquisition of any land or goodwill or financial instrument is not allowed
as deduction under this section.
V. Any expenditure in respect of which the payment or aggregate of payments made to a person
in a day, otherwise than by an account payee cheque drawn on a bank or an account payee
bank draft or use of electronic clearing system through a bank account or through such other
electronic mode as may be prescribed , exceeds `10000.FA-2017

SALEEM QURAISHEE Mo: 9175664444 INSPIRE ACADEMY-8888881719


PROFITS AND GAINS OF BUSINESS OR PROFESSION- Page 6.47

C. CONSEQUENCES OF CLAIMING DEDUCTION UNDER SECTION 35AD:


I. If deduction is claimed and allowed under section 35AD, the assessee shall not be allowed any
deduction in respect of the specified business under the provisions of Chapter VIA under sections
80HH to 80RRB for the same or any other assessment year.
II. No deduction in respect of the expenditure in respect of which deduction has been claimed shall
be allowed to the assessee under any other provisions of the Income-tax Act.
III. Any sum received or receivable on account of any capital asset, in respect of which deduction has
been allowed under section 35AD, being demolished, destroyed, discarded or transferred shall be
treated as income of the assessee and chargeable to income-tax under the head "Profits and
gains of business or profession".
IV. Any loss computed in respect of the specified business shall not be set off except against profits
and gains, if any, of any other specified business.
V. To the extent the loss is unabsorbed, the same will be carried forward for set off against profits
and gains from any specified business in the following assessment year and so on (no time-limit
for carry forward such loss).
VI. If the assessee owns two units one of them qualifies for deduction under section 35AD and the
other one is not eligible for the same and there is inter-unit transfer of goods or services between
the two units, then for the purpose of section 35AD calculation will be made as if such
transactions are made at the market value.

D. Asset for which deduction has been claimed u/s 35AD to be used only for specified business for a
period of 8 years. (Section 35AD (7A) and (7B).
I. Any asset in respect of which a deduction is claimed and allowed under section 35AD, shall
be used only for specified business for a period of 8 years beginning with the previous year
in which such asset is acquired or constructed.
II. If such asset is used for any purpose other than the specified business during the period of 8
year otherwise by any mode referred to in section 28(vii) ,the total amount of deduction so
claimed and allowed in any previous year in respect of such asset as reduced by the amount
depreciation allowable in accordance with the provisions of section 32 as if no deduction
has been allowed under section 35AD shall be deemed to income under the head Profits
and gains.

PROBLEM:171 Win Limited commenced the business of operating a three star hotel in Tirupati on 1-4-
2019. It furnishes you the following information:
I. Cost of land (acquired in June 2018) ` 60 lakhs
II. Cost of construction of hotel building
Financial year 2018-19 ` 30 lakhs ,Financial year 2019-20 `150 lakhs
III. Plant and Machineries (all new) acquired during financial year 2019-20 `30 lakhs
[All the above expenditures were capitalized in the books of the company]
IV. Net profit before depreciation for the financial year 2019-20 `80 lakhs
Determine the amount eligible for deduction under section 35AD of the Income-tax Act, 1961, for the
assessment year 2020-21.
ANSWER:171 ` 210

SALEEM QURAISHEE Mo: 9175664444 INSPIRE ACADEMY-8888881719


PROFITS AND GAINS OF BUSINESS OR PROFESSION- Page 6.48

PROBLEM:172 Mr. A commenced operations of the businesses of setting up a warehousing facility for
storage of food grains, sugar and edible oil on 1.4.2019. He incurred capital expenditure of ` 80 lakh, `
60 lakh and ` 50 lakh, respectively, on purchase of land and building during the period January, 2019 to
March, 2019 exclusively for the above businesses, and capitalized the same in its books of account as on
1st April, 2019. The cost of land included in the above figures is ` 50 lakh, ` 40 lakh and ` 30 lakh,
respectively. Further, during the P.Y.2019-20, he incurred capital expenditure of ` 20 lakh, ` 15 lakh & `
10 lakh, respectively, for extension/ reconstruction of the building purchased and used exclusively for
the above businesses.
Compute the income under the head “Profits and gains of business or profession” for the A.Y.2020-21
and the loss to be carried forward, assuming that Mr. A has fulfilled all the conditions specified for claim
of deduction under section 35AD and has not claimed any deduction under Chapter VI-A under the
heading “C – Deductions in respect of certain incomes”.
The profits from the business of setting up a warehousing facility for storage of food grains, sugar and
edible oil (before claiming deduction under section 35AD and section 32) for the A.Y. 2020-21 is ` 16
lakhs, ` 14 lakhs and ` 31 lakhs, respectively. Also, assume in respect of expenditure incurred, the
payments are made by account payee cheque or use of ECS through bank account.
ANSWER:172

PROBLEM:173 MNP Ltd. Commenced operations of the business of a new four-star hotel in Chennai on
01.04.2019. The company incurred capital expenditure of Rs.40 lakh during the period January, 2019 to
March, 2019 exclusively for the above business, and capitalized the same in its books of account as on
1st April, 2019. Further, during the Previous Year 2019-20, it incurred capital expenditure of Rs. 2.5
crore (out of which Rs. 1 crore was for acquisition of land) exclusively for the above business. Compute
the income under the heading “profit and gains of business or profession” for the assessment year
2020-21, assuming that MNP Ltd. has fulfilled all the conditions specified for claim of deduction under
section 3SAD. The profits from the business of running this hotel (before claiming deduction under
section 35AD) for the assessment year 2020-21 is Rs. 80 lakhs. Assume that the company also has
another existing business (specified business) of running a four-star hotel in Kanpur, which commenced
operations 2 years back, the profits from which was Rs. 130 lakhs for Assessment Year 2020-21.
ANSWER:173

PROBLEM:174 X had a capital asset on which he claimed deduction u/s 35AD. Due to an earthquake, this
capital asset was destroyed. The insurance company gave him compensation amounting Rs.
5,00,000. This income would be taxable as ................
(a) Income from Capital Gains (b) Profits and gains of business or profession
(c) Salaries (d) Income from other sources

PROBLEM:175 Which of the following business’s setting up and operation are not eligible for deduction
u/s 35AD?
(a) Cold chain facility (b) Warehousing facility for storage of agriculture produce
(c) Warehousing facility for storage of sugar
(d) Warehousing facility for storage of commercial produce

SALEEM QURAISHEE Mo: 9175664444 INSPIRE ACADEMY-8888881719


PROFITS AND GAINS OF BUSINESS OR PROFESSION- Page 6.49

PROBLEM:176 Which of the following business are not eligible for deduction u/s 35AD?
(a) Building and operating a hospital in India with at least 100 beds
(b) Building and operating a three-star hotel in India
(c) Developing and building a housing project under a scheme for slum redevelopment framed by
Central Government
(d) Developing and building a housing project under a scheme for slum redevelopment or
rehabilitation framed by renowned Builder

PROBLEM:177 Which of the following business are not eligible for deduction u/s 35AD?
(a) Setting up and operating an Inland Container Depot notified and approved under Customs Act,
1962 (b) Laying and operating slurry pipeline to transport iron ore
(c) Bee-keeping and production of honey (d) None of the above.

PROBLEM:178 Which of the following business are not eligible for deduction u/s 35AD?
(a) Developing and building a housing project under affordable housing scheme framed by the
Central Government (b) Production of fertilizer in India
(c) Laying and operating a cross country natural gas pipeline network for distribution, including
storage facilities therein (d) Production of fertilizer outside India.

PROBLEM:179 Which of the following business are not eligible for deduction u/s 35AD?
(a) Setting up and operating a semiconductor wafer fabrication manufacturing unit notified by the
Board
(b) Developing or maintaining and operating a new infrastructure facility
(c) Laying and operating a slurry pipeline for transportation of water (d) None of the above

PROBLEM:180 NFL Ltd., engaged in fertilizer production in India, purchased additional plant for Rs. 35
lakhs, to be exclusively used for production of fertilizer. The deduction u/s 35AD shall be Rs. ..........
(a) 35 lakhs (b) Nil (c) 52.5 lakhs (d) None of the above

PROBLEM:181 As per section 35AD of Income- tax Act 1961, 100% deduction shall be allowed on account
of any ................ expenditure, incurred wholly and exclusively for the purpose of building notified
three-star hotel in India.
(a) Capital (b) Revenue (c) Both (a) & (b) (d) None of the above

PROBLEM:182 TAJ Hotels, constructed a three-star hotel in India and claimed deduction u/s 3 5 AD, but
maintenance and operation of the hotel was given to XYZ Ltd. The deduction allowable u/s 35AD for
capital expenditure shall be allowed to ................
(a) TAJ Hotels (b) XYZ Ltd. (c) Both (a) & (b) (d) None of the above

SALEEM QURAISHEE Mo: 9175664444 INSPIRE ACADEMY-8888881719


PROFITS AND GAINS OF BUSINESS OR PROFESSION- Page 6.50

PROBLEM:183 MED LIFE Ltd. built a hospital in India during F. Y. 2018-19, and incurred Rs. 50 lakhs
towards hospital building and medical equipment. The hospital started operations on 1-4-2019. It
further incurred additional expenditure of Rs. 80 lakhs for purchase of additional equipment during
2019-20. Deduction u/s 35AD for F.Y. 2019-20 would be Rs. ................
(a) 80 lakhs (b) 130 lakhs (c) 50 lakhs (d) None of the above

PROBLEM:184 FORTIS Ltd. built a hospital in India during FY 2018-19 and incurred Rs. 50 lakhs towards
hospital building and medical equipment, which was capitalized in this FY and not on
commencement of operations of hospital on 1 -4-2019. It further incurred an expenditure of Rs. 80
lakhs for purchase of additional equipment during 2019-20. Deduction u/s 35AD for FY 2019-20
would be ................
(a) 80 lakhs (b) 130 lakhs (c) 50 lakhs (d) None of the above

PROBLEM:185 AGRO Ltd. provided warehousing facilities exclusively for commercial goods. During FY
2019-20, it demerged its business into two-unit, commercial produce unit warehouse and
warehousing for agriculture produce (for which it incurred additional capital expenditure of Rs. 50
lakhs). Deduction u/s 35AD for FY 2019-20 would be? ................
(a) 50 lakhs (b) Nil (c) 75 lakhs (d) None of the above

PROBLEM:186 PEPSI Ltd. had set up a cold chain facility and it purchased certain old equipment’s from an
existing company for Rs. Rs.
Rs. 5 lakhs during 2018-19 for setting up the business. Deduction u/s
35AD for F.Y. 2019-20 would be Rs. ................
(a) 50 lakhs (b) Nil (c) 75 lakhs (d) None of the above

PROBLEM:187 PEPSI Ltd. had set up a cold chain facility, and it purchased certain equipment of Rs. Rs.
5 lakhs during 2019-20 on which it claimed deduction u/s 35AD. Later, in FY 2020-21, it transferred
such equipment to one of its other businesses. Such transaction would result in ................
(a) Deduction claimed u/s 35AD as reduced by depreciation that would have been allowable u/s 32
on such equipment, if no deduction was given u/s 35AD, is deemed to be income of year in which
such equipment is used for other purpose
(b) Deduction claimed u/s 35AD is deemed as income of year when equipment is transferred
(c) 50% of the deduction claimed u/s 3 5AD is deemed as income of year when equipment is
transferred (d) None of the above

PROBLEM:188 Sun care Ltd. had set up a cold chain facility and it purchased certain equipment for Rs.
Rs. 5 lakhs during 2019-20 on which it claimed deduction u/s 35AD. Depreciation that would have
been allowable u/s 32, would have been Rs. 11.25 lakhs in FY 2019-20 if no deduction was claimed
u/s 35 AD. Later, in FY 2020-21, the company transferred such equipment to one of its other
business. The amount that would be deemed to be income in FY 2020-21 Rs. ................
(a) 63.75 lakhs (b) 75 lakhs (c) 37.50 lakhs (d) None of the above

SALEEM QURAISHEE Mo: 9175664444 INSPIRE ACADEMY-8888881719


PROFITS AND GAINS OF BUSINESS OR PROFESSION- Page 6.51

20. PAYMENT TO ASSOCIATIONS AND INSTITUTIONS FOR CARRYING OUT


RURAL DEVELOPMENT [SEC. 35CCA]
Any sum paid to the following is eligible for deduction under section 35CCA -
I. any association or institution to be used for carrying out any program of rural development
approved before March 1, 1983;
II. an association or institution which has its object the training of persons for implementation of a
rural development program approved before March 1, 1983;
III. National Fund for Rural Development.
IV. Notified National Urban Poverty Eradication Fund.

PROBLEM:189 Section 35CCA provides a deduction for payment made to ................


(a) National fund for rural development set up by the Central Government
(b) National Urban Poverty Eradication Fund setup and notified by the Central Government
(c) Both (a) & (b) (d) None of the above

21. AGRICULTURAL EXTENSION PROJECT [SEC. 35CCC] –


I. Where an assessee incurs any expenditure on notified agricultural extension project, then he will
be eligible to claim a weighted deduction of 150 per cent of such expenditure.
II. Projects shall be undertaken by an assessee for training, education and guidance of farmers.
III. Projects shall have prior approval of Ministry of Agriculture.
IV. For claiming weighted deduction application should submitted to CIT
V. 100% shall be allowed as deduction from AY-2021-22

PROBLEM:190 AGRO Ltd. incurred an expenditure of Rs. 50 lakhs on agriculture extension project
notified by the CBDT during FY 2019-20. Deduction available u/s 35CCC in respect of such
expenditure would be Rs. ................
(a) 75 lakhs (b) 50 lakhs (c) 10 lakhs (d) None of the above

22. EXPENDITURE FOR SKILL DEVELOPMENT [SEC. 35CCD]


I. It provides that where a company incurs any expenditure (not being expenditure in the nature of
cost of any land or building) on any notified skill development project, then such company can
claim a weighted deduction of 150 per cent of such expenditure.
II. 100% shall be allowed as deduction from AY-2021-22

PROBLEM:191 Alpha Inc., a foreign company, incurred an expenditure of Rs.50


Rs. lakhs on skill
development project notified by the CBDT during FY 2019-20. It has a permanent establishment in
India, which has taxable income. The deduction available u/s 35CCD of Income-tax Act, 1961 to Alpha
Inc. shall be Rs. ................
(a) 75 lakhs (b) 50 lakhs (c) 10 lakhs (d) None of the above

PROBLEM:192 STAR LLP, has incurred an expenditure of Rs. 50 lakhs on purchase of Land & Building
which is to be used for skill development project notified by the CBDT during FY 2019-20 . The
deduction available u/s 35CCD of Income-tax Act, 1961 to Bell Weather shall be Rs. ................
(a) 75 lakhs (b) 50 lakhs (c) 10 lakhs (d) None of the above
SALEEM QURAISHEE Mo: 9175664444 INSPIRE ACADEMY-8888881719
PROFITS AND GAINS OF BUSINESS OR PROFESSION- Page 6.52

23. AMORTIZATION OF PRELIMINARY EXPENSES [SEC. 35D] –


A. ALLOWED TO-
I. An Indian company or a resident non-corporate assessee can claim deduction under section 35AD
in respect of preliminary expenses.
II. Such expenditure may be incurred before commencement of the business or after
commencement of the business in connection with extension of an undertaking or in connection
with setting up a new unit.
B. QUALIFYING EXPENDITURE -The heads of qualifying expenditure are the following:
The work should be carried on by the assessee The work can be carried on by the assessee itself or by any
itself or by a concern approved by the board concern approved or not.
I. Feasibility report Legal charges for
II. Project report I. Agreement between assessee and any person
III. Market Survey II. Drafting MOA/AOA
IV. Engineering services. III. Printing charges for MOM/AOA
IV. Registration of a Company
V. Expenses in connection with public issue of shares

C. QUALIFYING EXPENDITURE - MAXIMUM CEILING - The aggregate expenditure cannot exceed the
following:
In the case of a corporate assessee In the case of a non-corporate assessee
a. 5 per cent of cost of project; or 5 per cent of cost of project
b. 5 per cent of capital employed, whichever is more

D. Cost of project-
I. It means the actual cost (or additional cost incurred after commencement of business in connection
with extension or setting up an undertaking) of fixed assets, namely, land, buildings, leaseholds,
plant, machinery, furniture, fittings and railway sidings (including expenditure on development of
land and buildings), which are shown in the books of the assessee as on the last day of the previous
year in which the business of the assessee commences.

E. Capital employed in the business of a company –


I. It is the aggregate of the issued share capital, amount outstanding as share premium account,
debentures and long-term borrowings, as on the last day of the previous year in which the business
of the company commences (in the case of an existing company only capital, debentures and long-
term borrowing issued or obtained in connection with the extension of the undertaking or the
setting up of the new unit of the company, shall be considered).

F. AMOUNT OF DEDUCTION:

G. AUDIT REPORT:
I. In the case of a person (other than a company/co-operative society), deduction is available only if a
report of audit is obtained in Form No. 3AE from a chartered accountant.

H. CONSEQUENCES IN THE CASE OF AMALGAMATION OR DEMERGER


I. The benefit of deduction under section 35D is not lost in a case where the undertaking of an Indian
company which is entitled to the amortization is transferred to another Indian company in a
scheme of amalgamation or demerger within the 5-year period of amortization. In that event, the
transferee-company can claim the deduction for unexpired period.
SALEEM QURAISHEE Mo: 9175664444 INSPIRE ACADEMY-8888881719
PROFITS AND GAINS OF BUSINESS OR PROFESSION- Page 6.53

PROBLEM:193 X Ltd. is incorporated in Bangalore on September 6, 2019. It commences production on


March 15, 2020. The following expenses are incurred by the company before commencement of
business-
a. expenses on incorporation, issue of shares, etc. : ` 92,000.
b. preparation of feasibility report, project report and conducting market survey (the work is completed
by the taxpayer itself) : ` 1,40,000.
c. engineering services (work is carried on by a concern which is not approved by the Board) : ` 1,30,000.
Determine the amount of deduction under section 35D assuming the following figures of fixed assets and
capital on March 31, 2020(i.e., the last day of the year in which the taxpayer starts production)-
Particulars ` in lakhs
Cost of fixed asset 55
Share capital 40
Debentures 12
Long-term borrowing from a financial institution (Repayable for not less than 7 years) 8
ANSWER:193 Rs. 46,400

PROBLEM:194 Preliminary expenditure is allowed as a deduction in ................


(a) 10 equal instalments (b) Five equal instalments (c) 100% is allowed as a deduction
(d) Depreciation is allowed on entire amount.

PROBLEM:195 Which of the following criterion are considered to ascertain deduction for preliminary
expenditure?
(a) Actual amount incurred towards project cost
(b) Higher of 5% of the cost of the project or capital employed in case of an Indian company
(c) 5% of the cost of the Project in case of assessee other than company (d) All of the above

PROBLEM:196 Sky Ltd. started business in FY 2017-18. It paid ROC fees of Rs. 5,000 for increasing
authorized capital in FY 2019-20 not related to extension of a project. Deduction for preliminary
expenditure for such ROC fees shall be Rs. ................
(a) Nil (b) 5,000 (c) 1,000 (d) None of the above

PROBLEM:197 Deduction u/s 35D is not available to ................


(a) Indian Company (b) A non-corporate resident in India
(c) A foreign company who has a permanent establishment in India (d) All of the above.

PROBLEM:198 SONY Limited incurred expenditure of Rs. 10 lakhs to conduct market survey for
assessing demand of product in which business is proposed to be carried out during FY 2018-19. They
started business in FY 2019-20. The deduction ................ available u/s 35D for such expenditure.
(a) Shall be (b) Shall not be (c) Is not (d) Can’t say

PROBLEM:199 STAR Ltd. has a capital employed of Rs. 46,00,000. It incurred preliminary expenses of
Rs. 2,40,000. The deduction available u/s 35D for such expenditure to STAR Ltd. shall be Rs.
................
(A) 2,30,000 (b) 2,40,000 (c) 48,000 (d) None of the above

SALEEM QURAISHEE Mo: 9175664444 INSPIRE ACADEMY-8888881719


PROFITS AND GAINS OF BUSINESS OR PROFESSION- Page 6.54

24. EXPENDITURE IN THE CASE OF AMALGAMATION/ DEMERGER [SEC.


35DD]
I. If an Indian company incurs any expenditure for the purpose of amalgamation or demerger, it is
allowed as deduction in five successive years in five equal installments.
II. The first installment is deductible in the previous year in which amalgamation or demerger takes
place.
III. A similar provision is applicable in the case of amalgamation or demerger of co-operative banks.

PROBLEM:200 Deduction for expenditure incurred by Indian company on Amalgamation or demerger is


allowed as a deduction to the extent of ................
(a) 1/10th of the expenditure, in 10 equal instalments
(b) 1 / 5th of the expenditure, in five equal instalments
(c) Entire expenditure (d) Depreciation

25. EXPENDITURE UNDER VOLUNTARY RETIREMENT SCHEME [SEC. 35DDA]


I. Expenditure by way of payment of any sum to an employee in connection with his voluntary
retirement (under any scheme of voluntary retirement), is deductible in 5 successive years in 5
equal installments.
II. The first installment is deductible in the year in which such amount is actually paid (deduction is
available on payment basis and not on accrual basis).

PROBLEM:201 Expenditure incurred on voluntary retirement scheme is allowed as a deduction to the


extent of
(a) 1/10th of the expenditure in 10 equal instalments
(b) 1/5th of the expenditure in five equal instalments, beginning with the previous year in which
payment is made (c) Entire expenditure is allowed as a deduction (d) Depreciation

SALEEM QURAISHEE Mo: 9175664444 INSPIRE ACADEMY-8888881719


PROFITS AND GAINS OF BUSINESS OR PROFESSION- Page 6.55

26. EXPENDITURE ON PROSPECTING, ETC., FOR DEVELOPMENT OF


CERTAIN MINERALS -SEC. 35E
A. Condition-
I. Deduction under section 35E is allowed only in the case of Indian companies and resident
assessee other than companies.
II. The qualifying expenditure should be incurred during the "year of commercial production" and
four years immediately preceding that year.
III. Expenditure incurred wholly and exclusively on any operations relating to prospecting for any
mineral (or group of associated minerals) specified in the Seventh Schedule or on the
development of a mine or other natural deposit of any such mineral or group of associated
minerals, is "qualifying expenditure".
IV. Few expenses (like expenses met by any other person, expenditure on acquisition of site, capital
expenses on acquiring building, plant, machinery and furniture) are excluded.

B. Amount and period of deduction


I. The amortization of qualifying expenditure is allowed in equal installments over a period of 10
years against the profits arising from commercial exploitation of any mine or natural deposit.
II. If the assessee is a person, other than a company/co-operative society, then books of account of
the relevant year(s) in which the expenditure is incurred should be audited.
III. In the case of amalgamation/ demerger of Indian companies, the above benefit for the unexpired
period will be available to the transferee.

PROBLEM:202 GAIL Ltd., incurred expenditure of Rs. 2,40,000 towards extraction of specified minerals
in year of commercial production and Rs. 5,00,000 within 4 years immediately preceding such year.
Deduction available u/s 35E in the year of production shall be Rs. ................
(a) 74,000 (b) 50,000 (c) 24,000 (d) None of the above

PROBLEM:203 X Ltd. is engaged in mining business and has profits of Rs. 5,00,000 before the deduction
allowable u/s 35E amounting Rs. 8,00,000. The amount which could not be claimed as deduction,
can be carried forward for ................
(a) 10th previous year, as reckoned from year of commercial production
(b) 5th previous year, as reckoned from year of commercial production
(c) 8th previous year, as reckoned from year of commercial production (d) None of the above

27. DEDUCTION UNDER SECTION 36

SALEEM QURAISHEE Mo: 9175664444 INSPIRE ACADEMY-8888881719


PROFITS AND GAINS OF BUSINESS OR PROFESSION- Page 6.56

A. EXPENDITURE INCURRED BY EMPLOYER FOR BENEFITS OF EMPLOYEE-


SECTION PARTICULARS DEDUCTION CONDITIONS
ALLOWED
36(1)(ib) Premium paid by Yes I. An employer can claim deduction in respect of
employer on health of premium paid by him by any mode other than cash
employee for insurance on the health of his employees in
accordance with the scheme framed by the General
Insurance Corporation and approved by the Central
Government or any other insurer and approved by
IRDA.

36(1)(ii) Bonus and commission YES I. Admissible only if not payable as profit or dividend
II. Bonus or commission is allowed as deduction only
where payment is made during the previous year
or on or before the due date of furnishing return
of income under section 139

36(1)(iv) Employers Contribution to YES I. Employer's contribution towards a recognized


recognized provident fund provident fund or an approved superannuation
and approved fund is allowable as deduction.
superannuation fund II. Fund should be for the benefit of employee.
III. Within due date u/s 139(1)

36(1)(v) Contribution towards YES I. Employer's contribution towards an approved


approved gratuity fund gratuity fund created by him exclusively for the
benefit of his employees under an irrevocable
trust is allowable as deduction.
II. Within due date u/s 139(1)

36(1)(iva) Employer's contribution to YES I. Employer's contribution towards NPS is deductible


notified pension scheme (to the extent of 10 per cent of "salary" of
(NPS) employees) with effect from the assessment year
2012-13.

36(1)(va) Employees' contribution YES I. Section 2(24) defines income. Clause (x) of section
towards staff welfare 2(24) provides that any sum received by an
schemes [Sec employer from his employees as contribution to
provident fund (or any fund for the welfare of such
employees) shall be included in the employer's
income.

36(1)(ix) Family planning YES I. Allowed to company assessee only.


expenditure by COMPANY II. Revenue expenditure-100%
III. Capital expenditure-1/5th
NOTE- Unabsorbed C/E carried forward and sett off as
unabsorbed deprecation.

SALEEM QURAISHEE Mo: 9175664444 INSPIRE ACADEMY-8888881719


PROFITS AND GAINS OF BUSINESS OR PROFESSION- Page 6.57

B. CHARGES RELATING TO FINANCE-

SECTION PARTICULARS DEDUCTION CONDITIONS


ALLOWED
36(1)(iii) Interest on borrowed YES I. The assessee must have borrowed money
capital II. The money so borrowed must have been used for the
purpose of business
III. Interest is paid or payable on such borrowing

36(1) INTEREST ON CAPITAL I. Interest liability pertaining to the period beginning


BORROWED FOR from the date on which capital is borrowed by an
ACQUIRING A CAPITAL existing concern for the acquisition of an asset till the
ASSET date, when such asset is first put to use, cannot be
claimed as deduction under section 36.
II. It has to be capitalized.
Only interest on capital borrowed to purchase a capital
asset for business purposes pertaining to the period after
the asset is put to use, is deductible under section 36
36(1)(iiia) Discount on ZERO Coupon YES I. Discount on notified zero coupon bonds (being the
Bond. difference between amount received and the
amount payable on redemption/maturity by the
issuing company) is allowed as deduction on pro rata
basis.
II. The pro rata deduction is available under section
36(1)(iiia) having regard to the period of life of such
bond.(For issuer)
III. "Period of life of the bond" means the period
commencing from the date of issue of the bond and
ending on the date of the maturity or redemption of
such bond.
ZERO COUPON BOND-
I. It is a bond issued by any infrastructure capital
company or infrastructure capital fund or public
sector company or scheduled bank on or after
June 1, 2005.
II. In respect of such bond, no payment/benefit is
received (or receivable) before maturity/
redemption from infrastructure capital company
or infrastructure capital fund or public sector
company or scheduled bank.
III. Such bond is specified by the Central
Government by notification in the Official
Gazette.

SALEEM QURAISHEE Mo: 9175664444 INSPIRE ACADEMY-8888881719


PROFITS AND GAINS OF BUSINESS OR PROFESSION- Page 6.58

C. DEDUCTION IN CASE OF LIVE STOCK-


SECTION PARTICULARS DEDUCTION CONDITIONS
ALLOWED
36(1)(ia) Insurance premium paid yes I. Insurance premium paid by a federal milk co-
by a federal milk co- operative society on the lives of cattle, owned by the
operative society members of a primary milk co-operative society
affiliated to it, is allowable as deduction

36(1)(vi) Write off of allowance for yes I. In respect of animals which are used for the purposes
animals of business or profession (not as stock-in-trade) and
have died or become useless, the difference between
the actual cost of the animals to the assessee and the
amount realized (if any) in respect of carcasses or sale
of animals, is allowable as deduction.

D. DECUTION FOR TAXES TO FINANCIAL SECTOR-


SECTION PARTICULARS DEDUCTION CONDITIONS
ALLOWED
36(1)(xiii) Banking Cash Transaction YES Actually, paid BCTT during the PY will allowed as a
Tax. deduction.
36(1)(xv) Securities Transaction Tax YES I. Taxable securities /Commodities transactions
(xvi) /Commodities Transaction entered into course of business.
Tax II. Income from such transactions chargeable under the
head PGBP.

SALEEM QURAISHEE Mo: 9175664444 INSPIRE ACADEMY-8888881719


PROFITS AND GAINS OF BUSINESS OR PROFESSION- Page 6.59

E. OTHERS
SECTION PARTICULARS DEDUCTION CONDITIONS
ALLOWED
36(1)(i) STOCK INSURANCE YES I. The amount of any premium paid in respect of
insurance against risk of damage or destruction
of stocks or stores, used for the purposes of
business or profession, is allowable as deduction.
36(1)(xiv) Contribution to credit YES I. A public financial institution can claim deduction
guarantee trust fund in respect of its contribution to the Credit
Guarantee Fund Trust for Micro and Small
Enterprises.
36(1)(xiii) Revenue expenditure YES I. Any revenue expenditure incurred by a notified
incurred by entities corporation or a body corporate (by whatever
established under any name called) constituted (or established) by a
Central, State or Provincial Central, State or Provincial Act for the objects and
Act purposes authorized by the Act, shall be allowed
as a deduction.
36(1)(xvii) Expenditure by Co- YES I. With effect from 2016-17, deduction will be
OPERATIVE SOCIETY for allowed in respect of expenditure incurred by a
purchase of sugarcane- co-operative society (engaged in the business of
manufacturing sugar) for purchase of sugarcane
at a price which is equal to or less than the price
fixed or approve by the Government

36(1)(xviii) Marked to market loss YES Marked to market loss or other expected loss as computed in
accordance with the income computation and disclosure
standards notified under sub-section (2) of section 145.

28. BAD DEBTS [SEC. 36(1)]-


I. There must be debt.
II. Debt must be incidental to the business or profession of assesses.
III. Debt must have been taken into account in computing assessable income.
IV. Debt must have been written off in the books of accounts of the assessee.
Other points –
I. It is not necessary to establish that debt has become bad during the relevant previous year.
II. For this purpose, transfer to "provision for bad and doubtful debts account" shall not be taken as bad
debts written off.
III. Debts of a discontinued business not deductible
IV. Allowable in the hands of successor

SALEEM QURAISHEE Mo: 9175664444 INSPIRE ACADEMY-8888881719


PROFITS AND GAINS OF BUSINESS OR PROFESSION- Page 6.60

29. PROVISION FOR BAD AND DOUBTFUL DEBTS RELATING TO RURAL BRANCHES
OF SCHEDULED COMMERCIAL BANKS [SEC. 36(1)(VIIA)] –
A deduction is allowed while computing the taxable profits in respect of any provision for bad and doubtful debts
made by a bank and financial institutions. The amount of deduction is given below:
Amount deductible in respect of provision for bad and doubtful debts

In the case of a scheduled In case of public financial In the case In case of


bank [other than a foreign institution, State financial of a NBFC (FA-
bank], a non-scheduled bank corporation, State Industrial foreign 2016)
and a co-operative bank investment corporation bank

Total income (computed 8.5 per cent of such income 5 per cent of such income 5 per cent 5% of such
before this deduction and (FA-2017) of such Income
amount deductible under income
sections 80C to 80U)

Aggregate average 10 per cent of such advances - -


advances made by rural
branches

30. TRANSFER TO SPECIAL RESERVE [SEC. 36(1) (VIII)] –


A financial corporation, banking company, co-operative bank and a housing finance company can claim deduction
under section 36(1)(viii) as follows, if a few conditions are satisfied:
I. the amount transferred during the previous year to the special reserve account created for the purpose of
section 36(1)(viii); or
II. 20 per cent of the profits derived from the business of providing long-term finance before claiming
deduction under section 36(1)(viii); or
III. 200 per cent of (paid-up share capital and general reserve as on the last day of the previous year) minus
the balance of the special reserve account on the first day of the previous year,
Whichever is lower-

Amount withdrawn from reserve account –


I. If any amount is withdrawn from the aforesaid reserve account [in respect of which deduction was allowed
under section 36(1)(viii)], it will be chargeable to tax in the year in which the amount is withdrawn, under
section 41(4A), regardless of the fact whether the business is in existence in that year or not.

PROBLEM:204 X Ltd. is a financial corporation for the purpose of section 36(1)(viii). Income of the
taxpayer for the previous year 2019-20 from different sources is as follows -(`` in lakh)
a. from providing long-term finance for industrial/agricultural development or development of
infrastructure facility (before any deduction under section 36) 560
b. business income from other activities 105
Compute the amount of deduction under section 36(1)(viii) for the assessment year 2020-21 taking
into consideration the following data -
1. Paid up capital and general reserve on March 31, 2020 ` 610 lakh
2. Balance standing to the credit of special reserve account as on April 1, 2019: ` 1,150 lakh (and the
same was allowed as deduction in the earlier years).
3. Amount transferred to special reserve account during 2019-20 is ` 220 lakh.
ANSWER:204 Rs. 70 lakhs

SALEEM QURAISHEE Mo: 9175664444 INSPIRE ACADEMY-8888881719


PROFITS AND GAINS OF BUSINESS OR PROFESSION- Page 6.61

PROBLEM:205 Ramesh was engaged in the business of trading of Jewellery. During the previous year 2018-19 debt
taken into account by him in computing the income amounted to Rs. 2,50,000. However, during the previous
year 2019-20 it was acknowledged that Rs. 1,35,000 is to be considered as irrecoverable in the accounts of the
assessee. Calculate the amount of bad-debts written off as irrecoverable to be allowed as a deduction in the
previous year 2019-20:
(a) Rs. 2,50,000 (b) Rs. 1,35,000 (c) Rs. 1,15,000 (d) No deduction available

PROBLEM:206 Suresh was engaged in the business of trading of jewellery. During the previous year 2018-19 debt
accrued to him amounted to Rs. 2, 50,000 which was not taken into account by him while computing the income
of the previous year. However, during the year itself it was acknowledged that 1,00,000 is to be considered as
irrecoverable from this debtor. Calculate the amount of bad-debts acknowledged as irrecoverable to be allowed
as a deduction in the previous year 2019-20 .
(a) Rs. 2, 50,000 (b) 1, 00,000 (c) Rs. 1, 50,000 (d) No deduction available -

PROBLEM:207 Jaipur Co-operative Bank made a provision for bad and doubtful debts account against the advances
made by it. In regard to advances made by rural branches it made a provision amounting to 25,00,000 and in
regard to advances made by urban branches it made a provision amounting to Rs. 35,00,000. The total credit
balance provision for bad and doubtful debts account was of Rs. 60, 00,000. The actual bad debts for the
previous year against the urban advances only amounted to Rs. 68,00,000. Calculate the amount of bad-debts to
be allowed as a deduction in the previous year 2019-20 :
(a) Rs. 33, 00,000 (b) 68, 00,000 (c) 8, 00,000 (d) Rs. 43, 00,000

PROBLEM:208 A scheduled bank incorporated in India had Gross Total Income of A.Y. 2019-20 [before deduction
under section 36(1) (viia)] of Rs. 750 lakhs and aggregate average advances made by rural branches of the bank
was of Rs. 150 lakhs. Calculate the amount of Provisions for doubtful debts to be made in the previous year.
(a) Rs. 90 lakhs (b) 67.5 lakhs (c) Rs. 150 lakhs (d) Rs. 78.75 lakhs

PROBLEM:209 A scheduled bank incorporated in India had Gross Total Income of A.Y. 2020-21 [before deduction
under section 36(1) (viia)] of Rs. 750 lakhs and aggregate average advances made by rural branches of the bank
was of 150 lakhs. Provision for bad and doubtful debts under section 36(l)(viia) upto A.Y. 2019-20 was of Rs. 50
lakhs. Calculate the amount of Provisions for doubtful debts to be made in the previous year.
(a) Rs. 90 lakhs (b) Rs. 67.5 lakhs (c) Rs. 150 lakhs (d) Rs. 78.75 lakhs

SALEEM QURAISHEE Mo: 9175664444 INSPIRE ACADEMY-8888881719


PROFITS AND GAINS OF BUSINESS OR PROFESSION- Page 6.62

31. ADVERTISEMENT EXPENSES [SEC. 37(2B)] –


I. Deduction is not available in respect of expenditure incurred by an assessee on advertisement in
any souvenir, brochure, tract, pamphlet or the like published by a political party

32. GENERAL DEDUCTION [SEC. 37(1)]-


I. The expenditure should not be of the nature described under sections 30 to 36.
II. It should not be in the nature of capital expenditure.
III. It should not be personal expenditure of the assessee
IV. It should have been incurred in the previous year.
V. It should be in respect of business carried on by the assessee
VI. It should have been expended wholly and exclusively for the purpose of such business.
VII. It should not have been incurred for any purpose which is an offence or is prohibited by any law.

SALEEM QURAISHEE Mo: 9175664444 INSPIRE ACADEMY-8888881719


PROFITS AND GAINS OF BUSINESS OR PROFESSION- Page 6.63

 INSTANCES OF EXPENSES ALLOWED/DISALLOWED-


1) Litigation expenditure incurred in order to defend or maintain an existing title to the assets is allowed.
2) Legal expenditure incurred to alter the Articles of Association of the company, in conformity with the
amendments in the law is allowed.
3) Expenditure incurred on stamp fee, registration fee etc. on raising loan is allowed.
4) Expenses on registration of trademark are allowed.
5) Compensation paid to a worker in order to dismiss him is allowed.
6) Annual listing fees paid to stock exchanges is allowed
7) Payment for obtaining tenancy right was in the nature of premium is disallowed.
8) Expenditure on valuation of shares is allowed.
9) Contribution to trade syndicate with a view to prevent uneconomic competition is an allowed expenditure.
10) Contribution given for construction of road around factory building is allowed.
11) Deposit made under "Tatkal Telephone Deposit" scheme is allowed.
12) Expenditure incurred on plantation in the business premises is allowed.
13) Expenditure incurred on modernizing and repairing the hotel is allowed.
14) Expenditure incurred on shifting of administrative office as a result of amalgamation is allowed.
15) Substantial repair charges on plant and machinery, provided the repair is of current nature, is allowed.
16) Fees paid for increase of authorized capital is disallowed.
17) Registration expenses paid by the company at the time of registration are disallowed.
18) Expenditure on raising equity and preference share capital is disallowed.
19) Litigation expenditure incurred to protect trade mark of the assessee is allowed.
20) Royalty paid by the assessee for use of trademark of another company is allowed.
21) Amount paid for purchase of loom hours is allowed.
22) Expenditure incurred for dismantling of building for construction of hotel is disallowed.
23) Payment to ward off competition in the business to a rival, being a capital expenditure is disallowed.
24) Expenditure incurred for vacating the property of the assessee shall not be allowed.
25) A reasonable expense on Diwali &Mahurat is allowed.
26) Salary paid to an employee by a partner to look after his interest in the partnership firm is allowed.
27) Professional tax is allowed expenditure
28) Anticipated future expenditure or loss is disallowed.
29) Loss or expenditure relating to any business or profession discontinued before the commencement of previous
year is disallowed.
30) Expenditure on management of temple in factory premises for recreation of employees being treated as
incidental to business and is allowed as deduction.
31) Expenditure incurred by the company on foreign visit of a director and his wife in connection with medical
treatment of the director is allowed.
32) Insurance premium paid by a firm on life insurance policies of its partners is disallowed.
33) Bank guarantee commission for payment of income tax is not allowed.
34) Premium for keyman insurance policy is allowed.
35) Payment to widow of chairman of the board of directors (not an employee of the company) without any
agreement is disallowed.
36) Expenditure incurred on sports tournament organized for publicity is allowed expenditure.
37) Penalty and damages paid in connection with infringement of law is disallowed.
38) Legal expenditure incurred to file a suit against income tax authorities is allowed.
39) Interest on delayed payment of sales tax is allowed.
40) Penalty payable for non-payment of sales tax is disallowed.
41) Contribution to a Union formed for opposing nationalization of business of assessee is allowed.
42) Donation made to a Chief Minister's Drought Relief Fund or District Welfare Fund etc. established by the District
Collector for benefit of public with an intention to secure benefit for the business is allowed.
43) Penalty or damages for breach of contract is allowed but for breach of Contract Act is disallowed
44) Premium paid for loss of profit policy is allowed.
45) Commission paid as a % of profit to General Manager is allowed.
46) Interest on loan paid to proprietor is disallowed.
47) Salary paid to proprietor is disallowed.

SALEEM QURAISHEE Mo: 9175664444 INSPIRE ACADEMY-8888881719


PROFITS AND GAINS OF BUSINESS OR PROFESSION- Page 6.64

PROBLEM:210 Expenditure incurred on family planning amongst the employees is allowed to


a) Any assessee b) a company assessee c) an assessee which is a company or Cooperative Society

PROBLEM:211 Capital expenditure incurred on family planning amongst employees of the company assessee is
allowed as deduction
a) In full b) in 5 equal installments c) in 10 equal installments

PROBLEM:212 Assessee firm incurred revenue expenditure of Rs. 15,000 for promoting family planning amongst its
employees. Calculate the amount of deduction available……………………………..

PROBLEM:213 Assessee company incurred capital expenditure of Rs. 75,000 for promoting family planning
amongst its employees. Calculate the amount of deduction available…………………………………….

PROBLEM:214 Assessee company incurred capital expenditure of Rs. 50,000 and revenue expenditure of Rs. 7,000
for promoting family planning amongst its employees. Calculate the amount of deduction available………………

PROBLEM:215 Assessee company incurred revenue expenditure of 15,000 for promoting family planning amongst
its employees. Calculate the amount of deduction available………………………………………

PROBLEM:216 Contribution was made by PFI towards Credit Guarantee Fund Trust Such expenditure shall be
considered as: ………………………………

PROBLEM:217 Which of the following amounts is allowed as a deduction u/s 36(1)?


(a) Insurance premium against risk of damage or destruction of stocks
(b) Insurance on lives of cattle, owned by members of primary milk cooperative society
(c) Sum paid to an employee as bonus (d) All of the above

PROBLEM:218 SANTOSH Ltd. took a loan of Rs. 50 lakhs for business purpose during FY 2019-20 and
paid Rs. 5 lakhs as interest on such loan. Deduction in respect of such interest in computation of
business income shall be Rs. ................
(a) 5 lakhs (b) Nil (c) 7.5 lakhs (d) None of the above

PROBLEM:219 MAYUR Ltd. took a loan of Rs. 50 lakhs for purchase of assets on 31-3-2018, which were
put to use on 1-4-2019. If annual interest payable on such loan is Rs. 2 lakhs. Deduction in respect
of such interest in computation of business income for FY 2019-20 shall be Rs. ................
(a) 2 lakhs (b) Nil (c) 4 Lakhs (d) None of the above

SALEEM QURAISHEE Mo: 9175664444 INSPIRE ACADEMY-8888881719


PROFITS AND GAINS OF BUSINESS OR PROFESSION- Page 6.65

PROBLEM:220 ROY Ltd. took a loan of Rs. 50 lakhs on 30-6-2018 at interest @ 9% per annum and uses it
to invest in shares on which it earned dividend of Rs. 2 lakhs, which is received in FY 2019-20.
Deduction in respect of such interest in computation of business income for F.Y. 2019-20 shall be Rs.
................
(a) 4.5 lakhs (b) Nil (c) 2.5 lakhs (d) None of the above

PROBLEM:221 A non-scheduled private bank issued 1000 zero coupon bond at a discount of Rs. 10 per
bond and the life of bond is 10 years. The amount of deduction available to such bank is ?
(a) 1,000 (b) 10,000 (c) Nil (d) None of the above

PROBLEM:222 Which assessee can claim deduction in respect of discount on issue of zero-coupon bond?
(a) Infrastructure capital company or fund (b) Public sector or scheduled bank
(c) Private sector bank (other than scheduled bank) (d) Both (a) and (b)

PROBLEM:223 INSPIRE Ltd. contributed Rs. 4,95,000 towards recognized provident fund for welfare of
employees during FY 2019-20 for 11 months. However, contribution of March 2020 of Rs. 45,000
was paid in October 2020. What would be amount of deduction available in respect of provident
fund contribution when INSPIRE Ltd ?
(a) Rs. 4,95,000 (b) Nil (c) Rs. 5,40,000 (d) None of the above

PROBLEM:224 SONY Ltd. contributed Rs. 5 lakhs towards pension scheme (as referred to in section
80CCD) during FY 2019-20. If the salary of employees is Rs. Rs. 5 lakhs, what would be the amount
of deduction available to company in respect of such contribution?
(a) Rs. 5 lakhs (b) Rs. 5 lakhs (c) Nil (d) None of the above

PROBLEM:225 Sunset Ltd. contributed Rs. 6 lakhs towards an approved gratuity fund during FY 2019-20
for 11 months. However, contribution amounting Rs. 50,000 for the month of March 2019 was paid
in October 2020. What would be amount of deduction available in respect of contribution to gratuity
fund during the PY 2019-20 ?
(a) Rs. 6,00,000 (b) Nil (c) Rs. 6,50,000 (d) None of the above

PROBLEM:226 Sunrise Ltd. received an amount of Rs. 60 lakhs towards contribution for provident fund
from its employees during the PY 2019-20. The company has paid all such contribution except Rs. 3
lakhs which was paid in September 2020 before the due date of filing return of income. What would
be the amount of disallowance, if any, for the PY 2019-20?
(a) Rs. 3 lakhs (b) Nil (c) Rs. 57 lakhs (d) None of the above

PROBLEM:227 RAJU has purchased cows and buffalos for its dairy business for Rs. 5 lakhs during PY
2019-20. Calculate the amount of depreciation available to RAJU?
(a) Nil (b) Rs. 5,000 (c) Rs. 50,000 (d) None of the above

SALEEM QURAISHEE Mo: 9175664444 INSPIRE ACADEMY-8888881719


PROFITS AND GAINS OF BUSINESS OR PROFESSION- Page 6.66

PROBLEM:228 Shyam has purchased cows and buffalos for his dairy business for Rs. 5 lakhs during the
PY 2018-19. During the PY 2019-20 these animals becomes permanently unproductive and sold for
Rs. 1 lakh. What would be the amount of deduction available in PY 2019-20?
(a) Nil (b) Rs. 4 lakh (c) Rs. 5 lakhs (d) Rs. 1 lakh

PROBLEM:229 Which of the following is not a mandatory condition to claim deduction of bad debt?
(a) Debtor should be written off as irrecoverable in the books of accounts of the assessee;
(b) Business should be discontinued during the year in which deduction of bad debt is claimed
(c) Debts so written off has been treated as income of the assessee in any earlier year or years
(d) Business is continued during the year in which deduction of bad debt is claimed

PROBLEM:230 Where a deduction has been allowed in respect of a bad debt, then what would be its
taxability in case of its subsequent recovery in succeeding year?
(a) Such recovery shall be taxed as income under the head “Profits and gains of business or
profession”
(b) Such recovery shall be treated as capital receipt of the year when it is recovered
(c) Both (a) and (b)
(d) None of the above

PROBLEM:231 AJIT & Associates has claimed deduction of bad debts of Rs. 50,000 in PY 2018-19,
Thereafter in the P Y 2019-20, its business was shut down due to some adverse market conditions. If
they recover bad debts of Rs. 30,000 out of Rs. 50,000 in the PY 2019-20, what would be the
amount taxable in the PY 2019-20?
(a) Rs. 20,000 (b) Rs. 30,000 (c) Nil (d) None of the above

PROBLEM:232 Aman & Associates made a provision of Rs. 50,000 in respects of Bad & Doubtful debts
for PY 2019-20. What would be the deduction available in respect of such provision?
(a) Rs. 50,000 (b) Nil
(c) Rs. 20,000
(d) None of the above

PROBLEM:233 Which of the following assessee cannot claim deduction of provision for bad and doubtful
debts?
(a) Schedule bank or Co-operative bank
(b) Bank incorporated by or under any Foreign Laws
(c) Primary agriculture credit society or Primary co-operative agriculture and rural development
bank
(d) Non-banking financial company

PROBLEM:234 A proprietorship firm had incurred Rs. 5 lakhs of revenue expenditure, in connection
with family planning amongst employees. The amount allowable as deduction would be ................
(a) Rs. 5 lakhs (b) Rs. 1 lakh (c) Nil (d) None of the above

SALEEM QURAISHEE Mo: 9175664444 INSPIRE ACADEMY-8888881719


PROFITS AND GAINS OF BUSINESS OR PROFESSION- Page 6.67

PROBLEM:235 ABC India Ltd. had incurred Rs. 5 lakhs of revenue expenditure, in connection with
family planning amongst employees. What amount would be allowable as deduction ?
(a) Rs. 5 lakhs (b) Rs. 1 lakh (c) Nil (d) None of the above

PROBLEM:236 What shall be the deduction available to a company for capital expenditure incurred in
connection with family planning amongst employees?
(a) 1 /10th of the amount of expenditure in 10 equal instalments
(b) 1 / 5th of the amount of expenditure in five equal instalments
(c) Entire amount is allowed as a deduction (d) None of the above

PROBLEM:237 What is the time limit on carry forward of unabsorbed capital expenditure on family
planning amongst employees which is incurred by a company?
(a) Indefinite period of time (b) 8 years
(c) 4 years (d) None of the above

PROBLEM:238 ABC Ltd. engaged in the business of share trading. It paid Rs. 1 lakh on account of
Securities Transaction Tax during the PY 2019-20. It had included income from securities as its
income from business or profession. What would be the allowable deduction allowed for STT?
(a) Rs. 1 lakh (b) Nil (c) Rs. 50,000 (d) None of the above

PROBLEM:239 RPL Ltd. is engaged in the business of commodities trading. It paid Rs.
Rs. 1 lakh on account
of Commodities Transaction Tax during P.Y. 2019-20. It included income from commodities as income
from business or profession. What would be the amount of deduction available for Commodities
Transaction Tax?
(a) Rs. 1 lakh (b) Nil (c) Rs. 50,000 (d) None of the above

PROBLEM:240 In order to claim a deduction u/s 37(1), which of the following conditions should be
satisfied?
(a) Expenditure should be incurred during the previous year
(b) Expenditure should be incurred wholly and exclusively for business purpose
(c) Expenditure should not be in the nature of capital expenditure (d) All of the above

PROBLEM:241 XYZ India Ltd. has paid penalty of Rs. 2,000 for late filing of GST return during the PY
2019-20. Select the correct statement from the following?
(a) Rs. 2,000 shall be allowed as deduction
(b) Rs. 2,000 shall be disallowed
(c) Only Rs. 1,000 shall be allowed as deduction
(d) None of the above

PROBLEM:242 GTL India Private Ltd. incurred Rs. 1,000 towards late filing of TDS return during the PY
2019-20, What would be the disallowance, if any for such late filing?
(a) Nil (b) Rs. 1,000 (c) Rs. 500 (d) None of the above

SALEEM QURAISHEE Mo: 9175664444 INSPIRE ACADEMY-8888881719


PROFITS AND GAINS OF BUSINESS OR PROFESSION- Page 6.68

PROBLEM:243 ACC India Private Ltd. paid Rs. 1 lakh for delay in completion of contract during the PY
2019-20. What would be the amount of disallowance, if any?
(a) Nil (b) Rs. 1,00,000( c) Rs. 50,000 (d) None of the above

PROBLEM:244 SAGAR & Associates incurs expenditure of Rs. 30,000 towards corporate social
responsibility (CSR) for the welfare of society. What shall be the amount of deduction allowable u/s
37?
(a) Rs. 30,000 (b) Rs. 3,000 (c) NIL (d) Rs. 45,000

PROBLEM:245 Which of the following conditions are not a pre-condition for allowance of expenditure
u/s 37?
(a) Expenditure should not be covered u/ss 30 to 36
(b) Expenditure should not be of personal nature
(c) Expenditure should be of capital nature
(d) Expenditure should have been incurred wholly or exclusively for business or profession

PROBLEM:246 Which of the following business expenses are not allowable as deduction?
(a) Diwali and Mahurat expenses (b) Payment of telephone charges
(c) Annual listing fee to stock exchange (d) Income tax

PROBLEM:247 What shall be the tax treatment of Rs. 60,000, incurred by ABC Ltd. on advertisement in
a souvenir published by political party?
(a) Rs. 60,000 shall be allowed as deduction (b) Rs. 60,000 shall be disallowed
(c) Rs. 60,000 shall be allowed in the year of payment u/s 43B (d) None of the above

PROBLEM:248 In case the assessee follows mercantile system of accounting, bonus or commission to the
employee are allowed as deduction on:
a) Due basis b) payment basis c) due basis but subject to section 43B.

PROBLEM:249 Interest incurred before the commencement of the production is to be:


a) Capitalized b) treated as revenue expenditure c) either capitalized or treated as revenue
expenditure.

PROBLEM:250 Interest on money borrowed for acquiring an asset by a existing concern for expansion of the
existing business, pertaining to a period prior to the date on which the asset is put to use is to be:
a) Capitalized b) treated as revenue expenditure
c) Either capitalized or treated as revenue expenditure at the option of the assessee till the asset is put to use.
PROBLEM:251 Interest on money borrowed for the purpose of acquiring a capital asset pertaining to the period
after the asset is put to use is to be:
a) Capitalized b) treated as revenue expenditure c) Either capitalized or treated as revenue expenditure

SALEEM QURAISHEE Mo: 9175664444 INSPIRE ACADEMY-8888881719


PROFITS AND GAINS OF BUSINESS OR PROFESSION- Page 6.69

PROBLEM:252 Expenditure incurred on purchase of animals to be used by the assessee for the purpose of
carrying on his business & profession is subject to
a) Depreciation b) deduction in the previous year in which animal dies or become permanently useless
c) Nil deduction

PROBLEM:253 Deductions under section 37(1) shall be allowed of those expenditure which are of:
a) Revenue nature b) capital nature c) both revenue and capital nature

PROBLEM:254 Expenditure incurred by an assessee on the activities relating to corporate social responsibility
referred to in section 135 of the Companies Act, 2013 is :
(a) An allowable expenditure (b) Illegal expenditure (c) Deferred revenue expenditure (d) Not an allowable
expenditure

PROBLEM:255 An assessee paid insurance premium against risk of damage or destruction of stocks or stores used
for the purpose of his business or profession. Such expenditure shall be considered as :
(a) Revenue expenditure (b) Capital expenditure
(c) Deferred revenue expenditure (d) Illegal expenditure

PROBLEM:256 Insurance premium was paid by a Federal Milk Co-operative Society on the life of cattle owned by
member of such co-operative society. Such society was engaged in supplying milk raised by its members to
such federal milk co-operative society. Such expenditure shall be considered as :
(a) Revenue expenditure (b) Capital expenditure
(c) Deferred revenue expenditure (d) None of the above

PROBLEM:257 An assessee purchased a computer on which depreciation is admissible. Such expenditure shall be
considered as :
(a) Revenue expenditure (b) Capital expenditure (c) Deferred revenue expenditure (d) None of the above

PROBLEM:258 One of the employees of the organization was terminated in the interest of business and was paid
one-time compensation of Rs. 75,000. For the organization such expenditure shall be considered as:
(a) Revenue expenditure (b) Capital expenditure (c) Deferred revenue expenditure (d) None of the above

PROBLEM:259 M/s. K and Company paid Rs. 84,000 as advertisement in the annual magazine of Bhartiya Janta
Party. For the organization such expenditure shall be :
(a) Considered as revenue expenditure (b) Considered as capital expenditure (c) Considered as deferred revenue
expenditure (d) Disallowed under Section 37(2B)

SALEEM QURAISHEE Mo: 9175664444 INSPIRE ACADEMY-8888881719


PROFITS AND GAINS OF BUSINESS OR PROFESSION- Page 6.70

PROBLEM:260 M/s. R & Company paid Rs. 75,000 as customs duty and Rs. 84,000 as income tax during the
previous year. Calculate the amount of expenditure allowable :
(a) Rs. 1,59,000 (b) Rs. 84,000 (c) Rs. 75,000 (d) Nil

PROBLEM:261 An employer paid an amount of Rs. 30,000 as insurance premium on the health of his employees
under a scheme framed in this behalf by GIC. Such payment was made by cash. The insurance amount was of
Rs. 3,00,000. The amount of deduction available to employer.
(a) Rs. 30,000 (b) Rs. 3,00,000 (c) Rs. 2,70,000 (d) Nil

PROBLEM:262 An employer paid an amount of Rs. 10,000 as insurance premium on the health of his employees
under a scheme framed by GIC. Such payment was made by cheque. The insurance amount was of Rs.
10,00,000. The amount of deduction available to employer.
(a) Rs. 10,000 (b) Rs. 10,00,000 (c) Rs. 1,00,000 (d) Nil

PROBLEM:263 XYZ Ltd. paid a sum of Rs. 25,000 to Mrs. Seema as employee bonus for services rendered by her
which was otherwise have been payable to her as profit or dividend. The amount allowable as per section
40A(2) was 20,000. Calculate the amount of deduction available.
(a) Rs. 25,000 (b) Rs. 20,000 (c) Rs. 5,000 (d) Nil

PROBLEM:264 XYZ Ltd. paid a sum of Rs. 25,000 to Mrs. Seema as employee bonus for services rendered by her.
Such amount was actually paid to her on 15h June 2019. The amount allowable as per section 40A(2) was Rs.
20,000. Calculate the amount of deduction available.
(a) Rs. 25,000 (b) Rs. 20,000 (c) Rs. 5,000 (d) NIL

PROBLEM:265 XYZ Ltd. paid a sum of Rs. 25,000 to Mrs. Geeta as employee bonus for services rendered by her
during the Previous Year 2019-20. Such amount was actually paid to her on 15 October 2020. The amount
allowable as per section 40A(2) was Rs. 20,000. Calculate the amount of deduction available to her in
Previous Year 2019-20.
(a) Rs. 25,000 (b) Rs. 20,000 (c) Rs. 5,000 (d) Nil

PROBLEM:266 XYZ Ltd. paid a sum of Rs. 25,000 to Mrs. Geeta as employee bonus for services rendered by her
during the Previous Year 2019-20. Such amount was actually paid to her on 15lh August 2020. The amount
allowable as per section 40A(2) was Rs. 28,000. Calculate the amount of deduction available to her in
Previous Year 2019-20.
(a) Rs. 25,000 (b) Rs. 28,000 (c) Rs. 3,000 (d) Nil

SALEEM QURAISHEE Mo: 9175664444 INSPIRE ACADEMY-8888881719


PROFITS AND GAINS OF BUSINESS OR PROFESSION- Page 6.71

PROBLEM:267 The employer made a contribution of Rs. 25,000 to recognized provident fund for the previous
year 2019-20. Such payment was made on 12 March, 2020. Such expenditure shall be considered as:
(a) Revenue expenditure (b) Capital expenditure (c) Deferred revenue expenditure (d) None of the above

PROBLEM:268 Mr. Ram made a contribution of Rs. 45,000 to a Pension Scheme referred under section 80CCD.
The salary of the employee was of Rs. 4,00,000 in the previous year. Calculate the amount of deduction
available to employer in Previous Year 2019-20.
(a) NIL (b) Rs. 45,000 (c) Rs. 40,000 (d) Rs. 4,00,000

PROBLEM:269 The assessee employer received a sum from his employee as contributions to Provident Fund or
Employee State Insurance Fund or Superannuation fund or any other employee-welfare fund. Such sum is first
treated as income of the employer and later claimed as expenditure. Such expenditure shall be considered a
(a) Revenue expenditure (b) Capital expenditure
(c) Deferred revenue expenditure (d) Illegal expenditure

PROBLEM:270 An assessee was engaged in the business of cattle rearing. He incurred a loss in respect of animals
which were used for -the purposes of his business (otherwise than as stock-in trade) and which have died.
Such expenditure shall be considered as:
(a) Revenue expenditure (b) Capital expenditure (c) Deferred revenue expenditure (d) Illegal expenditure

PROBLEM:271 An assessee was engaged in the business of dealing in securities. He had paid Securities
Transaction Tax of Rs. 25,000 on the securities. Income arising from taxable securities transactions computed
under the head "Profits and Gains of Business or Profession" was of Rs. 2,50,000. Such expenditure of
payment of Securities Transaction Tax shall be considered as:
(a) Revenue expenditure (b) Capital expenditure (c) Speculative transaction expenditure (d) Illegal expenditure

PROBLEM:272 Expenses incurred for any purposes, which is an offence or is prohibited under any law are
................
(a) Allowed as a deduction (b) Not allowed as a deduction
(c) Capitalized, to claim tax depreciation (d) None of the above

PROBLEM:273 An assessee was engaged in the business of dealing in commodities. He had paid Commodities
transaction tax of Rs. 15,000 in respect of the taxable commodities transactions. Income arising of Rs.
3,00,000 from such taxable commodities transactions was included in the income computed under the head
"Profits and gains of business or profession". Such expenditure of payment of Commodities transaction tax
shall be considered as:
(a) Revenue expenditure (b) Capital expenditure

SALEEM QURAISHEE Mo: 9175664444 INSPIRE ACADEMY-8888881719


PROFITS AND GAINS OF BUSINESS OR PROFESSION- Page 6.72

PROBLEM:274 To claim deduction of an expenditure u/s 37, the expenditure incurred must be:
(a) In respect of the business or profession carried on by the assessee (b) Not capital in nature
(c) Not of nature described u/s 30 to 36. (d) All of the above.

PROBLEM:275 An assessee paid penalty of 42,000 paid for non-compliance of the provisions of Customs Act.
Such expenditure Shall be considered as:
(a) Revenue expenditure (b) Capital expenditure (c) Speculative transaction expenditure
(d) Disallowed under Section 37(1)

PROBLEM:276 An assessee made an expenditure on issue of shares. Such expenditure, including fees paid to
Registrar of Companies, was incurred to increase the authorized share capital, resulting in expansion of the
capital base. Such expenditure shall be considered as:
(a) Revenue expenditure (b) Capital expenditure (c) Deferred revenue expenditure
(d) Expressly disallowed

PROBLEM:277 An assessee incurred an expenditure on stamp duty and registration fees for the issue of bonus
shares. Such expenditure shall be considered as:
(a) Revenue expenditure (b) Capital expenditure (c) Deferred revenue expenditure
(d) Expressly disallowed

PROBLEM:278 XYZ & Co. incurred a liability by giving discount on issue of debentures. Such expenditure shall be
considered as:
(a) Revenue expenditure (b) Capital expenditure (c) Deferred revenue expenditure (d) Illegal expenditure

PROBLEM:279 Assessee Company redeemed its debentures on premium. Such expenditure of paying premium
shall be considered as:
(a) Revenue expenditure (b) Capital expenditure (c) Deferred revenue expenditure (d) Illegal expenditure

PROBLEM:280 Expenditure incurred by a hotelier on replacement of linen and carpets in his hotel. Such
expenditure shall be considered as:
(a) Revenue expenditure (b) Capital expenditure (c) Deferred revenue expenditure (d) Illegal expenditure

PROBLEM:281 An assessee made a payment of Rs. 25,000 as a secret commission, prohibited by law, for some
offensive purpose. Such expenditure shall be considered as:
(a) Revenue expenditure (b) Capital expenditure (c) Deferred revenue expenditure (d) Non deductible
expenditure

SALEEM QURAISHEE Mo: 9175664444 INSPIRE ACADEMY-8888881719


PROFITS AND GAINS OF BUSINESS OR PROFESSION- Page 6.73

PROBLEM:282 An assessee incurred a sum of Rs. 35,000 for perfecting title or removing defects in title. Such
expenditure shall be considered as:
(a) Revenue expenditure (b) Capital expenditure
(c) Deferred revenue expenditure (d) Non deductible expenditure

PROBLEM:283 An assessee incurred a sum of Rs. 1,10,000 for alteration of the memorandum and articles of
association. Such expenditure shall be considered as:
(a) Revenue expenditure (b) Capital expenditure (c) Deferred revenue expenditure (d) Non deductible
expenditure

PROBLEM:284 An assessee incurred a loss of Rs. 50,000 on account of foreign exchange fluctuations on loans
taken from foreign banks for revenue purposes or trading liabilities. Such loss/expenditure shall be considered
as:
(a) Revenue expenditure (b) Capital expenditure (c) Deferred revenue expenditure (d) Non deductible
expenditure

PROBLEM:285 An assessee incurred an expenditure of Rs. 35,000 on shifting of its administrative office. Such
loss/expenditure shall be considered as:
(a) Revenue expenditure (b) Capital expenditure (c) Deferred revenue expenditure (d) Non deductible
expenditure

PROBLEM:286 Mr. Rakesh paid the income tax due of the previous year 2019-20 on 15 May 2020. When
deduction of this expenditure shall be allowed to him. …………………………………….

SALEEM QURAISHEE Mo: 9175664444 INSPIRE ACADEMY-8888881719


PROFITS AND GAINS OF BUSINESS OR PROFESSION- Page 6.74

33. AMOUNTS NOT DEDUCTIBLE UNDER SECTION 40(A) –


Amounts not deductible (Section 40)
Notwithstanding anything contained in sections 30 to 38, the following amounts shall not be deducted in
computing the income chargeable under the head profits and gains of business or profession.
(1) In the case of any assessee [Section 40(a)]
A. Any interest, royalty, fees for technical services, or other sums chargeable under Income tax Act
which is payable:
a) Outside India; or
b) In India to a non-resident or to a foreign company on which tax has not been deducted before
the end of the previous year or, after deduction, has not been paid before the due date
specified under section 139(1).

POINT TO BE NOTED-

1) However, if TDS is done in subsequent year or has been done in PY but paid in any subsequent year
after the due date of filing of ITR then such sum shall be allowed as deduction in the PY in which
TDS has been paid.
2) However, if the payer has not done TDS or after deducting has not deposited TDS with the
government but payee
I. has furnished his return of income under section 139
II. has taken such income while computing his taxable income and
III. has paid the tax due on the income declared by him in such return of income
Then the payer will be allowed deduction in the PY in which such sum has been paid. (Amended for AY
2020-2021)

 30% of any sum payable to a resident on which tax is deductible at source under sections 192 to
194LA shall not be allowed as deduction in the previous year in which the expense is incurred, while
computing the income chargeable under the head Profits and Gains of business or profession, if in
respect of such expense-
a) tax has not been deducted during the previous year or
b) After deduction tax has not been paid on or before the due date mentioned under section 139(1).

POINT TO BE NOTED-
1) However, if TDS is done in subsequent year or has been done in PY but paid in any subsequent year
after the due date of filing of ITR then such sum shall be allowed as deduction in the PY in which
TDS has been paid.
2) However, if the payer has not done TDS or after deducting has not deposited TDS with the
government but payee
I. has furnished his return of income under section 139
II. has taken such income while computing his taxable income and
III. has paid the tax due on the income declared by him in such return of income
Then the payer will be allowed deduction in the PY in which such sum has been paid. (Amended for AY
2020-2021)

SALEEM QURAISHEE Mo: 9175664444 INSPIRE ACADEMY-8888881719


PROFITS AND GAINS OF BUSINESS OR PROFESSION- Page 6.75

PROBLEM:287 INSPIRE Private Ltd. has paid Rs. 10,000 to Finder India Consultancy, for rendering
certain professional services without deduction of tax at source. Calculate the amount of
disallowance?
(a) Rs. 3000 (b) Rs. 10,000 (c) Rs. 5,000 (d) None of the above

PROBLEM:288 NEW India Ltd. paid Rs. 1,00,000 to an Indian consultancy firm for professional services
without deducting tax at source during the PY 2019-20. Calculate the amount of disallowance if any
in PY 2019-20?
(a) Rs. 30,000 (b) Nil (c) Rs. 1,00,000 (d) None of the above

PROBLEM:289 HLL India Ltd. has paid Rs. 1,00,000 to Beta Inc, a foreign consulting firm without
deducting TDS during the PY 2019-20. What would be the amount disallowed, if any, while computing
income under PGBP?
(a) Rs. 1,00,000 (b) Rs. 30,000 (c) Nil (d) None of the above

PROBLEM:290 NEW India Ltd. paid Rs. 1,00,000 to Boston LLP for royalty charges during PY 2018-19,
which was disallowed due to non-deduction of TDS. Thereafter, NEW India has deducted TDS and
deposited the same in December, 2020. What deduction shall be allowed in P Y 2019-20 on such
amount?
(a) Rs. 1,00,000 (b) Rs. 30,000 (c) Nil (d) None of the above

PROBLEM:291 NFL India Ltd. paid Rs. 1,00,000 to Sigma LLC UK, towards fee for technical services in PY
2019-20 without deduction of TDS. Thereafter, it has deducted TDS and deposited the same in
August, 2020, before due date of filing ITR of PY 2019-20. What would be the amount of disallowance
in PY 2019-20?
(a) Rs. 1,00,000 (b) Rs. 30,000 (c) Nil (d) None of the above

PROBLEM:292 KGN India Ltd. has made payment to a resident contractor amounting Rs. 3,40,000 on
15-06-2019, but tax was deducted at source on 31-03-2020. What shall be the amount of
disallowance if such TDS is deposited on 18-09-2019?
(a) Rs. 3,40,000 (b) Rs. 1,02,000 (c) Nil (d) None of the above

PROBLEM:293 Delta Ltd. credited the following amounts to the account of resident payees in the month
of March, 2020 without deduction of tax at source. What would be the consequence of non-
deduction of tax at source by Delta Ltd. on these amounts during the financial year 2019-20,
assuming that the resident payees in all the cases mentioned below, have not paid the tax, if any,
which was required to be deducted by Delta Ltd.?

Particulars Amount in `

1) Salary to its employees (credited and paid in March, 2020) 12,00,000


2) Directors’ remuneration (credited in March, 2020 and paid in 28,000
April, 2020)
Would your answer change if Delta Ltd. has deducted tax on directors’ remuneration in April, 2020 at the
time of payment and remitted the same in July, 2020?
ANSWER:293

SALEEM QURAISHEE Mo: 9175664444 INSPIRE ACADEMY-8888881719


PROFITS AND GAINS OF BUSINESS OR PROFESSION- Page 6.76

Date on which TDS Actual Date of Time limit as per Date of payment of Previous year in
should have been Deduction section 200(1) for TDS which deductible
deducted depositing TDS

B. DISALLOWANCE OF CONSIDERATION ON NON DEDUCTION/NON PAYMENT OF EQUILISATION


LEVY-F.A 2016
I. If equalization levy is not deducted or if deducted but not been paid before the due date of
filing return of income under section 139(1) the consideration shall not be allowed as
deduction in the PY as per the provision of section 40(ib) of Income tax act.
II. The amount of consideration paid or payable shall be allowed as deduction in the PY in
which payment is made.

C. Tax levied on profits or gains [Section 40(a)(ii)]: Any sum paid on amount of any rate of 'tax' levied
on the profits or gains of any business or profession or assessed at a proportion of, or otherwise on
the basis of any such profits or gains shall not be allowed as deduction;
Taxes which are not deductible
I. Indian income – tax
II. Agricultural income – tax
III. Interest payment in relation to income – tax

D. Taxes paid on income earned outside India Any sum paid outside India and eligible for relief of tax
under section 90 or 90A or deduction from the income – tax payable under section 91 is not
allowable, and is deemed to have never been allowable, as a deduction under section 40 of the
Income – tax Act. However, the tax payers will continue to be eligible for tax credit in respect of
income – tax paid in a foreign country in accordance with the provisions of section 90 or 90A or
section 91, as the case may be.

E. Wealth Tax [Section 40(a)(iia)]: Not allowed as deduction.

SALEEM QURAISHEE Mo: 9175664444 INSPIRE ACADEMY-8888881719


PROFITS AND GAINS OF BUSINESS OR PROFESSION- Page 6.77

F.Disallowance of certain fee, charge, etc. in the case of State Government Undertakings [Section
40(a)(ii)] [W.e.f. A.Y. 2014-15]
I. any amount paid by way of royalty, licence free, servie fee, privilege fee, service charge
or any other fee of charge, by whatever name called, which is levied exclusively on, or
II. any amount appropriated, directly or indirectly, from a State Government undertaking
by the State Government.
shall not be allowed as deduction for the purposes of computation of income of such undertakings under
the head "Profits and gains of business or profession"
G. Salary payable outside India or to a non – resident [Section 40(a)(iii)]: Any payment which is
chargeable under the head "Salaries" shall not be allowed as deduction, where it is payable:
I. outside India, or
II. to a non – resident in India
And if tax at source has not been paid thereon nor deducted as per the provision of the Income tax act.
Points to be noted-
I. If TDS is deposited late even by one day, then salary paid outside India or to a non resident in
India shall not be allowed as deduction.
II. If TDS is not deducted but paid by the assessee from his own pocket, then also salary payable
outside India or to a non resident in India shall not be allowed as deduction.

H. Payment to provident fund or other funds [Section 40(a)(iv)]: Any payment to provident fund or
other fund established for the benefit of employees of the assessee shall not be allowed as
deduction, unless the assessee has made effective arrangements to secure that tax shall be
deducted at source from any payment made from the funds which are chargeable to tax under the
head salaries.

I. Tax paid by employer or non – monetary perquisites provided to employees [Section 40(a)(v)]:
Any tax actually paid by any employer on the perquisites not provided by way of monetary
payment shall not be eligible for deduction while computing the business income of the employer.

SALEEM QURAISHEE Mo: 9175664444 INSPIRE ACADEMY-8888881719


PROFITS AND GAINS OF BUSINESS OR PROFESSION- Page 6.78

34. AMOUNTS NOT DEDUCTIBLE IN RESPECT OF PAYMENT TO RELATIVES


[SEC. 40 A(2)] -
A. Where the assessee incurs any expenditure, in respect of which payment has been made or is to be
made to relatives or close associates of the assessee, and the Assessing Officer is of the opinion that
such expenditure is excessive or unreasonable having regard to the fair market value of the goods,
services or facilities for which the payment is made or the legitimate needs of the business or
profession of the assessee or the benefit derived or accruing to him there from , so much of the
expenditure, as it so considered by him to be excessive or unreasonable, shall not be allowed as a
deduction.

B. Provided that no disallowance on account of any expenditure being excessive or unreasonable having
regard to the fair market value shall be made in respect of a specified domestic transactions referred
to in section 92BA ,if such transaction is at arm’s length price as defined in section 92E.

C. Therefore, for an amount to be disallowed under this Section, three conditions have to be fulfilled:
I. the payment is in respect of any expenditure
II. The payment has been made or is to be made to relative or close associates of the assessee in
respect of such expenditure.
III. the payment for the expenditure is considered excessive or unreasonable having regard to:
(a) the market value of the goods, services or facilities; or
(b) the legitimate business needs of the assessee's business or professions; or
(c) The benefit derived by or accruing to the assessee from the payment.
If the above condition are fulfilled the Assessing Officer can disallow the expenditure to the extent
the considers if excessive or unreasonable by the above objective standards or otherwise.
D. SUBSTANTIAL INTEREST

PROBLEM:294 In which of the following cases, deduction of section 40A(2) relating to disallowance of
excess payment to related party not attracted ?
(a) Payment is in respect of any expenditure (b) Payment have not been made to a specified person
(c) Payment is considered excessive or unreasonable as per fair market value of such services
(d) All of the above

PROBLEM:295 XYZ Ltd. buys 100 chairs @ 3,500 from Raju, one of the Director of company. The market
value of chair is Rs. 3,000. What would be the disallowance as per section 40A(2) ?
(a) Rs. 50,000 (b) Rs. 5,000 (c) Nil (d) Rs. 2,50,000

SALEEM QURAISHEE Mo: 9175664444 INSPIRE ACADEMY-8888881719


PROFITS AND GAINS OF BUSINESS OR PROFESSION- Page 6.79

35. AMOUNTS NOT DEDUCTIBLE IN RESPECT OF EXPENDITURE EXCEEDING


` 10,000 [SEC. 40A(3)] –
A. RULE - The following conditions should be satisfied:
I. The assessee incurs any expenditure which is otherwise deductible under the other provisions of
the Act for computing business /profession income (e.g., expenditure for purchase of raw
material, trading goods, expenditure on salary, etc.). The amount of expenditure exceeds `
10000
II. A payment (or aggregate of payments made to a person in a day) in respect of the above
expenditure exceeds ` 10,000
III. The payment is made otherwise than by an account payee cheque or an account payee demand
draft or use of ECS OR ANY OTHER MODE AS MAY BE PRESCRIBED.
If all the above conditions are satisfied, 100 per cent of such payment will be disallowed.

B. EXCEPTIONS - Rule 6DD prescribes the following circumstances under which no disallowance will
be made of the expenditure even if the payment exceeding ` 10,000 is made otherwise than by an
account payee cheque or demand draft:
I. Payment made to banking and other credit institutions,
II. Payment made to Government (both Central and State Governments)
III. Payment through the banking system.
IV. Payment made by book adjustment by an assessee in the account of the payee against money
due to the assessee for any goods supplied or services rendered by him to the payee
V. Payment to a cultivator, grower or producer in respect of the purchase of agricultural or forest
produce or product of animal husbandry (including livestock, meat, hides and skins) or dairy or
poultry farming or fish or fish products or products of horticulture or apiculture
VI. Payment made to a producer in respect of the purchase of the products manufactured or
processed without the aid of power in a cottage industry
VII. Payment made to a person who ordinarily resides or carries on business in a village not served by
any bank
VIII. Payment of terminal benefits, such as gratuity, retrenchment compensation, etc., not exceeding
` 50,000
IX. Payment made by an assessee by way of salary to his employee after deducting the income-tax
from salary in accordance with the provisions of section 192 and when such employee -
 is temporarily posted for a continuous period of 15 days or more in a place other than his
normal place of duty or on a ship; and
 does not maintain any account in any bank at such place or ship
X. Payment required to be made on a day on which the banks were closed either on account of
holiday or strike
XI. Payment made by any person to his agent who is required to make payment in cash for goods or
services on behalf of such person
XII. Payment made by an authorized dealer or a money changer against purchase of foreign currency
or travelers cheques in the normal course of his business

 POINT TO BE NOTED-

SALEEM QURAISHEE Mo: 9175664444 INSPIRE ACADEMY-8888881719


PROFITS AND GAINS OF BUSINESS OR PROFESSION- Page 6.80

PROBLEM:296 Will the provision of section 40A(3) be attracted in the following cases:

Questions Answers

P.Y. 2019-20(A.Y. 2020-21)

1) R purchases goods worth `30,000 from G against


one bill but makes payment of `18,000 & `12,000 at
different times on the same date.
2) R makes a payment of `40,000 as donation by
crossed cheque to National Defence Fund.
3) R makes a purchase of goods of `60,000 and makes
payment of `45,000 by account payee cheque and
`15,000 in cash.
4) R makes a purchase of goods of `60,000 on 14-2-
2020 and makes the payment as under;
(i) `30,000 by account payee cheque on 15-2-2020
(ii) `15,000 in cash on 15-2-2020
(iii) `15,000 in cash on 16-2-2020

5) R purchases a building for `10,00,000 and makes


the payment in cash
6) R, a dealer in real estate purchases buildings for
`10,00,000 and makes the payment by crossed
cheque.
7) R makes an advance of `30,000 on 14-8-2019 by
crossed cheque for purchase of goods and delivery
of the goods is made on 20-9-2019.

8) R pays a salary of `24,000 by crossed cheque to an


employee.
9) R purchases goods in cash from his brother for
`40,000, whose market value is `35,000.

10) R purchases goods in cash from his brother for


`21,000, whose market value is `19,000.

11) R purchases goods in cash for `40,000 from X, a


villager and makes payment to X in his village where
no banking facility is available.
12) In case given in (11) what will be answer, if R makes
the payment to X (the villager) in town where
banking facility is available.
13) R makes a payment in cash amounting to `35,000
to a transporter
ANSWER:296
SALEEM QURAISHEE Mo: 9175664444 INSPIRE ACADEMY-8888881719
PROFITS AND GAINS OF BUSINESS OR PROFESSION- Page 6.81

PROBLEM:297 Determine the amount of disallowance in the cases given below-


1. X pays salary to his employees by account payee cheques. Salary of December 2019 is, however, paid
to three employees A, B and C by bearer cheques (payment being ` 6,000, ` 10,000 and ` 10,500,
respectively).
2. X Ltd. purchases goods on credit from Y Ltd. on May 6, 2019 for ` 86,000 which is paid as follows—
a. ` 5,000 in cash on May 11,2019;
b. ` 30,000 by a bearer cheque on May 31, 2019;
c. ` 51,000 by an account payee cheque on May 16, 2019.
3. Z Ltd. purchases goods on credit from A Ltd. on May 10, 2019for ` 6,000 and on May 30, 2019 for `
5,000. The total payment of ` 11,000 is made by a crossed cheque on June 1, 2019.
4. A Ltd. purchases goods on credit from a relative of a director on June 20, 2019 for ` 50,000 (market
value :`42,000). The amount is paid in cash on June 25, 2019.
5. B Ltd. purchases raw material on credit from A who holds 20 per cent equity share capital in B Ltd. (the
amount of bill being ` 36,000, market price being ` 19,000). It is paid in cash on July 26, 2019.
ANSWER:297

PROBLEM:298 Mr. A has made cash payments for purchase of goods to Mr. B in month of April 2019. In
which of the following cases, such payment would be disallowed?
(a) Cash payment of Rs. 5,000 each was made on April 1, April 2 and April 3, 2019
(b) Cash payment of Rs. 10,000 each is made to Mr. B on April 1, April 2 and April 3, 2019
(c) Cash payment of Rs. 10,000 each is made three times on April 5,2019 (d) None of the above

PROBLEM:299 INSPIRE India Ltd. made cash payment of Rs. 30,000 to Life Insurance Corporation (LIC).
The disallowance in respect of such payment shall be ................
(a) Rs. 30,000 (b) Nil (c) Rs. 9,000 (d) None of the above

36. PROVISION FOR UNAPPROVED GRATUITY FUND [SEC. 40A(7)]:


I. Provision for gratuity fund (for meeting future liability) is deductible only if such gratuity fund is
an approved gratuity fund.
II. In other words, any provision for unapproved gratuity fund (for meeting future liability) is not
deductible.

PROBLEM:300 MV India Ltd. contributed Rs. 20 lakhs during the PY 2019-20 towards an approved
gratuity fund. Calculate the amount of allowable deduction?
(a) Rs. 20 lakhs (b) Rs. 6 lakhs (c) Nil (d) None of the above

PROBLEM:301 V India Ltd. made provision of Rs. 20 lakhs as contribution towards an approved gratuity
fund. It paid Rs. 10 lakhs out of such provision before due date of filing ITR. Calculate the amount of
allowable deduction?
(a) Rs. 20 lakhs (b) Rs. 10 lakhs (c) Nil (d) None of the above

PROBLEM:302 MM India Ltd. made provision for gratuity of Rs. 10 lakhs during PY 2019-20. Calculate
the amount of deduction allowable for such provision, where the amount has not become payable
and is not for an approved gratuity fund?
(a) Rs. 10 lakhs (b) Rs. 3 lakhs (c) Nil (d) None of the above

SALEEM QURAISHEE Mo: 9175664444 INSPIRE ACADEMY-8888881719


PROFITS AND GAINS OF BUSINESS OR PROFESSION- Page 6.82

37. CONTRIBUTIONS TO NON-STATUTORY FUNDS


I. The contribution/payment is made by an assessee as an employer.
II. It is paid towards setting up (or formation of) any trust, company, association of persons, body of
individuals, society or it is paid by way of contribution to any fund.
III. The contribution or payment is not required by any law

38. MARKED TO MARKET LOSS OR OTHER EXPECTED LOSS [SEC.40A(13)]


I. No deduction or allowance shall be allowed in respect of any marked to market loss or other
expected loss except as allowable u/s 36(1)(xviii).
II. In other words, loss can be allowed only if it is computed accordance with the provisions of
ICDS.

39. SPECIAL PROVISIONS FOR DEDUCTION IN CASE FOR BUSINESS FOR


PROSPECTING CERTAIN MINERAL OIL SECTION 42
A. The section grants certain special allowance to the oil-producing assessee who undertake the
business is the agree area und an agreement and in association with the Central Government, etc.
Unless it is specially provided for in the agreement, no special benefit under this section is allowable.
"Mineral oil" in section 42 shall include petroleum and natural gas.

B. SECTION 42(1) IS APPLICABLE WHEN THE FOLLOWING CONDITIONS ARE SATISFIED:


I. There is business consisting of the prospecting for or extraction or production of mineral oils.
II.in relation to such business, the Central Government has entered into an agreement with such assessee
for the association of participation of the Central Government or any person authorized by the
Central Government in such business: and
III.Such agreement has been laid on the Table of each House of Parliament.

C. If the condition mentioned above are satisfied, the section provides for the following allowance
shall be allowed, in lieu of or in addition to the normal allowance admissible under other provisions
of the Act:
I. if, in the course of exploration in respect of an area surrendered prior to the beginning of
commercial production, any expense has been incurred by the exploration proved to be
infractions or abortive, such expenditure is to be allowed as deduction the computation of
the business profits of the assessee;
II. Expenditure incurred, whether before or after the beginning of commercial production, by
the assessee in respect of drilling or exploration activities or services in respect of physical
assets used in that connection.

SALEEM QURAISHEE Mo: 9175664444 INSPIRE ACADEMY-8888881719


PROFITS AND GAINS OF BUSINESS OR PROFESSION- Page 6.83

40. CHANGES IN THE RATE OF EXCHANGE OF CURRENCY SECTION 43A


Section 43A shall be applicable if the following conditions are satisfied.
I. The asset is acquired from a country outside India.
II. It is either acquired through a foreign supplier's credit or a loan in a foreign currency.
III. It is acquired for the purpose of assessee's business and profession.
If the above conditions are satisfied and there is a change in the rate of exchange which results into an
increase or decrease in the liability of the assessee as expressed in Indian currency, such increase or
decrease shall be made only in the previous year in which actual payment is made to the foreign supplier
or to repay the foreign currency loan.
POINT TO BE NOTED-
1) Adjustment referred to in section 43A shall not be made if the supplier's credit or the foreign
currency loan is reinstated at the rates of exchange prevailing on 31st March every year but no
actual payment is made.
2) As per section 43A, the increase or decrease in liability at the time of payment has to be adjusted
from the actual cost of the asset. Cost referred to in section 43A should be read as actual cost
minus depreciation allowed till date.
3) Where foreign exchange fluctuations is on revenue account i.e. debtors for exports, creditors for
imports or creditors for other expenses, such fluctuation if results into profits/loss, it shall be given
effect to while computing income under the head Profits and Gains of Business or Profession.
4) Where the fluctuation in foreign exchange arises after the asset has been transferred by the block
of assets still exists, such fluctuation shall be added/deducted from WDV of the block of assets.
However, if such fluctuation arises after the block ceases to exist, there will be no tax treatment for
such fluctuation/ Such gain/loss shall be treated as capital receipt/capital loss having no tax
treatment

41. TAXATION OF FOREIGN EXCHANGE FLUCTUATION-SECTION 43AA


1) Subject to the provisions of section 43A, any gain or loss arising on account of any change in foreign
exchange rates shall be treated as income or loss, as the case may be, and such gain or loss shall be
computed in accordance with the income computation and disclosure standards notified under
sub-section (2) of section 145.
2) For the purposes of sub-section (1), gain or loss arising on account of the effects of change in
foreign exchange rates shall be in respect of all foreign currency transactions, including those
relating to—
I. monetary items and non-monetary items;
II. translation of financial statements of foreign operations;
III. forward exchange contracts;
IV. Foreign currency translation reserves.

SALEEM QURAISHEE Mo: 9175664444 INSPIRE ACADEMY-8888881719


PROFITS AND GAINS OF BUSINESS OR PROFESSION- Page 6.84

42. AMOUNT NOT DEDUCTIBLE IN RESPECT OF CERTAIN UNPAID


LIABILITIES [SEC. 43B]:
 Section 43B is applicable only if the taxpayer maintains books of account on the basis of
mercantile system of accounting.
 GENERAL RULE - CERTAIN EXPENSES ARE DEDUCTIBLE ON PAYMENT BASIS –
 The following expenses (which are otherwise deductible under the other provisions of the
Income-tax Act) are deductible on payment basis—
a) Any indirect tax, duty, cess or fees under any law in force.
b) Contribution by employer to any PF or superannuation fund or gratuity fund or ESI or any other
fund for the welfare of employees
c) Any sum payable to an employee as bonus or commission
d) Interest on loan or borrowings from any public financial institution or state financial corporation or
state industrial financial corporations
e) any sum payable by the assessee as interest on any loan or borrowing from a deposit taking non-
banking financial company or systemically important non-deposit taking non-banking financial
company.
 Further, deduction shall be allowed if such interest has been actually paid and any interest which
has been converted into a loan or borrowing shall not be deemed to have been actually paid.
(Amended from A Y 2020-2021)
 Note: Systemically important non-deposit taking non-banking financial company means a non-
banking financial company which is not accepting or holding public deposits and having total
assets of not less than five hundred crore rupees as per the last audited balance sheet and is
registered with the Reserve Bank of India (Amended from AY 2020-2021)
f) Interest on loan or advances from a scheduled bank or a co-operative bank other than a primary
agricultural credit society or a primary co-operative agricultural and rural development bank Note:
Deduction of interest shall be allowed if such interest has been actually paid and if interest has
been converted into a loan or advance then it shall not be considered to have been actually paid
g) Payment of leave encashment to employee.
h) any sum payable by the assessee to the Indian Railways for use of Railway assets.

The above expenses are deductible in the year in which payment is actually made. There is, however,
one exception
EXCEPTION - WHEN DEDUCTIBLE ON ACCRUAL BASIS - The exception is applicable if the following two
conditions are satisfied-
I. Payment in respect of the aforesaid expenses is actually made on or before the due date of
submission of return of income.
II. The evidence of such payment is submitted along with the return of income

PROBLEM:303 Which of the following payments are deductible on payment basis u/s 43B?
(a) Employer’s contribution to RPF, subject to the limits set up for approving the said fund
(b) Employer’s contribution to an approved gratuity fund (c) Both (a) and (b)
(d) None of the above

SALEEM QURAISHEE Mo: 9175664444 INSPIRE ACADEMY-8888881719


PROFITS AND GAINS OF BUSINESS OR PROFESSION- Page 6.85

PROBLEM:304 YAHOO India Ltd. made provision of Rs. 10 lakhs for payment of bonus in PY 2019-20. If it
paid bonus of Rs. 5 lakhs in July 2020, calculate the amount of deduction allowable in PY 2019-20 ?
ANSWER:304

PROBLEM:305 FAST India Ltd. was liable to pay Rs. 50,000 towards income tax to Government for FY
2019-20. What would be the amount of disallowance, if such amount is paid after due date of filing
ITR?
(a) Rs. 50,000 (b) Rs. 15,000 (c) Rs. 10,000 (d) Nil

PROBLEM:306 Mr. RONY , had a contribution of Rs. 50,000 due as employer’s contribution towards
provident fund. Out of such, Rs. 25,000 was contributed during PY 2019-20, while remaining Rs.
25,000 was contributed in October, 2020, after due date of filing of return of income. Calculate the
disallowance, if any?
(a) Rs. 25,000 (b) Rs. 50,000 (c) Nil (d) None of the above

PROBLEM:307 In which of the following cases, deduction shall be allowed during the year of accrual for
any sum in nature of tax, duty, cess or fee under any law which is accrued during the financial year
but not paid by end of financial year?
(a) When payment is made before the due date of filing return u/s 139(1)
(b) When payment is made before the end of assessment year
(c) When payment is made after the end of assessment year (d) None of the above

PROBLEM:308 Which of the following expenditures are not covered u/s 43B?
(a) Employer’s contribution to any provident fund
(b) Interest on any loan or borrowing from public financial institution
(c) Leave encashment from employer (d) Salary

PROBLEM:309 From the information given below, find out how much amount will be allowed. These
expenses relate to previous year 2019-20. The due date of furnishing the return of income is 30-9-
2020.
Sr.No. Expenses Amount & Date of actual payment

1. Interest on term loan from bank ` 3,00,000 on 4-6-2020


` 2,00,000 on 31-12-2020

2. Employer's contribution to Provident Fund ` 45,000 on 22-4-2020


(due date 15-4-2018+ 5 days grace)

3. Advance Income tax ` 37,000 on 15-9-2020

4. GST ` 3,50,000 paid on 5-11-2020


ANSWER:309

SALEEM QURAISHEE Mo: 9175664444 INSPIRE ACADEMY-8888881719


PROFITS AND GAINS OF BUSINESS OR PROFESSION- Page 6.86

43. SPECIAL PROVISION FOR COMPUTATION OF COST OF ACQUISITION OF


CERTAIN ASSETS (SEC 43C)
I. Where an amalgamating company transfers a capital asset as stock in trade to the amalgamated company
and such stock in trade is sold by the amalgamated company, then the cost of acquisition of stock in trade
shall be the aggregate of the following:
a) Cost of acquisition of the said asset to the amalgamating company,

b) Expenditure on improvement of the said asset incurred by the amalgamating company and
amalgamated company and

c) Expenditure incurred wholly and exclusively in connection with the transfer by the amalgamating
company,

II. Where a capital asset is transferred as stock in trade under a gift, will, irrevocable trust or on partition of
HUF and such stock in trade is sold, then the cost of acquisition of stock-in-trade shall be the aggregate of
the following:
a) Cost of acquisition of the said asset to the transferor.

b) Expenditure on improvement of the said asset incurred by transferor and transferee and

c) Expenditure incurred wholly and exclusively in connection with the transfer by the transferor including
any gift-tax, probate fees, expenditure incurred to effect the partition of HUF or to create a trust.

Note 1: Option to take FMV as on 01.04.2001 to be the cost is not available for section 43C.
Note 2: Cost of Improvement incurred before 01.04.2001 shall be considered for section 43C.

SALEEM QURAISHEE Mo: 9175664444 INSPIRE ACADEMY-8888881719


PROFITS AND GAINS OF BUSINESS OR PROFESSION- Page 6.87

44. STAMP DUTY VALUE OF LAND AND BUILDING TO BE TAKEN AS THE


FULL VALUE OF CONSIDERATION IN RESPECT OF TRANSFER, IF EVEN IF
SAME ARE HELD BY THE TRANSFEROR AS STOCK IN TRADE- SECTION
43CA
1) Where the consideration received or accruing as a result of the transfer by an assessee of an asset (other
than a capital asset), being land or building or both, is less than the value adopted or assessed or assessable
by any authority of a State Government for the purpose of payment of stamp duty in respect of such
transfer, the value so adopted or assessed or assessable shall, for the purposes of computing profits and
gains from transfer of such asset, be deemed to be the full value of the consideration received or accruing as
a result of such transfer.

Following proviso shall be inserted in sub-section (1) of section 43CA by the Finance Act, 2018, w.e.f. 1-4-
2019 : Provided that where the value adopted or assessed or assessable by the authority for the purpose of
payment of stamp duty does not exceed one hundred and five per cent of the consideration received or
accruing as a result of the transfer, the consideration so received or accruing as a result of the transfer shall,
for the purposes of computing profits and gains from transfer of such asset, be deemed to be the full value
of the consideration.

2) The provisions of sub-section (2) and sub-section (3) of section 50C shall, so far as may be, apply in relation
to determination of the value adopted or assessed or assessable under sub-section (1).

3) Where the date of agreement fixing the value of consideration for transfer of the asset and the date of
registration of such transfer of asset are not the same, the value referred to in sub-section (1) may be taken
as the value assessable by any authority of a State Government for the purpose of payment of stamp duty in
respect of such transfer on the date of the agreement.

4) The provisions of sub-section (3) shall apply only in a case where the amount of consideration or
a part thereof has been received by WAY of an account payee cheque or an account payee
demand draft or by use of ECS through a bank account OR ANY OTHER ELECTRONIC MODE AS
MAY BE PRESSCRIBED on or before the date of agreement for transfer of the asset

SALEEM QURAISHEE Mo: 9175664444 INSPIRE ACADEMY-8888881719


PROFITS AND GAINS OF BUSINESS OR PROFESSION- Page 6.88

PROBLEM:310 An assessee transferred his land (stock in trade) on 15"' May 2019 for Rs. 75,00,000. However, the
value adopted by State Government authority for the purpose of payment of stamp duty in respect of such
transfer was Rs. 90,00,000. What shall be the full value of the consideration received or accruing as a result of
such transfer for the purposes of computing profits and gains from transfer of such asset…………………………….

PROBLEM:311 Mahesh transferred his land (stock in trade) on 21" June 2019 for Rs. 80,00,000. However, the value
adopted by State Government authority for the purpose of payment of stamp duty in respect of such transfer
was Rs. 72,00,000. What shall be the full value of the consideration received or accruing as a result of such
transfer for the purposes of computing profits and gains from transfer of such asset………………………….

PROBLEM:312 Mukesh transferred his land (stock in trade) on 1St June 2019 for 80,00,000. However, the value
adopted by State Government authority for the purpose of payment of stamp duty in respect of such transfer on
the date of agreement fixing the value of consideration for transfer of the asset was Rs. 85,00,000 and on the
date of registration of such transfer of asset was Rs. 83,00,000. He received consideration of Rs. 25,00,000 by
cheque before the date of agreement. What shall be the full value of the consideration for the purposes of
computing profits and gains from transfer of such asset…………………………………………

PROBLEM:313 Hari transferred his land on 1st September 2019 for 80,00,000. However, the value adopted by State
Government authority for the purpose of payment of stamp duty in respect of such transfer on the date of
agreement fixing the value of consideration for transfer of the asset was Rs. 75,00,000 and on the date of
registration of such transfer of asset was Rs. 73,00,000. He received consideration of Rs. 25,00,000 by cheque
before the date of agreement. What shall be the full value of the consideration for the purposes of computing
profits and gains from transfer of such asset.

45. COMPUTATION OF INCOME FROM CONSTRUCTION AND SERVICE


CONTRACTS. SECTION 43CB
1) The profits and gains arising from a construction contract or a contract for providing services shall be
determined on the basis of percentage of completion method in accordance with the income computation and
disclosure standards notified under sub-section (2) of section 145 43c :
Provided that profits and gains arising from a contract for providing services, —
I. with duration of not more than ninety days shall be determined on the basis of project completion
method;
II. Involving indeterminate number of acts over a specific period of time shall be determined on the basis of
straight-line method.

2) For the purposes of percentage of completion method, project completion method or straight line method
referred to in sub-section (1)—
I. the contract revenue shall include retention money;
II. The contract costs shall not be reduced by any incidental income in the nature of interest, dividends or
capital gains. ]

SALEEM QURAISHEE Mo: 9175664444 INSPIRE ACADEMY-8888881719


PROFITS AND GAINS OF BUSINESS OR PROFESSION- Page 6.89

46. DEEMED PROFITS AND HOW THEY ARE CHARGED TO TAX-


Deduction Nature of Receipt treated as Deemed income Year in which taxable
already
allowed u/s
30-38 Recovery of loss or Expenditure or trading liability which Year in which recovered or
was already allowed, including remission or cessation of written off by the assessee
liability effected by a unilateral act. by remission or cessation.
32(1)(i) Balancing charge on assets in respect of which Taxable in the year in which
depreciation is claimed, is sold / discarded / demolished / amount becomes due.
destroyed
Balancing charge = Net Consideration Less WDV.
35(2) Amount realized on sale of Capital Assets used for Year in which transfer takes
Scientific Research. place
36(1)(vii) Bad Debts earlier allowed subsequently recovered by the Year in which it is recovered
assessee.
36(1)(viii) Amount withdrawn from Special Reserve created. Year in which it is
withdrawn
- Benefit of set-off loss: Unabsorbed loss pertaining to the Deemed Business Income =
year in which the business or profession was discontinued, Income u/s
is permitted to be set off against Deemed Business Income 41(1)/(3)/(4)/(4A)
u/s 41(1)(3)(4)(4A) Less : Loss of Discounted
Business.

PROBLEM:314 Which of the following shall be taxed under the head ‘Profit and gains of business or
profession’, even when there is no business existence for which such expenditure pertains?
(a) Recovery against any loss, expenditure or trading liability earlier allowed as a deduction
(b) Balancing charge for electricity companies
(c) Recovery against bad debts (d) All the above

PROBLEM:315 Which of the following conditions need to be satisfied in order to tax the trading liability
written back subsequently in next year?
(a) Business of assessee should be in existence in that year
(b) Deduction should have been claimed in the earlier year in respect of such trading liability
(c) None of the above

PROBLEM:316 Mr. Rajesh has purchased material worth Rs. 5 lakhs on credit basis from Mr. Bharat
during the PY2018-19. Due to some dispute regarding the defective material Mr. Bharat has made
final payment of Rs. 3 lakhs in the PY 2019-20. What would be the amount which is taxable in
hands of Mr. Rajesh?
(a) Rs. 5 lakhs (b) Rs. 2 lakhs (c) Rs. 3 lakhs (d) None of the above

PROBLEM:317 After amalgamation of A Ltd. with B Ltd., certain trading liability was waived off in respect
of an expenditure which was claimed as deduction by A Ltd. Who would be liable to pay tax on such
waiver?
(a) A Ltd. (b) B Ltd. (c) Both (d) None of the above

SALEEM QURAISHEE Mo: 9175664444 INSPIRE ACADEMY-8888881719


PROFITS AND GAINS OF BUSINESS OR PROFESSION- Page 6.90

PROBLEM:318 Mayur & Associates has claimed deduction of bad debts of Rs. 50,000 in PY 2018-19.
Thereafter, in the PY 2019-20, such business has been closed due to some adverse market conditions.
However, it has recovered Rs. 30,000 out of such bad debts in the PY 2019-20. In which of the
following year, would such recovery be taxable?
(a) PY 2019-20 (b) PY 2018-19 (c) AY 2017-18 (d) None of the above

47. UNDISCLOSED INCOME/INVESTMENTS-


The following are treated as income from undisclosed sources:
I. Cash credit [Sec. 68]-
II. Unexplained investments [Sec. 69]-
III. Unexplained money, etc. [Sec. 69A] –
IV. Amount of investments, etc., not fully disclosed in books of account [Sec. 69B]-
V. Unexplained expenditure, etc. [Sec. 69C] –
VI. Amount borrowed or repaid on hundi [Sec. 69D] –

PROBLEM:319 In which of the following cases, cash credits shall be treated as income of the assessee?
(a) Where an assessee does not offer an explanation about the nature and source of such income
(b) Where an assessee offers an explanation about such credit, which is not found adequate by the
AO (c) Where an assessee offers an explanation about such credit and satisfies the AO
(d) Both (a) and (b)

PROBLEM:320 Aakash, a non-resident assessee invested Rs. 5 lakhs in share of X Pvt. Ltd. a closely held
company. However, he was unable to explain the source of such investment. What amount would be
taxable in hands of Aakash u/s 68?
(a) Rs. 5 lakhs (b) Nil (c) Rs.5 lakhs (d) None of the above

PROBLEM:321 X Pvt. Ltd. a closely held company received Rs. 1 lakh as share application from Yogesh
(a resident assessee), who was unable to explain the source of such investment. Which of the
following person are liable to pay tax on such unexplained income?
(a) X Pvt. Ltd. (b) Yogesh (c) Both (a) and (b) (d) None of the above

PROBLEM:322 Income-tax Department surveyed the premises of Mr. Amar during the FY 2018-19 and
found bullion of Rs. 50 lakhs. Mr. Amar had acquired such bullion during FY 2016-17, but he failed
to offer an explanation about of the source of investment in such bullion. In which of the following
AY would such income be taxable?
(a) AY 2017-18 (b) AY 2019-20 (c) Either (a) or (b), at the option of assessee (d) None of the above

PROBLEM:323 Mr. Rohit is found owner of cash money of Rs. 40 lakhs for which he is unable to explain
the source of income. What would be the tax liability, where Mr. Rohit claimed that he had incurred
Rs. 10 lakhs to earn such income?
(a) Rs. 30 lakhs (b) Rs. 31.20 lakhs (c) Rs. 40 lakhs (d) Nil

SALEEM QURAISHEE Mo: 9175664444 INSPIRE ACADEMY-8888881719


PROFITS AND GAINS OF BUSINESS OR PROFESSION- Page 6.91

48. SPECIAL PROVISION IN CASE OF INCOME OF PUBLIC FINANCIAL


INSTITUTIONS SECTION 43D-AMENDED W.E.F 1-4-2018
"Notwithstanding anything contained in any other provision of this Act,
a) In case of an assessee being a public financial institution or State Financial Corporation or State
Industrial Investment Corporation or a deposit taking non-banking financial company or a
systemically important non-deposit taking non-banking financial company or a Scheduled bank,
the income by way of interest in relation to specified categories of bad or doubtful debts
prescribed by the Reserve Bank of India;
b) In case of a public company, the income by way of interest in relation to specified categories of
bad and doubtful debts, prescribed by National Housing Bank;
 shall be chargeable to tax
I. in the previous year in which it is credited by the assessee to its profit and loss account for that
year or
II. in the previous year in which it is actually received by the assessee whichever is earlier.

49. INSURANCE BUSINESS 44


Notwithstanding anything to the contrary contained in the provisions of the Income – tax Act relating to
the computation of income chargeable under the head. "Income from house property". "Capital gains" or
"Income from other sources", or in section 199 or in section 28 to 43B, the profits and gains of any
business of insurance, including any such business carried on by a mutual insurance company or by a co –
operative society, shall be computed with the rules contained in the First Schedule of the Income – tax .The
Profits and gains derived from Life Insurance business included in TI of an assess shall be taxable @
12.5%.

SALEEM QURAISHEE Mo: 9175664444 INSPIRE ACADEMY-8888881719


PROFITS AND GAINS OF BUSINESS OR PROFESSION- Page 6.92

50. SPECIAL PROVISIONS IN CASE OF CERTAIN ASSOCIATIONS 44A


I. Applicable only to that trade, professional or similar association, the income of which is not distributed to its members.
II. Amount Received by the association from its members by way of contribution or otherwise (other than amount
received from performing specific services) i.e.,

General receipts from members

Less: Expenditure incurred for the purposes of protection or advancement of interest of members (other than expenditure
which is otherwise deductible under the Act and other than capital Expenditure), i.e., General expenditure on member
If negative - call it deficiency - If positive - surplus exempt from tax

III. Such deficiency will be allowed as a deduction in computing the income under the head P/G/B/P.
IV. If deficiency is greater than income under the head P/G/B/P, then the balance deficiency will be allowed as deduction
in computing income under other heads of income.
V. Before setting off the deficiency effect shall be first given to the deductions under this Act and brought forward losses
and allowances.
VI. The total deficiency which can be set off shall not exceed 50% of the Total income computed before giving deduction
of deficiency.

POINT TO BE NOTED:

• The tax rate applicable to a mutual concern shall be the same as applicable to an individual except where the mutual
concern is incorporated as a company.

PROBLEM:324 XYZ & Association (trade association) furnishes profits of business before allowing deficiency was of Rs.
80,000. The deficiency amount was of Rs. 75,000 and the total income of assessee before allowing deficiency was of Rs.
1,40,000. What shall be the amount of deduction available to assessee for the deficiency? You with the following
information –

PROBLEM:325 XYZ & Association (trade association) furnishes profits of business before allowing deficiency was of T 90,000.
The deficiency amount was of Rs. 75,000 and the total income of assessee before allowing deficiency was of Rs. 2,00,000.
What shall be the amount of deduction available to assessee for the deficiency?

SALEEM QURAISHEE Mo: 9175664444 INSPIRE ACADEMY-8888881719


PROFITS AND GAINS OF BUSINESS OR PROFESSION- Page 6.93

51. COMPULSORY MAINTENANCE OF ACCOUNTS [SEC. 44AA]


A. "Specified profession" - For the purpose of section 44AA and rule 6F legal, medical, engineering,
architectural, accountancy, technical consultancy, or interior decoration or any other notified
profession [i.e., authorized representative, film artist, company secretary and information
technology] are specified professions.
B. "Non-specified profession" - A non-specified profession is a profession other than a "specified
profession" mentioned above.

C. Maintenance of accounts by certain persons carrying on profession or business [Sec. 44AA]

BASIS SPECIFIED PROFESSIONS NON SPECIFEIED PROFESSIONS


CONDITIONS  If gross receipts in all three preceding  Total income exceeds ` 2,50,000 (I/HUF)
p/y exceeds ` 1,50,000 and other assessee ` 1,20,000
 If it is a new profession which is set up or
in p/y, it is likely to exceed ` 1, 50,000  Total sales exceeds `25,00,000 (I/HUF)
in that p/y. Other assess ` 10,00,000
 In any other case, books of accounts to
be maintained but these have not In any of the 3 p/y immediately preceding
been specified. relevant p/y or likely to exceed ` 1,20,000 or `
10,00,000 as the case may be, but in case of
I/HUF limit is 2,50,000/25,00,000(FA-2017)

Books of accounts not specified but to maintain


such books of accounts as will enable the AO to
compute total income.
OR
 Assessee covered under Section 44AE,
44BB, 44BBB
Books are required to be maintained if assessee
claims that his total income from the said
business is lower than the deemed profits or
gains OR
 Assessee covered u/s 44AD- Books are
required to be maintained if assessee claims
that his income from the said business is
lower than 8% of the turnover but total
income exceeds the exemption limit.
Rule 6F  Rule 6F applicable  Rule 6F not applicable
Applicability
BOOKS OF a) Cash book  Maintain such books of accounts and
ACCOUNTS AND b) Journal (if accounts are maintained documents as enable the AO to compute
DOCUMENTS according to mercantile system of his total taxable income under the act.
accounting)
c) Ledger
d) Carbon copies of bills excepting if the
bills or receipts of an amount less than
` 25.
e) Original bills whenever issued to the
person and receipts in respect of
expenditure incurred by the person.

SALEEM QURAISHEE Mo: 9175664444 INSPIRE ACADEMY-8888881719


PROFITS AND GAINS OF BUSINESS OR PROFESSION- Page 6.94

PLACE OF i. The books of accounts are to be  As prescribed by CBDT


KEEPING maintained at the principal place of
BOA/DOCUMENTS profession.

PERIOD i. The books of accounts should be kept  As prescribed by CBDT


and maintained for a period of 6 years
from the end of relevant A/y.

PROBLEM:326 Vinod is a person carrying on profession as film artist. His gross receipts from profession are as
under:
Financial year 2016-17 1, 15,000
Financial year 2017-18 1, 80,000
Financial year 2018-19 2, 10,000
What is his obligation regarding maintenance of books of accounts for each Assessment Year under section 44AA
of Income-tax Act, 1961?
ANSWER:326

PROBLEM:327 (i) A person carrying on specified profession is-


a) Required to maintain books of account b) required to maintain prescribed books of account
c) Not required to maintain books of account
(ii) A person carrying on specified profession is required to maintain the prescribed books of account of the
current previous year if the gross receipts of such profession in all the three preceding previous year exceeds:
a) Rs 1 crore b) Rs 25,00,000 c) Rs 1,50,000 d) Rs 60,000
(iii) A person carrying on specified profession is required to maintain:
a) Prescribed books of account in all cases
b) prescribed books of account if the gross receipt of all the three preceding previous years exceeds Rs
1,50,000, otherwise no account books of are to be maintained:
c) Prescribed books of account if the gross receipt of all the three preceding previous years exceeds Rs 1,50,000
otherwise such books of account as will enable the Assessing Officer to compute him business income:

PROBLEM:328 If a person sets up a specified profession during the current previous year, he is:
a) Required to maintain prescribed books of account b) Not required to maintain books of account
c) Required to maintain prescribed books of account if the gross receipts of such profession are likely to exceed
Rs 1, 50,000 otherwise such books of account which will enable the Assessing Officer to compute his total income

SALEEM QURAISHEE Mo: 9175664444 INSPIRE ACADEMY-8888881719


PROFITS AND GAINS OF BUSINESS OR PROFESSION- Page 6.95

PROBLEM:329 A person who has been carrying on non-specified profession is:


a) Not required to maintain any books of account
b) Required to maintain book of account of the current previous year if the gross receipts of such profession
exceed Rs 1,50,000
c) Required to maintain books of account of the current previous year if the gross receipts of such profession of
any of three preceding previous year exceeded Rs 10 lakhs
d) Required to maintain book of account of the current previous year if in any of the preceding 3 previous years
his total income exceeded Rs 1,20,000 or gross receipts exceeded Rs 10 lakhs

PROBLEM:330 (i) A person, who has been carrying on business, is required to maintain books of account of the
current previous year if:
a) his total income of any of 3 preceding previous years exceeded Rs 25,0000
b) his gross turnover or sales of any of 3 preceding previous year exceeded Rs 25 lakh
c) if condition mentioned either in (a) or (b) is satisfied.
(ii) A person, who sets up a non-specified profession or commences a business during the current previous
year is required to maintain books of account if his:
a) Total income of the current year exceeds or is likely to exceed Rs 250000
b) Gross receipt/turnover of the current year exceeds or likely to exceed Rs 25 ,00,000
c) Condition mentioned in either (a) or (b) is satisfied

PROBLEM:331 For persons carrying on business or non-specified profession, the books account to be maintained
have been:
a) Prescribed b) not prescribed c) none of these

PROBLEM:332 Which amongst the following are specified books of account?


(a) Cash Book (b) Carbon copies or counterfoils of bills. (c) Original bills issued. (d) All of the above

PROBLEM:333 The books of accounts are to be kept and maintained for a period of how many years from the end
of the relevant assessment year.
(a) 6 years (b) 5 years (c) 8 years (d) Unlimited period

PROBLEM:334 In case of specified professions, what is the minimum amount the Gross receipts should exceed in all
of the three immediately preceding previous years or, where the business is newly setup, the amount that gross
receipts are likely to exceed during current previous year, so as to maintain the books of accounts.
(a) Rs. 1,00,000 (b) Rs. 1,20,000 (c) 1,50,000 (d) Rs. 10,00,000

SALEEM QURAISHEE Mo: 9175664444 INSPIRE ACADEMY-8888881719


PROFITS AND GAINS OF BUSINESS OR PROFESSION- Page 6.96

52. TAX AUDIT [SEC. 44AB]


a) A person carrying on business- If the total sales, turnover or gross receipt in business for the
previous year(s) relevant to the assessment year exceed or exceeds ` 1 crore.

b) A person carrying on profession--- If his gross receipts in profession for the previous year(s)
relevant to the assessment year exceeds `50 lakh (Finance act 2016)

c) A person covered under section 44AE, 44BB or 44BBB-- If such person claims that the profits and
gains from the business are lower than the profits and gains computed under these sections
(irrespective of his turnover).

d) A Person covered under section 44ADA- Where assessee is covered u/s 44ADA & he claims that
such income is lower than such profits and gains computed on a presumptive basis and his
income exceeds the maximum amount not chargeable to tax in any PY.

e) A person covered under section 44AD- If the provisions of section 44AD(4) are applicable
and his income exceeds maximum amount not chargeable to income tax in any PY.(w.e.f AY
2017-18)
[Provided that this section shall not apply to the person, who declares profits and gains for the
previous year in accordance with the provisions of sub-section (1) of section 44AD and his total
sales, turnover or gross receipts, as the case may be, in business does not exceed two crore
rupees in such previous year: ]

f) Form No. - These provisions are given in the table below-


Different taxpayers Audit Form No. Statement
particulars

In the case of a person who carries on business or profession and Form No. 3CA Form No. 3CD
who is required by or under any law to get his accounts audited

In the case of a person who carries on business or profession but Form No. 3CB Form No. 3CD
not being a person referred to above

g) Due date for obtaining audit report


I. Audit report should be obtained on or before the due date of submission of return of income under
section 139(1)*.
II. The audit report is not to be submitted with return of income. It should be retained by the
assessee.
III. It may be furnished in original, whenever the Assessing Officer wants to examine it in assessment
proceedings or otherwise.

SALEEM QURAISHEE Mo: 9175664444 INSPIRE ACADEMY-8888881719


PROFITS AND GAINS OF BUSINESS OR PROFESSION- Page 6.97

PROBLEM:335 For person carrying on profession, tax audit is compulsory, if the gross receipts of the previous year
exceeds:
a) Rs 10 lakhs b) Rs 1 crore c) Rs 15 lakhs d)50,00,000

PROBLEM:336 Tax audit is compulsory in case a person is carrying on business whose gross turnover/sales/receipts, as the
case may be, exceeds:
a) Rs 60 lakhs b) Rs 1 crore c) Rs 40 lakhs

PROBLEM:337 A person carrying on profession is required to get his accounts compulsorily audited by a Chartered
Accountant if his gross receipts from profession for the previous year exceed –
(a) Rs. 10, 00,000 (b) Rs. 25, 00,000 (c) Rs. 50, 00,000 (d) Rs. 60, 00,000

PROBLEM:338 The due date of furnishing audit report under section 44AB shall be:
a) 31 st July of the relevant assessment year
b) 30th September of the relevant assessment year
c) 30th November of the relevant assessment year in case the assessee has under taken any international transaction or
specified domestic transaction and 30th September in case of any other assessee

PRESUMTIVE INCOME

SALEEM QURAISHEE Mo: 9175664444 INSPIRE ACADEMY-8888881719


PROFITS AND GAINS OF BUSINESS OR PROFESSION- Page 6.98

53. COMPUTATION OF INCOME ON ESTIMATED BASIS IN THE CASE OF


TAXPAYERS ENGAGED IN A BUSINESS [SEC. 44AD]
A. CONDITIONS -
I. The assessee should be an eligible assessee.
II. Eligible assessee for this purpose is a resident individual, a resident Hindu undivided family or a
resident partnership firm (not being a limited liability firm).
III. The assessee has not claimed any deduction under sections 10A, 10AA, 10B, 10BA, 80HH to
80RRB in the relevant assessment year.
IV. The assessee should be engaged in any business (whether it is retail trading or civil construction
or any other business). However, the following persons are not eligible to avail any benefit under
section 44AD -
 a person carrying on profession as referred to in section 44AA(1);
 a person earning income in the nature of commission or brokerage;
 a person carrying on any agency business; or
 a person who is in the business of plying, hiring or leasing goods carriages.
V. Total turnover/gross receipt in the previous year of the eligible business should not exceed ` 2
crore. (F.A 2016)

B. CONSEQUENCES IF THE ABOVE CONDITIONS ARE SATISFIED –


I. If the above conditions are satisfied, the income from the eligible business is estimated at 8 percent of
the gross receipt or total turnover or "six per cent” in respect of the amount of total turnover or
gross receipts which is received by an account payee cheque or an account payee bank draft or use
of electronic clearing system through a bank account during the previous year or before the due
date specified in sub-section (1) of section 139 in respect of that previous year. FA-2017
II. The assessee can voluntarily declare a higher income in his return.
III. All deductions under sections 30 to 38, including depreciation and unabsorbed depreciation,
are deemed to have been already allowed and no further deduction is allowed under these
sections.
IV. Salary and interest paid to partners is not allowed as deduction to partnership firm from
estimated income - Finance Act, 2016 w.e.f. AY 2017-18].
V. The written down value is calculated, where necessary, as if depreciation as applicable has
been allowed.
VI. It will be assumed that disallowance, if any, under sections 40, 40A and 43B has been
considered while calculating the estimated income @ 8 per cent.

VII. PAYMENT OF ADVANCE TAX- An assessee opting for section 44AD is required to pay entire
tax on estimated income on Advance basis w.e.f. AY 2017-18 (earlier assessee opting for
section 44AD was exempt from provisions of Advance tax) .He can pay advance tax pertaining
to his business income in one installment (ie on or before 15th March of the FY)

VIII. MAINTENANCE OF BOOKS OF ACCOUNTS & TAX AUDIT-

 An assessee opting for the above scheme shall be exempted from maintenance of books of account
related to such business as required under section 44AA AND Tax audit under section 44AB.

SALEEM QURAISHEE Mo: 9175664444 INSPIRE ACADEMY-8888881719


PROFITS AND GAINS OF BUSINESS OR PROFESSION- Page 6.99

IX. Consequence of not declaring profit as per section 44AD-Where an eligible assessee declares
profit for any previous year in accordance with the provisions of this section and he declares
profit for any of the five consecutive assessment years relevant to the previous year
succeeding such previous year not in accordance with the provisions of sub-section (1), he
shall not be eligible to claim the benefit of the provisions of this section for five assessment
years subsequent to the assessment year relevant to the previous year in which the profit has
not been declared in accordance with the provisions of sub-section (1). This is provided in
new sub-section (4).(FA-2016)

X. An eligible assessee to whom the provisions of sub-section (4) are applicable and whose
total income exceeds the basic exemption limit has to maintain books of account under
section 44AA and get them audited and furnish a report of such audit under section 44AB.
This is provided in new sub-section (5) of section 44AD.(FA-2016)

CASE STUDY-
Let us consider the following particulars relating to a resident individual, Mr. A, being an eligible assessee
whose gross receipts do not exceed Rs. 2 crore in any of the assessment years between A.Y.2018-19 to
A.Y.2020-21
Particulars A.Y.2018-19 A.Y.2019-20 A.Y.2020-21
Gross receipts (Rs.) 1,80,00,000 1,90,00,000 2,00,00,000

Income offered for taxation (Rs.) 14,40,000 15,20,000 12,00,000

% of gross receipts 8% 8% 6%

Offered income as per presumptive Yes Yes No


taxation scheme u/s 44AD

 In the above case, Mr.A, an eligible assessee, opts for presumptive taxation under section 44AD for
A.Y.2018-19 and A.Y.2019-20 and offers income of Rs.14.40 lakh and Rs.15.20 lakh on gross receipts
of Rs.1.80 crore and Rs.1.90 crore, respectively.
 However, for A.Y.2020-21, he offers income of only Rs.12 lakh on turnover of Rs.2 crore, which
amounts to 6% of his gross receipts.
 He maintains books of account under section 44AA and gets the same audited under section 44AB.
 Since he has not offered income in accordance with the provisions of section 44AD(1) for five
consecutive assessment years, after A.Y. 2018-19, he will not be eligible to claim the benefit of section
44AD for next five assessment years succeeding A.Y.2020-21 i.e., from A.Y.2021-22 to 2025--26.

PROBLEM:339 Accounts of a person carrying on business referred to u/s 44AD are required to be audited
for previous year if –
(a) The assessee has claimed profits.
(b) Profits from such business to be lower than the profits and gains deemed u/s 44AD.
(c) The "total income" exceeds maximum amount, which is not chargeable to tax in any py
(d) All of the above,

SALEEM QURAISHEE Mo: 9175664444 INSPIRE ACADEMY-8888881719


PROFITS AND GAINS OF BUSINESS OR PROFESSION- Page 6.100

PROBLEM:340 For computation of profits of business on presumptive basis under section 44AD, the
deemed profits shall be calculated at the rate of _______ % of the total turnover or gross receipts of
such business :
(a) 8 (b) 10 (c) 20 (d) 5

PROBLEM:341 The total turnover of the business of assessee was of 30,00,000. The assessee declared a
profit of Rs. 2, 80,000. What shall be the deemed profits of assessee under section 44AD.
(a) Rs. 2, 80,000 (b) Rs. 2, 40,000 (c) Rs. 40,000 (d) Rs. 2, 60,000

PROBLEM:342 The provisions of section 44AD shall not apply to :


(a) Person carrying on specified profession as referred to Section 44 AA
(a) A .person earning income in the nature commission or brokerage.
(c) Person carrying on any agency business. (d) All of the above

54. PRESUMPTIVE TAXATION FOR PROFESSIONALS- SEC 44ADA


A. CONDITIONS-
I. New Section 44ADA has been inserted to provide for estimating the income of an assessee
(individual, HUF, partnership firm but not limited liability partnership firms) engaged in any
profession referred in section 44AA(1) such as legal, medical, engineering or architectural,
accountancy, technical consultancy, interior decoration or any other profession as is notified by the
Board.
II. Total Gross receipts should not exceed Rs. 50 lakhs.

B. CONSEQUENCES IF THE ABOVE CONDITIONS ARE SATISFIED –


I. Income shall be estimated @ 50% of the total gross receipts.
II. Deductions u/s 30 to 38 deemed to have been allowed (Including interest and remuneration to
partners in case of partnership firm)
III. The written down value is calculated, where necessary, as if depreciation as applicable has been
allowed.
IV. It will be assumed that disallowance, if any, under sections 40, 40A and 43B has been considered
while calculating the estimated income @ 50 per cent.

C. IS IT POSSIBLE TO DECLARE LOWER INCOME – Yes


Consequence-
I. The taxpayer will have to maintain the books of account as per section 44AA (irrespective of
income or turnover) if his total income exceeds the exemption limit.
II. The taxpayer will have to get his books of account audited under section 44AB (irrespective of
turnover) if his total income exceeds the exemption limit.

SALEEM QURAISHEE Mo: 9175664444 INSPIRE ACADEMY-8888881719


PROFITS AND GAINS OF BUSINESS OR PROFESSION- Page 6.101

PROBLEM:343 For computation of profits of profession on presumptive basis under section 44ADA the
deemed profits shall be calculated at the rate of ……….% of the gross receipts of such profession

PROBLEM:344 The Provisions of section 44ADA are applicable if gross receipts from profession duos not
exceed……………….

PROBLEM:345 Mrs. X is carrying of profession of CS. Hiss gross receipts from profession is 45,00,000 in PY
2019-20 .His deemed profits as per provision of section 44ADA shall be…………

PROBLEM:346 Mr. Praveen engaged in retail trade, reports a turnover of ` 1 98, 50,000 for the financial
year 2019-20. His income from the said business as per books of account is computed at ` 13,20,000.
Retail trade is the only source of income for Mr. Praveen.
I. Is Mr. Praveen eligible to opt for presumptive determination of his income chargeable to tax for the
assessment year 2020-21?
II. If so, determine his income from retail trade as per the applicable presumptive provision.
III. In case Mr. Praveen does not opt for presumptive taxation of income from retail trade, what are his
obligations under the Income-tax Act, 1961?
IV. What is the due date for filing his return of income under both the options?
ANSWER:346

SALEEM QURAISHEE Mo: 9175664444 INSPIRE ACADEMY-8888881719


PROFITS AND GAINS OF BUSINESS OR PROFESSION- Page 6.102

55. COMPUTATION OF INCOME ON ESTIMATED BASIS IN THE CASE OF


TAXPAYERS ENGAGED IN THE BUSINESS OF PLYING, LEASING OR HIRING
TRUCKS [SEC. 44AE] –
A. Conditions-Section 44AE is applicable only if the following conditions are satisfied-
I. The taxpayer may be an individual, HUF, AOP, BOI, firm, company, co-operative society or any
other person. He or it may be a resident or a non-resident.
II. Taxpayer is engaged in the business of plying, hiring or leasing goods carriages
III. The taxpayer owns not more than 10 goods carriages at any time during the previous year.
For this purpose, a taxpayer, who is in possession of a goods carriage, whether taken on hire
purchase or on installments and for which the whole or part of the amount payable is still
due, shall be deemed to be the owner of such goods carriage

B. CONSEQUENCES IF THE ABOVE CONDITIONS ARE SATISFIED

I. Presumptive Income = Higher of sum claimed to have been actually earned from the Vehicle or an
amount as computed below -
 Heavy Goods Vehicle:: `1,000 per ton of gross vehicle weight or unladen weight per month or
part of a month during which vehicle is owned by the assessee in the PY.
 Other Vehicle:: `7,500 per month or part of a month during which vehicle is owned by the
assessee in the PY.

Meaning of Terms:
 Heavy Goods vehicle” means any goods carriage, the gross vehicle weight of which exceeds
12000 kilograms (more than 12mt gross vehicle weight).
 Goods carriage”, “Gross vehicle weight” and “unladen weight shall have the respective
meanings assigned to them u/s 2 of the Motor Vehicles Act, 1988.

II. A taxpayer can claim his income from the aforesaid business at a higher amount then that specified.
III. All deductions under sections 30 to 38 including depreciation and unabsorbed depreciation are
deemed to have been already allowed and no further deduction is allowed under this section.
IV. However, in the case of a firm, the normal deduction in respect of salary and interest to partners
under section 40(b) shall be allowed.

C. IS IT POSSIBLE TO DECLARE LOWER INCOME – Yes


Consequence-
I. The taxpayer will have to maintain the books of account as per section 44AA (irrespective of
income or turnover) if his total income exceeds the exemption limit.
II. The taxpayer will have to get his books of account audited under section 44AB (irrespective of
turnover) if his total income exceeds the exemption limit.

PROBLEM:347 Assessee who owns not more than ___ goods carriages at any time during the previous year
and engaged in the business of plying/ hiring or leasing such goods carriages shall be eligible to
compute profits under Section 44AE:
(a) 10 (b) 8 (c) 6 (d) 15

SALEEM QURAISHEE Mo: 9175664444 INSPIRE ACADEMY-8888881719


PROFITS AND GAINS OF BUSINESS OR PROFESSION- Page 6.103

PROBLEM:348 For computing deemed profits under section 44AE in case of goods carriage being a goods
vehicle the amount with which per vehicle per month has to be multiplied is:
(a) Rs. 10,000 (b) Rs. 7,500 (c) Rs. 50,000 (d) Rs. 1,50,000

PROBLEM:349 For computing deemed profits under section 44AE in case of goods carriage other than a
heavy goods vehicle the amount with which per vehicle per month has to be multiplied is:
(a) Rs. 10,000 (b) Rs. 5,000 (c) Rs. 50,000 (d) Rs. 7,500

PROBLEM:350 An assessee was engaged in the business of plying, hiring or leasing of goods carriages. He
held 4 heavy goods vehicles for the entire year and three goods carriage other than heavy goods
vehicle which were acquired on 15"' July 2019. Compute the deemed profits u/s
44AE…………………………..

PROBLEM:351 An assessee was engaged in the business of plying, hiring or leasing of goods carriages. He
owned 6 heavy goods vehicle for the entire year. He claimed amount to have been actually earned
from such vehicle(s) Rs. 3,80,000. Compute the deemed profits u/s 44AE………………………………

PROBLEM:352 Ramamurthy had 4 heavy goods vehicles as on 1.4.2019. He acquired 7 heavy goods vehicles
on 27.6.2019. He sold 2 heavy goods vehicles on 31.5.2019.He has brought forward business loss
of`50,000 relating to assessment year 2016-17 of a discontinued business. Assuming that he opts for
presumptive taxation of income as per section 44AE, compute his total income chargeable to tax for
the assessment year 2020-21.
ANSWER:352

PROBLEM:353 Mr. X commenced the business of operating goods vehicles on 1.4.2019. He purchased the
following vehicles during the P.Y.2019-20. Compute his income under section 44AE for A.Y.2020-21.

Gross Vehicle Weight Number Date of purchase


(in kilograms)
1) 7,000 2 10.04.2019
2) 6,500 1 15.03.2020
3) 10,000 3 16.07.2019
4) 11,000 1 02.01.2020
5) 15,000 2 29.08.2019
6) 15,000 1 23.02.2020

Would your answer change if the goods vehicles purchased in April, 2019 were put to use only in July,
2019?
ANSWER:353

SALEEM QURAISHEE Mo: 9175664444 INSPIRE ACADEMY-8888881719


PROFITS AND GAINS OF BUSINESS OR PROFESSION- Page 6.104

PROBLEM:354 Mr. Sukhvinder is engaged in the business of plying goods carriages. On 1st April, 2019, he
owns 10 trucks (out of which 6 are heavy goods vehicles). On 2nd May, 2019, he sold one of the heavy
goods vehicles and purchased a light goods vehicle on 6th May, 2019. This new vehicle could however
be put to use only on 15th June, 2019.Compute the total income of Mr. Sukhvinder for the assessment
year 2020-21, taking note of the following data:
Particulars ` `

Freight charges collected 12,70,000

Less: Operational expenses 6,25,000

Depreciation as per section 32 1,85,000

Other office expenses 15,000 8,25,000

Net Profit 45,000

Other business and non - business income 70,000

ANSWER:354

56. ASSESSMENT OF NON-RESIDENTS


Special provisions for computing profits and gains in case of Non-Residents engaged in certain
businesses
Section 44B Section 44BB Section 44BBA Section 44BBB
This section Non-resident Non-resident engaged in Non-resident Foreign company
applies to- engaged in providing services or engaged in engaged in civil
operation of facilities in connection operation of construction or
SHIPS with, or supplying plant aircraft. erection, testing or
and machinery on hire commissioning of
used, or to be used, in the plant or machinery in
prospecting for, or connection with an
extraction or production approved turnkey
of, mineral oils (including power project
petroleum and natural
gas)
Deemed 7 ½ % of gross 10% of gross receipts 5% of gross receipts 10% of gross receipts
Income receipts
LOWER
INCOME

SALEEM QURAISHEE Mo: 9175664444 INSPIRE ACADEMY-8888881719


PROFITS AND GAINS OF BUSINESS OR PROFESSION- Page 6.105

57. DEDUCTION OF HEAD OFFICE EXPENDITURE IN THE CASE OF NON-RESIDENTS [SECTION


44C]:
The expenditure in nature of head office expenditure is allowable to the extent of the lower of the
following-
a) 5% of adjusted total income (in case If adjusted total income is a loss, then 5% of average
adjusted total income); or
b) The amount of so much of the expenditure in the nature of head office expenditure incurred by
the assessee as is attributable to the business or profession of the assessee in India.

PROBLEM:355 Which type of the assessee are required to maintain books of account u/s 44AA of
Income-tax Act, 1961?
(a) Person carrying on legal profession (b) Person carrying on medical profession
(c) Person carrying on architectural profession (d) All of the above

PROBLEM:356 Which type of the assessee are not required to maintain books of account u/s 44AA of
Income-tax Act, 1961?
(a) Person engaged in accountancy profession
(b) Person engaged in profession of interior decoration
(c) Person engaged in profession of engineering profession
(d) Person engaged in running a dance school

PROBLEM:357 Which of the following are the prescribed books of Account, which are required to be
maintained by persons carrying on specified profession u/s 44AA?
(a) Cashbook (b) Ledger (c) Journal (d) All of the above

PROBLEM:358 Which of the following are not the prescribed books of account, which are required to be
maintained by persons carrying on specified profession u/s 44 AA?
(a) Cashbook (b) Ledger
(c) Journal, in case where mercantile method is followed (d) Cash Flow Statement

PROBLEM:359 Person who is engaged in ................ shall also maintain additional documents in addition
to prescribed books of account as per rule 6F of Income-tax Rules, 1962.
(a) Medical profession (b) Legal profession
(c) Engineering profession (d) All of the above

SALEEM QURAISHEE Mo: 9175664444 INSPIRE ACADEMY-8888881719


PROFITS AND GAINS OF BUSINESS OR PROFESSION- Page 6.106

PROBLEM:360 Person who is engaged in medical profession shall also maintain the following books in
addition to prescribed books of account?
(a) Daily case register in Form No. 3C
(b) Inventory under broad heads as on the first and the last day of the previous year
(c) Both (a) and (b) (d) None of the above

PROBLEM:361 In which of the following place, specified books of account should be kept by specified
persons?
(a) Place where he is carrying on the profession
(b) Principal place, if profession is carried on at more than one place
(c) Registered office of the company (d) Both (a) and (b) above

PROBLEM:362 Books of account are required to be kept and maintained for a period of ................
(a) Six years from end of relevant previous year
(b) Eight years from end of relevant previous year
(c) Six years from end of relevant assessment year
(d) Eight years from end of relevant assessment year

PROBLEM:363 Ajay is engaged in medical profession and his gross receipts in the preceding 3 years are of
Rs. 1,45,000, Rs. 1,30,000, Rs. 1,20,000 respectively. Find the correct statement in context of
section 44AA.
(a) Ajay has to maintain prescribed books of account in current previous year
(b) Ajay is not required to maintain prescribed books of account in current previous year
(c) Ajay is required to keep his books of account for 6 years from end of relevant assessment year
(d) Both (a) and (c)

PROBLEM:364 ROY engaged in medical profession and his gross receipts in the preceding 3 years are of
Rs. 1,45,000, Rs. 1,30,000, Rs. 1,75,000 respectively. Find the correct statement in context of
section 44AA?
(a) ROY has to maintain prescribed books of account in current previous year
(b) ROY is not required to maintain prescribed books of account in current previous year
(c) ROY is required to keep his books of account for 6 years from end of relevant assessment year
(d) Both (a) and (c)

PROBLEM:365 MOHIT is engaged in trading business and his sales in preceding 3 years are of Rs.
20,00,000, Rs. 22,00,000, Rs. 25,00,000 respectively. His income in preceding 3 years are of Rs.
1,45,000, Rs. 2,30,000, Rs. 2,50,000. Find the correct statement in context of section 44AA?
(a) MOHIT has to maintain prescribed books of account in current previous year
(b) MOHIT is not required to maintain prescribed books of account in current previous year
(c) MOHIT is required to keep his books of account for 6 years from end of relevant assessment
year
(d) Both (a) & (c)

SALEEM QURAISHEE Mo: 9175664444 INSPIRE ACADEMY-8888881719


PROFITS AND GAINS OF BUSINESS OR PROFESSION- Page 6.107

PROBLEM:366 ANIL Ltd. is engaged in trading business and his sales in preceding 3 years are of Rs.
20,00,000 22,00,000, 25,00,000 respectively and his income in preceding 3 years are of Rs. 1,45,000,
2,30,000, 2,50,000. In that case find the correct statement in context of section 44AA?
(a) ANIL Ltd. have to maintain prescribed books of account in current previous year
(b) ANIL Ltd. is not required to maintain prescribed books of account in current previous year
(c) ANIL Ltd. is required to kept his books of account for 6 years from end of relevant assessment
year (d) Both (a) & (c)

PROBLEM:367 When assessee fails to maintain the books of account, then consequences of such failure
would be ................
(a) Penalty of Rs. 25,000 (b) Penalty of Rs. 30,000 (c) Penalty of Rs. 35,000 (d) Nil

PROBLEM:368 As per section 44AB, a person is required gets his account audited by a chartered
accountant when ................
(a) Engaged in business and total sales or turnover as the case may be exceeds Rs. 1 crore in such
previous year
(b) Engaged in profession and gross receipt exceeds Rs. 50 lakhs in such previous year
(c) Engaged in business and covered u/s 44 AE and claims that income from such business is lower
than deemed profits and gains computed under the relevant sections (d) All of the above

PROBLEM:369 As per section 44AB, in which of the following case a person is required to gets his account
audited by a chartered accountant?
(a) Engaged in business, which is covered u/s 44AD relating to special provision for computing
profits and gains of business on presumptive basis and his income exceeds the maximum amount
which is not chargeable to income tax
(b) Engaged in profession and gross receipt exceeds Rs. 50 lakhs in such previous year
(c) Both (a) and (b) above
(d) Engaged in business and covered u/s 44AD relating to special provision for computing profits
and gains of business on presumptive basis and his total sales, turnover or gross receipt as the case
may be does not exceeds Rs. 2 Cr in such previous year

PROBLEM:370 Profits of gains of any business, except business of plying, hiring or leasing goods carriages
referred to in section 44AE, are computed at what percentage of gross receipts (paid or payable) to
an assessee?
(a) 8% (b) 6% (c) 5% (d) 12%

PROBLEM:371 Scheme of section 44AD cannot be availed by an assessee being a ................


(a) Person carrying on specified profession as referred in 44AA( 1)
(b) Person earning income in the nature of commission or brokerage
(c) Person carrying on any agency business (d) All of the above

SALEEM QURAISHEE Mo: 9175664444 INSPIRE ACADEMY-8888881719


PROFITS AND GAINS OF BUSINESS OR PROFESSION- Page 6.108

PROBLEM:372 In context of section 44AD which of the following statement are correct?
(a) Where assessee is a firm, it is not entitled to deduction on account of interest and salary paid to
partners
(b) Assessee shall be required to pay the advance tax even if he opt for section 44AD
(c) Assessee who opt for this scheme are not required to maintain books of account as required u/s
44AA(d) All of the above

PROBLEM:373 Presumptive rate of 6% in respect of computing profits and gains of business on


presumptive basis shall be applicable only if the amount of such total turnover or gross receipt is
received by an account payee cheque or account payee bank draft ................
(a) During the previous year
(b) Before the due date specified in section 139 in respect of that previous year
(c) Either (a) or (b)(d) None of the above

PROBLEM:374 If the assessee claims that his profits are lower than deemed profits u/s 44AE, then he has
to ................
(a) Maintain books of account u/s 44 AA (b) Get his accounts Audited u/s 44 AB
(c) Both (a) and (b) (d) None of the above

PROBLEM:375 What would be the rate of presumptive income u/s 44AE in case business of plying, hiring
or leasing goods carriages?
(a) Rs. 1,000 per ton of gross vehicle weight for per month/part of a month, from a heavy goods
vehicle;
(b) Rs. 7500 per month/part of the month from other than heavy goods vehicle
(c) Both (a)&(b) (d) None of the above

PROBLEM:376 Provision of section 44 ADA can be availed by resident assessee engaged in ................
(a) Profession (b) Business (c) Both(a)&(&) (d) None of the above

PROBLEM:377 In context of section 44 ADA which of the following statement are correct?
(a) Assessee being a firm is not entitled to deduction on account of interest and salary paid to
partners
(b) Assessee who opt for this scheme and claims that his profits is less than 5096 of gross receipts is
required to get his books of account audited as required u/s 44AB
(c) Assessee who opt for this scheme and claims that his profits is less than 50% of gross receipts is
required to maintain books of account as required u/s 44AA
(d) All of the above

SALEEM QURAISHEE Mo: 9175664444 INSPIRE ACADEMY-8888881719


PROFITS AND GAINS OF BUSINESS OR PROFESSION- Page 6.109

58. SECTION SUMMARY

SECTION PARTICULAR

SALEEM QURAISHEE Mo: 9175664444 INSPIRE ACADEMY-8888881719


PROFITS AND GAINS OF BUSINESS OR PROFESSION- Page 6.110

PRACTICAL PROBLEMS
PROBLEM:378 X (age: 26 years), a leading tax consultant, who maintains books of account on cash basis
furnishes the following particulars of income and expenditure for the assessment year 2020-21:
Receipt and Payment Account for the year ending March 31, 2020.
Particulars ` Particulars `

Balance brought down 12,400 Purchase of a typewriter 6,000

Fees from clients : Car expenses 18,000

of 2019-20 7,30,500 Office expenses 40,000

of 2018-19 1,11,500 Salary to staff :

of 2017-18 1,13,000 of 2019-20 32,000

Presents from clients 24,000 of 2018-19 11,000

Interest-free loan from a client for 2,38,000 Expenses in respect of let out 6,000
purchase of a car property [municipal tax : ` 2,000,
repairs : ` 1,000, insurance : `
Winnings from lottery 46,000 3,000]

Interest from UTI (received on 12,000 Car purchased on December 10, 2,40,000
September 11, 2019) 2019

Rent of a let out property 60,000 Repairs of office 12,000

Share of income from a firm 15,000 Interest on loan 10,000

Income-tax payment 2,000

Life insurance premium 2,08,000

Balance credit down 7,77,400

Total 13,62,400 Total 13,62,400


Car is partly used for official purposes (40%) and partly for private purposes (60%). Determine the
taxable income and tax liability of X for the assessment year 2020-21.
ANSWER:378 G.T.I `

SALEEM QURAISHEE Mo: 9175664444 INSPIRE ACADEMY-8888881719


PROFITS AND GAINS OF BUSINESS OR PROFESSION- Page 6.111

PROBLEM:379 Dr. Gupta is a renowned medical practitioner who maintains books of account on cash basis. The
following is the receipts and payments A/c for the financial year 2019-20.
Particulars ` Particulars `

Balance brought forward 44,000 Rent of clinic


Consultation fees – 2018-19 5,000 2019-20 10,24,800
2019-20 51,35,000 2018-19 51,200

Visiting fees 30,000 Water & Electricity Bills 2,000

Loan from bank 1,25,000 Purchase of professional books 40,000

Sale of medicines 60,000 Household expenses 47,800

Gift and presents 5,000 Collection charges for dividend 100


income

Royalties for articles published in various journals 6,000 Motor car purchased 1,30,000

Dividend 10,000 Surgical equipment purchased 24,800

Interest on Government Securities 7,000 Income-tax 7,000

Audit fees 53,000

Salary to staff 15,15,000

Life Insurance Premium 15,000

Gift to son 5,000

Interest on loan 11,000

Car expenses 15,000

Purchase of medicines 40,000

Balance c/d 24,45,300

Total 54,27,000 Total 54,27,000


Compute his income profession for the assessment year 2020-21 after taking into account the following
information:
1. Books worth `25,000 were purchased on 15-5-2019, which were annual publications and the balance on 5-2-
2020 which were books other than annual publication.
2. Car was purchased on 1-1-2020 and the surgical equipment on 4-9-2019.
3. It is estimated that 1/3 of the use of car is for his personal use.
4. Gifts and presents include ` 2,000 from patients in appreciation of his medical service and ` 3,000 received as
birthday gifts.
5. Opening and closing stock of medicines amounted to ` 10,000 and ` 6,000 respectively.
ANSWER:379 `

SALEEM QURAISHEE Mo: 9175664444 INSPIRE ACADEMY-8888881719


PROFITS AND GAINS OF BUSINESS OR PROFESSION- Page 6.112

PROBLEM:380 The following is the Receipts and Payments account of Mr. Raj a practicing chartered
accountant, for the year ended 31-3-2020.
Receipts `

Audit fees 6,80,000

Consultation 40,000

Appellate Tribunal appearance 25,000

Miscellaneous receipts 20,000

Interest on Government Securities (Gross) 8,000

Rent received for house No. 1 24,000

Present from clients 10,000

Payments

Stipend to article clerks 12,000

Office expenses 24,000

Office rent 18,000

Salaries and wages 20,500

Printing and stationery 4,000

Payments `
Subscription of C.A. institute 1,500
Purchase of books for professional purposes 15,000
Traveling expenses 5,000
Interest on bank loan 12,000
Donation to National Defense Fund 5,000
Additional information
Loan from bank was taken for the construction of the second house in which he lives. Municipal value of
this house is ` 18,000 and the local taxes ` 1,800 p.a. 1/4th of traveling expenses are not allowable.
Compute professional income and income form house property for the assessment year 2020-21 assuming
50% of the books which were annual publication were purchased in August 2019 and balance 50% in
February 2020.
ANSWER:380 `

SALEEM QURAISHEE Mo: 9175664444 INSPIRE ACADEMY-8888881719


PROFITS AND GAINS OF BUSINESS OR PROFESSION- Page 6.113

PROBLEM:381 Mr. Roy is a registered medical practitioner. He keeps his books on cash basis, and his
summarized cash account for the year ended 31-3-2020 is as under:
Particulars ` Particulars `

Balance b/d 2,24,000 Cost of medicines 10,000

Loan from bank for private purpose 3,000 Surgical equipment’s 4,000

Sale of medicines 25,250 Motor car 2,20,000

Consultation fees 50,55,000 Car expenses 6,000

Visiting fees 5,24,000 Salaries 2031,600

Interest on Govt. securities 4,500 Rent of dispensary 5,51,600

Rent from property (not subject to 3,600 General expenses 20300


local taxes)
Personal expenses 11,600

Life insurance premium 3,000

Interest on loan from bank 300

Insurance of property 200

Telephone expenses 33,000

Balance c/d 29,47,750

Total 58,39,350 Total 58,39,350


Compute his income from profession for the assessment year 2020 taking into account the following
further information:
i. 1/4th of motor car expenses are in respect of his personal use.
ii. Depreciation allowable on car is 15% and surgical equipment is 15%. Both the assets were purchased in
December, 2019
iii. Opening stock and closing stock of medicines was ` 20,000 and ` 12,000, respectively.
ANSWER:381 `

SALEEM QURAISHEE Mo: 9175664444 INSPIRE ACADEMY-8888881719


PROFITS AND GAINS OF BUSINESS OR PROFESSION- Page 6.114

PROBLEM:382 The following is the Receipts and Payments Account of a medical practitioner for the year
ending 31-3-2020.
Receipts ` Payment `

Balance b/f 1,60,000 Clinic rent 24,000

Visiting fees 1,40,000 Staff salaries 1,20,000

Consultation fees 1,60,000 Rent and taxes 6,000

Sale of medicines 40,000 Electricity and water 7,000

Operation theatre rent 25,000 Purchase of medical books 20,100

Interest on bank deposit 13,000

Dividends from UTI 10,000 Purchase of surgical 40,000


equipment’s

Sale of surgical equipment 36,000 Motor car expenses 24,000

Medical association 4,000


membership

Life insurance premium 10,000

Audit fees 14,000

Staff welfare expenses 3,000

Diwali expenses 2,000

Entertainment expenses 8,400

Medicines purchased 26,000

Balance c/d 2,75,500

Total 5,84,000 Total 5,84,000


Additional information:
1. A cash payment of `15,000 was given to him by a patient in appreciation of his medical service but was
not accounted for in the books of account.
2. ¼th of motor-car expenses relate to his personal use, depreciation on motor car allowable under the
Income-tax Act is ` 9,000 for professional us.
3. Audit fees include income-tax appeal expenses of ` 12,000.
4. The rate of depreciation on surgical equipment is 15%. The written down-value of equipment’s
brought forward from earlier year was ` 26,000. He sold equipment for ` 36,000 during the current
year. The new surgical equipment were purchased on 1-11-2019
5. His taxable income from house properties was ` 3, 00,000.
6. Opening and closing stock of medicines were ` 10,000 & 15,000 respectively. Compute his income for
AY 2020-21.
ANSWER:382 424250
SALEEM QURAISHEE Mo: 9175664444 INSPIRE ACADEMY-8888881719
PROFITS AND GAINS OF BUSINESS OR PROFESSION- Page 6.115

PROBLEM:383 Kumar has prepared the following Profit and Loss Account for the year ended 31-3-2020. You are required to
ascertain his gross total income for the year ended on that date:
Particulars ` Particulars `

Salaries 60,000 Gross profit 1,50,000

Rent 26,400 Rent from house property 36,000

Loss by theft 20,000

Discount 10,000

Charity 500

VAT paid 6,000

Reserve for bad debts 10,000

Gift and presents 500

Life insurance premium 1,000

Interest on loan 12,000

Interest on capital 3,000

Repairs to house 500

Income-tax 2,000

Net profit 34,100

Total 1,86,000 Total 1,86,000


ANSWER:383 `40800
PROBLEM:384 R gives you the following particulars for the year ended 31-3-2020
Particulars `

Net profit as per P & L account (without allowing the following items) 5,20,000

Capital expenditure on family planning 70,000

Lump sum consideration for purchase of technical know-how developed in government laboratory 1,20,000

Entertainment expenditure 40,000

Expenditure on acquisition of patent right 80,000

Expenditure on advertisement paid in cash 25,000

Provision for excise duty 60,000

He paid ` 30,000 on 27-6-2020, and ` 12,000 on 1-11-2020 Due date of filing of return is 31-7-2020

Amount paid to Delhi University a notified institution for an approved research program in the field of
social sciences not connected with his business 60,000
Compute his business income for the assessment year 2020-21.
ANSWER:384 ` 325000

SALEEM QURAISHEE Mo: 9175664444 INSPIRE ACADEMY-8888881719


PROFITS AND GAINS OF BUSINESS OR PROFESSION- Page 6.116

PROBLEM:385 The following is the Profit and Loss Account of R for the year ended 31-3-2020.Compute
his taxable income from business for that year:
Particulars ` Particulars `

Opening stock 15,00,000 Sales 10700000

Purchases 75,20,000 Closing Stock 20,00000

Wages 20,00,000 Gift from father 10,0000

Rent 6,0000 Sale of car 17,0000

Repair of car 3,0000 Income-tax refund 3,0000

Wealth-tax paid 2,0000

Medical expenses 3,0000

General expenses 11,0000

Depreciation of car 3,0000

Advance income-tax paid 1,00,000

Profit for the year 1600000

Total 13000000 Total 13000000


Following further information is given:
1. R carries on his business from rented premises half of which is used as his residence
2. R bought a car during the year for `20, 0000. He charged 15% depreciation on the value of car. The
car was sold during the year for ` 17, 0000. The use of the car was ¾th for the business and ¼th for
personal purposes.
3. Medical expenses were incurred during sickness of R for his treatment
4. Wages include ` 60000 on account of R's driver.
ANSWER:385 `

PROBLEM:386 Following is the P&L A/c of Mr. Q for the year ended 31st March 2020:
Particulars Amount (Rs) Particulars Amount (Rs)
To repairs on building 1,81,000 By Gross Profit 6,01,000
To interest 1,10,000 By income tax refund 8,100
To travelling 1,30,550 By interest on company 6,400
deposits
To amount paid to IIT, Mumbai for an 1,00,000
approved scientific research program
To Net Profit 93,950
6,15,500 6,15,500
Following additional information is furnished:
1) Repairs on building includes Rs 1,00,000 being cost of building a new toilet.
2) Interest payments include Rs 50,000 on which tax has not been deducted and penalty for contravention
of GST Act of Rs 24,000.
3) Compute the PGBP income of Mr Q for the year ended 31st March 2020 ignoring depreciation.
ANSWER:386

SALEEM QURAISHEE Mo: 9175664444 INSPIRE ACADEMY-8888881719


PROFITS AND GAINS OF BUSINESS OR PROFESSION- Page 6.117

PROBLEM:387 From the Profit and Loss Account of X (age: 31 years) for the year ending March 31, 2020
ascertain his total income and tax liability for the assessment year 2020-21
Particulars ` Particulars `

General expenses 13,400 Gross profits 4,15,500

Bad debts 22,000 Commission 8,600

Advance tax 2,000 Brokerage 37,000

Insurance 600 Sundry receipts 2,500

Salary to staff 26,000 Bad debt recovered (earlier 11,000


allowed as deduction)
Salary to X 51,000

Interest on overdraft 4,000 Interest on debentures (i.e., net 25,000


amount ` 22,500 + tax deducted
Interest on loan to Mrs. X 42,000 at source : ` 2,500)

Interest on capital of X 23,000

Depreciation 48,000 Interest on deposit with a 13,000


company (non-trade) (net interest
Advertisement expenditure 7,000 : ` 11,700 + tax deducted of
source ` 1,300)
Contribution to employees' 13,000
recognized provident fund

Net profit 2,60,600

Total 5,12,600 Total 4,12,600


Other information-
1. The amount of depreciation allowable is ` 37,300 as per the Income-tax Rules. It includes depreciation
on permanent sign board.
2. Advertisement expenditure includes ` 3,000, being cost of permanent sign board fixed on office
premises .
3. Income of ` 4,500, accrued during the previous year, is not recorded in the Profit and Loss Account.
4. X pays ` 6,000 as premium on own life insurance policy of ` 70,000.
5. General expenses include (a) ` 500 given to Mrs. X for arranging a party in honor of a friend who has
recently come from Canada (b) ` 1 ,000 being contribution to a political party.
6. Loan was taken from Mrs. X for payment of arrears of income-tax.
ANSWER:387 `

SALEEM QURAISHEE Mo: 9175664444 INSPIRE ACADEMY-8888881719


PROFITS AND GAINS OF BUSINESS OR PROFESSION- Page 6.118

PROBLEM:388 X (age: 34 years) is businessman in Mumbai. Determine his net income and tax liability on
the basis of the following profit and loss account for the year ending March 31, 2020 :
Particulars ` Particulars `

Opening stock 1,04,000 Sales agency business 92,51,000

Purchases 80,08,750 Closing stock 2,10,000

Salaries and wages 1,75,000

Rent and rates 1,31,000

Commission 21,500

Household expenses 20,000

Income-tax for 2019-20 36,100

Advertisement 5,000

Postage and telegram 4000

Interest on own capital 84000

Reserve for bad debts 3400

Depreciation on furniture 18000

Net profit 850250

Total 9461000 Total 9461000


Other particulars
1. Closing stock and opening stock has consistently been valued at 10 per cent below cost price.
2. Depreciation on furniture, as per tax provisions, is ` 17,200.
3. Amount of sales includes a sum of ` 41,250 representing the value of goods withdrawn for the use of
X's family members. These goods were purchased at cost of ` 27,850. Market value of these goods is `
45,240.
4. Purchases include a consultancy bill of Rs. 1, 00,000 (out of Rs.1, 00,000, tax of Rs. 10,000 was
deducted at source but it is deposited on January 10, 2020)
5. Household expenses include a contribution of ` 6,000 towards public provident fund.
6. On September 20, 2019, X has received a gift of ` 96,000 from a friend settled in UK. The entire
amount is invested in NSC IX issue.
6. X purchases notified bonds of an infrastructure company on April 2, 2020 for ` 1, 02,000.
ANSWER:388 `

SALEEM QURAISHEE Mo: 9175664444 INSPIRE ACADEMY-8888881719


PROFITS AND GAINS OF BUSINESS OR PROFESSION- Page 6.119

PROBLEM:389 Mr. Sivam, a retail trader of Cochin, gives the following Trading and P&L A/c for the year
ended 31.03.2020:
Particulars Amount (Rs) Particulars Amount (Rs)
To Opening stock 90,000 By Sales 1,12,11,500
To Purchases 1,10,04,000 By Closing stock 1,86,100
To Gross Profit 3,03,600
1,13,97,600 1,13,97,600
To Salary 60,000 By Gross Profit b/d 3,03,600
To Rent and rates 36,000 By Income from UTI 2,400
To Interest on loan 15,000
To Depreciation 1,05,000
To Printing & stationery 23,200
To Postage & telegram 1,640
To Loss on sale of shares (ST) 8,100
To Other general expenses 7,060
To Net Profit 50,000
3,06,000 3,06,000
Additional Information:
1) It was found that some stocks were omitted to be included in both the opening and closing stock,
the values of which were:
a) Opening stock: Rs 9,000
b) Closing stock: Rs 18,000
2) Salary includes Rs 10,000 paid to his brother, which is unreasonable to the extent of Rs 2,000.
3) The whole amount of printing and stationery was paid in cash by way of one-time payment
4) The depreciation of Rs 1, 05,000 provided in P&L A/c was based on the following information "The
WDV of P&M is Rs 4, 20,000 as on 01.04.2019. A new plant falling under the same block of depreciation
was bought on 1.7.2019 for Rs 70,000. Two old plants were sold on 1.10.2019 for Rs 50,000"
5) Rent and rates include GST liability of Rs 3,400 paid on 07.04.2020.
6) Other general expenses include Rs 2,000 paid as donation to a Public Charitable Trust.
You are required to advise Mr. Sivam whether he can offer his business income u/s 44AD.
ANSWER:389

SALEEM QURAISHEE Mo: 9175664444 INSPIRE ACADEMY-8888881719


PROFITS AND GAINS OF BUSINESS OR PROFESSION- Page 6.120

PROBLEM:390 X (age: 64 years), a resident individual, furnishes the following particulars relevant for the
assessment year 2020-21
Profit and Loss Account for the year ending March 31, 2020
Particulars ` Particulars `

Salary to staff 34,000 Gross profit 16,86,000

General expenses 48,000 Commission and discount 2,17,200

Bad debts written off 15,000 Sundry receipts 43,000

Reserve for losses 2,000 Short-term profit on sale of 31,000


investment

Fire insurance premium (office 4,200


premises)

Advertisement ` 2400

Add : Outstanding ` 1,600 4,000

Interest on X's capital 3,500

Interest on bank loan 14,500

Expenditure on acquisition of a patent 17,000


right acquired and put to use on June
30, 2019

Lump sum consideration for acquiring 60,000


know-how on March 3, 2020

Depreciation on plant and machinery 28,000

Provision for outstanding sales tax and 13,000


excise duty

Net profit 17,34,000

Total 19,77,200 Total 19,77,200


Other information
1. Advertisement expenditure includes ` 3,400, being cost of 2 diaries (cost of each being ` 1,700)
presented to customers.
2. Depreciation on plant and machinery according to income-tax provision comes to ` 29,700.
3. Salary to staff includes payment of ` 8,000 to a relative which is unreasonable to the extent of `
3,000.
4. General expenses include (a) expenditure of ` 4,800, incurred by X on training of his employees, (b)
commission of ` 10,000 for securing a business order, and (c) compensation of ` 6,000 paid to on
employee while terminating his service in the business interest
5. Out of outstanding sales tax and excise duty, ` 3,000 is paid on July 10, 2020 and ` 8,000 is paid on
October 3, 2020. The balance is not paid as yet. Due date of filing return of income is July 31, 2020.

SALEEM QURAISHEE Mo: 9175664444 INSPIRE ACADEMY-8888881719


PROFITS AND GAINS OF BUSINESS OR PROFESSION- Page 6.121

6. Income of X from company deposit is ` 12,000, which is not shown in the Profit and Loss Account.
Determine the taxable income and tax liability of X for the assessment year 2020-21, assuming that
insurance premium paid by X on the life insurance policy (since 2011) of Mrs. X is ` 1,03,200(sum
1,0
assured Rs. 10,00,000.
ANSWER:390 `

PROBLEM:391 Mr. Gupta is having a trading business and his Trading and P&L A/c for PY 2019-20 as
under:
Particulars Amount (Rs) Particulars Amount (Rs)
To Opening stock 1,00,000 By Sales 2,70,00,000
To Purchases 2,49,00,000 By Closing stock 50,000
To Gross profit 20,50,000
Total 2,70,50,000 Total 2,70,50,000
Salary to employees (including PF 5,00,000 By Gross Profit b/d 20,50,000
contribution)
donation to Prime Minister's National 1,00,000
Relief Fund
Provision for bad debts 50,000
Bonus to employees 50,000
Interest on bank loan 50,000
Family planning expenditure incurred 20,000
on employees
depreciation 30,000
Income Tax 1,00,000
To Net Profit 11,50,000
Total 20,50,000 Total 20,50,000
Other information:
1) Depreciation allowable Rs 40,000 as per Income Tax Rules, 1962.
2) No deduction of tax at source on payment of interest on bank loan has been made.
3) Payment of bonus to workers made in the month of October 2019 on the occasion of Diwali festival.
4) Out of salary, Rs 25,000 pertains to employer's contributions to RPF which were deposited after the
due date of filing return of income. Further, employees' contribution of Rs 25,000 was also
deposited after the due date of filing ROI. Employees' contribution to PF has not been recorded in
the above P&L A/c
5) Calculate gross total income of Mr. Gupta for AY 2020-21.
ANSWER:391

PROBLEM:392 State with reasons, the allow ability of the following expenses incurred by MN Limited, a
wholesale dealer of commodities, under the Income Tax Act, 1961 while computing PGBP income for
Assessment Year 2020-21
1) Construction of school building in compliance with CSR activities amounting to Rs 5, 60,000.
2) Purchase of building for setting up a warehousing facility for storage of food grains amounting
to Rs 4, 50,000.
3) Interest on loan of Rs 50,000 paid to Mr. X (a resident) on which tax has not been deducted.
4) Commodities transaction tax paid Rs 20,000 on sale of bullion.
ANSWER:392

SALEEM QURAISHEE Mo: 9175664444 INSPIRE ACADEMY-8888881719


PROFITS AND GAINS OF BUSINESS OR PROFESSION- Page 6.122

PROBLEM:393 State, with reasons, the allow ability of the following expenses under the Income Tax Act,
1961, as deduction, while computing income from business/profession for Assessment Year 2020-21:
a) On 14.5.2019, ABC Ltd paid Rs 45,000 to Indian Railways for the use of railway assets pertaining to
PY 2019-20.
b) MNO Ltd paid Rs 55,000 as tax on non-monetary perquisite provided to an employee.
c) Rs 32,000 paid by 5 Ltd in cash on 28.3.2020 to a transporter (owning 8 goods carriages throughout
the previous year) for carriage of goods, without deduction of tax at source.
d) P Ltd paid Rs 80,000 in cash for purchase of wheat from a farmer on a banking day.
ANSWER:393

PROBLEM:394 Mr. B.A. Patel, a non-resident, operates an aircraft between London to Ahmedabad. For the
Financial year ended on 31st March, 2020, he received the amounts as under:
(i) For carrying passengers from Ahmedabad`50 lacs.
(ii) For carrying passengers from London`75 lacs received in India.
(iii) For carrying of goods from Ahmedabad`25 lacs.
The total expenditure incurred by Mr. B.A. Patel for the purposes of the business for the financial year 2018-19 was
`1.4 crores. Compute the income of Mr. B.A. Patel under the head "Profits and Gains from business or profession"
for the financial year ended on 31st March 2020 relevant to AY 2020-21.
ANSWER:394
PROBLEM:395 State with reasons, for the following sub-divisions, whether the following statements are true or
false having regard to the provisions of the Income-tax Act, 1961:
I. For a dealer in shares and securities, securities transaction tax paid in a recognized stock exchange is
permissible business expenditure.
II. Where a person follows mercantile system of accounting, an expenditure of`25,000 has been allowed on
accrual basis and in a later year, in respect of the said expenditure, assessee makes the payment of`25,000
through a cheque crossed "& Co.", disallowance of 25,000 under section 40A(3) can be made in the year of
payment.
III. It is mandatory for an assessee to claim depreciation under section 32 of the Income-tax Act, 1961.
IV. The Mediclaim premium paid to GIC by Mr. Lomesh for his employees, by a draft, on 27.12.2019 is a
deductible expenditure under section 36.
V. Under section 35DDA, amortization of expenditure incurred under eligible Voluntary Retirement Scheme at
the time of retirement alone, can be done.
VI. An existing assessee engaged in trading activities, can claim additional depreciation under Section 32(1)(iia)
in respect of new plant acquired and installed in the trading concern, where the increase in value of such
plant as compared to the approved base year is more than 10%.
ANSWER:395

PROBLEM:396 State, with reasons, the allow ability of the following expenses under the Income-tax Act,
1961 while computing income from business or profession for the Assessment Year 2020-21.
I. Provision made on the basis of actuarial valuation for payment of gratuity `5, 00,000. However, no
payment on account of gratuity was made before due date of filing return.
II. Purchase of oil seeds of`50,000 in cash from a farmer on a banking day.
III. Tax on non-monetary perquisite provided to an employee`20,000.
IV. Payment of`50,000 by using credit card for fire insurance.
V. Salary payment of 2, 00,000 outside India by a company without deduction of tax.
VI. Sales tax deposited in cash`50,000 with State Bank of India.
VII. Payment made in cash `30,000 to a transporter in a day for carriage of goods
ANSWER:396

SALEEM QURAISHEE Mo: 9175664444 INSPIRE ACADEMY-8888881719


PROFITS AND GAINS OF BUSINESS OR PROFESSION- Page 6.123

PROBLEM:397 Ramji Ltd., engaged in manufacture of medicines (pharmaceuticals), furnishes the


following information for the year ended 31.03.2019.
I. Municipal tax relating to office building `51,000 not paid till 30.09.2019
II. Patent acquired for `20, 00,000 on 01.09.2019 and used from the same month.
III. Capital expenditure on scientific research `10, 00,000 which includes cost of land `2, 00,000.
IV. Amount due from customer X, outstanding for more than 3 years, written off as bad debt in the
books `5, 00,000.
V. Income-tax paid `90,000 by the company in respect of non-monetary perquisites provided to its
employees.
VI. Provident fund contribution of employees `5, 50,000 remitted in July ,2019.
VII. Expenditure towards advertisement in souvenir of a political party `1,50,000.
VIII. Refund of sales tax`75,000 received during the year, which was claimed as expenditure in an earlier
year.
State with reasons the taxability or deductibility of the items given above under the Income-tax Act,
1961.Note: Computation of total income is not required.
ANSWER:397

PROBLEM:398 Raghav Industries Ltd. furnishes you the following information for the year ended 31-03-2020.
I. Scientific research expenditure related to its business `2, 40,000 fully revenue in nature.
II. Building acquired for scientific research (including cost of land`5, 00,000) in June 2019 for `12, 00,000.
III. Amount paid to Indian Institute of Science, Bangalore for scientific research `50,000.
IV. Demerger expenses incurred in financial year 2019-20 `5, 00,000.
V. Contribution to the account of employees as per pension scheme referred to in section 80CCD amounted to
`30, 00,000. Amount above 10% of the salary of employees is`7, 00,000.
VI. Amount recovered from employees towards provident fund contribution`12,00,000 of which amount
remitted upto the end of the year was`7,00,000 and the balance was remitted before the 'due date' for filing
the return prescribed in Section 139(1).
VII. Tax on non-monetary perquisites provided to the employees, borne by the employer`4, 50,000.
VIII. Gain due to change in the rate of exchange of foreign currency`1, 00,000 related to import of machinery.
The machinery was acquired two years ago and put to regular use since then. Explain in brief how the above
said items would be dealt with for the A. Y. 2020-21
ANSWER:398

PROBLEM:399 Answer the following with reference to the provisions of the Income-tax Act, 1961:
a) Bad debt claim disallowed in an earlier assessment year, recovered subsequently. Is the sum
recovered, chargeable to tax?
b) Tax deducted at source on salary paid to employees not remitted till the (due date' for filing the
return prescribed in section 139. Is the expenditure to be disallowed under section 40(a)(ia)?
c) X Co. Ltd. paid`120 lakhs as compensation as per approved Voluntary Retirement Scheme (VRS)
during the financial year 2019-20. How much is deductible under section 35DDA for the assessment
year 2020-21?
d) Bad debt of`50,000 written off and allowed in the financial year 2016-14 recovered in the financial
year 2019-20.
ANSWER:399

SALEEM QURAISHEE Mo: 9175664444 INSPIRE ACADEMY-8888881719


INCOME FROM OTHER SOURCES- Page 7.1

1. CHARGEABILITY- SEC 56(1)

The following conditions must be satisfied before an income can be taxed under the head "Income from
Other Sources"
I. There must be an INCOME;
II. Such income is not EXEMPT under the provisions of this Act,
III. Such income is not chargeable to tax under any first four heads viz, "Income from Salary", "Income
from House Property", 'Profits and Gains of Business or Profession" and "Income from Capital
Gain". Income from other sources is, therefore, a residuary head of income.

2. SPECIFIC INCOMES [SECTION 56(2)]

I. Dividends.
II. Winnings from lotteries, crossword puzzles, races including horse races, card games and other
games of any sort, or from gambling or betting of any form or nature whatsoever;
III. Any sum received by the assessee from his employees as contribution to any provident fund, or
any other welfare fund for the employees provided it is not taxable under the head 'Profits and
Gains of Business or Profession':
IV. Income by way of interest on securities provided the income is not chargeable to Income-tax
under the head profits, and gains of business or profession.

V. Income from machinery, plant or furniture belonging to the assessee and let on hire, provided
the income is not chargeable to Income-tax under the head profits and gains of business or
profession.

SALEEM QURAISHEE Mo: 9175664444 INSPIRE ACADEMY-8888881719


INCOME FROM OTHER SOURCES- Page 7.2

VI. Where the assessee lets on hire, the machinery, plant or furniture belonging to him and also
buildings, and letting of buildings, is inseparable from the letting of the said machinery, plant or
furniture the income from such letting, if it is not chargeable to income-tax under the head
profits and gains of business or profession.

VII. Any sum received under a Keyman Insurance Policy, including the sum allocated by way of bonus
on such policy, if such income is not taxable under the head "Salaries" or "Profits and gains of
business or profession”.
VIII. Gift-56(2)(X)-
IX. Income by way of interest received on compensation or on enhanced compensation to be taxed
in the year in which such interest is received.

X. Any compensation or other payment, due to or received by any person, by whatever name
called, in connection with the termination of his employment or the modification of the terms
and conditions relating thereto.(w.e.f 1-04-2019)
XI. Forfeiture of advance received for transfer of a capital asset to be taxed under the head income
from other sources

3. OTHER INCOMES -

I. Insurance commission;
II. Family pension (payments received by the legal heirs of a deceased employees);
III. Director's sitting fee for attending board meetings;
IV. Interest on bank deposits/deposits with companies, interest on loans
V. Income from undisclosed sources;
VI. Remuneration received by Members of Parliament;
VII. Interest on securities of foreign governments;
VIII. Examiner ship fees received by a teacher from an institution other than his employer;
IX. Total interest till date on employee's contribution to an unrecognized provident fund at the time
when the payment of lump sum amount from the unrecognized provident fund is due;
X. Rent from a vacant piece of plot of land; Ground rent
XI. Agricultural income from agricultural land situated outside India;
XII. Income from royalty, if it is not income from business or profession;
XIII. Director's commission for standing as a guarantor to bankers/ underwriting shares of a new
company;
XIV. Gratuity received by a director who, under the relevant contract is not an employee or servant of
the company, is assessable as income from other sources;
XV. Income from racing establishment;/Income from granting of mining rights;
XVI. Income from markets, fisheries, rights of ferry or moorings; Income from grant of grazing rights;
XVII. Interest paid by the Government on excess payment of advance tax, etc.;
XVIII. Income received after discontinuance of business
SALEEM QURAISHEE Mo: 9175664444 INSPIRE ACADEMY-8888881719
INCOME FROM OTHER SOURCES- Page 7.3

4. WHAT IS DIVIDEND?

I. Dividend in its ordinary connotation means the sum paid to or received by a shareholder
proportionate to his shareholding in a company out of the total sum distributed.
II. Section 2(22) of the Income-tax Act 1961 has devised a special inclusive definition of dividend. As
per the definition given in section 2(22), dividend includes the following disbursements by the
company to the shareholders, to the extent of accumulated profits.

SALEEM QURAISHEE Mo: 9175664444 INSPIRE ACADEMY-8888881719


INCOME FROM OTHER SOURCES- Page 7.4

1) TAX ON CERTAIN DIVIDENDS RECEIVED FROM DOMESTIC COMPANY- SECTION 115BBDA.


1) Applicable only to Specified assessee. -
2) Specified assessee means a person OTHER THAN,—
I. A domestic company; or
II. A fund or institution or trust or any university or other educational institution or any hospital or
other medical institution referred to in sub-clause (iv) or sub-clause (v) or sub-clause (vi) or sub-
clause (via) of clause (23C) of section 10; or
III. A trust or institution registered under section 12A or section 12AA.

2) Aggregate amount of dividend received from one or more domestic company during PY exceeds `
1,00,0000 at the rate of 10% (+SC+HEC).
3) The taxation of dividend income in aggregate exceeding ten lakh rupees shall be on gross basis.
However, this rule is not applicable in case of deemed dividend under section 2(22)(e)
4) No deduction is allowed from dividend income.

a) Any distribution by a company to the extent of accumulated profits involving the release of the assets
of the company. [Section 2(22)(a)]:
Two conditions are essential for treating a distribution to be deemed dividend:
I. the company should possess accumulated profits;
II. Such accumulated profits are distributed in cash or in kind. Where the distribution is in kind, the
market value of the asset (and not the book value) shall be deemed dividend in the hands of the
shareholders
Points to be noted-
However, mere capitalization of accumulated profits by issue of bonus shares to equity shareholders does
not entail the release of assets of the company and hence shall not be deemed to be dividend.

b) Distribution of Debentures/Deposit Certificates to shareholders and bonus shares to preference


shareholders [Section 2(22)(b)]
I. Any distribution to its shareholders (equity or preference) by a company of debentures,
debenture-stock, or deposit certificates m any form, whether with or without interest to the
extent of accumulated profits, whether capitalized or not, and
II. Any distribution to its preference shareholders of shares by way of bonus, to the extent to
which the company possesses accumulated profits, whether capitalized or not.

c) Distribution to shareholders on liquidation of the company [Section 2(22)(c)]


I. Any distribution made to the shareholders of a company on its liquidation, to the extent to which
the distribution is attributable to the accumulated profits of the company immediately before its
liquidation, whether capitalized or not.

d) Distribution on reduction of share capital [Section 2(22)(d)] :


I. Any distribution to its shareholders by a company on the reduction of its capital, to the extent to
which the company possesses accumulated profits, whether such accumulated profits have been
capitalized or not.

SALEEM QURAISHEE Mo: 9175664444 INSPIRE ACADEMY-8888881719


INCOME FROM OTHER SOURCES- Page 7.5

e) Loans/advances to certain shareholders/concerns [Section 2(22)(e)]:


I. Any payment by a company, (other than a company in which the public are substantially
interested), of any sum (whether as representing a part of the assets of the company or otherwise)
by way of advance or loan, to the extent of accumulated profits (excluding capitalized profits) to:-
• An equity shareholder, who is beneficial owner of shares holding not less than 10% of the
voting power, or
• any concern in which such shareholder (holding not less than 10% voting power) is a
member or a partner and in which he has a substantial interest i.e. holding 20% voting
power or 20% shares in the concern; or
• Any person, on behalf, or for the individual benefit, of any such shareholder. Such
shareholder here means a shareholder who is beneficial owner of shareholding not less than
10% voting power.

Dividends not to include the following:


I. Any advance or loan to a shareholder or specified concern by a company in the ordinary course
of its business where the lending of money is a substantial part of the business of the company.
'Ordinary course of business' shall mean that the loan or advance should be given to such
shareholder at the same rate and terms as it is given to other borrowers.
II. any dividend paid by a company which is set off by the company against the Whole or any part
of loan or advance previously paid by it and which has been treated as deemed dividend under
section 2(22)(e). In this case the dividend so set off shall not be treated as dividend in the hands
of shareholder

3) BASIS OF CHARGE OF DIVIDEND


I. Any income by way of dividends, referred to under section 115-O, is excluded from the total
income of the shareholder [Section 10(34)].
II. Under section 115-O, any dividend declared, distributed or paid by a domestic company,
whether out of current or accumulated profits, shall be charged to additional income-tax at a
flat rate of 15% in addition to normal income-tax chargeable on the income of the company.
This is known as corporate dividend tax
III. Corporate dividend tax @30% is leviable on deemed dividend under section 2(22)(e).
IV. Dividends received from a company, other than a domestic company, is still liable to tax in the
hands of the shareholder. For example, dividend received from a foreign company is liable to tax
in the hands of the shareholder.
V. It may, however, be noted that the exemption available under section 10(34) would not
be allowable in respect of dividend income chargeable to tax in accordance with the
provisions of section 115BBDA, even if the dividend distribution tax is paid by the
domestic company on such amount of dividend

SALEEM QURAISHEE Mo: 9175664444 INSPIRE ACADEMY-8888881719


INCOME FROM OTHER SOURCES- Page 7.6

PROBLEM:1 Rahul holding 28% of equity shares in a company, took a loan of ` 5,00,000 from the same
company. On the date of granting the loan, the company had accumulated profit of ` 4,00,000. The
company is engaged in some manufacturing activity.
I. Is the amount of loan taxable as deemed dividend, if the company is a company in which the
public are substantially interested?
II. What would be your answer, if the lending company is a private limited company (i.e. a
company in which the public are not substantially interested)?
ANSWER:1

PROBLEM:2 A Ltd., a domestic company, declared dividend of ` 170 lakh for the year F.Y. 2018-19 and
distributed the same on 10.7.2019. Mr. X, holding 10% shares in A Ltd., receives dividend of ` 17 lakh
in July, 2019. Mr. Y, holding 5% shares in A Ltd., receives dividend of ` 8.50 lakh. Discuss the tax
implications in the hands of Mr. X and Mr. Y, assuming that Mr. X and Mr. Y have not received
dividend from any other domestic company during the year.
ANSWER:2

PROBLEM:3 Find out the tax liability for the AY 2020-21 in the following cases
X (40 YEARS) Y (45 YEARS) Z FIRM ALTD B(61 YEARS)(NR)
BUSINESS INCOME 3,00,000 40,00,000 40,00,000 40,00,000 23,00,000
DIVIDEND FROM DC
a) D LTD 40,00,000 960000 15000 11,00,000 40,00,000
b) E LTD 10000 NIL 3000 18,00,000 10000
c) F LTD 890000 NIL 2982000 21,00,000 890000
Expenditure for 260000 90000 80000 1200000 260000
earning income
Deduction under 4,10,000 2,15,000 7,00,000 8,18,000 4,10,000
section 80C to 80U
ANSWER:3

PROBLEM:4 Dividend paid by MATRIX India Private Limited, an Indian company, was received by A.
Such dividend shall be exempt from tax in the hands of the shareholders provided they are not
covered u/s.
(a) 115BBDA (b) 115BDA (c) 115BAD (d) None of the above

PROBLEM:5 Which of the following categories of dividend, requires deduction of tax at source by the
company paying such dividend?
(a) 2(22)(e) (b) 2(22)(a) (c) 2(22)(d) (d) None of the above

PROBLEM:6 STAR India Private Limited, an Indian company, declared dividend amounting to Rs.
5,00,000 out of which 50% were received by Sumit. He wants to claim deduction of Rs. 50,000
against dividend income. The taxable income in the hands of Sumit would be
(a) 2,50,000 (b) 2,00,000 (c) Nil (d) 5,00,000

SALEEM QURAISHEE Mo: 9175664444 INSPIRE ACADEMY-8888881719


INCOME FROM OTHER SOURCES- Page 7.7

PROBLEM:7 ABC Mutual Fund is covered u/s 10(23D). It declares a dividend of Rs. 1,00,000 to
SANJAY. The amount taxable in hands of Saarthak on account of such dividend would be ................
(a) 2,50,000 (b) 2,00,000 (c) Nil (d) 5,00,000

PROBLEM:8 SONY India Private Limited, an Indian company, has accumulated profits of Rs. 5,00,000
out of which it capitalized profits of Rs. 1,00,000, by issuing bonus shares. Sumit, an equity
shareholder received, bonus shares of Rs. 50,000 out of such issue. The amount which would be
construed as dividend in the hands of SONY India, liable to DDT would be ................
(a) 50,000 (b) 1,00,000 (c) Nil (d) 75,000

PROBLEM:9 GAGAN India Private Limited, an Indian company, has accumulated profits of Rs.
5,00,000 out of which it capitalised profits of Rs. 1,00,000, by issuing bonus shares. Sumit received
bonus shares of Rs. 50,000 out of such issue. The amount which would be construed as taxable
dividend in the hands of the hands of Sumit, would be ................
(a) 50,000 (b) 1,00,000 (c) Nil (d) 75,000

PROBLEM:10 MATRIX India Private Limited, an Indian company, has accumulated profits of Rs.
5,00,000 out of which it distributed profits of Rs. 1,00,000, to its shareholders in cash. Sumit
received Rs. 50,000 out of such payment. The amount which would be construed as dividend in
the hands of MATRIX India, liable to DDT would be ................
(a) 50,000 (b) 1,00,000 (c) Nil (d) 75,000

PROBLEM:11 MATRIX India Private Limited, an Indian company, has accumulated profits of Rs.
5,00,000 out of which it distributed profits of Rs. 1,00,000, to its shareholders in cash. Sumit
received Rs. 50,000 out of such payment. The amount which would be construed as dividend in
the hands of Sumit, liable to income-tax would be ................
(a) 50,000 (b) 1,00,000 (c) Nil (d) 75,000

PROBLEM:12 TOY India Private Limited, an Indian company, has accumulated profits of Rs. 5,00,000.
It distributed assets of Rs. 1,00,000, with a market value of Rs. 2,00,000, to its shareholders.
Sumit received assets worth Rs. 50,000 of book value out of such distribution. The amount which
would be construed as dividend in the hands of TOY India, liable to DDT would be ................
(a) 1,00,000 (b) 2,00,000 (c) Nil (d) 50,000

PROBLEM:13 STAR India Private Limited, an Indian company, has accumulated profits of Rs. 5,00,000.
It distributed assets of Rs. 1,00,000, with a market value of Rs. 2,00,000, to its shareholders.
Sumit received assets worth Rs. 50,000 of book value out of such distribution. The amount which
would be construed as dividend in the hands of Sumit, liable to Income-tax would be ................
(a) 50,000 (b) 1,00,000 (c) Nil (d) 75,000

PROBLEM:14 Power India Private Limited, an Indian company, has accumulated profits of Rs.
5,00,000. It distributed debenture certificate of Rs. 1,00,000 and bonus shares of Rs. 50,000 to
preference shareholders. TONY received debenture certificate worth Rs. 50,000 out of such
distribution and bonus shares of 120,000. The amount which would be construed as dividend in the
hands of Powerplay India, liable to DDT would be ................
(a) 1,50,000 (b) 1,00,000 (c) Nil (d) 75,000

SALEEM QURAISHEE Mo: 9175664444 INSPIRE ACADEMY-8888881719


INCOME FROM OTHER SOURCES- Page 7.8

PROBLEM:15 Power India Private Limited, an Indian company, has accumulated profits of Rs.
5,00,000. It distributed debenture certificate of Rs. 1,00,000 and bonus shares of Rs. 50,000 to
preference shareholders. TARU received debenture certificate worth Rs. 50,000 out of such
distribution and bonus shares of Rs. 20,000. The amount which would be construed as dividend in
the hands of TARU , liable to income tax would be ................
(a) 50,000 (b) 70,000 (c) Nil (d) 25,000

PROBLEM:16A, a resident assessee received dividend of Rs. 15,00,000 from a domestic company.
Assuming his other income is equal to exemption limit, the tax payable by him shall be ................
(a) Rs. 50,000 (b) Rs. 25,000 (c) Normal slab rate (d) Nil

PROBLEM:17X held 30% shares in SHUBHAM Private Limited, which gave a loan of Rs. 2,00,000 to him
and had accumulated profits of Rs. 9,00,000. Such loan would be ................
(a) Liable to DDT in the hands of SHUBHAM Private Limited(b) Liable to tax in the hands of X
(c) Either (a) or (b) (d) Neither (a) nor (b)

PROBLEM:18STAR Private Limited, which had accumulated profits, gave a loan to Y, who holds 40%
equity shares in STAR Private Limited. Such loan would ................
(a) Not have any tax implications in the hands of STAR Private Limited
(b) Be liable to tax in the hands of Y (c) Exempt from tax in the hands of Y
(d) Either (a) or (c)

5. CASUAL INCOME-

1) Taxable @ flat rate 30% + (SC + HEC)


2) Winning from a motor car rally: Winning from a motor car rally is a return for skill and effort and
cannot be treated as casual income but taxable as normal income
3) Lottery held as stock in trade: Winning from lottery to an agent or trader out of its unsold stock
(tickets) shall be treated as incidental to business and taxed under the head "Profits & gains of
business or profession"
4) Income of jockey: Income of jockey from such profession is not treated as winning from horse
races.
5) Expenditure to be deducted: No deduction can be claimed from such income even if such
expenditure is incurred exclusively and wholly for earning such income.
6) Deduction: Deduction u/s 80C to 80U is not available from such income.

 Method of Grossing up-


Sometime in the problem, lottery income received is given rather than lottery income. In such case,
students are required to gross up the lottery income received. Relation between lottery income
earned and lottery income received is as under –
Lottery income received = Lottery income earned - Tax deducted at source on such income
Procedure of grossing up, in case of resident individual or HUF, is as follow-
Lottery Income Received = Gross Lottery Income -TDS @ 30% on Gross Lottery Income.
Lottery Income Received = 70% of Gross Lottery Income
Gross Lottery Income = Lottery Income Received
70%

SALEEM QURAISHEE Mo: 9175664444 INSPIRE ACADEMY-8888881719


INCOME FROM OTHER SOURCES- Page 7.9

TAX DEDUCTED AT SOURCE


SEC 194B 194BB
PAYER Any person paying winnings from Any person paying winning from
lotteries/cross word puzzle/card horse race
games/other games
RECEPIENT Any person Any person
PAYMENT COVERED Winnings from Winning from horse race
lotteries/crossword puzzle/card
games
AT WHAT TIME TAX HAS TO BE At the time of payment At the time of payment
DEDUCTED AT SOURCE
MAX AMT WITHOUT TDS ` 10000 or less than ` 10000 ` 10000 or less than ` 10000
RATE 30% (no SC or HEC) 30% (no SC or HEC)
WHEN PROV DO NOT APPLY - -
IS IT POSSIBLE TO GET PAYMENT Not possible Not possible
WITHOUT TDS OR LOWER TDS

PROBLEM:19 The rate of tax in case of winning from lotteries, crossword puzzles, races etc. is ................

PROBLEM:20 A earned winning from lotteries amounting Rs. 4,50,000 and has incurred expenditure of
Rs. 50,000 in respect of such winnings. The taxable income on such account would be ................

PROBLEM:21 B won Rs. 5,000 from horse races. The TDS shall be deducted at the rate of ................

PROBLEM:22 C won 3 10,000 from crossword puzzle. The TDS shall be deducted at the rate of ................

PROBLEM:23 Shyam won Rs. 20,000 from lottery. The amount of tax to be paid by him is ................

PROBLEM:24 Winnings from lotteries (gross) Rs. 90,000. Calculate the amount to be included while computing
income under the head 'Income from Other Sources'………………………….

PROBLEM:25 Winnings from horse race (net) Rs. 35,000. Calculate the amount to be included while computing
income under the head 'Income from Other Sources'……………………

PROBLEM:26 Miss. Z received an amount of Rs. 27,760 on 1-12-2019 in connection with winning from camel
races. Cost of race tickets purchased was ` 2,000. Calculate amount to be included under the head 'Income
from Other Sources':

SALEEM QURAISHEE Mo: 9175664444 INSPIRE ACADEMY-8888881719


INCOME FROM OTHER SOURCES- Page 7.10

6. INTEREST ON SECURITIES-

1) Income, by way of interest on securities, is chargeable under the head "income from other sources", if
such income is not chargeable to income-tax under the head, "Profits and Gains of Business or
Profession".

2) According to section 2(28B) "Interest on securities" means:


 interest on any security of the Central Government or a State Government;
 interest on debentures or other securities for money issued by, or on behalf of a local authority
or a company or a corporation established by Central, State or Provincial Act.

3) Chargeability of Interest on Securities:


I. Interest on securities may be taxed on receipt basis or on due basis, depending upon the system of
accounting if any, adopted by the assessee.
II. If the assessee follows the cash system of accounting interest is taxable on receipt basis otherwise
it shall be taxable on due basis.
III. If no system of accounting is followed, it will always be taxable on 'due' basis.

4) Accrual of Interest:
I. Interest on securities accrues or becomes due on a specified date and not on a day-to-day basis.
II. The date on which the interest shall become due is specified by the issuing authority.
III. Interest may become due on quarterly basis, half yearly basis or annual basis, depending upon the
term of the issue.

5) Grossing up of Interest:
I. Tax is also to be deducted at source on interest on securities at the prescribed rates of tax
II. Tax is charged at gross amount of interest.
III. If net is given-Gross up…(Gross up=Net + TDS )
IV. If net-interest is given, it has to be grossed up to arrive at the taxable amount.

6) TDS ON INTEREST ON SECURITIES (SEC 193)

Payer Payer of interest on securities


Recipient A resident person holding securities
Payment covered Interest on securities
At what time payment has to be deducted at source At the time of payment or at the time of credit
whichever is earlier.
Maximum amount which can be paid without TDS -
Rate of TDS 10%
When provision are not applicable Interest on Central/state government securities
Is it possible to get the payment without TDS or with Yes
lower TDS

SALEEM QURAISHEE Mo: 9175664444 INSPIRE ACADEMY-8888881719


INCOME FROM OTHER SOURCES- Page 7.11

7. AVOIDANCE OF TAX SEC 94-

A. BOND WASHING TRANSACTION SEC- 94(1)


I. A bond washing transaction is narrated as a transaction which consist of selling securities to a
friend or relative before due date and reacquire back the same or similar securities after due date
of interest is over.
II. This practice is adopted by high income class assessee who transfers the security to low income
class.
III. Interest is received by the transferee.
IV. By virtue of section 94(1) interest is taxable in the hands of transferor.

B. SALES CUM INTEREST SEC-94(2)


I. If an assessee having beneficial interest in securities sells them in such a way that either no income
is received or income received is less than the sum he would have received if interest had accrued
from day to day
II. Income from such securities for such year would be deemed as income of such person.

C. EXCEPTIONS-SEC 94(1), (2) are not applicable if the security owner proves to AO.
I. There has been no avoidance of tax
II. The avoidance of tax was exceptional and not systematic and there was not any avoidance of tax
during last three years preceding the PY.

PROBLEM:27As per section 2, interest on securities includes ................


(a) Interest on security of the Central/State Government
(b) Interest on debentures or securities issued by local authority, company or corporation
established by Central or State Act (c) Both (a) and (b) (d) None of the above

PROBLEM:28Interest on securities is ................


(a) Taxable only receipt basis (b) Taxable only due basis
(c) Either (a) or (b), as per method of accounting adopted by assessee (d) Not taxable

PROBLEM:29 TDS @ 10% is deducted on income from which of the following securities?
(a) Government Securities except 8% Saving (Taxable) Bonds
(b) Government Securities in nature of 8% Saving (Taxable) Bonds
(c) Both (a) and (b) (d) None of the above

PROBLEM:30 Bond Washing Transaction ensure that ................


(a) Shifting of interest income is not allowed in certain cases
(b) Shifting of interest income is allowed in certain cases
(c) Either (a) or (b) (d) None of the above

SALEEM QURAISHEE Mo: 9175664444 INSPIRE ACADEMY-8888881719


INCOME FROM OTHER SOURCES- Page 7.12

PROBLEM:31Arun owned certain securities as on 31.3.2019, on which interest of 1,00,000 was due on
that date, he sold such securities on April 1,2019 to Varun. The interest on such securities was
received by Varun on April 21,2019 and bought back such securities on 1.05.2019. In such a case,
the interest income will be taxable in the hands of ................
(a) Arun (b) Varun (c) Exempt from tax (d) Equally in the hands of Arun and Varun

PROBLEM:32Arun owned certain securities as on 31.3.2019, on which interest of Rs. 1,00,000 was
received. The deductions which shall be allowed to Arun against such interest income would be
................
(a) Collection charges paid to banker
(b) Interest on money borrowed to make such Investments
(c) Both (a) and (b) (d) Either (a) or (b)

SALEEM QURAISHEE Mo: 9175664444 INSPIRE ACADEMY-8888881719


INCOME FROM OTHER SOURCES- Page 7.13

8. GIFT- [SECTION 56(2) (X)]


1) Any sum of money or value of property received without consideration or for inadequate
consideration to be subject to tax in the hands of the recipient [Section 56(2)(x)]
2) In order to prevent the practice of receiving sum of money or the property without consideration or
for inadequate consideration, section 56(2)(x) brings to tax any sum of money or the value of any
property received by any person without consideration or the value of any property received for
inadequate consideration.
3) Applicability of section 56(2)(x): The provisions of section 56(2)(x) would apply only to property
which is the nature of a capital asset of the recipient and not stock-in-trade, raw material or
consumable stores of any business of the recipient. Therefore, only transfer of a capital asset,
without consideration or for inadequate consideration would attract the provisions of section
56(2)(x).
 PROPERTY means-
i. Immovable property being land or building or both,
ii. shares and security
iii. Jewellery, archaeological collections, drawings, paintings, sculptures, any work of art, bullion.

 RELATIVE FOR THE PURPOSES MEANS


i. spouse of the individual/brother or sister of the individual,
ii. brother or sister of the spouse of the individual,
iii. brother or sister of either of the parents of the individual,
iv. any lineal ascendant or descendant of the individual,
v. any lineal ascendant or descendant of the spouse of the individual, and
vi. Spouse of a person referred to in items (ii) to (vi) mentioned above.

A. Sum of Money: If any sum of money is received without consideration and the aggregate value of
which exceeds ` 50,000, the whole of the aggregate value of such sum is chargeable to tax.

B. Immovable property [Land or building or both]:

I. If an immovable property is received


a) Without consideration: The stamp duty value of such property would be taxed as the income of the
recipient if it exceeds ` 50,000.
b) For Inadequate consideration: If consideration is less than the stamp duty value of the property
and the difference between the stamp duty value and consideration is more than the higher of –
i. ` 50,000 and
ii. 5% of consideration,
The difference between the stamp duty value and the consideration shall be chargeable to
tax in the hands of the assessee as “Income from other sources”.

SALEEM QURAISHEE Mo: 9175664444 INSPIRE ACADEMY-8888881719


INCOME FROM OTHER SOURCES- Page 7.14

II. Value to be considered where the date of agreement is different from date of registration: Taking
into consideration the possible time gap between the date of agreement and the date of
registration, the stamp duty value may be taken as on the date of agreement instead of the date of
registration, if the date of the agreement fixing the amount of consideration for the transfer of the
immovable property and the date of registration are not the same, provided whole or part of the
consideration has been paid by way of an account payee cheque or an account payee bank draft or
by use of electronic clearing system (ECS) through a bank account or through such prescribed
electronic mode on or before the date of agreement.

III. If the stamp duty value of immovable property is disputed by the assessee, the Assessing Officer
may refer the valuation of such property to a Valuation Officer. If such value is less than the stamp
duty value, the same would be TAKEN for determining the value of such property, for computation
of income under this head in the hands of the buyer.

 STAMP DUTY VALUE-Means the value adopted or assessed or assessable by any authority of the
Central Government or a State Government for the purpose of payment of stamp duty in respect of
an immovable property.

CASE1

Particulars SITUATION-1 SITUATION-2

Declared consideration 200 200

Stamp duty value 220 208

Difference as a percentage of declared consideration 20/200 x 100 = 10% 8/200 x 100 = 5%

Stamp Duty Value > 5% of declared consideration Yes No

For the purpose of computing income under section 56(2)(x)


a) Situation -1. Rs. 20 lakhs (Rs. 220 lakhs less Rs. 200 lakhs) will be regarded as income, since the
stamp duty value exceeds the declared consideration by more than 5% of the consideration and
also Rs. 50,000. This Rs. 20 lakhs will be taxable for the recipient as income of other sources.
b) Situation -2, since the difference is less than 5%, it will not be treated as income and nothing will
be taxable for the recipient.

C. Movable Property [Property, other than immovable property]-If movable property is received

a) Without consideration: The aggregate fair market value of such property on the date of receipt
would be taxed as the income of the recipient, if it exceeds ` 50,000.

b) For inadequate consideration: If the difference between the aggregate fair market value and
such consideration exceeds `50,000, such difference would be taxed as the income of the
recipient.

SALEEM QURAISHEE Mo: 9175664444 INSPIRE ACADEMY-8888881719


INCOME FROM OTHER SOURCES- Page 7.15

POINT TO BE NOTED-
1) EXEMPTED CATEGORY-Any sum of money or property received from the following shall not be
chargeable
I. Received from relative
II. Received on the occasion of marriage of Individual.
III. Received by way of will or inheritance
IV. Received in the contemplation of death of payer
V. Received from a local authority, fund, foundation, university, other educational institution, and
hospital/Received from a charitable institute
VI. By way of transaction not regarded as transfer under section 47 .
VII. From an individual by a trust created solely for the benefit of the relative of the individual.

2) GIFT RECEIVED BY MEMBER FROM HUF- If HUF has given gift to its member, it will be taxable for
the receiver as income of other sources.

3) GIFT RECEIVED BY EMPLOYEE FROM EMPLOYER- Gift, Voucher or Gift Token: Section 17(2)(viii):
Tax treatment in hands of employee

Nature of gift Amount/value of gift which is less Amount/value of gift which is


than or equal to Rs. 5,000 more than Rs. 5,000

Gift in cash Fully taxable Fully taxable

Gift in kind or gift voucher or gift Fully exempt Fully taxable


token

CASE2 Mr. JOHN is employed in GAGAN Ltd. He has received a cash gift of Rs. 11,000 from his
employer on the occasion of his birthday. In this case full amount of Rs. 11,000 shall be taxable for
Mr. JOHN as his income under the head Salary.

CASE3 Mr. JATIN is employed in SANTOSH Ltd. He has received a wrist watch as a gift which has
Fair Market Value of Rs. 4,000 from his employer on the occasion of his birthday. In this case full
amount of Rs. 4,000 shall be exempt for Mr. JATIN.

CASE4 Mr. MANOJ is employed in XYZ Ltd. He has received a wrist watch as a gift which has Fair
Market Value of Rs. 11,000 from his employer on the occasion of his birthday. In this case full
amount of Rs. 11,000 shall be taxable for Mr. MANOJ as his income under the head Salary.

SALEEM QURAISHEE Mo: 9175664444 INSPIRE ACADEMY-8888881719


INCOME FROM OTHER SOURCES- Page 7.16

4) GIFT RECEIVED IN CONNECTION WITH BUSINESS/PROFESSION.- Gifts or Perquisites from Clients


which are in connection with business or profession are covered under section 28. If any person
has received any gift or perquisite or benefit either in cash or in kind from any of his clients, it will
be considered to be business receipt and shall be taken into consideration while computing PGBP
income. Further asset received may be capital asset or may not be a capital asset.

CASE5 A Doctor has received an ambulance as gift from one of his patients and its Fair Market
Value is Rs. 16,00,000. Doctor has been fully compensated for his services and this has been given
to him over and above the normal professional fees. In this case it will be considered to be PGBP
income of doctor and taxable will be full Rs. 16,00,000.

5) GIFT BY RESIDENT TO NON RESIDENT- If gift has been paid/given outside India on or after 5/7/2019
by resident to non-resident or to foreign company then it shall be deemed to accrue or arise in
India and shall be taxable in India for that non-resident or foreign company as income of other
sources: Section 9(1)(viii)

CASE6 Mr. X has received three gifts from his three friends
I. Rs.55000 in cash
II. Land with market value Rs.5,00.000 but the value for the purpose of charging stamp duty
Rs.4,00,000.
III. Jewellery with market value Rs.3.00.000
In this case, taxable amount shall be 55,000 + 4,00,000 + 3,00,000 = 7,55,000

CASE7 Mr. X has received gift of Rs.50,000 in cash friend his friend, in this case it will not be
considered to be his income.

CASE8 Mr. X has received gift of Rs.1,50,000 in cash from his brother, in this case it will not be
considered to be his income.

CASE9 Mr. X has received gift of Rs. 1,50,000 in cash from his mother’s sister, in this case it will not
be considered to be his income

CASE10 Mr. X has received gift of Rs. 1,50,000 in cash from his father’s brother, in this case it will not
be considered to be his income

CASE11 Mr. X has received gift of Rs. 1,50,000 in cash from his cousin, in this case it will be
chargeable to tax.

CASE12 Mr. X has received gift of Rs. 1,50,000 in cash from brother of his spouse, in this case it will
not be considered to be his income.

CASE13 Mr. X has received gift of Rs.1,50,000 in cash from his grand father, in this case it will not be
considered to be his income

SALEEM QURAISHEE Mo: 9175664444 INSPIRE ACADEMY-8888881719


INCOME FROM OTHER SOURCES- Page 7.17

CASE14 Mr. X has received gift of Rs.1,50,000 in cash from spouse of his brother, in this case it will
not be considered to be his income

CASE15 Mr. X has received gift of Rs.1,50,000 in cash from husband of his sister, in this case it will
not be considered to be his income

CASE16 Mr. X has received gift of Rs.1,50,000 in cash from sister of his brother’s wife, in this case it
will be considered to be his income.

CASE17 Mr. X has received gift of Rs.1,50,000 in cash from the sister of his spouse, in this case it will
not be considered to be his income.

CASE18 Mr. X has received gift of Rs.5,000 in cash on his birthday from each of his eleven friends, in
this case it will be considered to be his income because the total amount is exceeding Rs.50,000.

CASE19 Mr. X has received gift of property valued Rs.1,50,000 from his friend, in this case it will be
considered to be his income.

CASE20 Mr. X has received gift of Rs.1,50,000 in cash from his friend on the occasion of his
marriage, in this case it will not be considered to be his income.

CASE21 Mr. X has received gift of Rs.75,000 in cash and property Rs.75,000 from his fiancee, in this
case gift in cash will be considered to be his income and the gift as property shall also be considered
to be his income

SALEEM QURAISHEE Mo: 9175664444 INSPIRE ACADEMY-8888881719


INCOME FROM OTHER SOURCES- Page 7.18

SALEEM QURAISHEE Mo: 9175664444 INSPIRE ACADEMY-8888881719


INCOME FROM OTHER SOURCES- Page 7.19

PRACTICAL PROBLEMS
PROBLEM:33 R, the friend of A has gifted an immovable property to A whose stamp duty value on the date of
gift is ` 45, 00,000.
1) What shall be the value of Gift taxable under section 56?
2) What shall be the amount taxable if A purchased the above property from R for a consideration of ` 36,
00,000?
PROBLEM:34 If any person has purchased immovable property for Rs.20 lakh but stamp duty value is Rs.23
lakhs, this case taxable amount of gift shall be
(a)3 lakh (b) 1 lakh (c) Nil (d) 23 lakh

PROBLEM:35 Mr. Kashyap has acquired a building from his friend on 10.10.2019 for Rs. 15,00,000. The stamp
duty value of the building on the date of purchase is Rs. 15,70,000. Income chargeable to tax in the hands of
Mr. Kashyap is
(a) Rs. 70,000 (b) Rs. 50,000 (c) Nil (d) Rs. 20,000

PROBLEM:36 Anil took possession of property on 31st August 2019 booked by him three years back at Rs.25
lakhs, The Stamp Duty Value (SDV) of the property as on 31st August 2019 was Rs.3l lakh and on date of
booking it was Rs.29 lakh. He had paid Rs.2 lakh by A/c payee cheque as down payment on date of booking.
Which of the following will be considered as income, if any, and in which previous year
(a) Rs.4 lakhs in P.Y. 2019-20 (b) Rs.4 lakhs in P.Y. 2016-17 (c) Rs.6 lakhs in P.Y. 2019-20
(d) No income shall be taxable, since down payment was paid by A/c cheque while booking the property

PROBLEM:37 Mr. Karan celebrated his 50th marriage anniversary. On this occasion, his wife received a diamond
necklace worth Rs.5,00,000 from Karan's brother. Karan’s son gifted him a luxurious car worth Rs.15,00,000,
His grandchildren gifted them a new furniture set worth Rs.3,00,000. Also, he received cash gifts from his
friends amounting collectively to Rs.80,000. Which of them the following statements stand true on
taxability?
a) Neither Mr. Karan nor Mrs. Karan will be liable for tax for any gifts since they have been received on
occasion of marriage anniversary
b) Mr. Karan & Mrs. Karan will jointly share the tax liability on all the gifts
c) Mrs. Karan will be liable to pay tax on diamond set and Mr. Karan will bear tax for the cash gifts received
d) Mr. Karan will be liable for tax on cash gifts only.

PROBLEM:38 Mr. X received cash gift Rs. 51,000 and gift of jewelry valued Rs. 49,000, in this case taxable
amount shall be
(a) Rs. 51.000 (b) Rs. 49.000 (c)Rs. 1.00.000 (d)Nil (e) none of these

PROBLEM:39 Mr. X received cash gift Rs. 40,000, gift of land stamp duty value Rs. 40,000 and gift of building
stamp duty value Rs. 40,000, in this case taxable amount shall be
(a) Rs. 40,000(b) Rs. 80,000 (c) Rs. 1,20,000 (d) Nil (e) none of these

PROBLEM:40 Mr. X purchased one house property for Rs. 3,00,000 market value Rs. 7,00,000 stamp duty value
Rs. 3,40,000, in this case taxable amount shall be
(a) Rs. 4.00.000 (b) Rs. 441.000 (c) Nil (d) Rs. 3.40.000 (e)none of these

SALEEM QURAISHEE Mo: 9175664444 INSPIRE ACADEMY-8888881719


INCOME FROM OTHER SOURCES- Page 7.20

PROBLEM:41 R receives ` 2, 00,000 from his father, ` 50000 from his friend S and ` 20,000 from another friend
T on his occasion of birthday on 29.05.2018. Compute the sum taxable in the hands of R?

PROBLEM:42 R receives a sum of ` 15000 each from his friends S and T and another sum of ` 20000 from his
friend U. Discuss the taxability of the sum received.

PROBLEM:43 R receives ` 10 lakhs from his relative and parents of his wife on occasion of their marriage .He
also receives one car and huge amount of jewellery from parents of his wife. He receives ` 600,000 from
person other than relative on the said occasion .Discuss the taxability of such gifts?

PROBLEM:44 R receives a gift of car valuing ` 600,000 from his friend. Discuss the taxability of such gifts?

PROBLEM:45 An individual has received a gift of Rs30,000 each during the previous year from his two friends,
the amount taxable under the head income from the other sources shall be:
a) Rs 10,000 b) Rs60, 000 c) Nil

PROBLEM:46 Cash gifts exceeding ____ shall be chargeable under the head income from other sources:
(a) Rs. 5,000 (b) Rs. 20,000 (c) Rs. 50,000 (d) Rs. 25,000

PROBLEM:47 The above clause shall not apply if this gift is received from:
(a) Relative (b) Local authority (c) By way of inheritance (d) All of these

PROBLEM:48 Mrs. X received the following gifts during the year. Which gifts shall be included in computing the
income from other sources?
(a) Gift of Rs. 26,000 from her employer.
(b) Gift of 21,000 on 15th December from her mother's friend.
(c) Gift of Rs. 1, 21,000 from her husband's brother.
(d) Gift of `60,000 on 25 November from her father's brother.

PROBLEM:49 Mr. X received the following gifts during the year. Which gifts shall not be included in computing
the income from other sources?
(a) Scholarship of Rs. 1, 20,000 from a charitable institution registered under section 12AA.
(b) Gifts of Rs. 51,000 each received from her four friends on the occasion of her marriage on 21st October.
(c) Gift of Rs. 1, 41,000 from her husband's brother. (d) None of the above.

PROBLEM:50 X received gift of jewellery, fair market value of which is Rs. 3, 00,000 on 17th October from her
fiancée. What will be the taxable amount?
(a) Nil (b) Rs. 3, 00,000 (e) Rs. 2, 50,000 (d) Rs. 50,000

SALEEM QURAISHEE Mo: 9175664444 INSPIRE ACADEMY-8888881719


INCOME FROM OTHER SOURCES- Page 7.21

PROBLEM:51 Rohan received a watch worth Rs. 60,000 from his cousin grandfather (brother of his grandfather).
What will be the taxable amount?
(a) Nil (b) Rs. 60,000 (c) Rs. 10,000 (d) Rs. 50,000

PROBLEM:52 If A receives Rs. 31,000 from B and Rs. 20,000 from C, then, what will be the taxable amount.
(a) Nil (b) Rs. 1,000 (c) Rs. 51,000 (d) Rs. 50,000

PROBLEM:53 Sohan received a share of 60,000 from his cousin grandfather (brother of his grandfather). What
will be the amount?
(a) Nil (b) Rs. 60,000 (c) Rs. 10,000 (d) Rs. 50,000

PROBLEM:54 Nisha received a gift from his sister in Netherlands of Rs. 2,50,000. What will be the taxable
amount.
(a) Exempt (b) Rs. 2,50,000 (c) Rs. 2,00,000 (d) Rs. 50,000

PROBLEM:55 Rakesh received Rs. 70,000 from his friend on the occasion of his birthday.
(a) The entire amount of Rs. 70,000 is taxable (b) Rs. 20,000 is taxable
(c) The entire amount is exempt (d) None of the above.

PROBLEM:56 Hemali received a cash gift of Rs. 80,000 from her friend on her 25th wedding anniversary,
Amount taxable is:
(a) Exempt (b) Rs. 80,000 (c) Rs. 30,000 (d) Rs. 50,000

PROBLEM:57 Gift of Rs. 5,00,000 received on 1 July, 2019 through account payee cheque from a non-relative
regularly assessed to income-tax, is –
(a) A capital receipt not chargeable to tax
(b) Chargeable to tax as income from other sources
(c) Chargeable to tax as business income
(d) Exempt Upto Rs. 50,000 and balance chargeable to tax as income from other sources.

PROBLEM:58 Mr. A received cash gift worth Rs. 55,000 from his grand father's brother Mahesh, on the occasion
of the marriage of his son. What will be the taxable amount –
(a) Rs. 55,000 (b) Nil (c) Rs. 50,000 (d) Rs. 5,000

PROBLEM:59 Mr. J received a Watch worth Rs. 55,000 from his employer on the occasion of his birthday. What
will be the tax consequences?..................................

SALEEM QURAISHEE Mo: 9175664444 INSPIRE ACADEMY-8888881719


INCOME FROM OTHER SOURCES- Page 7.22

PROBLEM:60 On 5th February, 2019 Rajat gets a gift of motor car from a relative Madan. Fair market value of the
car is Rs. 3,60,000. The amount taxable in the hands of Rajat under section 56(2)(X) is –
(a) Rs.3,60,000 (b) Rs.3,10,000 (c) Nil (d) 50,000

PROBLEM:61 On 30 December, 2018, Raju gets by gift a commercial flat from the elder brother of his father-in-
law (stamp duty value is 25, 00,000). The amount chargeable to tax in the hands of Raju is – ………………………..

PROBLEM:62 Rohan received gift of immoveable property from his friend stamp duty value 6, 00,000. What will
be the taxable amount. …………………………

PROBLEM:63 Ramesh purchased an immovable property for a consideration ` 11,00,000. The stamp duty value
is Rs. 4,00,000. What will be the taxable amount……………………………….

PROBLEM:64 Manju purchased bullion for Rs. 4,40,000 whose fair market value is Rs. 4,85,000. What will be the
taxable amount. ………………………………………

PROBLEM:65 Rajesh gifted an immovable property for a consideration of ` 70,00,000. The stamp duty value as
on the date of agreement is ` 81,00,000 whereas stamp duty value on the date of registration was `
80,50,000. Part of the amount of consideration of ` 34,00,000 has been received by cheque on the date of
agreement for transfer of the asset. What will be the taxable amount………………………………………..

PROBLEM:66 Rajni received a movable property without consideration amounting to Rs. 3,00,000 on the
occasion of her marriage. The aggregate fair market value of the property was of Rs. 4,50,000. What will be
the taxable amount…………………………………

PROBLEM:67 Rakhi received a movable property without consideration amounting to Rs. 3,00,000. The
aggregate fair market value of the property was of Rs. 4,50,000. What will be the taxable
amount…………………..

PROBLEM:68 Raima received a movable property for d consideration amounting to Rs. 3,00,000. The aggregate
fair market value of the property was of Rs. 4,50,000. What will be the taxable amount………………………..

SALEEM QURAISHEE Mo: 9175664444 INSPIRE ACADEMY-8888881719


INCOME FROM OTHER SOURCES- Page 7.23

PROBLEM:69 Ms R who draws a salary of ` 20,000 p.m. received the following gifts during the previous year
2019-20.
i. Gift of ` 5,00,000 on 16.04.2019 from a friend.

ii. Gift of jewellery fair market value of which is ` 3,00,000 on 17.05.2019 from her fiancée.

iii. Gifts of ` 51,000 each received from her 4 friends on the occasion of her marriage on 21.10.2019.

iv. Gift of ` 1,00,000 on 22.11.2019 from her mother’s sister.

v. Gift of ` 60,000 on 25.11.2019 from her father’s brother.

vi. Gift of ` 50,000 from her husband’s friend on 1.12.2019.

vii. Gift of ` 21,000 on 15.12.2019 from her mother’s friend.

viii. Gift of ` 26,000 on 25.12.2019 from her brother’s father- in- law.

ix. Gift of ` 1,21,000 from her husband’s brother.

x. Gift of ` 26,000 from her employer.

xi. Scholarship of ` 1,20,000 from a charitable institution registered under section 12AA.

xii. He has purchased a immovable property from B who is not his relative from a sum of ` 24,50,000. The
stamp duty value of the property is ` 26,00,000.

xiii. She purchased bullion for ` 4,40,000 whose fair market value is ` 4,85,000.

xiv. Gift of immovable property from her friend whose stamp duty value is ` 5,00,000.

Compute her total income for the assessment year 2020-21.

ANSWER:69

9. DEDUCTIONS ALLOWABLE- (SEC 57)

I. Commission or remuneration or bank charges for realizing dividend or interest on securities.


II. Deduction in respect of employee’s contribution towards staff welfare scheme
III. Repairs, depreciation, Insurance in case of letting out of plant, machinery, furniture and building
IV. Standard deduction in case of family pension- 1/3rd of pension or ` 15000 (whichever is less)
V. Any other expense for earning income-if following four conditions are satisfied.
 Expenditure must be wholly and exclusively for earning the income
 Expenditure must not be in nature of capital/personal nature.
 Must be laid out or expended in PY.

VI. Amount deductible from interest on compensation- 50% of interest is deductible

SALEEM QURAISHEE Mo: 9175664444 INSPIRE ACADEMY-8888881719


INCOME FROM OTHER SOURCES- Page 7.24

10. AMOUNT NOT DEDUCTIBLE UNDER SECTION 58-

a) Any personal expense of the assessee.


b) Any interest chargeable to tax under the Act which is payable out India on which tax has not
been paid or deducted at source.
c) Any payment chargeable to tax under the head “Salaries”, if it is payable outside India unless tax has
been paid thereon or deducted at source there from.
d) 30% of sum payable to a resident on which tax is deductible at source, if such tax has not been
deducted or after deduction has not been paid on or before the due date of return specified in
section 139(1)
e) Any expenditure in respect of which a payment is made to a person, to the extent the same is
considered excessive or unreasonable by the Assessing Officer, having regard to the FMV.
f) Any expenditure in respect of which payment or aggregate payments exceeding ` 10,000 is made to a
person in a day otherwise than account payee cheque/bank draft or ECS through bank account.
g) Expenditure in respect of winning from lottery

11. DEEMED PROFIT- SEC 59


Loss or expenditure already allowed in computation of Income from other sources u/s 57 and subsequently
recovered shall be treated as income of the PY in which it is recovered.

12. RECEIPTS OF SHARES BY A FIRM OR A CLOSELY HELD COMPANY

I. Recipient is a firm or a closely held company.


II. The asset (which is received )is in the form of shares in a closely held company
III. These shares are received from any person on or after June 1,2010.
IV. Such shares are received without consideration or inadequate consideration.
V. Such shares are not received by way of a transaction referred to in sec 47
If these conditions are satisfied then value of such shares are taxable in the hands of recipient (i.e. firm
or closely held company) if aggregate value exceeds ` 50000.

13. SHARE PREMIUM IN EXCESS OF FAIR MARKET VALUE

I. Recipient is a company(not being a company in which public are substantially interested)


II. It receives consideration for issue of shares from a resident person
III. The consideration for issue of shares exceeds the face value of such shares. In other words shares
are issued at premium.
 If above conditions are satisfied the aggregate consideration received for such shares exceeds the
FMV of the shares, shall be chargeable to income tax in the hands of recipient company under the
head income from other sources.
 ABOVE PROVISIONS DO NOT APPLY-
 Where consideration for issue of shares received by a venture capital undertaking from a
venture capital company or a venture capital fund OR SPECIFIED FUND
 Where the consideration for issue of shares is received by a company from class of a person as
notified by the Central Government.

SALEEM QURAISHEE Mo: 9175664444 INSPIRE ACADEMY-8888881719


INCOME FROM OTHER SOURCES- Page 7.25

CASE22 The following are the details of the shares issued by the following closely held companies.
Discuss the applicability of provisions of Section 56(2)(viib) in the hands of these companies:
Company No of Face Value FMV of Issue Price Applicability of Section 56(2)(viib)
Shares of Shares Shares of Shares
A Pvt Ltd 10,000 100 120 130 I. The provisions of Section 56(2)(viib) are
attracted in this case since the shares are
issued at premium and the issue price is also
more than FMV.
II. The excess of the issue price of the shares over
the FMV would be taxable u/h income from
other sources in the hands of A Pvt Ltd.
III. Taxable amount = Rs 1,00,000 [10,000 shares x
Rs 10 (Rs 130 - Rs 120)].
B Pvt Ltd 20,000 100 120 110 I. Although the shares are issued at premium but
the issue price of shares is less than their FMV.
II. Therefore, no sum shall be chargeable to tax in
the hands of B Pvt Ltd u/s 56(2)(viib).
C Pvt Ltd 30,000 100 90 98 I. Since the shares are issued at discount, the
provisions of Section 56(2)(viib) are not
attracted.
II. No sum shall be chargeable to tax in the hands
of the company even though the issue price is
greater than the FMV.
D Pvt Ltd 40,000 100 90 110 I. The provisions of Section 56(2)(viib) are
attracted in this case since the shares are
issued at premium and the issue price is also
more than FMV.
II. The excess of the issue price of the shares over
the FMV would be taxable u/h income from
other sources in the hands of D Pvt Ltd.
III. Taxable Amount = Rs 8,00,000 [40,000 shares x
Rs 20 (Rs 110 - Rs 90)].

PROBLEM:70 Where a firm or closely held company receives from any person any property being shares of closely
held company without consideration:
a) The whole of the fair market value of the shares shall be taxable
b) The whole of the FMV shall be taxable if it exceeds Rs 50,000 c) The whole of FMV shall be exempt

PROBLEM:71 Where a closely held company receives any consideration for issue of shares:
a) Such consideration in excess of face value of shares shall be treated as income under the head "income from
other sources"
b) Such consideration in excess of fair market value of shares shall be treated as income under the head
"income from other sources"

PROBLEM:72 X & Co. received a gift of 700 shares of RZX Pvt. Ltd. for nil consideration. The aggregate fair market
value of shares is Rs. 2, 30,000. What will be the taxable amount under the head Income from Other Sources.
(a) Nil (b) 50,000 (c) Rs. 2, 30,000 (d) Rs. 70,000

SALEEM QURAISHEE Mo: 9175664444 INSPIRE ACADEMY-8888881719


INCOME FROM OTHER SOURCES- Page 7.26

PROBLEM:73 X & Co. received a gift of 900 shares of RST Pvt. Ltd. at a consideration of Rs. 1,60,000. The aggregate
fair market value of shares is Rs. 2,30,000. What will be the taxable amount under the head IFOS-
(a) Nil (b) 50,000 (c) Rs. 70,000 (d) Rs. 1,60,000

PROBLEM:74 VSC Pvt. Ltd. issued 1,00,000 shares. The face value of shares is 200, Fair Market value Rs. 190 and
issue price Rs. 210. Calculate the amount to be considered as income in hands of VSC Pvt. Ltd. as per section
56(2)(viib).
(a) Nil (b) Rs. 20,00,000 (c) Rs. 10,00,000 (d) Rs. 5,00,000

PROBLEM:75 If in the above case, face value of shares is Rs. 200, Fair Market value Rs. 190 and issue price 198.
Calculate the amount to be considered as income in hands of VSC Pvt. Ltd. as per section 56(2)(viib
(a) Nil (b) Rs. 20,00,000 (c) Rs. 10,00,000 (d) Rs. 5,00,000

PROBLEM:76 USC Pvt. Ltd. issued 1,00,000 shares. The face value of shares is 200, Fair Market value Rs. 220 and
issue price Rs. 230. Calculate the amount to be considered as income in hands of USC Pvt. Ltd. as per section
56(2)(viib).
(a) Nil (b) Rs. 20,00,000 (c) Rs. 10,00,000 (d) Rs. 5,00,000

PROBLEM:77 XYZ Pvt. Ltd. issued shares. Mr. K, resident of India, purchased the shares at a consideration of Rs.
5,00,000. The fair market value of such shares was of Rs. 4,50,000. What will be the taxable amount under the
head Income from Other Sources?
(a) Nil (b) Rs.50,000 (c) Rs. 4,50,000 (d) 5,00,000

14. EXEMPTED INCOME-

SALEEM QURAISHEE Mo: 9175664444 INSPIRE ACADEMY-8888881719


INCOME FROM OTHER SOURCES- Page 7.27

15. SECTION SUMMARY


SECTIONS PARTICULAR

PRACTICAL PROBLEMS

PROBLEM:78From the following particulars of Pankaj for the previous year ended 31st March, 2020,
compute the income chargeable under the head “Income from other sources”:
Sl. Particulars `
No.
(i) Directors fee from a company 10,000
(ii) Interest on bank deposits 3,000
(iii) Income from undisclosed source 12,000
(iv) Winnings from lotteries (Net) 35,000
(v) Royalty on a book written by him 9,000
(vi) Lectures in seminars 5,000
(vii)Interest on loan given to a relative 7,000
(viii) Interest on debentures of a company (listed in a recognized 3,600
stock exchange) net of taxes
(ix) Interest on Post Office Savings Bank Account 500
(x) Interest on Government Securities 2,200
(xi) Interest on Monthly Income Scheme of Post Office 33,000
He paid ` 1,000 for typing the manuscript of book written by him.
ANSWER:78

SALEEM QURAISHEE Mo: 9175664444 INSPIRE ACADEMY-8888881719


INCOME FROM OTHER SOURCES- Page 7.28

PROBLEM:79 Compute income u/h other sources with the help of following information:
Dividend from Indian Companies Rs 10,500
Winning from lotteries net of TDS (TDS Rs 3,000) Rs 7,000
Additional information:
(i) The assessee has received Rs 24,000 being honorarium for valuing examination answer books.
(ii) The assessee has paid Rs 2,500 for purchase of lottery tickets.
ANSWER:79

PROBLEM:80 Compute income u/h other sources with the following information:
Dividend on shares of Indian Companies Rs 10,524
Income from Unit Trust of India Rs 7,600
Honorarium received from various institutions for valuation of answer sheets Rs 15,800
ANSWER:80

PROBLEM:81 Mr. Tejas is a businessmen dealing in cloth. On 1-4-2019 the position of his investments -
1) ` 24,000 7.5% Government of India Loan 2002.
2) ` 36,000 10% Debentures of Birla Jute Mills Ltd. (listed).
3) ` 10,000 12% Debentures of G Ltd.
4) Interest on National Savings Certificate (VIII issue) due ` 3,860.
5) ` 25,000 10% Rajasthan Government Loan.
6) ` 10,000 7% Capital Investment Bonds.
The due dates of interest of all the above securities are 1st May and 1st November. Calculate income from other
sources for the assessment year 2020-21.
ANSWER:81 ` 12,960

PROBLEM:82 Following are the particulars of income of Rohit. Compute her income under the head Income
from other sources for the assessment year 2020-21.
1. Dividend received from an Indian Company ` 11,000.
2. Winnings from lottery: amount received ` 70,000. Purchased lottery tickets of ` 500.
3. Winning from card games (gross) ` 25,000.
4. Interest received on Government securities held as investments ` 14,000.
5. Family pension received ` 24,600.
R incurred the following expenses:
1. Interest paid on amount borrowed for purchasing shares ` 7,000.
2. Collection charges in respect of interest on Government securities @ 2½% on amount collected.
ANSWER:82 ` 1, 55,050

PROBLEM:83 Mr. Sagar furnishes the following particulars of his income for the financial year ending 31-3-2020.
a) Dividends in September, 2019 from UTI ` 2,000.
b) Dividends received in July, 2019 from Assam Tea Co. Ltd., ` 3,600 (60% of income of the company is
agricultural income).
c) Amount received on 1-12-2019 in connection with winning from a horse race ` 14000
d) Amount received on 1-12-2019 in connection with winning from lottery ` 28,000. Cost of lottery tickets
purchased `2,000.
e) Director’s fees received in August, 2019 ` 20,000.
f) He has rented a residence of ` 250 per month. Half portion of this house was sub-let on a monthly rent of `
250 p.m. Compute his taxable income for the assessment year 2020-21
ANSWER:83

SALEEM QURAISHEE Mo: 9175664444 INSPIRE ACADEMY-8888881719


INCOME FROM OTHER SOURCES- Page 7.29

PROBLEM:84 Mr. Anil furnishes the following particulars of his incomes for the previous year 2019-20. Compute
income under the head "Income from Other Sources".

i) Dividend on equity shares 600

ii) Dividend on preference shares 3,200

iii) Income from letting on hire of building and machinery under one composite lease 17,000

iv) Interest on Bank Deposits 2,500

v) Directors sitting fees received 1,200

vi) Ground rent received 600

vii) Income from undisclosed sources 10,000

viii) Amount received on account of winnings from lotteries 14000

The following deductions are claimed by him

a) Collection charges of preference dividend 200

b) Allowable depreciation on Building and Machinery 4,000

c) Fire Insurance on Building and Machinery 100

ANSWER:84 `

PROBLEM:85 Check the taxability of the following gifts received by Mrs. Rashmi during the previous year 2019-
20 and compute the taxable income from gifts for Assessment Year 2020-21:
i) On the occasion of her marriage on 14.8.2019, she has received `90,000 as gift out of which`70,000 are from
relatives and balance from friends.
ii) On 12.9.2019 she has received gift of `18,000 from cousin of her mother.
iii) A cell phone worth ` 21,000 is gifted by her friend on 15.8.2019.
iv) She gets a cash gift of`25,000 from the elder brother of her husband's grandfather on 25.10.2019.
v) She has received a cash gift of `12,000 from her friend on 14.4.2019.
ANSWER:85

PROBLEM:86 State with brief reasoning whether the following receipts are chargeable to Income-Tax
or are exempt (if chargeable, the amount taxable is to be mentioned) for the AY-2020-21.
Nature of Receipts `
I. Interest on Enhanced Compensation received in 12-03-2020 for acquisition of 96,000
Urban Land, of which 40% relates to the earlier year.
II. Rent Received for letting out Agricultural Land for a movie shooting 72,000
ANSWER:86

SALEEM QURAISHEE Mo: 9175664444 INSPIRE ACADEMY-8888881719


INCOME FROM OTHER SOURCES- Page 7.30

PROBLEM:87 From the following particulars submitted by R, compute his income from other sources for
assessment year 2020-21.

Director's meeting fees received from Y Ltd. 3,000

Agricultural income from land situated in India 10,000

Agricultural income from Nepal 15,000

Interest received-

a) from bank on FDR (Net) 10,800

b) on post office saving account 600

c) on Government securities 1,200

d) on Public Provident Fund a/c 4,000

e) on National saving Certificate VIII issue 3,000

Dividend from A Limited declared on 25-8-2019 8,000

Lottery prize received after T.D.S. 28,000

Rent from sub-letting of a flat (rent paid to landlord for the flat is ` 6,000) 12,000

R spent ` 600 for realizing the rent. He had also spent ` 10,000 for the purchase of lottery tickets and received
the prize on one ticket.
ANSWER:87 `

PROBLEM:88 Compute income under the head other sources from the following particulars:
1. 8% Taxable Saving Bonds of Reserve Bank of India `3,00,000.
2. 6% Tax-free notified Railway's Bonds of ` 2, 60,000.
3. 12% Bonds of Industrial Development Bank of India of ` 3, 50,000.
4. 12% Debentures of ABC Ltd. listed on Ahmadabad Stock Exchange purchased at ` 96 (Face Value ` 100) `
1,92,000.
5. Interest received from debentures issued by X Ltd. listed on Stock Exchange – ` 59,400.
6. Interest received from debenture issued by a Ltd. Company not listed on Stock Exchange – ` 10,800.
7. Dividend received from A Ltd. on 27-6-2019 – ` 17,900.
8. Dividend declared by the company on 5-8-2019 on shares of B Ltd. @ 50% on 1,000 shares of ` 10 each, which
were purchased at ` 40 per share.
9. 10% Dividend on preference shares of ` 10 each amounting to ` 1,50,000 paid on 31-3-2020
ANSWER:88 `

PROBLEM:89 On 10.10.2019, Mr. Govind (a bank employee) received `5,00,000 towards interest on enhanced
compensation from State Government in respect of compulsory acquisition of his land effected during the
financial year 2015-16. Out of this interest,`1,50,000 relates to the financial year 2016-17; `1,65,000 to the
financial year 2017-18; and `1,85,000 to the financial year 2018-19. He incurred `50,000 by way of legal
expenses to receive the interest on such enhanced compensation. How much of interest on enhanced
compensation would be chargeable to tax for the assessment year 2020-21?
ANSWER:89

SALEEM QURAISHEE Mo: 9175664444 INSPIRE ACADEMY-8888881719


INCOME FROM OTHER SOURCES- Page 7.31

PROBLEM:90 The following details have been furnished by Mrs. Hemali pertaining to the year ended 31.3.2020
i) Cash gift of `51,000 received from her friend on the occasion of her "Shastiaptha Poorthi", a wedding function
celebrated on her husband completing 60 years of age. This was also her 25th wedding anniversary.
ii) On the above occasion, a diamond necklace worth `2 lacs was presented by her sister living in Dubai.
iii) When she celebrated her daughter's wedding on 21.2.2020, her friend assigned in Mrs. Hemali's favour, a fixed
deposit held by the said friend in a scheduled bank; the value of the fixed deposit and the accrued interest on
the said date was`51,000.Compute the income, if any, assessable as income from other sources.
ANSWER:90

PROBLEM:91 Smt. Laxmi reports the following transactions to you:


i) Received cash gifts on the occasion of her marriage on 18-7-2019 of `1,20,000. It includes gift of`20,000
received from non-relatives.
ii) On 1-8-2019, being her birthday, she received a gift by means of cheque from her mother's maternal uncle, the
amount being `40,000.
iii) On 1-12-2019 she acquired a vacant site from her friend for ` 1,05,000. The State stamp valuation authority
fixed the value of site at ` 1,80,000 for stamp duty purpose.
iv) She bought 100 equity shares of a listed company from another friend for`60,000. The value of share in the
stock exchange on the date of purchase was `1,15,000. Determine the amounts chargeable to tax in the hands
of Smt. Laxmi for the A. Y. 2020-21.
ANSWER:91

PROBLEM:92 Discuss the taxability or otherwise in the hands of the recipients, as per the provisions of
the Income-tax Act, 1961:
(i) Mr. A received an advance of ` 50,000 on 1-09-2019 against the sale of his house. However, due to
non-payment of installment in time, the contract has cancelled and the amount of ` 50,000 was
forfeited.
(ii) Mr. N, a member of his father’s HUF, transferred a house property to the HUF without consideration.
The value of the house is ` 10 lakhs as per the Registrar of stamp duty.
(iii) Mr. Kumar gifted a car to his sister’s son (Sunil) for achieving good marks in CA Final exam. The fair
market value of the car is ` 5,00,000.
ANSWER:92

PROBLEM:93 X ( 42 years) gives the following information for the PY 2018-19 Calculate IFOS.
1) On 1-12-2019, he gets gift of house A from his friend B (SDV is determined at ` 600,000.
2) On 03-12-2019, he gets gift of house B from C (who is father in law of his elder brother) SDV `40000, however
current market value is `65000.
3) On 7-12-2019, X purchases a second hand car for `70000 from D whose market value 300,000.
4) On 14-12-2019, X purchases a work of art for 5,00,000 from E (FMV is 530000)
5) On 20-12-2019 , X purchases jewellery for 700,000 from F (FMV is 725000.F is not a registered dealer.
6) On 21-12-2019 X purchases a painting for 400,000 from G (who is brother of Mrs X FMV 7,00,000)
7) On 24-12-2019, X purchases a commercial property for 72,00,000 from H ( FMV 900000)
8) On 25-12-2019, X gets a gift of 100 preference shares in A Ltd from J (on 25-12-2019 stock exchange are closed
,the lowest quotation on immediate preceding working day in NSE is 45)
9) On 25-1-2020, X gets a gift cheque of 1,00,000 from his friend L on his birthday.
10) On 28-12-2019, minor son of X gets the gift of 55000 from elder of Xs grandfather.
ANSWER:93

SALEEM QURAISHEE Mo: 9175664444 INSPIRE ACADEMY-8888881719


INCOME FROM OTHER SOURCES- Page 7.32

PROBLEM:94 Discuss the taxability or otherwise in the hands of the recipients, as per the provisions of
the Income-tax Act, 1961:
I. ABC Private Limited, a closely held company, issued 10,000 share at Rs. 130 per share. (The face
value of the share is Rs. 100 per share and the fair market value of the share is Rs. 120 per
share).
II. Mr. A received an advance of Rs. 50,000 on 01.09.2019 against the sale of his house. However,
due to non-payment of installment in time, the contract has cancelled and the amount of Rs.
50,000 was forfeited.
ANSWER:94

PROBLEM:95 Mr. Chezian is employed in a Company with Taxable Salary Income of `5,00,000. He
received a Cash Gift of `1,00,000 from Atma Chartiable Trust (registered u/s 12AA) in the month of
December for meeting his Medical Expenses. Is the Cash Gift so received from the Trust chargeable to
Tax in the hands of Mr. Chezian?
ANSWER:95

PROBLEM:96Discuss the taxability of the following receipts in the hands of Mr. Sanjay Kamboj under
the Income Tax Act, 1961 for A.Y. 2020-21:
I. `51,000 received from his sister living in US on 1.6.2019.
II. Received a car from his friend on payment of`2, 50,000, the FMV of which was `5, 50,000.
Provisions of taxability or Non-taxability must be discussed
ANSWER:96

PROBLEM:97Examine whether the following are chargeable to tax and the amount liable to tax :
I. A sum of ` 1,20,000 was received as gift from non-relatives by Raj on the occasion of the
marriage of his son Pravin.
II. Interest on enhanced compensation of ` 96,000 received on 12-3-2020 for acquisition of urban
land, of which 40% relates to the earlier year.
ANSWER:97

16. MULTIPLE CHOICE QUESTIONS-


PROBLEM:1 “Income from Other Sources” is chargeable to tax under ................
(a) Section 55(1) (b) Section 56(1) (c) Section 57(1) (d) Section 58(1)

PROBLEM:2 Which of the following conditions need to be satisfied, so that an income can be taxed
under the head “Income from Other Sources”?
(a) Assessee has an income (b) Income should not be tax exempt
(c) Income should not be taxable under any of the other heads (d) All of the above

PROBLEM:3 Aman earned certain interest income, which he wants to offer to tax under the head
“Income from Other Sources” on cash basis. However, he wants to claim deduction for certain
expenditure on accrual basis. He can claim deduction ................
(a) Only on accrual basis (b) Only on cash basis
(c) Accrual or cash basis, whichever is more beneficial to him (d) None of the above

SALEEM QURAISHEE Mo: 9175664444 INSPIRE ACADEMY-8888881719


INCOME FROM OTHER SOURCES- Page 7.33

PROBLEM:4 Raman earned certain interest income, which he wants to offer to tax under the head
“Income from Other Sources" on accrual basis. However, he wants to claim deduction for certain
expenditure on cash basis, since he has loss in the current year. He can claim deduction ................
(a) Only on accrual basis (b) Only on cash basis
(c) Accrual or cash basis, whichever is more beneficial to him (d) None of the above

PROBLEM:5 As per section 56(2), which of the following income are not chargeable to tax under the
head “Income from Other Sources”?
(a) Occasional one time gains derived from sale of capital asset
(b) Winning from lotteries, crossword puzzles and races including horse races
(c) Interest on securities which is not chargeable to tax under the head ‘PGBP’
(d) Sum received under Keyman insurance policy not taxable under Salaries or PGBP

PROBLEM:6 As per section 56(2), which of the following income are chargeable to tax under the head
“Income from Other Sources”?
(a) Winning from card games (b) Winning from gambling
(c) Winning from betting (d) All of the above

PROBLEM:7 F & S Private Limited, received certain contribution from its employees towards
provident fund. Such receipt is not taxable under the head “Profits and Gains of Business or
Profession” in the hands of HPL. Such contribution would be ................
(a) Taxable under the head “Income from Other Sources”
(b) Not taxable under the head “Income from Other Sources"
(c) Either (a) or (b) (d) Neither (a) nor (b)

PROBLEM:8 Which of the following incomes, if not chargeable to tax under the head “PGBP” in the
hands of the assessee, would be chargeable to Tax under the head “Income from Other Sources”?
(a) Plant, machinery or furniture let on hire (b) Income by way of interest on securities
(c) Both A and B (d) Neither A nor B

PROBLEM:9 A received Rs. 5,000 from a friend as a gift on friendship day. Such receipt is ................
as “Income from Other Sources”?
(a) Not taxable (b) Taxable (c) Taxable to the extent of 50% (d) Exempt

PROBLEM:10X receives Rs. 1,50,000 from Y on 01.09.2019 on account of sale of land (Stamp Duty
Value = Rs. 2,50,000). Compute the income chargeable to tax in the hands of Y under the head
“Income from Other Sources”, if X & Y are not relatives?
(a) Nil (b) Rs. 1,00,000 (c) Rs. 1,50,000 (d) 2,50,000

PROBLEM:11Interest received on compensation or enhanced compensation for compulsory acquisition


of land should be taxed in the year ................
(a) In which such compensation is declared (b) In which such compensation is disbursed
(c) In which such compensation is received
(d) Retrospectively from the year in which such compensation was actually due

SALEEM QURAISHEE Mo: 9175664444 INSPIRE ACADEMY-8888881719


INCOME FROM OTHER SOURCES- Page 7.34

PROBLEM:12Which of the following income is not chargeable to tax under the head “Income from
Other Sources”?
(a) Insurance Commission assuming not chargeable under the head "PGBP” (b) Family Pension
(c) Income Tax Refund (d) Both (b) and (c)

PROBLEM:13Aman is engaged in the business of manufacturing of garments. He is also working as an


agent of LIC and received insurance commission of Rs. 75,000. It is ................ under the head
“Income from Other Sources”.
(a) Chargeable (b) Not chargeable (c) May be chargeable (d) May not be chargeable

PROBLEM:14 A died in a car accident. His family received Pension from his employer. It is chargeable to
tax under Income from Other Sources.
(a) True (b) False

PROBLEM:15 Maninder is a non-employee director with XYZ Private Limited for which he received
Director fees. It is chargeable under Income from Other Sources.
(a) True (b) False

PROBLEM:16Whether the following income is chargeable to tax under the head "Income from Other
Sources”? Sumit has kept deposits of Rs. 2,00,000 with ABC Private Limited and received an
interest @ 10% on such deposits.
(a) True (b) False

PROBLEM:17A was married to B, and received a gift of Rs. Rs. 5,000 in cash from his brother at the
time of marriage. The money received is taxable to the extent of ................

PROBLEM:18A was married to S, and received a gift of Rs. 55,000 in cash from his friend at the time of
marriage. The money received is taxable to the extent of ................

PROBLEM:19As per section 56(2)(ix), amount received as advance and forfeited, under a negotiation
for transfer of capital asset will be taxable if ................
(a) Such capital asset is transferred and the advance for the same is forfeited
(b) Such capital asset is not transferred and the advance for the same is forfeited
(c) Such capital asset is transferred and the advance for the same is paid back
(d) Such capital asset is not transferred and the advance for the same is paid back

PROBLEM:20The deduction allowed on family pension received by the legal heir of the employee is
................
(a) 30 1/3% of such pension or Rs. 10,000 whichever is less
(b) 33 1/3% of such pension or Rs. 10,000 whichever is less
(c) 30 1/3% of such pension or Rs. 15,000 whichever is less
(d) 33 1/3% of such pension or Rs. 15,000 whichever is less

SALEEM QURAISHEE Mo: 9175664444 INSPIRE ACADEMY-8888881719


INCOME FROM OTHER SOURCES- Page 7.35

PROBLEM:21Mr. A borrowed Rs. 5,00,000 on which he needs to pay interest of Rs. 5,000 p.a. He
borrowed the amount for meeting his wealth-tax liability. Compute the amount ‘of eligible
deduction for Mr. A?
(a) Rs. 5,000 is allowed as deduction
(b) Rs. 5,000 is allowed as deduction only if the funds are borrowed from scheduled bank
(c) Rs. 37,500 (50% of Rs. Rs. 5,000) is allowed as deduction (d) No deduction is allowed

PROBLEM:22Which of the following is allowed as deduction while computing the income under the
head “Income from Other Sources”?
(a) Salaries paid outside India on which TDS is not deducted
(b) Any expenditure being made in connection with winning of lottery
(c) Interest paid outside India on which TDS has been deducted (d) Income-tax/wealth-tax paid

PROBLEM:23Compute the Income from Other Sources of RAJ for AY 2019-20 if he receives the
following gifts /amount apart from his monthly salary of Rs. 30,000.
I. Cash received Rs. 60,000 on his marriage anniversary from his friend
II. Gift of Rs. 90,000 from his mother’s brother
(a) Rs. 1,80,000 (b) Rs. 1,50,000 (c) Rs. 60,000 (d) Rs. 90,000

PROBLEM:24Who among the following is/are not included in the list of specified assessee u/s
115BBDA?
(a) Domestic Company (b) Foreign Company
(c) Trust or institution registered u/s 12A or section 12AA (d) Both (a) and (b)

PROBLEM:25Which of the following deduction is allowed from letting out of plant, machinery or
building?
(a) Amount paid as insurance premium against risk of damage and destruction
(b) Amount paid for current repairs to the machinery, plant or building
(c) Expenditure of capital nature, being made exclusively for the purpose of earning such income
(d) Both (a) and (b)

PROBLEM:26Composite rent is chargeable to tax under the head “Income from Other Sources” when
................
(a) It is not chargeable to tax under the head “PGBP”
(b) Letting out of property and asset is inseparable (c) Both A and B

PROBLEM:27 Reema received rent of Rs. 60,000 from letting a residential building (in Mumbai) along with plant
and machinery out building is inseparable from letting of plant and machinery). She expended Rs. 6,000 for
repairs and insurance of buildings and plant and machinery. The WDV of building was Rs. 2,00,000 as on 1-4-
2019 and the plant and was purchased on 9 May 2019 for ` 20,000. Amount taxable is:………………………….

SALEEM QURAISHEE Mo: 9175664444 INSPIRE ACADEMY-8888881719


INCOME FROM OTHER SOURCES- Page 7.36

PROBLEM:28 If interest on securities is received after deduction of tax at source then the amount to be included in
the total income is :
(a) Gross interest (b) Net interest (c) No amount to be included (d) None of these

PROBLEM:29 Sum received under a Keyman insurance policy including bonus shall be taxable under the head:
(a) Income from other sources (b) Income from house property
(c) Income from Business and profession (d) Any of the above

PROBLEM:30 Interest on compensation/enhanced compensation shall –


(a) Be taxable in the year of receipt (b) be taxable in the year of accrual
(c) Be taxable receipt/accrual, whichever is earlier, (d) not taxable.

PROBLEM:31 Adhoc deduction available in respect of income of interest on compensation/enhanced


compensation shall be:

PROBLEM:32 Assessee received interest on enhanced compensation of Rs. 50,000 as per court decree in
December 2019 by Mr. Yogesh. Out of die said amount a sum of 35,000 relates to preceding financial years.
The taxable income is:

PROBLEM:33 Incomes taxable under the head of Income from Other Source are :
(a) Interest on bank deposits and loans. (b) interest on foreign Government securities.
(c) Agricultural income received from outside India. (d) All of the above.

PROBLEM:34 Which of the following income is not taxable under the head income from other sources.
(a) Income from letting. (b) Income from sub letting.
(c) Director's fees. (d) Commission received by the director giving bank guarantee for the company.

PROBLEM:35 Royalty received from a publisher by Nina was of 42,700. She spent Rs. 2,700 on books, stationery,
typing, etc the amount of income chargeable to tax under head Income from other sources?

PROBLEM:36 Mr. Ram was earning income from sub-letting of motor car to his friend. Such income shall be
taxable under which head of income.
(a) Income from other sources (b) Income from house property
(c) Income from Business and profession (d) Any of the above

SALEEM QURAISHEE Mo: 9175664444 INSPIRE ACADEMY-8888881719


INCOME FROM OTHER SOURCES- Page 7.37

PROBLEM:37 J took a house on lease for 10 years and let it further to a tenant for his residence at a monthly rent
of Rs. 2,400. He incurred following expenses during year : Lease rent: Rs. 1000 p.m., Salary of Durban: Rs. 200
p.m. and Interest on loan taken to pay for acquisition of lease : Rs. 200 p.m.. Compute income chargeable
under head Income from other sources?

PROBLEM:38 An assessee earned income of interest on securities amounting to Rs. 65,000. He paid a reasonable
sum of Rs. 32,000 as commission to a banker for realizing such interest on behalf of the assessee. Such
amount of expenditure shall be
: (a) Allowed (b) Disallowed (c) Partly allowed and partly disallowed (d) Any of the above

PROBLEM:39 The deduction for family pension under section 57 can be determined as:
(a) One third of the family pension (b) Rs. 15,000 (c) Lower of (a) or (b) (d) Higher of (a) or (b)

PROBLEM:40 Family pension received by a widow of a member of the armed forces where the death of the
member has occurred in the course of the operational duties in the circumstances and subject of prescribed
conditions, is –
(a) Exempt upto Rs. 3,00,000 (b) Exempt upto Rs. 3,50,000 (c) Totally exempt under section 10(19)
(d) Totally chargeable to tax

PROBLEM:41 Family pension received by Mr. Ram from the Government of Madhya Pradesh was of Rs. 15,000.
Calculate the amount of income chargeable to tax under the head of income from other sources.

PROBLEM:42 Bahadur, a defense personnel, died in a war. His wife received the family pension of Rs. 7,500 per
month during the year 2018-19 . Calculate the amount of income chargeable to tax under the head of income
from other sources

PROBLEM:43 If in the above case bahadur did not die in a war then calculate the amount of income chargeable to
tax.
PROBLEM:44 Sunder died on 23rd July, 2004 while being in Central Government service. In terms of rules
governing his service, his widow Mrs. Sunder is paid a family pension of Rs. 10,000 per month and dearness
allowance of 40% thereof. Compute taxable income for the assessment year 2020-21.

PROBLEM:45 Amounts that are not deductible while computing Income from Other Sources:
(a) Personal expenses of the assessee; Wealth tax paid by assessee.
(b) AO disallowances under section 40A.
(c) Interest/Salaries payable outside India on which tax has not been paid or deducted.
(d) All of the above

SALEEM QURAISHEE Mo: 9175664444 INSPIRE ACADEMY-8888881719


INCOME FROM OTHER SOURCES- Page 7.38

PROBLEM:46 Which of the following amount is not deductible while computing income from other sources
(a) Any sum paid on account of income tax. (b) Any personal expenses of the assessee.
(c) Any sum paid on account of wealth tax. (d) (b) & (c) but not (a)

PROBLEM:47 Gagan received a royalty of Rs. 1,00,000 from a book of stories written by him. He claimed Rs.
12,000 as expenditure on stationery and typing. He let-out of his buildings along with plant, machinery and
furniture for Rs. 50,000 per month. He claimed the following expenses as deduction for this building -
Insurance : Rs. 10,000; repairs : Rs. 15,000; depreciation: Rs. 40,000. Interest credited to his recurring deposit
account and cumulative time deposit account in post office were Rs. 32,000 and Rs. 48,000 respectively.
Compute the income taxable.

PROBLEM:48 Compute income taxable under head income from other sources: Dividend from shares of Indian
company Rs. 3,000 Winnings from lotteries (net) 70,000 Rental Income of Plant and machinery Rs. 51,000.

PROBLEM:49 Compute income taxable under head income from other sources received by Mr. X : Cash gift
received from his brother on occasion of his marriage anniversary Rs. 75,000 Winnings from lotteries (net) Rs.
70,000 Forfeited advance money received on occasion of transfer of capital asset 151,000

PROBLEM:50 If no system of accounting is followed, interest on securities is taxable on:


(A) Due basis (B) receipts basis (C) due or receipt basis at the option of the assessee

PAST EXAMINATION MCQ-


PROBLEM:51Dec 2014: Which of the following is not included in taxable income ________
A. Income from smuggling activity
B. Casual income
C. Gifts of personal nature subject to a maximum of Rs. 50,000 received in cash
D. Income received in kind

PROBLEM:52Dec 2014: The amount deductible from family pension is up to _______

PROBLEM:53Dec 2014: Sameer received the following income during PY 2019-20 - Director’s fees Rs.
5,000, income from agricultural land in Pakistan Rs. 15,000 rent form let – out of land in Jaipur Rs.
20,000, interest on deposit with HDFC Bank Rs. 1,000 and dividend form Indian company Rs. 5,000.
His income from other sources is _______

PROBLEM:54Dec 2014: Akshat received a gift of Rs. 35,000 each on 22nd May, 2019 form his three
friends. The amount chargeable to tax in this case would be ______

SALEEM QURAISHEE Mo: 9175664444 INSPIRE ACADEMY-8888881719


INCOME FROM OTHER SOURCES- Page 7.39

PROBLEM:55June 2015: Which of the following income will be taxable as income from other sources?
A. Purchase of house form husband for inadequate consideration
B. Purchases of painting form registered dealer at invoice value less than fair market value
C. Cash gift form a non – resident friend on marriage anniversary
D. All of the above

PROBLEM:56June 2015: Hemant holds 100 shares of Gold Ltd, a domestic company. He has been paid
divided of Rs. 10,000 during the year 2019-20 . During distribution tax payable u/s 115-O will be ____

PROBLEM:57June 2015: John, engaged in fertilizer trade received rent by sub – letting a building. This will
be taxable under the head______

PROBLEM:58Dec 2015: A private limited company engaged in manufacturing activity had general reserve
of Rs. 20 Lakh. It granted a loan of Rs. 5 lakh to a director who held 13% shareholder cum voting rights
in the company. The said loan was re – paid by him before the end of the year. The amount of
deemed of dividend arising out of the above transaction is _____

PROBLEM:59Dec 2015 : Rajiv (aged 28 years) received cash gift of Rs. 2 lakh on the occasion of his
married. It includes gift form non- relative of Rs. 80,000. His income by way of lottery winnings is Rs. 3
lakh. His net income tax liability (ignoring TDS) would be _____

PROBLEM:60Dec 2015 : Ms. Sitara is in receipt of family pension of Rs. 15,000 p.m. during 2018-19 .
Income chargeable to tax for AY 2020-21 of Ms. Sitara is _______

SALEEM QURAISHEE Mo: 9175664444 INSPIRE ACADEMY-8888881719


CLUBBING OF INCOME- Page 8.1

1. WHY CLUBBING?
I. Income-tax is levied on a slab system on the total income of an individual.
II. There are progressive rates of tax and as the income goes up, the rates of tax also go up.
III. At present, the income-tax rates start from a minimum of 5 % and go up to 30%.
IV. There is a tendency amongst the tax-payers in higher tax brackets to divert a part of their income to
the hands of their relatives, in order to reduce the burden of tax.
V. It may, therefore, be observed that by diverting the income into different hands, he is able to make
a substantial reduction in the tax liability.
VI. In order to curb such practices of tax avoidance, provisions have been incorporated in the Income-
tax Act under which the income arising to certain persons is to be included in the income of another
person, for purposes of computation of his tax liability.
VII. These provisions are contained in sections 60 to 64 of the Income-tax Act

2. TRANSFER-

3. CONSIDERATION

SALEEM QURAISHEE Mo: 9175664444 INSPIRE ACADEMY-8888881719


CLUBBING OF INCOME- Page 8.2

4. TRANSFER OF INCOME WHERE THERE IS NO TRANSFER OF ASSET-SEC


60
I. The taxpayer owns an asset.
II. The ownership of asset is not transferred.
III. The income from asset is transferred to any person.
IV. The transfer may be revocable or not revocable.
V. The above transfer may be effected at any time
 If above conditions are satisfied, the income from the asset would be taxable in the hands of
transferor.

CASE-1 A owns Debentures worth Rs 1,000,000 of ABC Ltd., (annual) interest being Rs. 100,000. On April
1, 2018, he transfers interest income to B, his friend without transferring the ownership of these
debentures. In this particular case during 2019-20, interest of Rs. 100,000 is received by B; it will be
taxable in the hands of A as per Section 60.

CASE-2 Mr. A confers the right to receive rent in respect of his house property to his wife, Mrs. A, without
transferring the house itself to her. In this case, the rent received by Mrs. A will be clubbed with the
income of Mr. A.

PROBLEM:1 Mr. Vatsan has transferred, through a duly registered document, the income arising from a
godown to his son, without transferring the godown. In whose hands will the rental income from
godown be charged?
ANSWER:1 Section 60 expressly states that where there is transfer of income from an asset without
transfer of the asset itself, such income shall be included in the total income of the transferor. Hence,
the rental income derived from the godown shall be clubbed in the hands of Mr. Vatsan.

SALEEM QURAISHEE Mo: 9175664444 INSPIRE ACADEMY-8888881719


CLUBBING OF INCOME- Page 8.3

5. REVOCABLE TRANSFER OF ASSETS [SECTION 61]


I. Where there is a revocable transfer of an asset by a person to another person, any income
arising/derived from such assets shall be included in the total income of the transferor.
II. Transfer for sections 60, 61 and 62 includes any settlement, trust, covenant, agreement or
arrangement

WHAT IS REVOCABLE TRANSFER-?


I. If an asset is transferred under a trust and it is revocable during the lifetime of the beneficiary.
II. If an asset is transferred to a person and it is revocable during the life time of transferee.
III. If an asset is transferred before April 1 1961 and it is revocable within six years
IV. It contains any provision for the re-transfer, directly or indirectly of the whole or any part of the
income or assets to the transferor
V. it gives the transferor a right to re-assume power directly or indirectly over the whole or any part of
the income or assets during the life time of the beneficiary or the transferee as the case may be.

CONCLUSION:
I. There is an asset which is transferred under a revocable transfer.
II. Income from the aforesaid asset is taxable in the hands of transferor.
III. Such income is taxable as and when the power to revoke arises.
IV. The above rule is applicable even if power to revoke has not been exercised

CASE-3 X transfers a house property to a trust for the benefit of A and B. However, X has a right to revoke
the trust during the lifetime of A and/or B. It is a revocable transfer and income arising from the house
property is taxable in the hands of X

CASE-4 X transfers a house property to A. However, X has a right to revoke the transfer during the lifetime
of A. It is a revocable transfer and income arising from the house property is taxable in the hands of X.

CASE-5 X transfers an asset on March 31, 1961. It is revocable on or before June 6, 1963. It is a revocable
transfer. Income arising from the asset is taxable in the hands of X. Conversely, if X transfers an asset
before April 1, 1961 and it is revocable after 6 years (say, on April 10, 1967), it is not taken as a
revocable transfer.

CASE-6 X transfers an asset. Under the terms of transfer, on or after April 1, 1998, he has a right to utilize
the income of the asset for his benefit. However, he has not exercised this right as yet. On or after April
1, 1998, income of the asset would be taxable in the hands of X, even if he has not exercised the
aforesaid right.

CASE-7 X transfers an asset. Under the terms of transfer, he has a right to use the asset for the personal
benefits of his family members whenever he wants. Till date, he has not exercised this right. It is a
revocable transfer. The entire income from the asset would be taxable in the hands of X.

SALEEM QURAISHEE Mo: 9175664444 INSPIRE ACADEMY-8888881719


CLUBBING OF INCOME- Page 8.4

6. WHEN AN INDIVIDUAL IS ASSESSABLE IN RESPECT OF REMUNERATION


OF SPOUSE [SECTION 64(1)(II)]
A. Conditions:
I. The taxpayer is an Individual.
II. He/she has a substantial interest in the concern.
III. Spouse of the taxpayer is employed in the above mentioned concern.
IV. Spouse is employed in the concern without any technical or professional knowledge or experience.
 If aforesaid conditions are satisfied, then salary income of spouse will be taxable in the hands
of the taxpayer.

CASE-8 X has a substantial interest in A Ltd. and Mrs. X is employed by A Ltd. without any technical or
professional qualification to justify the remuneration. In this case, salary income of Mrs. X shall be taxable in the
hands of X.
B. No clubbing if remuneration is due to technical or professional qualifications:
I. on account of technical or professional qualifications possessed by the spouse, and
II. The income is solely attributable to the application of his/her technical or professional knowledge
or experience.

C. Where both husband and wife have substantial interest and both are getting remuneration from the
concern:
I. If the husband and wife both have substantial interest in the concern and both are in receipt of
remuneration from the, concern, then the remuneration of both shall be clubbed in the hands of that spouse
whose total income, before including such remuneration, is greater.
II. In this case, the clubbing will be done for the first time in the previous year in which the following three
conditions are satisfied :
 Both the husband and wife have a substantial interest in the concern.
 Both the husband and wife get remuneration from such a concern.
 The relationship of husband and wife subsists at the time of accrual of such income.
III. Where such income is once included in the hands of either spouse, any such income arising in any
succeeding year shall not be included in the total income of other spouse unless the Assessing Officer is
satisfied, after giving that spouse an opportunity of being heard, that it is necessary so to do.

D. MEANING OF SUBSTANTIAL INTEREST-

Substantial Interest

In the case of a In the case of a concern


company other than company

If an individual beneficially If an individual is entitled to


holds (individually or along 20% share in profit in the
with his relatives) 20% or more concern (individually or along
of equity shares in the with his relatives) at any time
company at any time during during the previous year.
the previous year.

SALEEM QURAISHEE Mo: 9175664444 INSPIRE ACADEMY-8888881719


CLUBBING OF INCOME- Page 8.5

PROBLEM:2 X holds 20 per cent equity share capital in Y Ltd. Mrs. X is employed by Y Ltd. (salary being
Rs. 1,40,000 per month) as general manager (finance). She does not have any professional qualification
to justify the remuneration. Ascertain in whose hands salary income is chargeable to tax. Does it make
any difference if Mrs. X was employed by Y Ltd. even prior to her marriage?
ANSWER:2

PROBLEM:3 Mr. P is employed as Public Relation Officer in a company where Mrs. P holds 21 per cent
equity shares. She has been holding the share before marriage with Mr. P., Mr. P gets a salary of Rs.
1,500 per month.
ANSWER:3 The whole salary of Rs. 18,000 will be included in the income of Mrs. P provided Mr. P has
no technical or professional qualification. It is immaterial that the remuneration so paid is genuine and
not excessive and that Mrs. P had substantial interest in the company even before her marriage.

PROBLEM:4 Mr. A holds shares carrying 25% voting power in X Ltd. Mrs. A is working as a
computer software programmer in X Ltd. at a salary of ` 30,000 p.m. She is, however, not
qualified for the job. The other income of Mr. A & Mrs. A is ` 7, 00,000 & ` 4, 00,000,
respectively. Compute the gross total income of Mr. A and Mrs. A for the A.Y.2020-21. Will
your answer be different if Mrs. A was qualified for the job?
ANSWER:4

PROBLEM:5 Mr. B holds shares carrying 30% voting power in Y Ltd. Mrs. B is working as accountant in Y
Ltd. getting income from salary (computed) of ` 3,44,000 without any qualification in accountancy.
Mr. B also receives ` 30,000 as interest on securities. Mrs. B owns a house property which she has let
out. Rent received from tenants is ` 6,000 p.m. Compute the gross total income of Mr. B and Mrs. B
for the A.Y. 2020-21.
ANSWER:5

PROBLEM:6 Ram and Mrs. Ram hold 20% and 30% equity shares in Anand Ltd. respectively. They are
employed in Anand Ltd. (monthly salary being ` 20,000 and ` 30,000 respectively) without any
technical or professional qualification. Other incomes of Ram and Mrs. Ram are ` 70,000 and `
1,00,000 respectively. Find out the net income of Ram and Mrs. Ram for the assessment year 2020-21.
ANSWER:6

SALEEM QURAISHEE Mo: 9175664444 INSPIRE ACADEMY-8888881719


CLUBBING OF INCOME- Page 8.6

PROBLEM:7 Mr. & Mrs. Om both are working in A Ltd. without possessing any technical or professional
qualification. From the following details compute their income for the A.Y. 2020-21:
Particulars Mr. Om Mrs. Om

Salary from A Ltd. ` 2,20,000 ` 70,000

Other income ` 50,000 ` 80,000

Share of holdings: Case 1 15% 6%

Case 2 3% 17%

Case 3 18% 1%
ANSWER:7

PROBLEM:8 Mr. & Mrs. X working in A Ltd. without possessing any qualification. From the following
details compute their income for the A.Y. 2020-21:

Particulars Mr. X Mrs. X

Share of holdings 15% 6%

Taxable salary from A Ltd. ` 1,20,000 ` 60,000

Case 1) Other income ` 50,000 ` 80,000

Case 2) Other income ` 90,000 ` 65,000

Case 3) Other income Nil Nil

ANSWER:8

PROBLEM:9 Income of X (age : 31 years) and Mrs. X (age : 30 years) for the previous year 2020-21 .
X Mrs. X
` `
Salary from B Ltd. 5,20,000 Nil
Business income 10,00,000 160,000
Bank interest (fixed deposit) 3,70,000 90,000
Total income 18,90,000 2,50,000
Tax
X is employed by B Ltd. (salary being Rs. 40,000 per month and one month's salary as bonus) without any
technical or professional or educational qualification. Mrs. X holds 20 per cent equity share capital in B
Ltd. from March 20, 2020. Find out the net income of X and Mrs. X for the assessment year 2020-21.
ANSWER:9

SALEEM QURAISHEE Mo: 9175664444 INSPIRE ACADEMY-8888881719


CLUBBING OF INCOME- Page 8.7

7. INCOME FROM ASSETS TRANSFERRED TO THE SPOUSE [SECTION


64(1)(IV)]:
When an individual is assessable in respect of income from assets transferred to spouse [Sec. 64(1) (iv)]
Condition 1 The taxpayer is an individual.
Condition 2 He/she has transferred an asset (other than a house property).
Condition 3 The asset is transferred to his/her spouse.
Condition 4 The transfer may be direct or indirect.
Condition 5 The asset is transferred otherwise than (a) for adequate consideration, or (b) in
connection with an agreement to live apart.
Condition 6 The asset may be held by the transferee-spouse in the same form or in a different
form.
CONSEQUENCES IF THE ABOVE CONDITIONS ARE SATISFIED -If the above conditions are satisfied, any
income from such asset shall be deemed to be the income of the taxpayer who has transferred the asset.
POINT TO BE NOTED-
A. HOW TO COMPUTE INCOME FROM TRANSFERRED ASSET – The income from asset transferred
must be calculated in the same way as it would be if the asset has not been transferred. Exemption,
deduction or tax incentives in respect of such income can be claimed by the transferor.

B. CAPITAL GAIN ON SALE OF TRANSFERRED ASSETS – If an individual transfers an asset without


consideration to his wife who sells it at a profit, capital gain arising to wife on sale of asset is
chargeable to tax in the hands of the transferor.

C. APPROPRIATION WHEN TRANSFERRED ASSET IS INVESTED IN A BUSINESS – An asset (maybe in cash


or kind) is transferred by husband to his wife (or vice versa) (directly or indirectly) without adequate
consideration. She invests the asset in a business. The amount of income that will be clubbed in the hands of
husband will be determined as follows-
Step one Find out total investment of transferee-spouse in the business on the first day of the previous
year.
Step two Find out the amount invested by the transferee-spouse out of the assets transferred to her
without adequate consideration by her husband on the first day of the previous year in the said
business.
Step three Find out the taxable income (exempt income is not included) of the transferee- spouse from the
business. If the transferee-spouse becomes a partner of a firm by investing the aforesaid asset
then only interest income from the firm is considered under Step three. Share of profit from the
firm is not considered under Step three as it is exempt under section 10(2A).
Step four The amount which shall be included in the hands of transferor is determined as follows—Step
three × Step two ÷ Step one.

SALEEM QURAISHEE Mo: 9175664444 INSPIRE ACADEMY-8888881719


CLUBBING OF INCOME- Page 8.8

D. WHEN TRANSFERRED ASSET IS INVESTED IN A FIRM – The aforesaid rule is also applicable in case
transferred asset is invested by the spouse to become partner in a firm. Share of profit is not
taxable in the hands of partners. Consequently, clubbing provisions are not attracted in respect of
share of profit from a firm where the transferred assets are invested by way of contribution
towards capital. Proportionate interest on capital will, however, be clubbed if transferred asset is
invested in a firm.

PROBLEM:10 X and Y form a partnership firm on April 1, 2019 (profit sharing ratio: 2: 3) by investing Rs.
10 lakh and Rs. 15 lakh respectively. The investment has been financed from the following sources—
X Y
Rs. Rs.
Gift from Mrs. X 6, 60, 000 -
Gift from Mrs. Y - 8, 00, 000
Past savings of X and Y 3, 40, 000 7, 00, 00
For the year ending March 31, 2020, share of profit from the firm is as follows—
X Y
Rs. Rs.
Interest on capital @ 12 per cent 1, 20, 000 1, 80, 000
Salary as working partner 24, 000 24, 000
Share of profit 1, 08, 000 1, 62, 000
Find out the income chargeable to tax in the hands of X and Mrs. X.
ANSWER:10

E. INCOME ARISING FROM ACCRETIONS TO TRANSFERRED ASSETS –


If an assessee gifts debentures of a company to the spouse and, subsequently, the company issues bonus
debentures to the spouse, interest on bonus debentures will not be includible in the hands of the assessee
under section 64(1) (iv) as there is no transfer of bonus debentures by the assessee to the spouse.

8. WHEN CLUBBING IS NOT APPLICABLE-


I. If assets are transferred before marriage.
II. If assets are transferred for adequate consideration.
III. If assets are transferred in connection with an agreement to live apart.
IV. If on date of accrual of income, transferee is not spouse of the transferor.
V. If property acquired out of pin money.

SALEEM QURAISHEE Mo: 9175664444 INSPIRE ACADEMY-8888881719


CLUBBING OF INCOME- Page 8.9

9. INCOME FROM ASSETS TRANSFERRED TO SON'S WIFE [SECTION


64(I)(VI)]:
Conditions:
I. The taxpayer is an individual
II. He/she transferred an asset.
III. The asset is transferred to his/her son’s wife.
IV. Transfer may be direct or indirect.
V. The asset is transferred otherwise than for adequate consideration.
VI. The asset may be held by the transferee in the same form or different form
If the above conditions are satisfied -----taxable in the hands of transferor.

CASE-9

10. INCOME FROM ASSETS TRANSFERRED TO ANY PERSON FOR THE


BENEFIT OF THE SPOUSE OF THE TRANSFEROR[SECTION64(I)(VII)]
Conditions:
I. The taxpayer is an individual
II. He/she transferred an asset.
III. The asset is transferred to a person or an association of persons.
IV. It is transferred for the immediate or defferd benefit of his/her spouse.
V. Transfer may be direct or indirect.
VI. The asset is transferred otherwise than for adequate consideration.
If the above conditions are satisfied -----taxable in the hands of transferor.

CASE-10

11. INCOME FROM ASSETS TRANSFERRED TO ANY PERSON FOR THE


BENEFIT OF SON'S WIFE[SECTION 64(1)(VIII)]:
Conditions:
I. The taxpayer is an individual
II. He/she transferred an asset.
III. The asset is transferred to a person or an association of persons.
IV. It is transferred for the immediate or defferd benefit of his/her sons wife
V. Transfer may be direct or indirect.
VI. The asset is transferred otherwise than for adequate consideration.
If the above conditions are satisfied ----- taxable in the hands of transferor.

SALEEM QURAISHEE Mo: 9175664444 INSPIRE ACADEMY-8888881719


CLUBBING OF INCOME- Page 8.10

PROBLEM:11 Mrs. Kasturi transferred her immovable property to ABC Co. Ltd. subject to a condition
that out of the rental income, a sum of ` 36,000 per annum shall be utilized for the benefit of her
son’s wife. Mrs. Kasturi claims that the amount of ` 36,000 (utilized by her son’s wife) should not
be included in her total income as she no longer owned the property. Examine with reasons
whether the contention of Mrs. Kasturi is valid in law.
ANSWER:11 The clubbing provisions under section 64(1)(viii) are attracted in case of transfer of any
asset, directly or indirectly, otherwise than for adequate consideration, to any person to the extent to
which the income from such asset is for the immediate or deferred benefit of son’s wife. Such income
shall be included in computing the total income of the transferor-individual. Therefore, income of `
36,000 meant for the benefit of daughter-in-law is chargeable to tax in the hands of transferor i.e., Mrs.
Kasturi in this case. The contention of Mrs. Kasturi is, hence, not valid in law

12. CLUBBING OF INCOME OF MINOR CHILD [SECTION 64(IA)]:


I. In computing the total income of an individual, there shall be included all such income as arises or
accrues to his minor child.
II. The income of a minor child is to be clubbed in the hands of either of his parents.
III. The income shall be clubbed in the hands of that parent whose total income (excluding the income
of the minor) is greater.
IV. If the marriage of his parents does not subsist, the income shall be clubbed in the hands of that
parent who maintains the minor child in the previous year.
V. Where any income is once included in the total income of either parent, any such income arising in
any succeeding year shall not be included in the total income of the other parent unless the
Assessing Officer is satisfied, after giving that parent an opportunity of being heard, that it is
necessary so to do.
VI. Where the income of a minor child has been included in the total income of a parent, such parent
shall be entitled to an exemption to the extent of such income or ` 1,500 whichever is less, in
respect of each minor child whose income, is so included.

When clubbing not attracted:


I. Any income of a minor child suffering, from any disability of the nature specified in section 80U like
physically disabled, totally blind, etc
II. Such income which accrues or arises to the minor child on account of any manual work done by him
III. Such income which accrues or arises to the minor child on account of any activity involving
application of his skills, talent or specialized knowledge and experience

Point to be noted:
I. Child in relation to an individual includes a step child and an adopted child of that individual.
II. Since 64(1A) does not exclude minor married daughter, even income arising to minor married
daughter would be clubbed. However where section 27 applies, clubbing of income from property
gifted by the parent does not arise.
III. If both the parents of the minor child are not alive then the income of minor child cannot be
clubbed and the guardian of the minor child shall file the return of such income on behalf of the
minor. It may be added that it will not be included in the income of guardian, if the guardian is not a
parent.
IV. Where the minor child attains majority during the previous year, then, the income till the date he
remained minor in that previous year shall be clubbed in the hands of the parent.

SALEEM QURAISHEE Mo: 9175664444 INSPIRE ACADEMY-8888881719


CLUBBING OF INCOME- Page 8.11

PROBLEM:12 Mr. Sharma has four children consisting 2 daughters and 2 sons. The annual income of 2
daughters were`9,000 and ` 4,500 and of sons were`6,200 and`4,300, respectively. The daughter who
has income of ` 4,500 was suffering from a disability specified under section 80U.Compute the amount
of income earned by minor children to be clubbed in hands of Mr. Sharma
ANSWER:12
PROBLEM:13 Mr. & Mrs. Mantri have income under the head “Profits & gains of business or profession”
of ` 3,00,000 and ` 4,00,000 respectively. They have 7 children. From the following details compute
taxable income of Mr. and Mrs. Mantri for the A.Y. 2020-21:

1) 1st child (aged 26 years) is a chartered accountant. His annual income from profession is `
4,00,000. His income from house property for the P.Y. 2019-20 is ` 30,000. He has a son (4 years
old) who has earned interest on fixed deposit of ` 5,000.

2) 2nd child (aged 17 years being a married daughter) who is a stage singer, earned income of `
1,00,000 during the P.Y. 2019-20. She earned interest on fixed deposit ` 8,000. Such fixed deposit
has been made out of such singing income.

3) 3rd child (aged 16 years) is suffering from disability specified u/s 80U (to the extent 55%) blind. He
has received interest income of ` 40,000 for loan given to a private firm. He is dependent on Mrs.
Mantri.

4) 4th child (aged 14 years) has earned income of ` 45,000 during the P.Y.2019-20 out of his physical
and mental effort. Expenditure incurred to earn such income is ` 15,000. His loss from house
property is ` 30,000.

5) 5th child (aged 12 years) is a partner in a partnership firm from which he earned interest income
(taxable) of 40,000 and share of profit of ` 35,000. Other two partner of the firm are Mr. & Mrs.
Mantri.

6) 6th child (aged 9 years) has 1,000 debentures of ` 100 each of a public sector company acquired
through will of his Grandfather. Interest income on such debenture is ` 10,000. Expenditure
incurred to collect such interest is 200. Such debenture was sold and long-term capital gain earned
` 25,000.

7) 7th child (aged 7 years) has earned dividend from domestic company ` 10,000 and interest on fixed
deposit ` 500.
ANSWER:13

PROBLEM:14 Mr. Mittal has four minor children consisting of three daughters and one son. The annual
income of all the children for AY 2020-21 were as follows:
Particulars Amount (Rs)
First daughter (including scholarship received Rs 5,000) 10,000
Second daughter 8,500
Third daughter (suffering from disability specified u/s 80U) 4,500
Son 40,000
Mr. Mittal gifted Rs 2,00,000 to his minor son who invested the same in business and derived income of
Rs 20,000 which is included above. Find the amount to be clubbed in the hands of Mr. Mittal.
ANSWER:14
SALEEM QURAISHEE Mo: 9175664444 INSPIRE ACADEMY-8888881719
CLUBBING OF INCOME- Page 8.12

PROBLEM:15 In whose total income, the income of a minor child is included -


(a) Father (b) Mother (c) Father and mother both (d) Parent whose total income is greater.

PROBLEM:16 When the income of an individual includes Rs. 20,000 as the income of his minor child in terms of
section 64(1A), taxable income in this respect will be –
(a) Nil (b) Rs. 20,000 (c) Rs. 18,500 (d) None of the above.

PROBLEM:17 Exemption of _____ will be available u/s 10(32) to the parent in respect of minor's income clubbed
with the parent
(a) Rs.1,500 (b) Rs. 5,000 (c) Rs. 15,000 (d) Rs. 1,00,000

PROBLEM:18 In case of clubbing of income of two minor children exemption of ____ will be available.
(a) Rs. 1,500 (b) Rs. 3,000 (c) Rs. 1,000 (d) None of these

PROBLEM:19 _____ Income of minor child has to be included in the income of the assessee for determining rate
of income tax applicable to income of the assessee.
(a) Agricultural (b) Gross total (c) Business (d) None of these.

PROBLEM:20 Income of a minor child shall be included in the income of –


(a) That parent whose total income (before this clubbing) is greater.
(b) Minor child (c) That parent whose total income (before this clubbing) is lower.
(d) That parent whose total income (after this is greater)

PROBLEM:21 Income of a minor child on account of _________ shall be taxable in his hands
(a) Any manual work done by him.
(b) Any activity involving application of his skill, talent or specialized knowledge and experience.
(c) Either (a) or (b) (d) None of these

PROBLEM:22 Minor child includes –


(a) Stepchild (b) Adopted child (c) Both (a) & (b) (d) None of these

PROBLEM:23 Mr. Y, who is a physically handicapped minor (suffering from a disability of the nature specified in
section 80 U), earns bank interest of Rs. 50,000 and Rs. 60,000 from making bags manually by himself. The total
income of Mr. Y shall be computed in his hands separately.
(a) Rs. 50,000 (b) Rs. 60,000 (c) Nil (d) Rs. 1,10,000

SALEEM QURAISHEE Mo: 9175664444 INSPIRE ACADEMY-8888881719


CLUBBING OF INCOME- Page 8.13

PROBLEM:24 The following income that accrue to a minor child will not be included in the total income of his
(a) Income earned from fixed deposits transferred by his grandfather.
(b) Income earned from house property transferred by his father
(c) Income earned from agricultural land competition
(d) Income from participation in dance transferred by his mother

PROBLEM:25 Mr. Sharma has four children consisting 2 daughters and 2 sons. The annual income of 2
daughters were`9,000 and ` 4,500 and of sons were`6,200 and`4,300, respectively. The daughter who
has income of ` 4,500 was suffering from a disability specified under section 80U.Compute the amount
of income earned by minor children to be clubbed in hands of Mr. Sharma
ANSWER:25

13. INCOME FROM SELF-ACQUIRED PROPERTY CONVERTED TO JOINT


FAMILY PROPERTY [SECTION 64(2)]
Where an individual, who is a member of the Hindu Undivided Family,:
I. converts, his separate property as the property of the HUF, or
II. throws the property into the common stock of the family, or
III. otherwise transfers his individual property to the family,
Otherwise than for adequate consideration, then the income from such property shall continue to be
included in the total income of the individual.

14. POINTS TO BE NOTED-


I. Income is to be clubbed but income on income is not to be clubbed.
II. Income includes loss.
III. Under which head of income will the clubbed income be assessed
 Such an income will first be computed in the hands of the recipient as if it was his income and
such recipient will compute this income under the relevant head after claiming exemptions/
allowances/ deductions permissible under the relevant head in which it falls.
 Such income computed, under the relevant head, will be included in the total income of the
individual under the same head of income.

IV. Recovery of tax-Sec 65


 The assessing officer has the power for the recovery of tax from the person to whom income
actually accrued if the AO so desires

SALEEM QURAISHEE Mo: 9175664444 INSPIRE ACADEMY-8888881719


CLUBBING OF INCOME- Page 8.14

15. SECTION SUMMARY


SECTION PARTICULAR

PRACTICAL PROBLEMS
PROBLEM:26 Mr. Sharma invests Rs 10 lakh in a fixed deposit (FD) at a bank, in his wife’s name. Interest of
Rs. 1 lakhs arises on this income. Mrs. Sharma invests the interest on periodic basis and interest for an
amount of Rs. 5,000 arises on the interest deposited by her in bank. Analyze the clubbing provisions
and find out the taxability of interest accrued.
ANSWER:26 Rs. 1 lakhs in the Now Interest income on FD will be clubbed with his (Mr. Sharma) income.
Interest of Rs. 5,000 aroused out of Investment made by Mrs. Sharma will be taxed as her own income.

PROBLEM:27 Red holds 40% of shares in a Company. Mrs. Red (a CS) is employed in the company as a
Company Secretary and is getting salary of ` 15,000 per month. Compute total income and tax
payable by Red and Mrs. Red for the Assessment Year 2020-21 assuming other income of Red is
`2,00,000 from a business and dividend income from company is ` 3,00,000.
ANSWER:27

PROBLEM:28 Mr. Amit is beneficially holding 21% equity shares of Essem Minerals Pvt. Ltd. Mrs. Amit is
employed as Manager (in accounts department) in Essem Minerals Pvt. Ltd. at a monthly salary of
`84,000. Mrs. Amit is not having any knowledge, experience or qualification in the field of accountancy.
Will the remuneration (i.e., salary) received by Mrs. Amit be clubbed with the income of Mr. Amit?

ANSWER:28 In this situation, Mr. Amit is having substantial interest in Essem Minerals Pvt. Ltd. and
remuneration of Mrs. Amit is not justifiable (i.e., she is employed without any technical or professional
knowledge or experience) and, hence, salary received by Mrs. Amit from Essem Minerals Pvt. Ltd. will
be clubbed with the income of Mr. Amit and will be taxed in the hands of Mr. Amit.

PROBLEM:29 Mr. Kapoor gifted ` 8, 40,000 to his wife. The said amount is invested by his wife in
debenture of a company. Will the income from the debenture purchased by Mrs. Kapoor from gifted
money be clubbed with the income of Mr. Kapoor?

SALEEM QURAISHEE Mo: 9175664444 INSPIRE ACADEMY-8888881719


CLUBBING OF INCOME- Page 8.15

ANSWER:29 `8, 40,000 is transferred to spouse. Fund is transferred via gift (i.e., without adequate
consideration) and, hence, the provisions of section 64(1)(iv)will be attracted. The provisions of
clubbing will apply even if the form of asset is changed by the transferee-spouse. In this case asset
transferred is money and, subsequently, the form of asset is changed to debentures, hence, income
from debentures acquired from money gifted by her husband will be clubbed with the income of her
husband. Thus, interest on debenture received by Mrs. Kapoor will be clubbed with the income of Mr.
Kapoor.

PROBLEM:30 Mrs. X received the following amounts during 2019-20


`
Gross salary 570000
Family pension received by wife (` 3250*12) 39000
Accumulated balance in PF of her husband after his death 1, 00,000
Gratuity received after the death of husband 100,000
Calculate taxable income of Mrs. X and tax liability for the assessment year 2020-21.
ANSWER:30

PROBLEM:31 State in whose income, the following incomes will be included:


i) M transferred ` 50,000 to his daughter-in-law in 2017 without any consideration. She received ` 12,000 as
interest on this amount during the previous year.
ii) R transferred his self-acquired property to the HUF, of which he is a member. During the previous year 2018-19
the HUF earned an income of ` 42,000 from this property.
iii) R transferred his self-acquired property to the HUF of which he is a member. The HUF earns an income of `
42,000 per annum. During the previous year 2019-20 the HUF is partitioned and the property is divided as
under:
R 1/5th share
R's minor son 1/5th share
R's major son 1/5th share
Mrs. R 1/5th share
R's brother 1/5th share

iv) R transferred debentures worth ` 5,00,000 to his wife on 1-4-2017. The debentures carry an interest of 12%
per annum. Mrs. R accumulates the interest of ` 60,000 per annum, which she receives on the debentures.
The accumulated amount of interest of ` 1, 80,000 is invested in a fixed deposit with a bank and Mrs. R receives
interest amounting to ` 15,000 on this FDR.
ANSWER:31

PROBLEM:32 Decide about the person in whose hands the following incomes shall be taxable:
i) R transfers 1000 debentures of ` 100 each carrying 12% interest to S on the condition that he will have a right
to receive 5% interest till his life-time.

ii) Master Pranay (Age 14 Years) received following incomes during 2019-20.
`
(a) Interest on Bank deposits 11,000
(b) Interest on Debentures 7,000
(c) Dividend on shares of a company 11,000
(d) Income from a singing concert held by him 60,000
(e) His father’s total income 50,000
(f) His mothers total income 8, 00,000

iii) R transfers a shop (monthly rent ` 7,000) to his relative S on the condition that shop will revert back to R on the
death of S.
ANSWER:32

SALEEM QURAISHEE Mo: 9175664444 INSPIRE ACADEMY-8888881719


CLUBBING OF INCOME- Page 8.16

PROBLEM:33 Compute the gross total income of Mr. & Mrs. A from the following information:
No. Particulars `

(a) Salary income (computed) of Mrs. A 2,30,000

(b) Income from profession of Mr. A 3,90,000

(c) Income of minor son B from company deposit 15,000

(d) Income of minor daughter C from special talent 32,000

(e) Interest from bank received by C on deposit made out of her special talent 3,000

(f) Gift received by C on 30.09.2019 from friend of Mrs. A 2,500


Brief working is sufficient. Detailed computation under various heads of income is NOT required.
ANSWER:33 GTI ` 4,05,000
PROBLEM:34 Mr. Dhaval and his wife Mrs. Hetal furnish the following information:
No. Particulars `

i) Salary income (computed) of Mrs. Hetal 4,60,000

ii) Income of minor son 'B' who suffers from disability specified in Section 80U 1,08,000

iii) Income of minor daughter 'C from singing 86,000

iv) Income from profession of Mr. Dhaval 7,50,000

v) Cash gift received by 'C on 2.10.2019 from friend of Mrs. Hetal on winning of singing 48,000
competition

vi) Income of minor married daughter 'A' from company deposit 30,000
Compute the total income of Mr. Dhaval and Mrs. Hetal for the Assessment Year 2020-21.
ANSWER:34 Mr Dhaval--` 7,78,500 ` 4,60,000
Mrs Hetal -`

PROBLEM:35 Mr. Dhaval has an income from salary of ` 3,50,000 and his minor children's income are as under:
Particulars `

Minor daughter has earned the following income:

From a TV show 50,000

From interest on FD with a bank (deposited by Mr. Dhaval from his income) 5,000

Minor son has earned the following income:

From the sale of a own painting 10,000

From interest on FD with a bank (deposited by Mr. Dhaval from his income) 1,000
Computation of Gross Total Income of Mr. Dhaval
ANSWER:35 ` 3,53,500

SALEEM QURAISHEE Mo: 9175664444 INSPIRE ACADEMY-8888881719


CLUBBING OF INCOME- Page 8.17

PROBLEM:36 During the previous year 2019-20, the following transactions occurred in respect of Mr. A.
(a) Mr. A had a fixed deposit of ` 5,00,000 in Bank of India. He instructed the bank to credit the
interest on the deposit @ 9% from 1-4-2019 to 31-3-2020 to the savings bank account of Mr. B, son
of his brother, to help him in his education.
(b) Mr. A holds 75% share in a partnership firm. Mrs. A received a commission of ` 25,000 from the
firm for promoting the sales of the firm. Mrs. A possesses no technical or professional qualification.
(c) Mr. A gifted a flat to Mrs. A on April 1, 2019. During the previous year 2019-20, Mrs. A’s “Income
from house property” (computed) was ` 52,000.
(d) Mr. A gifted ` 2,00,000 to his minor son who invested the same in a business and he derived
income of ` 20,000 from the investment.
(e) Mr. A’s minor son derived an income of ` 20,000 through a business activity involving application of
his skill and talent.
(f) During the year, Mr. A got a monthly pension of ` 10,000. He had no other income.
(g) Mrs. A received salary of ` 20,000 per month from a part time job.
Examine the tax implications of each transaction and compute the total income of Mr. A, Mrs. A and
their minor child.
ANSWER:36

SALEEM QURAISHEE Mo: 9175664444 INSPIRE ACADEMY-8888881719


CLUBBING OF INCOME- Page 8.18

16. MCQ
PROBLEM:1 Generally, clubbing of income provision deals with inclusion of ................
(a) Income of other persons in lower tax bracket in income of asses- see in higher tax brackets total
income
(b) Income of other person in lower tax bracket in the individual’s in higher tax brackets total
income (c) Both (a) and (b) (d) Neither (a) nor (b)

PROBLEM:2 A transferred his rent income from his owned house to B, without transferring the house.
Such rental income has to be included in the total income of …………………
(a) A (b) B (c) Both (a) and (b) (d) Neither (a) nor (b)

PROBLEM:3 In case of revocable transfer of asset by one person to another person, income derived
from such asset shall be included in the total income of ................
(a) Transferor (b) Transferee (c) Both the parties equally (d) None of the parties

PROBLEM:4 Which of the following is included under “Transfer” for the purpose of sections 60, 61 and
62?
(a) Settlement (b) Trust (c) Covenant (d) All of the above

PROBLEM:5 A transfer of an asset is said to be revocable under which of the following cases?
(a) If it contains provision for re-transfer of any part of the income from asset to the transferor
(b) If transferor has a right to re-assume power over any part of the assets during the life time of
beneficiary (c) Both (a) and (b) (d) None of the above

PROBLEM:6 As per section 62, clubbing provisions are applicable to which of the following transfers?
(a) Transfer under trust not revocable during the lifetime of beneficiary or transferee
(b) Transfer made before April 1, 1961, which is not revocable for a period exceeding 6 years
(c) Transfer made after April 1, 1961, not revocable for a period exceeding 6 years
(d) None of the above

PROBLEM:7 Ram owns a house property in Delhi. He transfers it to his wife Sita on the condition that
such transfer is irrevocable during the life time of Sita. Ram transfers the property on an agreement
that he will be paid 30% of the rent received by Sita on such property. The total income from such
house property will be included in the income of ................
(a) Ram (b) Sita (c) Ram & Sita proportionately
(d) Sita only up to the extent of income received by her from such property and balance in hands of
Ram

PROBLEM:8 Clubbing provisions will not be applicable where ................


(a) Remuneration arising to spouse on account of their technical or professional qualifications
(b) Remuneration arising to the spouse solely attributable to the application of professional
knowledge or experience
(c) Both (a) and (b) (d) Neither (a) nor (b)

SALEEM QURAISHEE Mo: 9175664444 INSPIRE ACADEMY-8888881719


CLUBBING OF INCOME- Page 8.19

PROBLEM:9 Where both husband and wife have substantial interest in a concern and both are getting
remuneration, then such remuneration will be included in the income of ................
(a) Husband (b) Wife
(c) Either husband or wife, whose-total income is higher before including such remuneration
(d) Either husband or wife, whose- total income is higher after including such remuneration

PROBLEM:10 Mrs. A holds 35% of equity shares of XYZ Ltd. Mr. A is also working in XYZ Ltd. as Finance
Head. He does not possess any qualification to hold the position in XYZ Ltd. The salary received by
Mr. A will be included in the total income of ................
(a) Mr. A (b) Mrs. A
(c) Either Mr. A or Mrs. A depending whose income is higher (d) None of the above

PROBLEM:11 Abhinav holds 80% of equity shares of A&A Ltd. Arti, his wife, who is a qualified Chartered
Accountant is also working in A&A Ltd. as Finance Head and draws fair remuneration based on her
qualification. The salary received by Arti will be included in the total income of ................
(a) Arti (b) Abhinav
(c) Either Abhinav or Arti depending whose income is higher (d) None of the above

PROBLEM:12 Mr. Y holds 18% of equity shares of SWIGY Ltd. Mrs. Y is also working in SWIGY Ltd. as
Finance Head. She does not possess any qualification to hold the position in SWIGY Ltd. The salary
received by Mrs. Y will be included in the total income of ................ , assuming neither Mrs. Y nor
any of her relatives are entitled to receive 20% or more profits in SWIGY Ltd.
(a) Mr. Y (b) Neither Mr. Y nor Mrs. Y
(c) Mrs. Y (d) Either Mr. Y or Mrs. Y, whose income is higher

PROBLEM:13 Ganesh holds 25% equity shares in Agni Pvt. Ltd. and his wife Geeta holds 21% equity
shares in it. Both are in receipt of salary from Agni Pvt. Ltd. and are not professionally qualified for
the job. The salary received by Ganesh will be included in the income of ................
(a) Ganesh (b) Geeta
(c) Neither Ganesh nor Geeta
(d) Either Ganesh or Geeta, whose total income is higher before including such income

PROBLEM:14 Where an income in form of remuneration is once included in the hands of either spouse
where both have substantial interest in a concern, any income arising in succeeding year shall
................
(a) Never be included in the income of other spouse
(b) Always be included in the income of other spouse
(c) Be included in the income of other spouse, if Assessing Officer is satisfied that it is necessary to
do so (d) None of the above

SALEEM QURAISHEE Mo: 9175664444 INSPIRE ACADEMY-8888881719


CLUBBING OF INCOME- Page 8.20

PROBLEM:15 As per the clubbing provisions, who among the following are not included in the definition
of “relative” of an individual?
(a) Spouse of individual (b) Brother or sister of individual
(c) Lineal ascendant or descendant of individual
(d) Lineal ascendant or descendant of spouse of individual

PROBLEM:16 As per the clubbing provisions, who among the following are included in the definition of
relative of an individual?
(a) Brother or sister of spouse of individual (b) Spouse of brother or sister of individual
(c) Grandparents of individual (d) Parents of spouse of individual

PROBLEM:17 There will be no clubbing of in come, arising from transfer of assets to a spouse, if
................
(a) The transfer is for adequate consideration
(b) The transfer is under an agreement to live apart
(c) The relationship of husband and wife does not exist, either at the time of transfer of such asset,
or at the time of accrual of such income (d) All of the above

PROBLEM:18 Avinash and his wife Anu, both are working in ZENITH Pvt. Ltd. with a salary of Rs.
9,60,000 p.a. and Rs. Rs. 750,000 p.a. respectively and total income of Rs. 10,00,000 and Rs.
9,50,000 before including such salary. They do not possess any professional qualification. Also,
Avinash holds 25% equity shares of the company and Anu holds 30% equity shares of the company.
The salary received by Anu will be included in the total income of ................
(a) Avinash (b) Anu (c) Neither Avinash nor Anu
(d) Both Avinash & Anu in proportion to the shares held by them

PROBLEM:19 Avinash and his wife Anu, both are working in Zenith Pvt. Ltd. with a salary of Rs.
9,60,000 p.a. and Rs. Rs. 7,50,000 p.a. respectively. They do not possess any professional
qualification. Also, Avinash holds 25% shares of the company and Anu holds 30% shares of the
company. Compute the total income of Avinash, assuming he does not has any other income during
the year, and his total income before including salary is higher than Anu?
(a) Rs. 9,60,000 p.a. (b) Rs. 17,10,000 p.a.
(c) Rs. 4,27,500 p.a. (d) None of the above

PROBLEM:20 As per section 64(1 )(iv), which of the following income is/are not included in the total
income of the transferor?
(a) Transfer income from asset to spouse for inadequate consideration
(b) Income from asset transferred to spouse under an agreement to stay together
(c) Income from asset transferred to spouse for adequate consideration
(d) All of the above

SALEEM QURAISHEE Mo: 9175664444 INSPIRE ACADEMY-8888881719


CLUBBING OF INCOME- Page 8.21

PROBLEM:21 Shyam transfers house property to his wife Hima for inadequate consideration. Hima
leases out the house property to ABC & Co. at a lease rent of Rs. 2,50,000 p.a. Compute the income
from house property which is to be clubbed in the total income of Shyam?
(a) Rs. 2,50,000 (b) Proportionate income arising from the leasing out of such property
(c) NIL (d) None of the above

PROBLEM:22 The income earned by transfer of an asset by Sunil to Alka, wife of his son Amar, without
adequate consideration is ................
(a) Taxable in the hands of Sunil (b) Taxable in the hands of Alka
(c) Taxable in the hands of Amar (d) Not taxable

PROBLEM:23 Income earned by Saarthak, who turns 18 years on 1.04.2018, from a work involving
application of his painting skills is
(a) Taxable in the hands of Samar, his father (b) Exempt as income earned by minor is not taxable
(c) Taxable in the hands of Saarthak (d) Either (a) or (b)

PROBLEM:24 Aadi, a 12-year-old boy, earns Rs. 20,000 as interest from a fixed deposit which has been
invested in his name by his father. The amount received by Aadi will be ................
(a) Assessed in the hands of Aadi (b) Assessed in the hands of Aadi’s father
(c) Assessed in the hands of Aadi’s mother
(d) Assessed in the hands of either of the parents of Aadi, whose income before including Aadi’s
interest income is higher

PROBLEM:25 As per section 64(1 A), how much exemption is allowed to the parents in case of clubbing
of minor’s income in their hands?
(a) Rs. 1,500 or minor’s income whichever is less, in respect of each minor child
(b) Rs. 500 or minor’s income, whichever is less in respect of each minor child
(c) Rs. 2,500 or minor’s income whichever is less in respect of each minor child
(d) Full income of minor child is exempt upto two children

PROBLEM:26 Interest on deposit, received by minor married daughter is ................


(a) Taxable in the hands of the father of minor (b) Taxable in the hands of minor
(c) Taxable in the hands of either of the parents of minor whose income before including minor’s
income is higher (d) Taxable in the hands of husband of such minor

PROBLEM:27 Akash, a 10-year-old boy who is a singer by profession invests Rs. 5,00,000 (which he
earns through his singing concerts) in a fixed deposit and he earns interest on such deposit. The
interest income received by Akash will be ................
(a) Included in the total income of either of the parents of Akash whose income before including
Akash’s income is higher
(b) Included in the total income of Akash (c) Included in the total income of Akash’s father
(d) Exempt from tax

SALEEM QURAISHEE Mo: 9175664444 INSPIRE ACADEMY-8888881719


CLUBBING OF INCOME- Page 8.22

PROBLEM:28 Mr. B converts his immovable property into property of HUF, in which he is a member.
The income from such property after the transfer will be ................
(a) Included in the total income of Mr. B (b) Included in the total income of HUF
(c) Included in the total income of Karta of HUF
(d) Included in the total income of all the members of HUF

PROBLEM:29 Mr. C converts his immovable property into the immovable property of HUF in which he is
a member. After such conversion the property has been partitioned. The share received by Mrs. C
will be ................
(a) Exempt from tax (b) Included in the income of Mrs. C
(c) Included in the income of Mr. C (d) Included in the income of HUF

PROBLEM:30 Mr. X has gifted the house property to his minor married daughter. The income arising
from such house property shall be taxable in the hands of ................
(a) Mr. X (b) Minor married daughter
(c) Either of the parents of minor whose income before including minor's income is higher
(d) Husband of such minor

PROBLEM:31 Master Pradeep, a minor child has earned Rs. 4,00,000 during the AY 2020-21. He does
not have any parents who’s alive. The income of such minor shall be taxable in the hands of
(a) Guardian (b) Minor, on whose behalf Guardian shall do necessary filings
(c) Proportionately in the hands of his Guardian and Minor (d) Not taxable

PROBLEM:32 Master Sumit, a minor child earned income of Rs. 10,00,000 till June 30, 2019. On July 1,
2019 he attains majority and earned income of Rs. 40,00,000 during the period July 1, 2019 to
March 31, 2020. The income of minor shall be taxable in the hands of ................
(a) The entire income of Rs. 50,00,000 shall be taxable in his hands
(b) The entire income of Rs. 50,00,000 shall be taxable in the hands of his parents
(c) Rs. 10,00,000 shall not be taxable and Rs. 40,00,000 shall be taxable in hands of minor
(d) Rs. 10,00,000 shall be taxable in the hands of his parents and Rs. 40,00,000 shall be taxable
in his hands

PROBLEM:33 The notice of demand in respect of tax paid on clubbed income can be served on
................
(a) Transferor (b) Transferee (c) Both (a) and (b) (d) Neither (a) nor (b)

PROBLEM:34 If there is a transfer of income by a person to another person without the transfer of the
asset from which the income arises, such income shall be included in the income of:
a) Transferor b) transferee c) transferor if transfer is revocable
d) Transferee if transfer is irrevocable.

SALEEM QURAISHEE Mo: 9175664444 INSPIRE ACADEMY-8888881719


CLUBBING OF INCOME- Page 8.23

PROBLEM:35 If there is revocable transfer of an asset by any person to another person, any income
arising from such asset shall be included in the income of:
a) transferor b) transferee c) both transferor & transferee

PROBLEM:36 If there is a transfer of asset which is not revocable during the life time of the transferee,
income arising from such asset shall be included in the income of:
a) Transferor
b) Transferee c) transferee till his death and thereafter in the hands of the transferor

PROBLEM:37 Where an individual has substantial interest in a concern, there shall be included in his
total income any remuneration paid by such concern to:
a) The wife of such individual b) the husband of such individual c) The spouse of such individual

PROBLEM:38 Substantial interest for the purpose of clubbing provision u/s 64(l) shall be of:
a) The individual only
b) The individual & his spouse taken together c) the individual along with his relatives

PROBLEM:39 Relative for the purpose of section 64(l) shall include:


a) Spouse, brother and sister of the individual
b) Spouse, brother, sister or any lineal ascendant or descendant of that individual
c) Spouse, children and dependent brothers and sisters of the individual
d) Spouse, children, dependent parents, dependent brothers and sisters of the individual

PROBLEM:40 Where spouse of an individual gets any remuneration from a concern in which such
individual has substantial interest, then such remuneration shall be included:
a) In all cases
b) Only when such remuneration is received by the spouse due to his/her technical or professional
qualification
c) In all cases except when remuneration is received by the spouse due to his/her technical or
professional qualifications

PROBLEM:41 As per section 64(i)(iv), there shall be included in the income of an individual, any income
arising from the gift to the spouse of:
a) Any capital asset b) Any asset c) any asset other than house property

SALEEM QURAISHEE Mo: 9175664444 INSPIRE ACADEMY-8888881719


CLUBBING OF INCOME- Page 8.24

PROBLEM:42 Where an individual transfers the house property to his wife without adequate
consideration, then income from such house property shall be subject to the provisions of:
a) Section 64(1 )(iv) i.e. income from such house property shall be clubbed in the hands of the
transferor
b) Section 27 i.e. the transferor shall be the deemed owner of such house property and taxable under
section 22. c) None of these

PROBLEM:43 Clubbing provisions under section 64(l)(v); are applicable where the asset is transferred by
an individual without an adequate consideration to:
a) daughter's husband b) sons wife c) major son d) major daughter

PROBLEM:44 R gifts Rs 5,00,000 to his wife who invested the same in the partnership business Mrs. R
receives Rs 2,05,000 as her share of profits from such firm. In this case amount to be clubbed in the
income of R shall be:
a) Rs 2,05,000 b) Rs 15,000 after giving maximum exemption of Rs 1,90,000 to Mrs. R
c) Nil as share of profit from firm is exempt

PROBLEM:45 If any income has to be clubbed under section 64, it will be clubbed under the:
a) head income from other sources b) relevant head to which it belongs c) none of these two

PROBLEM:46 Transfer of income without transfer of asset would be taxable in the hands of :
(a) Transferor only (b) Transferee only (c) Either transferor or transferee (d) Both
transferor and transferee

PROBLEM:47 A transfer which contains any provision for the re-transfer, directly or indirectly/ of the
whole or any part of the income or asset to the transferor, regarded as:
(a) Transfer of income without transfer of asset (b) Indirect transfer
(c) Revocable transfer (d) Irrevocable transfer

PROBLEM:48 X transfers his house property to a trust for benefit of Y till his death. In this case, till
death of Y, the income from house property shall be taxable in the hands of ____ and afterwards in
the hands of _______.
(a) X,Y (b) X, legal heirs of Y (c) Y, legal heirs of Y (d) Y, X

SALEEM QURAISHEE Mo: 9175664444 INSPIRE ACADEMY-8888881719


CLUBBING OF INCOME- Page 8.25

PROBLEM:49 Mr. 'X transfers his house property to Mr. Y with a condition that 25% of the income there
from should be handed over to him Mr. Y earns Rs. 1,00,000 from such house property. In this case –
(a) Total amount Rs. 1,00,000 shall be assessed in the hands of X.
(b) Only Rs. 25,000 will be assessed in the hands of X.
(c) Rs. 25,000 will be assessed in the hands of X and Rs. 75,000 will be assessed in the hands of Y.
(d) Total amount Rs. 1,00,000 shall be assessed in the hands of Y

PROBLEM:50 Mr. A, a fashion designer having lucrative business, pays salary to his wife, who is a
model. Remuneration received by Mrs. A shall be included in the total income of:
(a) Mrs. A (b) Mr. A (c) Mrs. A or Mr. A (whose total income is higher before this clubbing)
(d) Mr. A and Mrs. A proportionately.

PROBLEM:51 If Mr. A and Mrs. A both have substantial interest in & concern and both are in receipt of
remuneration from that concern, then what will be the tax consequences.
(a) Remuneration shall be clubbed in the total
(b) Remuneration shall be clubbed in the total income of Mr. A. income of Mrs. A.
(c) Remuneration shall be clubbed in the total income of that individual whose total income
(d) Clubbing shall not apply, (before this clubbing) is higher.

PROBLEM:52 Mr. A transferred a capital asset to Mrs. A in natural love and affection. She transferred
the capital asset to her friend and she earned a capital gains of Rs. 2,50,000. The capital gains income
shall be regarded :
(a) Mrs. A (b) Mr. A
(c) Mrs. A or Mr. A (whose total income is higher before this clubbing)
(d) Mr. A and Mrs. A proportionately.

PROBLEM:53 An individual is said to have substantial interest in a concern if he or she, along with his or
her relatives, is, at any time during the previous year –
(a) Beneficial owner of equity shares carrying 20% or more of the voting power.
(b) Entitled to 20% or more of the profits of such concern.
(c) Either (a) or (b) (d) Both (a) and (b)

SALEEM QURAISHEE Mo: 9175664444 INSPIRE ACADEMY-8888881719


CLUBBING OF INCOME- Page 8.26

PROBLEM:54 Income from assets transferred to spouse for inadequate consideration, clubbed in the
total income of –
(a) Transferor (b) Clubbing shall not apply (c) Transferee (d) More of these

PROBLEM:55 Income from assets transferred to son's wife for inadequate consideration, shall be
included in the total income of –
(a) Transferor (b) Son's wife (c) Both (a) and (b)
(d) Individual whose total income (before this clubbing) is higher.

PROBLEM:56 In the above case, relationship between transferor and transferee should subsist at the
time of –
(a) Transfer of the asset (b) Accrual of the income
(c) Either (a) or (b) (d) Both (a) and (b)

PROBLEM:57 Mr. A transfers his house property to his fiancée, in this case the income from house
property shall be taxable in the hands of –
(a) Mr. A (b) Mr. A's fiancée
(c) Such individual whose total income (before this clubbing) is higher.
(d) None of the above.

PROBLEM:58 Mr. A transfers Rs. 25,000 to Mrs. A. she purchases investments Rs. 20,000 in X Ltd. out of
such cash transferred to her by Mr. A. he earns interest Rs. 5,000 from such investment. Hence this is
a/an –
(a) Cross transfer (b) Indirect transfer (c) Revocable transfer (d) Irrevocable transfer

PROBLEM:59 In the above case interest income Rs. 5,000 shall be included in the total income of -
(a) Mrs. A (b) Mr. A
(c) Mrs. A or Mr. A (whose total income is higher before this clubbing) (d) Mr. A and Mrs. A

SALEEM QURAISHEE Mo: 9175664444 INSPIRE ACADEMY-8888881719


CLUBBING OF INCOME- Page 8.27

PROBLEM:60 Mr. Y gifts 12.75% debentures worth Rs. 10 lakhs to Mrs. X (wife of Mr. X, his brother) and
Mr. X gifted a flat worth same amount to Mrs. Y. The flat yields a rental income of Rs. 1,30,000 p.a.
Interest income will be included in the total income of ___ and rental income will be included in the
total income of ____.
(a) Mr. Y and Mrs. X respectively. (b) Mrs. Y and Mr. X respectively
(c) Mr. X and Mr. Y respectively. (d) Mrs. Y and Mrs. X respectively.

PROBLEM:61 Income arising to a minor married daughter is –


(a) To be assessed in the hands of the minor married daughter
(b) To be clubbed with the income of that parent whose total income, before including minor's income, is
higher (c) Completely exempt from tax (d) To be clubbed with the income of her husband.

PROBLEM:62 Income from asset transferred to spouse will be taxable in the hands of transferor if:
(a) asset has been transferred in pursuance of an agreement to live apart;
(b) asset was transferred for an adequate consideration; (c) asset was transferred before marriage;
(d) asset was transferred for inadequate consideration

PROBLEM:63 If the self acquired property of an individual (being a member of HUF) is ______ then the
income derived by the per family on account of such property shall be included in the total income of
the individual who was the owner of property.
(a) Converted into joint family property,
(b) Transferred by him directly or indirectly, to HUF otherwise than for adequate consideration.
c) Transferred by him, directly or indirectly to HUF for adequate consideration. (d) Either (a) or (b)

PROBLEM:64 Where the converted property has been the subject-matter of a partition amongst the
members of the family, the income derived from such converted property as is received by ____ on
partition shall be deemed to arise to the individual from assets transferred indirectly by the
individual to the ____ and shall be clubbed in the hands of such individual
(a) Minor child, minor child (b) Brother, brother (c) Spouse, spouse (d) Sister, sister

PROBLEM:65 For the purposes of clubbing of income of the specified person in the income of the
individual under section 64, the word 'income' includes _________.
(a) Salaries (b) Loss (c) Capital gains (d) Income from other sources

SALEEM QURAISHEE Mo: 9175664444 INSPIRE ACADEMY-8888881719


CLUBBING OF INCOME- Page 8.28

PROBLEM:66 Mr. A gifts cash of Rs. 1, 00,000 to his brother's wife Mrs. B. Mr. B gifts cash of Rs. 1,
00,000 to Mrs. A. From the cash gifted to her, Mrs. B invests in a fixed deposit, income there from is
Rs. 10,000. Aforesaid Rs. 10,000 will be included in the total income of _________.
(a) Mr. A (b) Mr. B (c) Mrs. A (d) Mrs. B

PROBLEM:67 Income of minor married daughter ________


A. Be clubbed in the hands her husband.
B. Is not to be clubbed and taxable in her hand
C. Be clubbed in the hands that parent whose total income excluding the income daughter is
greater
D. None of the above

PROBLEM:68 If income is clubbed as per Section 64, then it is clubbed_____________


A. Under the head “income from salary’
B. Under the head “income form other sources’
C. Under relevant head to which it belongs
D. None of the above

PROBLEM:69 X is owner of house on which he receives rent of RS. 8,000. He confers right to receive rent
on such house to his wife without transfer of house. Rental income in this is taxable in the hands of
A. Husband
B. Wife
C. Partly husband partly wife
D. None of the above

PROBLEM:70 In which of the following cases income from asset will be treated as income of transferor?
A. Income from asset transferred by husband to wife before marriage.
B. Income from asset transferred by husband wife after marriage for adequate consideration.
C. Income from asset transferred by husband after dissolution of marriage for inadequate
consideration.
D. None of the above

PROBLEM:71 Ram transfers 10% Debentures of Rs. 10,00,000 to his wife without consideration. His wife
earns Rs. 1,00,000 as interest on such debentures. She deposit Rs. 1,00,000 received on account of
interest and earns Rs. 10,000 as interest on such deposit. How the above income will be taxed?
A. Rs. 1,10,000 will ne taxed in the hands of wife.
B. Rs. 1,10,000 will be taxed in the hands of husband.
C. Rs. 1,00,000 will be taxed in the hands of wife and Rs. 10,000 will be taxed in the hands of
husband.
D. Rs. 1,00,000 will be taxed in the hands of husband and Rs. 10,000 will be taxed in the hands
of wife.

SALEEM QURAISHEE Mo: 9175664444 INSPIRE ACADEMY-8888881719


CLUBBING OF INCOME- Page 8.29

PROBLEM:72 Mrs. Varsha received the following amounts during PY 2019-20.


Gross salary 4,50,000
Family pension @ 1,125 p.m. 13,500
Income of minor child 6,000
Balance in PF received after 75,000
Death of her husband
Gratuity received after the death 60,000
Of her husband
Calculate taxable income of Mrs. Varsha .

PROBLEM:73 Total income of Mr. G and Mrs. G for the PY 2019-20 is Rs. 3,60,000 and Rs. 2,70,000
respectively. Their child is mentally retarded and has income of Rs. 1,80,000. What is the treatment
of income of child?
A. Income of child will be clubbed in the hands of father i.e. Mr. G.
B. Income of child will be clubbed in the hands of mother i.e. Mrs. G.
C. Income of child cannot be clubbed in the hands of father or mother.
D. Income of minor child who is mentally retarded is totally exempt.

PROBLEM:74 Mr. X is employed in PQR Ltd, at monthly salary of Rs. 25,000 and is holding 25% shares of
the company. His wife is employed in same company having salary of Rs. 10,000 p.m. Other income
of Mr. X and Mrs. X is respectively Rs. 30,000 and Rs. 1,00,800. They have twin daughter and their
income is Rs. 2,000 p.a. They have one son whose income is Rs. 1,200 p.a. compute the total income
of Mr. X & Mrs. X…………………………………………

PROBLEM:75 Ram transfers 1,000 debentures of Rs. 100 each carrying 10% interest to Raman on the
condition that he will get 3% interest during his life time. Which of the following shows correct
taxable implication for this transaction?
A. Interest income of Rs. 7,000 will be taxable in the hands of Raman and Rs. 3,000 interest will
be taxable in the hands of Ram as it is revocable transaction for Rs. 3,000 interest income
only.
B. Interest income of Rs. 10,000 will be taxable in the hands of Raman as it is not revocable
transfer.
C. Interest income of Rs. 10,000 will be taxable in the hands of Ram as it is revocable transfer.
D. Interest income of Rs. 3,000 will be taxable in the hand s of Raman and Rs. 7,000 interest
will be taxable in the hands of Ram.

PROBLEM:76 Y transferred agricultural land to Z with a condition that 60% of income from land should
be handed over to him. Z earned Rs. 1,00,000 form such agricultural land. What are the tax
implications?
A. Rs. 1,00,000 is taxable in the hands of Z
B. Rs. 1,00,000 is taxable in the hands of Y as it is revocable transfer
C. Rs. 60,000 is taxable in the hands of Y and Rs. 40,000 is taxable in the hands of Z.
D. Nothing is taxable Y or Z.

SALEEM QURAISHEE Mo: 9175664444 INSPIRE ACADEMY-8888881719


CLUBBING OF INCOME- Page 8.30

PAST EXAMINATION MCQ

PROBLEM:77 June 2015: Shyam transferred 2,000 shares of X Ltd, to Ms. Babita without any
consideration. Later, Shyam d Ms. Babita got married to each other. The dividend income from the
share transferred would be________
A. Taxable in the hands of Shyam both before and after marriage
B. Taxable in the hands of Shyam before marriage but not after marriage
C. Taxable in the hands of Shyam after marriage but not before marriage
D. Never taxable in the hands of Shyam

PROBLEM:78 Dec 2015: Rohit ( A Chartered Accountant) s working as Account Officer in Raj (P) Ltd, on a
salary of Rs. 20,000 p.m. He got married to Ms. Pooja who holds 25% shares if this company. What
will be the impact of salary paid to Rohit by the company in the hands of Ms. Pooja __________
A. 100% salary to be clubbed
B. 50% salary to be clubbed
C. No amount be clubbed
D. 25% salary be clubbed

SALEEM QURAISHEE Mo: 9175664444 INSPIRE ACADEMY-8888881719


SET OFF AND CARRY FORWARD OF LOSSES- Page 9.1

SET-OFF CARRY-FORWARD OF LOSSES


SECTION Subject Matter
Section 70 Set-off of loss from one source against income from other source under the same head of
income
Section 71 Set-off of loss from one head against income from another head
Section 71B Carry forward and set-off of loss from house property
Section 72 Carry forward & set-off of business losses
Section 72A Special cases (amalgamation, demerger, etc)
Section 73 Losses in speculation business

Section 73A Carry forward and set-off of losses by specified business referred in Section 35AD
Section 74 Losses under the head capital gains
Section 74A Loss from the activity of owning and maintaining race horses.
Section 78 Carry forward and set off of losses in case of change in constitution of firm or on
succession
SECTION 79 Carry-forward and set-off of losses of companies in which the public are not substantially
interested
SECTION 80 Compulsory filing of loss returns
The provisions for set off or carry forward and set off of losses are contained in sections 70 to 80 of
Income-tax Act and it involves the following three steps.
Step 1: Inter source adjustments under the same head of income also known as intra head adjustment.
Step 2: Inter-head adjustment in the same assessment year at the time of aggregation of income of
various heads.
Step 3: Carry forward of loss to the subsequent assessment years to claim it as set off if it could not be
set off under Step 1 and Step 2.

SALEEM QURAISHEE Mo: 9175664444 INSPIRE ACADEMY-8888881719


SET OFF AND CARRY FORWARD OF LOSSES- Page 9.2

1. INTRA HEAD ADJUSTMENT- [SECTION 70]


General Rule-If net result for any AY in respect of any source under any head of income, is a loss the
assessee is entitled to have the amount of such loss set off against his income from any other source under
the same head of income for the same AY.

CASE-1 If Mr. X has loss of ` 60,000 from one house and income of `1,00.000 from another house,
the loss of ` 60,000 can be set-off against the income of ` 1,00,000 and the net income u/h house
property would come out to `40,000.

EXCEPTION
I. Loss from a speculation business: Loss in a speculation business can be set off only against the
profit in a speculation business.
II. Loss of a specified business referred to in section 35AD: Any loss computed in respect of any
specified business referred to in section 35AD shall not be set off except against profit or gains, if
any, of any other specified business.
III. Loss from the activity of owning and maintaining race horses: As per section 74A, the loss incurred
by the assessee, in the activity of owning and maintaining race horses, shall only be set off against
the income of such activity.
IV. Loss on account of lottery, etc. cannot be set off against winnings from lotteries, crossword
puzzles, card games, etc.: No expenditure or allowance is allowed from winnings from lotteries or
crossword puzzle, etc
V. Loss from a source which is exempt: Loss incurred by an assessee from a source, income from
which is exempt, cannot be set off against income from a taxable source.
VI. Long-term capital losses-Long term capital loss can be set off only against Long term capital gain.

2. INTER-HEAD ADJUSTMENT [SECTION 71]


General rule-Where the net result of computation made for any AY in respect of any head of income is a
loss, the same can be sett off against income from another head.

EXCEPTION
I. Loss from a speculation business: Loss in a speculation business can be sett off only against the
profit in a speculation business.
II. Loss of a specified business referred to in section 35AD: Any loss computed in respect of any
specified business referred to in section 35AD shall not be set off except against profit or gains, if
any, of any other specified business
III. Business loss cannot be set off against salary income-
IV. House property loss exceeding ` 2,00,000- Sec 71(3A)- House property loss in excess of ` 2,00,000
cannot be sett of against income under the other heads of income. (FA-2017)
V. Loss from the activity of owning and maintaining race horses: As per section 74A, the loss incurred
by the assessee, in the activity of owning and maintaining race horses, shall only be set off against
the income of such activity.
VI. Loss an account of lottery, etc. cannot be set off against winnings from lotteries, crossword
puzzles, card games, etc.:
VII. Loss from a source which is exempt: Loss incurred by an assessee from a source, income from
which is exempt, cannot be set off against income from a taxable source.
VIII. Loss under the head capital gains- Losses under the head capital gains cannot be sett off against
any income except income under the head capital gains.

SALEEM QURAISHEE Mo: 9175664444 INSPIRE ACADEMY-8888881719


SET OFF AND CARRY FORWARD OF LOSSES- Page 9.3

3. CARRY FORWARD AND SET OFF OF LOSSES


I. If the losses could not be set off under the same head or under different heads in the same
assessment year, such losses are allowed to be carried forward to be claimed as set off from the
income of the subsequent assessment years.
II. All losses are not allowed to be carried forward.
III. The following losses are only allowed to be carried forward and set off in the subsequent
assessment years:
a) House property loss;
b) Business loss;
c) Speculation loss;
d) Loss on account of owning and maintaining race horses;
e) Capital loss;
f) Loss from a specified business referred to in section 35AD;

Point to be noted-
1) Under the Income Tax Act, 1961, no particular mode has been prescribed for set-off of losses. In the
absence of any specific provision, the assessee may choose to set-off the losses in the manner
which is most beneficial to him.
2) Set-off of losses is MANDATORY: The assessee does not have an option to set-off or not to set-off a
loss. If income is there, then the loss needs to be mandatorily set-off in that year itself, otherwise
the option of carried forward of such loss is lost.

4. COMPULSORY FILING OF LOSS RETURNS [SECTION 80]:


I. Although the above losses are allowed to be carried forward, but the carry forward is allowed only
when such loss has been determined in pursuance of a return of loss submitted by the assessee on
or before the due date for filing of the returns prescribed under section 139(1).

II. However loss under the head income from house property can be carried forward even if the
return is not filed within the due date, mentioned under section 139(l).

III. Although submission of return of loss, on or before the due date mentioned under section 139(1) is
compulsory for carry forward of losses mentioned in clause (b) to (e) above, but this provision is not
applicable for carry forward of unabsorbed depreciation which is covered under section 32(2).

IV. There are two conditions which are to satisfied before loss is allowed to be carried forward. Firstly
the return of loss must be submitted on or before the due date and secondly such loss has been
determined by the Assessing Officer.

SALEEM QURAISHEE Mo: 9175664444 INSPIRE ACADEMY-8888881719


SET OFF AND CARRY FORWARD OF LOSSES- Page 9.4

5. TREATMENT OF UNABSORBED DEPRICIATION- SECTION 32(2)


A. Meaning - Assessee carrying business/profession are allowed to debit depreciation expenditure while
calculating their income under the head PGBP. However, such expenditure can be debited only to
the extent income is available under the head PGBP. The balance amount of depreciation that
cannot be debited is referred to as ’unabsorbed depreciation'.

B. Treatment
I. Depreciation allowance of the PY is first deductible from the income chargeable under the head
PGBP.
II. If depreciation allowance is not fully deductible under the head PGBP because of absence of profit,
it is deductible from any other head except income under the salary.
III. If depreciation allowance is still unabsorbed, it can be carried forward to the subsequent
assessment years.
IV. Period of carry forward-No time limit

6. LOSSES CAN BE CARRIED FORWARD BY THE PERSON WHO INCURRED THE


LOSS-BUT THIS RULE HAS THE FOLLOWING EXCEPTIONS-
I. Loss of business can be carried forward by inheritance.
II. Accumulated business loss of an amalgamating company under section 72A or 72AA
III. Accumulated business loss of a proprietary concern or a firm when its business is taken over by a
company satisfying conditions of section 47.
IV. Accumulated business loss of a demerged company.
V.

7. ORDER OF SET OFF:


The order of set off will be as under:
S. No. Sec. Nature of Losses
1) 35(1) Current Scientific Research Capital Expenditure
2) 32(1) Current Depreciation
3) 36(1 )(ix) Current year Expenditure on Family Planning to the extent allowed
4) 72(1) Unabsorbed Business Losses of previous years
5) 32(2) Unabsorbed Depreciation of previous years
6) 35(4) Unabsorbed Scientific Research Capital Expenditure of previous years
7) 36(1 )(ix) Unabsorbed Family Planning Promotion Expenditure of previous year

8. BROUGHT FORWARD LOSSES MUST BE SET OFF IN THE IMMEDIATELY


SUCCEEDING YEARS-
I. The losses which are eligible to be carried forward must be set off against the income/profit of
the immediately succeeding year and if there is any balance still to be set off, it should be set of
in the immediately next succeeding year or years within the time allowed.
II. Where the losses incurred are not set off against the income/profits of the immediately
succeeding year/years, as the case may be, they cannot be set off at a later date

SALEEM QURAISHEE Mo: 9175664444 INSPIRE ACADEMY-8888881719


SET OFF AND CARRY FORWARD OF LOSSES- Page 9.5

9. CARRY FORWARD AND SET OFF OF LOSS FROM HOUSE PROPERTY-

10. CARRY FORWARD AND SET OFF OF BUSINESS LOSSES –

SALEEM QURAISHEE Mo: 9175664444 INSPIRE ACADEMY-8888881719


SET OFF AND CARRY FORWARD OF LOSSES- Page 9.6

11. CARRY FORWARD OF LOSSES UNDER THE HEAD 'CAPITAL GAINS' -

POINT TO BE NOTED-
With effect from 1 st April 2018, the long-term capital gain exceeding ` 1, 00,000 arising on sale of equity
shares or units of equity oriented fund or unit of business trust on which STT is paid
I. in respect of equity shares, both at the time of acquisition and sale and
II. in respect of units of equity oriented fund or unit of business trust, at the time of sale
Is taxable under section 112A @10%. Long-term capital loss on sale of such shares/units can, therefore,
be set-off and carried forward for set-off against long-term capital gains by virtue of section 70(3) and
section 74.

12. SET OFF AND CARRY FORWARD AND SET OFF OF LOSS FROM ACTIVITY
OF OWNING AND MAINTAINING RACE HORSES [SECTION 74A]

SALEEM QURAISHEE Mo: 9175664444 INSPIRE ACADEMY-8888881719


SET OFF AND CARRY FORWARD OF LOSSES- Page 9.7

13. CARRY FORWARD AND SET-OFF OF ACCUMULATED BUSINESS LOSS AND


UNABSORBED DEPRECIATION IN CERTAIN CASES OF AMALGAMATION OR
DEMERGER ETC (SECTION 72A)
Section 72A provides for carry forward and set off of accumulated loss and unabsorbed depreciation
allowance in case of:
Amalgamation - [Section 72 A(1), (2) and (3)], or
Demerger - [Section 72 A(4) and (5], or
Reorganization of business [Section 72 A(6)].

A. Carry forward and set off of accumulated loss and unabsorbed depreciation in case of
amalgamation [Section 72A(1), (2) and (3)]

1) Where there has been an amalgamation of a company


a) owning an industrial undertaking or
b) a ship or
c) a hotel with another company or
d) an amalgamation of a banking company with a specified bank,
 then, notwithstanding anything contained in any other provision of this Act, the accumulated loss
and the unabsorbed depreciation of the amalgamating company shall be deemed to be the loss or,
as the case may be, allowance for depreciation of the amalgamated company for the previous year
in which the amalgamation was effected, and other provisions of this Act relating to set off and carry
forward of loss and allowance for depreciation shall apply accordingly.

2) Notwithstanding anything contained in Sub-section (1), the accumulated loss shall not be set off or
carried forward and the unabsorbed depreciation shall not be allowed in the assessment of the
amalgamated company unless –

a) The amalgamating company –


I. has been engaged in the business, in which the accumulated loss occurred or depreciation
remains unabsorbed, for three or more years;
II. has held continuously as on the date of the amalgamation at least three-fourths of the book
value of fixed assets held by it two years prior to the date of amalgamation;

b) The amalgamated company –


I. holds continuously for a minimum period of five years from the date of amalgamation at least
three-fourths of the book value of fixed assets of the amalgamating company acquired in a scheme
of amalgamation;
II. continues the business of the amalgamating company for a minimum period of five years from the
date of amalgamation;
III. Fulfils such other conditions as may be prescribed to ensure the revival of the business of the
amalgamating company or to ensure that the amalgamation is for genuine business purpose.

Consequences if the above conditions are not satisfied [Section 72A(3)]:


I. In a case where the conditions laid down under Clause (b) above are not complied with, the set off
of loss or allowance of depreciation made in any previous year in the hands of the amalgamated
company shall be deemed to be in the income of the amalgamated company chargeable to tax for
the year in which such conditions are not complied with.
[Note: The carry forward and set off of loss and unabsorbed depreciation as per the above provisions
shall be allowed only when amalgamation is as per the provisions of Section 2(1B) of the Income-tax Act,
1961].
SALEEM QURAISHEE Mo: 9175664444 INSPIRE ACADEMY-8888881719
SET OFF AND CARRY FORWARD OF LOSSES- Page 9.8

B. Carry forward and set off of accumulated losses and unabsorbed depreciation in case of demerger
[Sections 72A (4) and (5] Notwithstanding anything contained in any other provisions of this Act in
the case of a demerger, the accumulated loss and the allowance for absorbed depreciation of the
demerged company shall –
I. where such loss or unabsorbed depreciation is directly relatable to the undertakings transferred to
the resulting company, be allowed to be carried forward and set off in the hands of the resulting
company;
II. where such loss or unabsorbed depreciation is not directly relatable to the undertakings
transferred to the resulting company, be apportioned between the demerged company and the
resulting company in the same proportion in which the assets of the undertakings have been
retained by the demerged company and transferred to the resulting company, and be allowed to
be carried forward and set off transferred to the resulting company, and be allowed to be carried
forward and set off in the hands of the demerged company or the resulting company, as the case
may be.
The Central Government may, for the purposes of this Act, by notification in the Official Gazette,
specify such conditions as it considers necessary to ensure that the demerger is for genuine
business purposes.
[Note: The carry forward and set off of accumulated loss and unabsorbed depreciation as per the
above provisions shall be allowed only when demerger is as per the provisions of Section 2(19AA)
of the Income-tax Act.]

C. Carry forward and set off of accumulated losses and unabsorbed depreciation in case of
reorganization of business [Section 72A(6)]-Where there has been reorganization of business,
whereby,
I. a firm is succeeded by a company fulfilling the conditions laid down in Clause (xiii) of Section 47
II. or a proprietary concern is succeeded by a company fulfilling the conditions laid down in Clause
(xiv) of Section 47,
III. a private company or unlisted public company is succeeded by a limited liability partnership
 then, notwithstanding anything contained in any other provisions of this Act, the accumulated loss and the
unabsorbed depreciation of the predecessor firm or the proprietary concern, as the case may be, shall be
deemed to be the loss or allowance for depreciation of the successor company for the purpose of previous
year in which business reorganization was effected and other provisions of this Act relating to set off and
carry forward of loss and allowance for depreciation shall apply accordingly.

14. TREATMENT OF CARRY-FORWARD OF LOSSES OF CERTAIN ASSESSEES


1) Carry-forward and set-off of losses in case of change in constitution of firm (Section 78) Where a
change has occurred in the constitution of a firm, the firm is not entitled to carry forward and set
off so much of the loss proportionate to the share of a retired or deceased partner as exceeds his
share of profits, if any, in the firm in respect of the previous year.

2) Carry-forward and set-off of losses in case of succession of business or profession- When a


business or profession is succeeded by another person, the brought forward losses by the
predecessor can be set-off against the income earned by the predecessor before the succession.
The successor is not entitled to carry forward the losses sustained by the predecessor and set them
off against the income earned by him. However, there is exception. If the succession is by
inheritance, the heir-at-law is entitled to carry-forward and set-off the losses sustained by the
predecessor provided the business in question continues to be carried on by the successor.

SALEEM QURAISHEE Mo: 9175664444 INSPIRE ACADEMY-8888881719


SET OFF AND CARRY FORWARD OF LOSSES- Page 9.9

15. CARRY-FORWARD AND SET-OFF OF LOSSES OF COMPANIES IN WHICH


THE PUBLIC ARE NOT SUBSTANTIALLY INTERESTED (SECTION 79)
1) Notwithstanding anything contained in this Chapter, where a change in shareholding has taken place
during the previous year in the case of a company, not being a company in which the public are
substantially interested, no loss incurred in any year prior to the previous year shall be carried forward
and set off against the income of the previous year, unless on the last day of the previous year, the
shares of the company carrying not less than fifty-one per cent of the voting power were beneficially
held by persons who beneficially held shares of the company carrying not less than fifty-one per cent of
the voting power on the last day of the year or years in which the loss was incurred:

Provided that even if the said condition is not satisfied in case of an eligible start-up as referred to in
section 80-IAC, the loss incurred in any year prior to the previous year shall be allowed to be carried
forward and set off against the income of the previous year if all the shareholders of such company
who held shares carrying voting power on the last day of the year or years in which the loss was
incurred, continue to hold those shares on the last day of such previous year and such loss has been
incurred during the period of seven years beginning from the year in which such company is
incorporated.

2) Nothing contained in sub-section (1) shall apply,—


a) To a case where a change in the said voting power and shareholding takes place in a previous
year consequent upon the death of a shareholder or on account of transfer of shares by way of
gift to any relative of the shareholder making such gift;
b) To any change in the shareholding of an Indian company which is a subsidiary of a foreign
company as a result of amalgamation or demerger of a foreign company subject to the
condition that fifty-one per cent shareholders of amalgamating or demerged foreign company
continue to be the shareholders of the amalgamated or the resulting foreign company;
c) To a company where a change in the shareholding takes place in a previous year pursuant to a
resolution plan approved under the Insolvency and Bankruptcy Code, 2016 (31 of 2016), after
affording a reasonable opportunity of being heard to the jurisdictional Principal Commissioner
or Commissioner;
d) To a company, and its subsidiary and the subsidiary of such subsidiary, where, —
I. the Tribunal, on an application moved by the Central Government under section 241 of the
Companies Act, 2013 (18 of 2013), has suspended the Board of Directors of such company and
has appointed new directors nominated by the Central Government, under section 242 of the
said Act; and
II. a change in shareholding of such company, and its subsidiary and the subsidiary of such
subsidiary, has taken place in a previous year pursuant to a resolution plan approved by the
Tribunal under section 242 of the Companies Act, 2013 (18 of 2013) after affording a
reasonable opportunity of being heard to the jurisdictional Principal Commissioner or
Commissioner.

SALEEM QURAISHEE Mo: 9175664444 INSPIRE ACADEMY-8888881719


SET OFF AND CARRY FORWARD OF LOSSES- Page 9.10

PRACTICAL PROBLEMS
PROBLEM:1 X submits the following particulars for PY
2019-20 2020-21
`)
(` `)
(`

Income from salary 1,00,000 2,00,000


Business profits (before depreciation) 16,000 18,000
Current depreciation 1,34,000 1,32,000
Income from other sources 10,000 80,000
Determine the taxable income of X for the assessment years 2020-21 and 21-22.
ANSWER:1

PROBLEM:2 X submits the following particulars:


2019-20 2020-21
`)
(` `)
(`

Business profits (before depreciation) (-) 50,000 45,000


Current depreciation 18,000 20,000
Income from other sources 20,000 70,000
Determine the net income of X for the assessment years 2020-21 and 21-22.
ANSWER:2

PROBLEM:3 From the following information submitted to you, compute the taxable income -
Situation I Situation II
`)
(` `)
(`

Long-term capital gain/loss (+) 1,90,000 (-) 3,50,000


Short-term capital gain/loss (-) 60,000 (+) 1,40,000
Business income/loss (-) 90,000 (+) 2,90,000
ANSWER:3

PROBLEM:4 Compute the taxable income in the following two situations.


Situation I Situation II
`)
(` `)
(`

Income/loss from manufacturing business 2,50,000 (-) 1,50,000


Income/loss from speculation business (-) 1,20,000 (+) 2,80,000
Capital gain/loss (-) 1,40,000 (-) 1,40,000
Agricultural income/loss (-) 50,000 (+) 70,000
ANSWER:4

SALEEM QURAISHEE Mo: 9175664444 INSPIRE ACADEMY-8888881719


SET OFF AND CARRY FORWARD OF LOSSES- Page 9.11

PROBLEM:5 Mr. A (aged 35 years) submits the following particulars pertaining to the A.Y. 2020-21:

Particulars `
I. Income from salary (computed) 4,00,000
II. Loss from self-occupied property (-) 70,000
III. Loss from let-out property (-)1,50,000
IV. Business loss (-)1,00,000
V. Bank interest (FD) received 80,000
Compute the total income of Mr. A for the A.Y.2021-22. ICAI
ANSWER:5

PROBLEM:6 Mr. B, a resident individual, furnishes the following particulars for the P.Y. 2019-20:

Particulars `
I. Income from salary (computed) 45,000
II. Income from house property (24,000)
III. Income from business – non-speculative (22,000)
IV. Income from speculative business (4,000)
V. Short-term capital losses (25,000)
VI. Long-term capital gains 19,000
What is the total income chargeable to tax for the A.Y.2020-21? ICAI
ANSWER:6

PROBLEM:7 During the P.Y. 2019-20, Mr. C has the following income and the brought forward losses:

Particulars `
I. Short term capital gains on sale of shares 1,50,000
II. Long term capital loss of A.Y.2018-19 (96,000)
III. Short term capital loss of A.Y.2019-20 (37,000)
IV. Long term capital gain 75,000
What is the capital gain taxable in the hands of Mr. C for the A.Y.2020-21? ICAI
ANSWER:7

PROBLEM:8 Mr. D has the following income for the P.Y. 2019-20:

Particulars `
I. Income from the activity of owning and maintaining the race horses 75,000
II. Income from textile business 85,000
III. Brought forward textile business loss 50,000
IV. Brought forward loss from the activity of owning and maintaining the 96,000
race horses (relating to A.Y. 2017-18)
What is the total income in the hands of Mr. D for the A.Y. 2020-21? ICAI
ANSWER:8

SALEEM QURAISHEE Mo: 9175664444 INSPIRE ACADEMY-8888881719


SET OFF AND CARRY FORWARD OF LOSSES- Page 9.12

PROBLEM:9 Mr. E has furnished his details for the A.Y.2020-21 as under:

Particulars `
I. Income from salaries (computed) 1,50,000
II. Income from speculation business 60,000
III. Loss from non-speculation business (40,000)
IV. Short term capital gain 80,000
V. Long term capital loss of A.Y. 2018-19 (30,000)
VI. Winning from lotteries 20,000
What is the taxable income of Mr. E for the A.Y.2020-21? ICAI
ANSWER:9

PROBLEM:10 From the following particulars, compute the gross total income of B for the assessment
year 2020-21.
`

i. Loss under the head 'Income from house property', from a house which is let out. 245,000

ii. Income from business 220,000

iii. Profit from speculation business 15,000

iv. Long-term capital gains from building 285,000

v. Short-term capital loss 27,000

vi. Loss under the head 'Income from Other Sources'. 12,000
ANSWER:10 `281000

PROBLEM:11 B submits the following particulars of his income and loss for the assessment year 2020-21
1) Income from house property (computed) 7,000

2) Income from interest from a firm 1,500

3) Profit from cloth business (before depreciation) 40,000

4) Income from speculation business 3,200

5) Long-term capital gain 9,100

6) Dividend from UTI(units are held as stock-in-trade))Gross 2,000

7) Current year's depreciation 2,000


The following items have been brought forward from the preceding year:
i) Loss from cloth business ` 10,000; ii) unabsorbed depreciation ` 7,500;
iii) Loss from speculation ` 7,000' iv) short-term capital loss ` 4,200;
v) Long-term capital loss ` 11,400.
You are required to compute his gross total income and deal with the carry forward losses.
ANSWER:11 ` 29000

SALEEM QURAISHEE Mo: 9175664444 INSPIRE ACADEMY-8888881719


SET OFF AND CARRY FORWARD OF LOSSES- Page 9.13

PROBLEM:12 X, a resident individual, submits the following information, relevant to the previous year
ending 31-3-2020.Calculate GTI
1) Income from salary (computed) 212000

2) Income from house property

House I 12,000

House II (-)250,000

House III (Self-occupied) (-)10,000

3) Profit and gains of business or profession

Business I 8,000

Business II (-)12,000

Business III (Speculative) (-)64,000

Business IV (Speculative) 36,000

4) Capital gains

Short-term capital loss (-) 6,000

Long-term capital gains on transfer of shares 5,400

5) Income from other sources (computed):

Income from card games 36,000

Income from betting 24,000

Loss on maintenance of race horses (-) 4,600


ANSWER:12`
ANSWER:12

PROBLEM:13 From the following particulars compute the total income of tax payable by R for the
assessment year 2020-21.
`
Income from House property (-) 208000
Short-term capital loss on sale of shares 95,000
Long-term capital gain on sale of buildings 3,65,000
Other Sources: Interest on Government Securities
15,000
The assessee has unabsorbed depreciation of `40,000 being brought forward from assessment year
2020-21 . Assessee had closed the business and all the assets have been disposed of.
ANSWER:13`
ANSWER:13

SALEEM QURAISHEE Mo: 9175664444 INSPIRE ACADEMY-8888881719


SET OFF AND CARRY FORWARD OF LOSSES- Page 9.14

PROBLEM:14 The Accountant of a company computed the income of the company for the assessment
year 2020-21 as ` 99,000 without taking into account the following.
`

(i) Carried forward depreciation 45,000

(ii) Depreciation for the current year 20,000

(iii) Capital Expenditure on research for the current year 20,000

(iv) Carried forward unabsorbed business loss relating to previous year 2016-17 12,000

(v) Carried forward unabsorbed business loss relating to previous year 2017-18 10,000

(vi) Carried forward unabsorbed business loss relating to previous year 2018-19 12,000
Compute the total income of the company for the assessment year 2020-21 after making necessary
adjustment of losses and also the amount of losses to be carried forward, if any.
ANSWER:14 Nil

PROBLEM:15 The Income of an assessee from business is determined as ` 1,50,000 for the assessment year
2020-21 without making the following adjustments.
`

(i) Depreciation allowances for the current year 30,000


(ii) Unabsorbed depreciation brought forward for the assessment year 2020-21 15,000
(iii) Long-term capital loss for the current year 12,000
(iv) Unabsorbed business loss brought forward from the assessment year 2009-10 50,000
(v) Unabsorbed speculation loss brought forward from the assessment year 2017-18 15,000
(vi) Short-term capital loss for the current year 24,000
Compute the total income of the assessee for the assessment year 2020-21 and indicate the amount of loss to be
carried forward to next year.
ANSWER:15 ` 1,05,000

SALEEM QURAISHEE Mo: 9175664444 INSPIRE ACADEMY-8888881719


SET OFF AND CARRY FORWARD OF LOSSES- Page 9.15

PROBLEM:16 R, a resident individual, submits the following information’s relevant for the previous year
ending on 31-3-2020
`

(a) Income from salary (computed) + 262,000

(b) Interest on securities + 2,000

(c) Income from House Property

House No. 1 + 12,000

House No. 2 - 220,000

House No. 3 - 10,000

(d) Profit and Gains from Business:

Business No. 1 + 16,000

Business No. 2 - 12,000

Business No. 3 (speculative) -64,000

Business No. 4 (speculative) + 36,000

(e) Capital gains:

Short-term capital gain (computed) - 60,000

Long-term capital gain (computed) + 54,000

(f) Income from card games and betting (gross) + 60,000

Loss from maintenance of race horses - 46,000

Income from maintaining horses and winning of races + 20,000


Determine the total income of R for the assessment year 2020-21.
ANSWER:16`
ANSWER:16

SALEEM QURAISHEE Mo: 9175664444 INSPIRE ACADEMY-8888881719


SET OFF AND CARRY FORWARD OF LOSSES- Page 9.16

PROBLEM:17 R furnishes the following particulars of his incomes and losses for the assessment year 2020-21.
`

1. Interest on securities (gross) 14,000


2. Income from house property (computed) 30,000
3. Profit from paints business 64,000
4. Speculation profits 15,000
5. Share of profit from a firm 23,000
6. Long-term capital gain 30,000
7. Short-term capital gain 18,000
Following are the brought forward items of the assessment year 2020-21
`

1. Loss from Hardware business (discontinued) 15,000


2. Unabsorbed depreciation allowance of paints business 11,000
3. Speculation loss 27,000
4. Loss from short-term capital assets 21,000
5. Loss from long-term capital assets 31,000
ANSWER:17 ` 82,000

SALEEM QURAISHEE Mo: 9175664444 INSPIRE ACADEMY-8888881719


SET OFF AND CARRY FORWARD OF LOSSES- Page 9.17

PROBLEM:18 Particulars of taxable income under various heads of R for the previous year 2019-20 are as under:
`

(i) Taxable income from salary 257,000

(ii) Taxable incomes from various houses:

(a) Income from House A 12,000

(b) Loss from House B 209,000

(c) Loss from House C 6,750

(iii) Business income:

(a) Profit from business 25,500

(b) Current year's depreciation 9,750

(c) Previous trade losses 6,750

(d) speculation profit 6,000

(iv) Capital Gains:

(a) Short-term capital profit 24,000

(b) Long-term capital profit on shares (Net) 7,800

(c) Long-term capital loss on Building (Net) 18,750


Compute gross total income for the assessment year 2020-21 after setting off the losses.
ANSWER:18 ` 92,250

SALEEM QURAISHEE Mo: 9175664444 INSPIRE ACADEMY-8888881719


SET OFF AND CARRY FORWARD OF LOSSES- Page 9.18

16. MCQ
PROBLEM:1 Loss from speculation business ................ be adjusted against the income from non-
speculation business.
(a) Can (b) Cannot (c) Partly can (d) None of the above

PROBLEM:2 Loss from lotteries is ................ to be set off from the income earned from winning of
lotteries.
(a) Allowed (b) Not allowed (c) Allowed up to 50% of the amount (d) None of the above

PROBLEM:3 Long-term capital loss can be set off against ................


(a) Long-term capital gain (b) Short-term capital gain (c) Both (a) and (b) (d) None of the above

PROBLEM:4 Loss from business cannot be set off against ................


(a) Income from House Property (b) Income from Capital Gains
(c) Income from Salaries (d) Income from Other Sources

PROBLEM:5 Loss under the head House Property is allowed to be set off from any other head ...........
(a) Maximum Up to Rs. 1,00,000 (b) Maximum Upto Rs. 2,00,000
(c) 50% of the loss (d) Without any ceiling limit

PROBLEM:6 RADHIKA , owner of a house property, incurs loss of Rs. 5,00,000 from such house
property. She has filed his return of loss u/s 139(1) after the due date. Will he be eligible to carry
forward his loss of Rs. 5,00,000?
(a) Yes, he can carry forward loss of Rs. 5,00,000
(b) No, he can’t carry forward the loss of Rs. 5,00,000
(c) He can carry forward only Rs. 2,00,000 (d) None of the above

PROBLEM:7 Loss from house property, if not set off in the same assessment year, can be carried
forward up to ................
(a) 2 years (b) 4 years (c) 6 years (d) 8 years

PROBLEM:8 As per section 32(2), which of the following can be carried forward in case the property is
inherited by the legal heir?
(a) Business Loss (b) Unabsorbed Depreciation (c) Both (a) and (b) (d) None of the above

PROBLEM:9 Business loss which was incurred in previous year 2019-20, can be carried forward till the
assessment year ................
(a) 2026-27 (b) 2027-28 (c) 2028-29 (d) Indefinitely

SALEEM QURAISHEE Mo: 9175664444 INSPIRE ACADEMY-8888881719


SET OFF AND CARRY FORWARD OF LOSSES- Page 9.19

PROBLEM:10 The correct order of set off is?


(i) Current year depreciation
(ii) Unabsorbed depreciation
(iii) Brought forward business or profession losses
(iv) Current year capital expenditure on scientific research and on family planning
(v) Unabsorbed capital expenditure on scientific research
(vi) Unabsorbed expenditure on family planning
(a) (i), (in), (iv), (ii), (v), (vi)
(b) (i), (iii), (iv), (it), (vi), (v)
(c) (i), (iv), (iii), (it), (v), (vi)
(d) (i), (it), (iii), (iv), (v), (vi)

PROBLEM:11 The loss from a speculation business is allowed to be set off from the profits or gains of
................ in the same year and if losses cannot be set off fully, then such losses can be carried
forward for a maximum period of ................
(a) Another speculation business, 4 years (b) Another speculation business, 8 years
(c) Any other business, 4 years (d) Any other business, 8 years

PROBLEM:12 A is engaged in derivative trading on a recognised stock exchange and incurs a loss of Rs.
5,00,000. He has other business profit of manufacturing business amounting to Rs. 3,00,000.
Taxable income will be treated as
(a) Rs. 5,00,000 (b) Rs. 3,00,000
(c) Rs. 2,00,000 c/f of loss (d) Rs. 3,00,000

PROBLEM:13 If in any assessment year, short term and long-term loss has been incurred under the head
“Capital Gains” then such loss, can be carried forward ................
(a) Separately (b) Collectively
(c) Either (a) or (b) (d) None of the above

PROBLEM:14 The loss arising from capital gain can be carried forward to a maximum of ................
(a) 4 years (b) 6 years (c) 8 years (d) Indefinitely

PROBLEM:15 As per section 74A, loss arising from owning and maintaining race horses can be carried
forward to a maximum of ................ immediately succeeding the assessment year for which such
loss was first calculated.
(a) 4 years (b) 6 years (c) 8 years (d) Indefinitely

PROBLEM:16 Section 72A, does not cover carry forward of business loss and unabsorbed depreciation in
case of ................
(a) Amalgamation of company (b) Demerger of company
(c) Reorganization of business (d) Liquidation of Company

SALEEM QURAISHEE Mo: 9175664444 INSPIRE ACADEMY-8888881719


SET OFF AND CARRY FORWARD OF LOSSES- Page 9.20

PROBLEM:17 As per section 72A, unabsorbed depreciation of an amalgamating company can be carried
forward for ................
(a) 4 years (b) 6 years (c) 8 years (d) Indefinite period

PROBLEM:18 AMIT incurred short-term capital loss of Rs. 20,000 on sale of shares through the
National Stock Exchange. Such loss can ................
(a) Be set off only against short-term capital gains (b) Be set off only against long-term capital gains
(c) Be set off against both short-term and long-term capital gains (d) Not allowed to be set off

PROBLEM:19 KABEER , who incurs a loss of Rs. 4,00,000 from his house property in the previous year
2019-20 will be allowed to set off such loss from the gain or income on house property of Rs.
3,50,000, to the extent of ................
(a) Rs. 2,00,000 (b) Rs. 3,50,000 (c) Rs. 4,00,000 (d) Not allowed to be set off

PROBLEM:20 Intra head adjustment mean


(a) Set off of loss against any other income under the same head
(b) Set off of loss against any other income under different head
(c) Both (a) and (b) (d) Neither (a) nor (b)

PROBLEM:21 In which of the following cases, would loss from one source, not be allowed to be set off
against other income falling under the same head of income?
(a) Loss from a speculation business
(b) Loss from the activity of owning and maintaining horse race
(c) Loss from winnings from lotteries (d) All of the above

PROBLEM:22 SAMEER has two business. Business trading of goods has a profit of Rs.
Rs. 5,00,000 and the
other one, for purchase and sale of shares which is settled otherwise than by actual delivery, has a
loss of Rs. 2,00,000. His taxable income would be ................
(a) Rs. 2,00,000 (b) Rs. 5,00,000 (c) Rs. 3,00,000 (d) Rs. 6 ,00,000

PROBLEM:23 Which of the following will not be treated as speculative business?


(a) Derivative trading on stock exchange (b) Derivative trading in foreign exchange
(c) Both (a) and (b) (d) Neither (a) nor (b)

PROBLEM:24 JOY has a loss of Rs. 5,00,000 from the activity of owning and maintaining race horses.
He also has a profit of Rs. 10,00,000 from the activity of trading in goods through delivery. His
taxable income would be ................
(a) Rs. 10,00,000 (b) Rs. 5,00,000 (c) Rs. 13,00,000 (d) Rs. 7,00,000

PROBLEM:25 SAMEER has a loss of Rs. 5,00,000 from the activity of owning and maintaining race
horses in Delhi and he also has a profit of Rs. 10,00,000 from the activity of owning and maintaining
race horses in Mumbai. His taxable income would be ................
(a) Rs. 10,00,000 (b) Rs. 5,00,000 (c) Rs. 13,00,000 (d) Rs. 7,00,000

SALEEM QURAISHEE Mo: 9175664444 INSPIRE ACADEMY-8888881719


SET OFF AND CARRY FORWARD OF LOSSES- Page 9.21

PROBLEM:26 SANJEEV has a short-term capital loss of Rs. 5,00,000 from Capital Asset A and he also
has a short-term capital gain of Rs. 10,00,000 from sale of Capital Asset B. His taxable income
would be ................
(a) Rs. 10,00,000 (b) Rs. 5,00,000 (c) Rs. 13,00,000 (d) Rs. 7 ,00,000

PROBLEM:27 MANOJ has a short-term capital loss of Rs. 5,00,000 from Capital Asset A and he also has
a long term capital gain of Rs. 10,00,000 from sale of Capital Asset B. His taxable income would be
................
(a) Rs. 10,00,000 (b) Rs. 5,00,000 (c) Rs. 13,00,000 (d) Rs.7,00,000

PROBLEM:28 SANTOSH has a long-term capital loss of Rs. 500,000 from Capital Asset A and he also has
a long-term capital gain of Rs. 10,00,000 from sale of Capital Asset B. His taxable income would be
................
(a) Rs. 10,00,000 (b) Rs. 5,00,000 (c) Rs.13,00,000 (d) Rs. 7 ,00,000

PROBLEM:29 SANJEEV has a long-term capital loss of Rs. 5,00,000 from Capital Asset A and he also has
a short-term capital gain of Rs. 10,00,000 from sale of Capital Asset B. His taxable income would be
................
(a) Rs. 10,00,000 (b) Rs. 5,00,000(c) Rs. 13,00,000 (d) Rs.7,00,000

PROBLEM:30 GAGAN has a business loss of Rs. 5,00,000. Certain income of his minor child amounting
Rs. 15,00,000 was also clubbed in his hands under the income tax provisions as business income.
His taxable income would be ................
(a) Rs. 10,00,000 (b) Rs. 15,00,000 (c) Rs. 20,00,000 (d) Rs. 8,00,000

PROBLEM:31 ANIL has a business loss of Rs. 5,00,000 and he has received the salary of Rs. 14,00,000
during the year. His taxable income would be ................
(a) Rs. 9,00,000 (b) Rs. 15,00,000 (c) Rs. 14,00,000 (d) Rs. 19,00,000

PROBLEM:32 KUNAL has a loss of Rs. 4,00,000 under the head “Income from house property". He also
has a business profit of Rs. 5,00,000 during the year. His taxable income would be ................
(a) Rs. 9,00,000 (b) Rs. 3,00,000 (c) Rs. 1,00,000 (d) Rs. 8,00,000

PROBLEM:33 Mr. X has IFS of Rs. 5,00,000 and he has suffered loss from house property amounting Rs.
1,50,000. Speculation business Joss - Rs. 1,00,000 Find out the gross total income.

PROBLEM:34 Mr. X has earned IFS of 5,00,000 and he has suffered loss from house property amounting Rs.
1,50,000. General business loss - 11,00,000 Find out the gross total income.

PROBLEM:35 Mr. X has earned general business income of Rs. 5,00,000 and he has suffered loss from house
property amounting Rs. 1,50,000. Specified business loss under Section 35AD - Rs. 1,00,000 Find out the gross
total income.

SALEEM QURAISHEE Mo: 9175664444 INSPIRE ACADEMY-8888881719


SET OFF AND CARRY FORWARD OF LOSSES- Page 9.22

PROBLEM:36 Mr. X has earned Long term capital gains on sale of equity shares listed in recognized stock
exchange on which STT -Rs. 7,20,000 Short term capital loss of Rs. 2,00,000. General business income of Rs.
5,00,000 Find out the gross total income.

PROBLEM:37 Find out the gross total income of Mr. A for assessment year 2019-20 from the following
information -Income from salaries Rs. 80,000; Loss from house property Rs. 50,000; Profit from textile trade
Rs. 40,000; Loss from Automotive trade Rs. 50,000.

PROBLEM:38 Mr. Rahul have income from cloth business Rs. 1,00,000; Loss from agriculture Rs. 50,000; Long-
term capital gain Rs. 60,000 and short term capital loss 180,000 find out his gross total income for assessment
year 2020-21.

PROBLEM:39 Mr. Ram had incurred loss in activity of owning and maintaining racehorses Rs. 90,000; Winnings
from lottery (net) Rs. 70,000; Loss in card game of assessment year 2005-06 Rs. 4,000 find out his gross total
income for assessment year 2020-21 .

PROBLEM:40 What can be the maximum amount of negative income of a self occupied house under the head
Income from house property.
(a) Rs. 30,000 (b) Rs. 1,00,000 (c) Rs. 1,50,000 (d) Rs. 2,00,000

PROBLEM:41 Business loss of an amalgamating company shall:


(a) be carried forward and set off in the hands of amalgamated company unconditionally
(b) be carried forward and set-off in the hands of amalgamated company subject to certain conditions
(c) not be carried forward (d) be allowed to be carried forward only by amalgamating company

PROBLEM:42 In case of amalgamation u/s 72A, after fulfilling prescribed conditions accumulated loss can be
carried forward for –
(a) Further 8 years in the hands of amalgamating company
(b) Further 8 years in the hands of amalgamated company (c) Further 4 years in the hands of amalgamating
company (d) Further 4 years in the hands of amalgamated company

PROBLEM:43 In case of demerger after fulfilling prescribed conditions u/s 72A, the accumulated loss can be
carried forward for the –
(a) Remaining period out of 8 years in the hands of resulting company.
(b) Further 8 years in the hands of resulting company. (c) Further 8 years in the hands of demerged undertaking.
(d) Remaining period out of 8 years in the hands of demerged undertaking

SALEEM QURAISHEE Mo: 9175664444 INSPIRE ACADEMY-8888881719


SET OFF AND CARRY FORWARD OF LOSSES- Page 9.23

PROBLEM:44 The accumulated loss shall not be set off or carried forward and the unabsorbed depreciation shall
not be allowed in the assessment of the amalgamated company unless the amalgamated company fulfills
which of the following conditions.
(a) Holds continuously for a minimum period of 5 years from the date of amalgamation at least 3/41 of the book
value of fixed assets of the amalgamating company acquired in a scheme of amalgamation;
(b) Continues the business of the amalgamating company for a minimum period of 5 years from the date of
amalgamation;
(c) Fulfils such other conditions as may be prescribed to ensure the revival of the business of the amalgamating
company or to ensure that the amalgamation is for genuine business (d) All of these

PROBLEM:45 In case of reorganization of business whereby firm is succeeded by a company fulfilling conditions
given under section 47(xiii)/(xiv), the accumulated loss and unabsorbed depreciation can be carried forward in
the hands of successor company for –
(a) Remaining period out of 8 years (b) Further 8 years (c) Remaining period out of 4 years (d) Further 4 years

PROBLEM:46 In case of amalgamation of banking company under section 72AA, accumulated loss and
unabsorbed depreciation can be carried forward in the hands of amalgamated company for –
(a) Remaining period out of 8year (b) Remaining period out of 4 years (c) Further 8 years (d) Further 4 years

PROBLEM:47 In case of amalgamation of co-operative banks, after fulfilling the prescribed conditions,
accumulated loss and unabsorbed depreciation can be carried forward and set-off for –
(a) Further 8 years in the hands of successor co- operative bank.
(b) Remaining period out of 8 years in the hands of amalgamated co-operative bank.
(c) Further 4 years in the hands of successor co- operative bank.
(d) Further 4 years in the hands of predecessor co- operative bank.

PROBLEM:48 In case of demerger of co-operative banks as per prescribed conditions accumulated loss can be
carried forward and set-off in the hands of resulting co-operative bank for
(a) Further 8 years (b) Further 4 years (c) Remaining period out of 8 years. (d) Remaining period out of 4 years.

PROBLEM:49 In case of succession of any business/ profession otherwise than by inheritance, the losses can be
carried forward set-off –
(a) Only by the person incurring the loss. (b) Only by the successor.
(c) Partly by the person incurring the loss and partly by the successor. (d) None of the above.

SALEEM QURAISHEE Mo: 9175664444 INSPIRE ACADEMY-8888881719


SET OFF AND CARRY FORWARD OF LOSSES- Page 9.24

PROBLEM:50 In which of the following case loss Can be carried forward without furnishing the return of loss?
(a) Loss from house property.
(b) Losses under the head Profits & Gains of Business or Profession except speculation business loss.
(c) Losses under Profits & Gains of Business or Profession including speculation business loss.
(d) Losses under the head Capital Gains.

PROBLEM:51 Mr. X has loss from house property Rs. 5,00,000 and LTCG Rs. 5,00,000, in this case his tax
liability shall be……………….

PROBLEM:52 Mr. X a senior citizen has loss from house property Rs. 5,00,000 and LTCG Rs. 5,00,000, in
this case his tax liability shall be………………………

PROBLEM:53 Mr. X a senior citizen has loss from house property Rs. 5,00,000 and as income from
business Rs. 12,00,000 in this case his tax liability shall be……………….

SALEEM QURAISHEE Mo: 9175664444 INSPIRE ACADEMY-8888881719


AGRICULTURE INCOME- Page 10.1

1. INCOME-

2. AGRICULTURAL INCOME- [SECTION 2(1A)]


Meaning of ‘Agricultural Income' Section
2(1 A)

Section 2(1 A)(a) Section 2(1 A)(b) Section 2(1 A)(c)


Income From Letting Out of Land For Income From Agricultural Income From Farm
Agricultural Purposes Operations Building

1) MEANING OF AGRICULTURE INCOME-

a) Income from Letting-Out of Land for Agricultural Purposes - Section 2(1A)(a)- Any rent or revenue
derived from land which is situated in India and is used for agriculture purpose.
The following three conditions have to be satisfied for income to be treated as agricultural income:
I. Rent or revenue should be derived from land;
II. Land has to be situated in India (If agricultural land is situated in a foreign country, the entire
income would be taxable); and
III. Land should be used for agricultural purposes.

POINT TO BE NOTED-
 The amount received in money or in kind, by one person from another for right to use land is
termed as Rent.
 The rent can either be received by the owner of the land or by the original tenant from the sub-
tenant. It implies that ownership of land is not necessary.
 Thus, the rent received by the original tenant from sub-tenant would also be agricultural income
subject the other conditions mentioned above.
 The scope of the term “Revenue” is much broader than rent. It includes income other than rent.
 Interest on delayed payment of rent received from the tenant is not an agricultural income in the
hands of the owner. Rather such interest would be taxable u/h ’income from other sources'

SALEEM QURAISHEE Mo: 9175664444 INSPIRE ACADEMY-8888881719


AGRICULTURE INCOME- Page 10.2

CASE-1 Mr. X has let out one land to Mr. Y for agricultural purposes. Mr. Y has given 1000 kgs of
wheat to Mr. X as rent payment. The market value of 1 kg of wheat is Rs 50. In this case, rent
income in the hands of Mr. X would amount to Rs 50,000 and income would be treated as
agricultural income of Mr. X.

b) Income from Agricultural Operations - Section 2(1 A)(b)-


 Any income derived from such land by agriculture operation or from processing of agricultural
produce.
 Income earned by any person from sale of crops which have been grown as a result of agricultural
operations is treated as agricultural income in the hands of the cultivator.
 (Agricultural Operations = Basic Operations + Subsequent Operations)

CASE-2 Mr. X has grown 10000 kg of potatoes on a piece of land owned by him after carrying out
basic & subsequent operations. The potatoes have been sold in the market for Rs 10/kg whereas
the cost of cultivation of such potatoes came out to Rs 40,000. The net income of Rs 60,000 earned
by Mr. X from sale of potatoes qualifies as agricultural income u/s 2(1 A)(b).

POINT TO BE NOTED-
A. MEANING OF ’AGRICULTURE’/'AGRICULTURAL PURPOSES’
I. The terms agriculture' and agricultural purposes' have not been defined under the Income Tax Act.
II. The Supreme Court of India in the case of Raja Benoy Kumar Sahas Roy vs CIT held that carrying out
an agricultural activity would involve combined performance of basic operations along with
subsequent operations.

B. Basic Operations: Basic operations involve application of human labour and skill on the land to make
the crop sprout from the land. Examples of basic operations are ploughing of land, sowing of seeds,
planting and other similar operations on land.

C. Subsequent Operations: Subsequent operations are carried out by agriculturalists after the crop
sprouts from the land such as weeding, digging the soil around the growth, protections of plants by
using insecticides A pesticides, watering of the plant at regular intervals, pruning, cutting,

D. MARKETING OPERATIONS/MARKETING PROCESS:


I. At times, a cultivator is required to perform additional operations on a crop to make it fit for sale in
the market because such crop cannot be sold in the market in the form in which it was grown. Such
additional operations which make the product fit for sale in the market are known as 'marketing
operations/marketing process'.

CASE-3 Wheat needs to be threshed before being sold, tobacco leaves need to be dried to make
them fit for sale, etc. The activity of threshing, drying, etc is referred to as marketing operations.
The gain in the value of the agricultural produce by such marketing process is also classified as
agricultural income.

SALEEM QURAISHEE Mo: 9175664444 INSPIRE ACADEMY-8888881719


AGRICULTURE INCOME- Page 10.3

c) INCOME FROM FARM BUILDING - SECTION 2(1 A)(C) Bonafide annual value of house property is
taxable under Section 22.However, income from House property which satisfy the following cumulative
conditions would be treated as agriculture income and would be exempt from tax under section 10(1).
I. Building is on or in the immediate vicinity of land situated in India which is used for agricultural
purposes.
II. It is occupied by cultivator or receiver of rent or revenue.
III. It is used as (a) dwelling house; (b) store house; or (c) other out building.
IV. The land is assessed to land revenue or a local rate. If it is not assessed to land revenue or local rate
then such land should be situated outside urban area.
Income derived from any building or land arising from the use of such building or land for any purpose
(including letting for residential purpose or for the purpose of any business or profession) other than
agriculture shall not be agricultural income.

POINT TO BE NOTED-

A. MEANING OF ‘URBAN AREA’-The following two points define the scope of urban area-
I. any area situated within the local limits of a municipality/cantonment board having a
population of 10,000 or more;
II. any additional area beyond the local limits of the above mentioned municipality/cantonment
board to the extent provided in the table given below:
Population of the area as per the latest census Shortest aerial distance from the local limits of
published before the commencement of the relevant the municipality/ cantonment board mentioned
previous year under point (i)
> 10,000 and ≤ 1,00,000 2 kilometers
> 1,00,000 and ≤ 10,00,000 6 kilometers
> 10,00,000 8 kilometers

CASE-4 Identify-
S. No. Area Shortest aerial distance Population according to the last Would income
from the local limits of preceding census of which the relevant derived from farm
a municipality or figures have been published before the building situated in
cantonment board first day of the previous year this area be treated
as AI
(i) A 1 km 9,000 YES
(ii) B 1.5 kms 12,000 NO
(iii) C 2 kms 11,00,000 NO
(iv) D 3 kms 80,000 YES
(v) E 4 kms 3,00,000 NO
(v) F 5 kms 12,00,000 NO
(vi) G 6 kms 8,000 YES
(vii) H 7 kms 4,00,000 YES
(viii) I 8 kms 10,50,000 NO
(ix) J 9 kms 15,00,000 YES

SALEEM QURAISHEE Mo: 9175664444 INSPIRE ACADEMY-8888881719


AGRICULTURE INCOME- Page 10.4

2) INCOMES TREATED/DEEMED AS AGRICULTURAL INCOME


I. Income from sale of all types of crops & spices grown through basic & subsequent operation
II. Income from sale of flowers, trees, bamboo, timber, grass, etc. grown through basic &
subsequent operations
III. Income of a nursery derived from saplings or seedlings grown in nursery irrespective of the
fact whether basic operations are carried out or not [Explanation 3 to Section 2(1 A)
IV. Compensation received from an insurance company on account of damage caused to the crop

3) INCOMES NOT TREATED AS AGRICULTURAL INCOME


I. Income from sale of forests, trees, wild grass, flowers, etc. grown spontaneously without any
human effort
II. Income from breeding of livestock (i.e. animal husbandry)
III. Income from dairy farming, butter, cheese making, poultry farming, fisheries, etc.
IV. Income from sale of agricultural land (such income is taxable u/h capital gains)

No. Whether Agricultural Income Yes/No


1 Dividend from company whose entire income is derived from agriculture No
2 Interest on arrear of rent receivable in respect of agricultural land No
3 Natural growth of trees and its sale No
4 Directors remuneration as fixed % of turnover No
5 Interest received by money lender in form of agriculture produce No
6 Income from supply of water for irrigation purpose No
7 Income from lease of land for grazing of cattle required for agriculture purpose Yes
Salary to an active partner from a firm whose entire income is derived from agricultural Yes
8
operations
Interest on capital to any partner from a firm whose entire income is derived from agricultural Yes
9
operations
10 Sale of trees replanted in forest and subsequent operations Yes
11 Compensation received from insurance company for damage of any agricultural crop Yes
12 Income from growing commercial product like jute and cotton Yes
13 Income from growing flowers Yes
14 Salary received by an employee of an agricultural company No
Income from fisheries/poultry/Dairy farming, butter and cheese making / producing salt by No
15
flooding with sea water
Income earned by a cultivator from conservation of sugarcane (raised on own land) to jaggery No
16 is non-agriculture income to the extent the income is related to such conservation only. This is
because sugarcane itself is marketable
17 Income from agricultural land situated outside India is treated as non-agricultural INCOME IFOS

SALEEM QURAISHEE Mo: 9175664444 INSPIRE ACADEMY-8888881719


AGRICULTURE INCOME- Page 10.5

3. TAX TREATMENT OF AGRICULTURAL INCOME-


A. Agricultural Income Earned Outside India: No exemption is available in respect of agricultural income
earned outside India. The taxability of agricultural income earned outside India depends upon the
residential status of the assessee:
 In case of ROR, such income would always be taxable.
 In case of NOR & NR, such income would be taxable only if it is received directly in India.

B. Agricultural Income Earned in India: Agricultural income earned in India is exempt in the hands of the
assessee as per Section 10(1) of the Income Tax Act, 1961. Section 10(1) provides that agricultural
income is not to be included in the total income of the assessee. The reason for total exemption of
agricultural income from the scope of central income -tax is that under the Constitution, the Central
Government has no power to levy a tax on agricultural income. The provisions relating to partial
integration of agricultural income may apply in some cases. (Partial integration of agricultural income
has been discussed later)

POINT TO BE NOTED-

CASE-1 Company Carrying on Agricultural Operations:


I. Income from agricultural operations would be treated as agricultural income in the hands of
the company and thus exempt u/s 10(1).
II. Dividend distributed by a company carrying on agricultural operations shall be treated as
normal dividend on which a domestic company would be liable to pay AIT u/s 115-O. Dividend
received from a domestic company is exempt in the hands of the shareholders u/s 10(34).
Provisions of Section 115BBDA shall apply in case of resident shareholders.
III. Provisions of Section 115-O don't apply in case of foreign companies due to which dividend
received from foreign companies is taxable in the hands of the shareholders.

CASE-2 Partnership Firm Carrying on Agricultural Operations:


I. Income from agricultural operations would be treated as agricultural income in the hands of
the partnership firm and thus exempt u/s 10(1).
II. Share of profit from a partnership firm is exempt in the hands of the partner’s u/s 10(2A).
III. Generally, interest on capital and salary received by the partners from a partnership firm are
treated as income in the hands of the partners u/h ‘income from business/profession'.
However, where such salary or interest on capital is paid by a partnership firm earning income
through agricultural operations, such salary/interest on capital would be treated as agricultural
income in the hands of the partners as per the decision of the Supreme Court in the case of
R.M. Chidambaram Pillai vs CIT.

CASE-3 Salary Received by a Person Employed in Agricultural University:-Where any person is


employed in any agricultural university, the salary income received by him shall be taxable u/h
’income from salary'. Such salary shall not be considered as agricultural income in the hands of such
person.

SALEEM QURAISHEE Mo: 9175664444 INSPIRE ACADEMY-8888881719


AGRICULTURE INCOME- Page 10.6

4. PARTIALLY AGRICULTURAL AND PARTIALLY FROM BUSINESS-


INCOME NON AGRICULTURAL AGRICULTURE INCOME TAX RULE
INCOME INCOME
1. Growing and manufacturing 40% 60% Rule 8
tea in India
2. Growing and manufacturing 35% 65% Rule 7A
rubber
3. Sale of coffee grown and 25% 75% Rule 7B(1)
cured by seller
4. Sale of coffee grown, cured, 40% 60% Rule 7B(1A)
roasted and grounded by
seller in India.

5. Any other case (Rule 7)- For disintegrating a composite business income which is partly agriculture
and partly non agriculture ,the market value of any agriculture produce, raised by the assessee or
received by him as rent in kind and utilized as raw material in his business , is deducted. No further
deduction is permissible in respect of any expenditure incurred by the assessee as a cultivator or
receiver of rent in kind

PROBLEM:1 X Ltd. grows sugarcane to manufacture sugar. The data for the financial year 2019-20 is
`

Cost of cultivation of sugarcane 6,00,000

Market value of sugarcane when transferred to factory 10,00,000

Other manufacturing cost 6,00,000

Sales of Sugar 25,00,000

Salary of managing director who looks after all the operation of the company 3,00,000
Determine the income of company.
ANSWER:1 Business Income- ` 6,00,000 Agricultural Income ` 4, 00,000

PROBLEM:2 Mr. Ankush grows sugarcane and uses the same for the purpose of manufacturing sugar in
his factory. 30% of sugarcane produce is sold for Rs. 10 lacs, and the cost of cultivation of such
sugarcane is Rs. 5 lacs. The cost of cultivation of the balance sugarcane (70%) is Rs. 14 lacs and the
market value of the same is Rs. 22 lacs. After incurring Rs. 1.5 lacs in the manufacturing process on the
balance sugarcane, the sugar was sold for Rs. 25 lacs. Compute Mr. Ankush business income and
agricultural income.
ANSWER:2

SALEEM QURAISHEE Mo: 9175664444 INSPIRE ACADEMY-8888881719


AGRICULTURE INCOME- Page 10.7

PROBLEM:3 Mr. Kamal grows paddy and uses the same for the purpose of manufacturing of rice in his
own rice mill. The cost of cultivation of 40% of paddy produce is Rs 7,00,000 which is sold for Rs
15,00,000; and the cost of cultivation of balance 60% of paddy is Rs 12,00,000 and the market value of
such paddy is Rs 24,00,000.To manufacture the rice, he incurred Rs 2,00,000 in the manufacturing
process on the balance (60%) paddy. The rice was sold for Rs 30, 00,000. Compute the business income
and agriculture income of Mr. Kamal.
ANSWER:3

PROBLEM:4 Mr. C manufactures latex from the rubber plants grown by him in India. These are then sold
in the market for ` 30 lacs. The cost of growing rubber plants is ` 10 lacs and that of manufacturing
latex is ` 8 lacs. Compute his total income.
ANSWER:4

PROBLEM:5 As per Rule 7A income derived from the sale of Latex/Cenex/Block rubbers manufactured or
processed from rubber plants grown by seller in India will be disintegrated between business and
agricultural income in the ratio of:
(a) 35:65 (b) 30:60 (c) 0:100 (d) 25:75

PROBLEM:6 As per Rule 7B income derived from the sale of coffee grown & cured by seller in India will
be disintegrated between business and agricultural income in the ratio of:
(a) 35:65 (b) 30:60 (c) 0:100 (d) 25:75

PROBLEM:7 As per Rule 7B income derived from the sale of Coffee grown, cured, roasted and grounded
by seller in India will be disintegrated between business and agricultural income in the ratio of:
(a) 35:65 (b) 30:60 (c) 40:60 (d) 25:75

PROBLEM:8 As per Rule 8 income derived from the sale of Tea grown & manufactured by seller in India
will be disintegrated between business and agricultural income in the ratio of:
(a) 35:65 (b) 30:60 (c) 40:60 (d) 25:75

SALEEM QURAISHEE Mo: 9175664444 INSPIRE ACADEMY-8888881719


AGRICULTURE INCOME- Page 10.8

6. TAX ON NON-AGRICULTURAL INCOME IF THE ASSESSEE EARNS


AGRICULTURAL INCOME ALSO-
i. There is no tax on agricultural income but if an assesses has non-agricultural income as well as
agricultural income such agricultural income is included in his Total Income for the purpose of
computation of Income-tax on non-agricultural income.
ii. This is also known as partial integration of agricultural income with non-agricultural income or
indirect way of taxing agricultural income.
iii. Such partial integration is done only in the case of: (i) individual; (ii) HUF; (iii) AOP/BOI; (iv) Artificial
juridical person.
iv. It is not done in the case of; (i) firm; (ii) company, (iii) co-operative society; (iv) local authority.

The partial integration is done to compute the tax on non-agricultural income only when the following
two conditions are satisfied:
Condition-1-
Condition-2-
Computation of tax where there is agricultural income also: The following steps should be followed to
calculate the tax:

PROBLEM:9 Mr. X, a resident, has provided the following particulars of his income for the P.Y. 2019-20.

I.Income from salary (computed) - ` 3,40,000


II. Income from house property (computed) - ` 3,00,000
III.Agricultural income from a land in Jaipur - ` 1,80,000
IV. Expenses incurred for earning agricultural income - ` 1,20,000
V. Compute his tax liability assuming his age is -
a) 45 years
b) 70 years
ANSWER:9

SALEEM QURAISHEE Mo: 9175664444 INSPIRE ACADEMY-8888881719


AGRICULTURE INCOME- Page 10.9

PROBLEM:10 Mr. Kunal submits the following information regarding his income for the previous year
2019-20.
`
i) Gross salary per annum 5,76,000
ii) Rent received @ ` 2,500 p.m. 30,000
iii) Agricultural Income 60,000
iv) NSC (VIII Issue) purchased on 25-3-2020 20,000
Compute his taxable income and tax liability for the assessment year 2020-21.
ANSWER:10

PROBLEM:11 From the following information, compute the tax liability of R, for the assessment year
2020-21
Particulars `
Business Income 6,60,000
Receipt from sale of trees of spontaneous growth 1,00,000
Agricultural income 60,000
ANSWER:11

PROBLEM:12 Compute the tax liability of Hemant who is 62 years old and resident in India for
assessment year 2020-21
`

a) Rent of Agricultural land 1,00,000

Land revenue paid to State Government 10,000

Collection charges on recovery of agricultural rent 5,000

b) Interest on arrears of land revenue received from tenants 12,000

c) Income from manufacturing business carried on by Hemant 5,50,000


ANSWER:12

PROBLEM:13 R earns the following income during previous year 2019-20


`

Income from farm building 50,000

Income from sale of standing crops 1,00,000

Income from Ginning operation carried on by the owner of land on unginned cotton 4,00,000

Compute his tax liability for the assessment year 2020-21


ANSWER:13 `

SALEEM QURAISHEE Mo: 9175664444 INSPIRE ACADEMY-8888881719


AGRICULTURE INCOME- Page 10.10

PROBLEM:14 Mr. X has estates in Rubber, Tea and Coffee. He derives income from them. He has also a
nursery wherein he grows plants and sells. For the previous year ending 31.03.2020, he furnishes the
following particulars of his sources of income from estates and sale of Plants. You are requested to
compute the taxable income and tax liability for the assessment year 2020-21.
Rs.
(i) Manufacture of rubber 5,00,000
(ii) Manufacture of coffee grown and cured 3,50,000
(iii) Manufacture of tea 7,00,000
(iv) Sale of plants from nursery 1,00,000
ANSWER:14

PROBLEM:15 Mr. X, a 60 years old individual, is engaged in the business of roasting and grinding of
coffee, derives income Rs. 10 lacs during the financial year 2019-20. Compute the tax payable by him
assuming he has not earned any other income during the financial year 2020-21.
ANSWER:15

PROBLEM:16 Gross total income of S as computed under Income-tax Act, for the assessment year 2020-
21 is ` 520000. He deposits ` 20,000 in a PPF account. Compute the tax payable by S assuming that
he has agricultural income of –
(a)Nil; (b) ` 4,000; and (c) ` 80,000.
ANSWER:16

PROBLEM:17 R, aged 60 years, earned agricultural income of ` 2, 50,000 during the previous year 2019-
20. He deposits ` 40,000 in PPF account. Compute his tax liability assuming that he has non-
agricultural income before claiming deduction u/s 80C of:
(a)` 550000 (b) ` 630,000; (c) ` 850000.
ANSWER:17

PROBLEM:18 R is employed with G Ltd. on a salary of ` 800000 p.a. During the previous year R earned
agricultural income of ` 82,000. Compute the total income of R and his tax liability for the
assessment year 2020-21 assuming he paid life insurance premium of ` 10,000.
ANSWER:18 `

PROBLEM:19 Mr. Suresh has non-agricultural income of Rs. 6,50,000. Agricultural income earned of Rs.
50,000. Compute the income tax payable by him
ANSWER:19

PROBLEM:20 Gross total Income of A, aged 67 years, computed for assessment year 2020-21 is `
7,50,000 which includes long-term capital gains of ` 40,000 and winning of lotteries ` 30,000. He
contributed ` 50,000 towards PPF during the previous year. Compute the tax payable assuming his
agricultural income for the previous year was ` 60,000.
ANSWER:20 `

PROBLEM:21 Mr. Suresh has non-agricultural income of Rs. 50,0000. Agricultural income earned of Rs.
7,000. On what amount the partial integration would be applicable.
ANSWER:21

SALEEM QURAISHEE Mo: 9175664444 INSPIRE ACADEMY-8888881719


AGRICULTURE INCOME- Page 10.11

PROBLEM:22 Mr. Asim, a 60 years old individual, is engaged in the business of roasting and grounding of
coffee, derives income Rs. 10 lacs during the year. Compute the tax payable by him assuming he has
not earned any other income during the year.
ANSWER:22

PROBLEM:23 From the following information, compute taxable income and tax liability of Mrs. X for
the assessment year 2020-21.
`.
Income from business - letting cycles on hire 2,40,000
Fixed deposit interest received from companies on deposits made of sale proceeds of 18,000
land
Dividends from an Indian company having rubber plantations 6,000
Salary received as a partner from a firm growing and manufacturing tea 40,000
Sale of agricultural produce 1,75,000
Payment of government tax on agricultural lands 6,000
Expenses on power, irrigation cess and farm labour 10,000
Purchase of seeds 1,000
Tractor hire charges (for agricultural operations) 2,500
ANSWER:23

PROBLEM:24 Miss Kavita a resident and ordinarily resident in India, has derived the following income
for the year ended 31-3-2020.

I. Income from sale of centrifuged latex processed from rubber plants grown in 1,00,000
Darjeeling.

II. Income from sale of coffee grown and cured in Chennai 2,00,000

III. Income from sale of coffee grown, cured, Roasted and grounded in Colombo. 5,00,000

Sale Consideration was received in Chennai.

IV. Income from sale of tea grown and manufactured in Shimla. 10,00,000

V. Income from sapling and seedling grown in a nursery at Cochin. 2,00,000

Basic operations were not carried out by her on land.

You are required to compute the business income and agricultural Income of Miss Kavita for the
Assessment Year 2020-21.
ANSWER:24

SALEEM QURAISHEE Mo: 9175664444 INSPIRE ACADEMY-8888881719


AGRICULTURE INCOME- Page 10.12

PROBLEM:25 Total income of Mrs. JAYA computed for the AY 2020-2021 i.e. PY 2019-2020 is ` 6,00,000
which includes:

Long-term capital gains on sale of gold Rs. 90,000

Winning from lotteries Rs. 60,000

Short-term capital gains covered by Section 111 A Rs. 30,000

Agricultural Income earned by her was Rs. 1,50,000. Compute the tax payable by Mrs. JAYA.
ANSWER:25

PROBLEM:26 Mr. JOHN, aged 58 years, has Income under the head House Property Rs. 3,25,000,
agricultural Income of Rs. 1,00,000, Long term capital gain amounting to Rs. 45,000 and casual
Income Rs. 35,000. He is eligible for deduction under section 80C for Rs. 20,000. Compute tax liability
for the AY 2020-2021 i.e. PY 2019-2020.
ANSWER:26

SALEEM QURAISHEE Mo: 9175664444 INSPIRE ACADEMY-8888881719


AGRICULTURE INCOME- Page 10.13

7. MCQ
PROBLEM:1 As per section 2(1 A), agricultural income includes ................
(a) Rent or revenue derived from land
(b) Income derived from land by processing of agricultural produce
(c) Income from farm building (d) All of the above

PROBLEM:2 Land is said to be used for agricultural purpose when, which of the following operations
are carried out?
(a) Basic Operations (b) Subsequent Operations (c) Primary Operations (d) Both (a) and (b)

PROBLEM:3 Which of the following will be considered as agricultural income?


(a) Share of agricultural produce received by a landlord from the tenant as rent
(b) Dividend received from a company carrying on agricultural operations
(c) Commission earned by a broker for selling agricultural produce
(d) Capital gain arising from sale of agricultural land

PROBLEM:4 The process which is performed to make an agricultural produce marketable or saleable is
known as ................
(a) Selling Process (b) Marketing Process (c) Both (a) and (b) (d) None of the above

PROBLEM:5 Which of the following condition/s is/are to be fulfilled in order to treat the income from
building as agricultural income?
(a) Building is occupied by cultivator or receiver of rent or revenue
(b) It is used as dwelling house or store house
(c) Building is in the immediate vicinity of the land which is used for agricultural purpose
(d) All of the above

PROBLEM:6 Income from farm house situated in ................ will not be considered as agricultural
income, unless the land on which farm house is situated is assessed to land revenue.
(a) Rural Area (b) Urban Area (c) Both (a) and (b) (d) None of the above

PROBLEM:7 Which of the following is not an agricultural income?


(a) Income from toddy received by actual cultivator of trees
(b) Compensation received from an insurance company on account of damage caused to crop
(c) Income from breeding of livestock (d) Income from growing creepers

PROBLEM:8 Profit on sale of standing crops purchased by the assessee is ................


(a) Agricultural income (b) Non-agricultural income
(c) Semi-agricultural income (d) None of the above

PROBLEM:9 Income from sale of seeds derived on account of cultivation by assessee I is ................
(a) Agricultural income (b) Non-agricultural income
(c) Semi-agricultural income (d) None of the above

SALEEM QURAISHEE Mo: 9175664444 INSPIRE ACADEMY-8888881719


AGRICULTURE INCOME- Page 10.14

PROBLEM:10 Agricultural income is exempt provided the:


a) Land is situated in India b) Land is situated in any rural area in India
c) Land is situated whether in India or outside India. d) None of the above

PROBLEM:11 Which of the following income is agricultural income —


(a) Rent received from agricultural land (b) Income from dairy farm
(c) Income from poultry farm (d) Dividend from a company engaged in agriculture.

PROBLEM:12 Which of the following income is an agricultural income -


(a) Income from brick making (b) Income from agriculture land situated in Pakistan
(c) Prize from government on account of higher
(d) Compensation received from insurance crop Yield Company on account of loss of crop.

PROBLEM:13 Agriculture income includes which of the following income –


(a) Rent derived from land.
(b) Income derived from agricultural land by agricultural operations.
(c) Income from farm building. (d) All of the above.

PROBLEM:14 Which of the following is the condition for applicability of partial integration of
agricultural income?
(a) The taxpayer is a firm. (b) Agricultural income exceeds Rs. 10,000.
(c) The non-agricultural 'income exceeds the maximum amount not chargeable to tax.
(d) All of the above.

PROBLEM:15 Partial integration is applicable when the taxpayer is:


(a) HUF (b) An Individual
(c) BOI or an AOP or artificial juridical person (d) Any of the above

PROBLEM:16 An assessee has incurred Rs1,00,000 on the cultivation of agricultural produce.50% of the
produce has been sold for Rs1,10,000 and the balance 50% has been used by the assessee for his self-
consumption, the agricultural income in this case shall be:
a) Rs 10,000 b) Rs 60,000 c) Rs 1, 20,000 d) Rs 1, 50,000

PROBLEM:17 The partial integration of agricultural income with non -agricultural income is done in case
of:
a) All assessee b) any assessee other than who is liable to be taxed at flat rate of income tax
c) Individual HUF, AOP or BOI d) artificial judicial person

PROBLEM:18 Agricultural income is:


a) Fully exempt b) partially taxable) fully taxable d) none of these

SALEEM QURAISHEE Mo: 9175664444 INSPIRE ACADEMY-8888881719


AGRICULTURE INCOME- Page 10.15

PROBLEM:19 If a firm earns agricultural income, it will be exempt:


a) In the hands of firm b) In the hands of firm but taxable in the hands of the partners
c) In the hands of firm as well its partners
d) In the hands of the firm as well its partners but would be included in the other income of partners
for computation of tax on his other incomes.

PROBLEM:20 If a company declares dividend out of agricultural income, such dividend declared by the
company shall be:
a) Exempt in the hands of the shareholder but dividend tax will be payable by the company
b) Not be subject to any income tax, either in the hands of company or the shareholder
c) Included in the total income of the shareholder d) None of these

PROBLEM:21 There will be no partial integration of agricultural income with non-agricultural income, if
the non-agricultural income of A who is less than 60 years does not exceed:
a) Rs 5,000 b) Rs 2, 50,000 c) Rs 1, 80,000

PROBLEM:22 There will be no partial integration, if the agricultural income does not exceed:
a) Rs 2, 50,000 b) Rs 2, 00,000 c) Rs 5,000

PROBLEM:23 Income derived from rubber plantation in Singapore but received in India shall be treated
as
a) Agricultural income and hence exempt
b) Agricultural income but taxable under the head income from other source
c) Exempt as earned outside India. d) None of these

PROBLEM:24 Dividend received by a shareholder from an Indian company the whole of whose income
is agricultural income shall be treated as:
a) Agricultural income in the hands of shareholder and thus exempt
b) Agricultural income and thus exempt but it will be subject to partial integration
c) Exempt under section 10(34) but taxable in the hands of the company
d) Income taxable under the head income from other sources

PROBLEM:25 Which of the following would be agricultural income -?


(a) Income from breeding of livestock (b) Income from poultry farming
(c) Rent received from land used for movie shooting
(d) Rent received from land used for grazing of cattle required for agricultural activities

PROBLEM:26 Mr. Karan earned income of Rs.4,00,000 from sale of tea grown and manufactured in
Shimla. Income from sapling and seedling grown in nursery at Cochin is Rs.80,000. The basic
operations were not carried out by her on land. Her agricultural income is
(a) Rs.4,80,000 (b) Rs.4,00,000 (c) Rs.2,40,000 (d) Rs.3,20,000

SALEEM QURAISHEE Mo: 9175664444 INSPIRE ACADEMY-8888881719


AGRICULTURE INCOME- Page 10.16

PROBLEM:27 Mr. Pawan earned income of Rs.22 lakhs from manufacture and sale of coffee grown,
cured, roasted and grounded by him in India. The business income chargeable to tax in his hands
would be -
(a) Rs.8,80,000 (b)Rs.5,50,000 (c) Rs.13,20,000 (d) Rs.16,50,000

PROBLEM:28 Mr. A has agricultural land which he has given on rent, in this case income shall be
(a) taxable under the head other Sources (b) taxable under the head house property
(c) taxable under the head Business/Profession (d) Agricultural income
(e) none of these

PROBLEM:29 Mr. JATIN has agricultural land in Delhi which he has sold and has earned gains of Rs. 10
Lakhs . This is his:
(a) Agriculture income (b) Income from salary
(c) Income from capital gains (d) Income from other sources

PROBLEM:30 Mr. J is employed in MP Agricultural University and getting a Basic Pay of Rs. 20,000 pm
and claims that it is his Agricultural Income? State whether it is:
(a) exempt as Agriculture income (b) chargeable under the head of salary
(c) chargeable under the head of business and profession (d) None of the above

SALEEM QURAISHEE Mo: 9175664444 INSPIRE ACADEMY-8888881719

You might also like